postgraduate.kaznmu.kz€¦ · web view58 Больная И, 28 лет, доставлена с...

240
С.Ж.АСФЕНДИЯРОВАТЫНДАҒЫҚАЗАҚ ҰЛТТЫҚ МЕДИЦИНА УНИВЕРСИТЕТІ КАЗАХСКИЙНАЦИОНАЛЬНЫЙМЕДИЦИНСКИЙ УНИВЕРСИТЕТ ИМЕНИ С.Д.АСФЕНДИЯРОВА КАФЕДРА ИНТЕРНАТУРЫ И РЕЗИДЕНТУРЫ ПО АКУШЕРСТВУ И ГИНЕКОЛОГИИ ТЕСТОВЫЕ ЗАДАНИЯ ДЛЯ ПРОВЕДЕНИЯ ИТОГОВОЙ ГОСУДАРСТВЕННОЙ АТТЕСТАЦИИ РЕЗИДЕНТОВ ВЫПУСКА 2012-2013 ГГ. ПО СПЕЦИАЛЬНОСТИ «АКУШЕРСТВО И ГИНЕКОЛОГИЯ» «СОГЛАСОВАНО» Директор высшей медицинской школы проф. __________ А.В. Балмуханова «_____»_______________________2014 г. «УТВЕРЖДАЮ» Проректор по постдипломному образованию и межрегиональному сотрудничеству проф. _____________ А.Д. Дуйсекеев «_____»_______________2014г. Тестовые задания для проведения итогового экзамена в рамках государственной аттестации резидентов выпуска 2013-2014 гг. по специальности «акушерство и гинекология».

Upload: others

Post on 07-Apr-2020

10 views

Category:

Documents


0 download

TRANSCRIPT

Page 1: postgraduate.kaznmu.kz€¦ · Web view58 Больная И, 28 лет, доставлена с жалобами на внезапно возникшую боль внизу живота

С.Ж.АСФЕНДИЯРОВАТЫНДАҒЫҚАЗАҚ ҰЛТТЫҚ МЕДИЦИНА

УНИВЕРСИТЕТІ

КАЗАХСКИЙНАЦИОНАЛЬНЫЙМЕДИЦИНСКИЙ УНИВЕРСИТЕТ ИМЕНИ С.Д.АСФЕНДИЯРОВА

КАФЕДРА ИНТЕРНАТУРЫ И РЕЗИДЕНТУРЫ ПО АКУШЕРСТВУ И ГИНЕКОЛОГИИТЕСТОВЫЕ ЗАДАНИЯ ДЛЯ ПРОВЕДЕНИЯ ИТОГОВОЙ ГОСУДАРСТВЕННОЙ АТТЕСТАЦИИ РЕЗИДЕНТОВ

ВЫПУСКА 2012-2013 ГГ. ПО СПЕЦИАЛЬНОСТИ «АКУШЕРСТВО И ГИНЕКОЛОГИЯ»

«СОГЛАСОВАНО»

Директор высшей медицинской школы проф. __________ А.В. Балмуханова

«_____»_______________________2014г.

«УТВЕРЖДАЮ»Проректор по постдипломному

образованию и межрегиональному сотрудничеству

проф. _____________ А.Д. Дуйсекеев «_____»_______________2014г.

Тестовые заданиядля проведения итогового экзамена в рамках

государственной аттестации резидентов выпуска 2013-2014 гг.по специальности «акушерство и гинекология».

Алматы- 2014г

Page 2: postgraduate.kaznmu.kz€¦ · Web view58 Больная И, 28 лет, доставлена с жалобами на внезапно возникшую боль внизу живота

С.Ж.АСФЕНДИЯРОВАТЫНДАҒЫҚАЗАҚ ҰЛТТЫҚ МЕДИЦИНА

УНИВЕРСИТЕТІ

КАЗАХСКИЙНАЦИОНАЛЬНЫЙМЕДИЦИНСКИЙ УНИВЕРСИТЕТ ИМЕНИ С.Д.АСФЕНДИЯРОВА

КАФЕДРА ИНТЕРНАТУРЫ И РЕЗИДЕНТУРЫ ПО АКУШЕРСТВУ И ГИНЕКОЛОГИИТЕСТОВЫЕ ЗАДАНИЯ ДЛЯ ПРОВЕДЕНИЯ ИТОГОВОЙ ГОСУДАРСТВЕННОЙ АТТЕСТАЦИИ РЕЗИДЕНТОВ

ВЫПУСКА 2012-2013 ГГ. ПО СПЕЦИАЛЬНОСТИ «АКУШЕРСТВО И ГИНЕКОЛОГИЯ»

Тестовые задания утверждены на заседании кафедры интернатуры и резидентуры по акушерству и гинекологии, протокол №___ от «___» ________ 2014 г.

Зав.кафедрой, проф. Кудайбергенов Т.К. ___________________

Тестовые задания утверждены на заседании КОП по хирургическим болезням, протокол №___ от «___» ________ 2014 г.

Председатель, к.м.н. Садуакасова Ш.М._____________

Тестовые задания утверждены на заседании методического совета университета, протокол №___ от «__» __________ 2014 г.

Председатель МС, проф. Тулебаев А.К.___________________

Декан резидентуры и докторантуры, д.м.н. Максутова Д.Ж._____________________

Page 3: postgraduate.kaznmu.kz€¦ · Web view58 Больная И, 28 лет, доставлена с жалобами на внезапно возникшую боль внизу живота

С.Ж.АСФЕНДИЯРОВАТЫНДАҒЫҚАЗАҚ ҰЛТТЫҚ МЕДИЦИНА

УНИВЕРСИТЕТІ

КАЗАХСКИЙНАЦИОНАЛЬНЫЙМЕДИЦИНСКИЙ УНИВЕРСИТЕТ ИМЕНИ С.Д.АСФЕНДИЯРОВА

КАФЕДРА ИНТЕРНАТУРЫ И РЕЗИДЕНТУРЫ ПО АКУШЕРСТВУ И ГИНЕКОЛОГИИТЕСТОВЫЕ ЗАДАНИЯ ДЛЯ ПРОВЕДЕНИЯ ИТОГОВОЙ ГОСУДАРСТВЕННОЙ АТТЕСТАЦИИ РЕЗИДЕНТОВ

ВЫПУСКА 2012-2013 ГГ. ПО СПЕЦИАЛЬНОСТИ «АКУШЕРСТВО И ГИНЕКОЛОГИЯ»

Гинекология 1 Больной 23 лет произведена гистеросальпингография по поводу первичного бесплодия. На рентгенограмме: полость матки Т-образной формы, маточные трубы укорочены, ригидные, с булавовидными расширениями в ампулярных отделах; выхода контрастного вещества в брюшную полость не наблюдается. Для какого заболевания наиболее характерны подобные изменения: A. рак маточных труб B. туберкулез половых органовC. аденомиоз и эндометриоз маточных труб D. хронический сальпингит гонорейной этиологииE. хронический сальпингит хламидийной этиологии2 У 18 летней девушки не было ни одной менструации. Рост 140 см, недоразвитые грудные железы, матка маленькая и гипоплазия наружных половых органов, отсутствует половое оволосение. Результаты гормонального исследования следующие: - эстрадиол сыворотки крови 8 пг/мл (норм 23-45)- ЛГ -105 МЕ/мл (норм 2-15)- ФСГ -120 МЕ/мл (норм 2-20)- ТТГ - 1,8 мкЕ/мл (норм 0,1-4,5) При каком их перечисленных диагнозов отмечается подобная клиническая картина?A. аденома гипофизаB. гиперпролактинемияC. гипофизарная недостаточностьD. гипофизарно-гипоталамическая недостаточностьE. яичниковая недостаточность (дисгенезия гонад)3 У.К., 25 лет после обследования по поводу вторичного бесплодия в течение одного года обнаружена непроходимость маточных труб в ампулярных отделах. Какое лечение необходимо провести в первую очередь?A. гидротубациюB. хирургическоеC. физиотерапевтическоеD. провокацию с пирогеналомE. провокацию гоновакциной в сочетании с антибиотикотерапией4 Пациентке 30 лет, жалобы на бесплодие во втором браке. Имела в первом браке нормальные роды и два больничных аборта. Менструальная функция без патологии. В зеркалах: шейка матки без изменений, светлые бели в небольшом количестве. РV: без патологии. Муж обследован, спермограмма не изменена. Сколько факторов женского бесплодия существует по протоколу ВОЗ?A. дваB. десятьC. восемнадцатьD. двадцать дваE. двадцать пять5 Больная 34 лет обратилась в женскую консультацию с жалобами на первичное бесплодие в течение 7 лет. Менструальная функция не нарушена. Половая жизнь регулярная, в браке, от беременности не предохранялась, беременности не наступали. С помощью базальной температуры установлен двухфазный цикл. Согласно результатам рентгенотелевизионной гистеросальпингографии маточные трубы проходимы. При исследовании спермограммы мужа

Page 4: postgraduate.kaznmu.kz€¦ · Web view58 Больная И, 28 лет, доставлена с жалобами на внезапно возникшую боль внизу живота

С.Ж.АСФЕНДИЯРОВАТЫНДАҒЫҚАЗАҚ ҰЛТТЫҚ МЕДИЦИНА

УНИВЕРСИТЕТІ

КАЗАХСКИЙНАЦИОНАЛЬНЫЙМЕДИЦИНСКИЙ УНИВЕРСИТЕТ ИМЕНИ С.Д.АСФЕНДИЯРОВА

КАФЕДРА ИНТЕРНАТУРЫ И РЕЗИДЕНТУРЫ ПО АКУШЕРСТВУ И ГИНЕКОЛОГИИТЕСТОВЫЕ ЗАДАНИЯ ДЛЯ ПРОВЕДЕНИЯ ИТОГОВОЙ ГОСУДАРСТВЕННОЙ АТТЕСТАЦИИ РЕЗИДЕНТОВ

ВЫПУСКА 2012-2013 ГГ. ПО СПЕЦИАЛЬНОСТИ «АКУШЕРСТВО И ГИНЕКОЛОГИЯ»

выявлены астено- и олигозооспермия II степени. После лечения мужа андрологом сохраняется олигозооспермия II степени. Укажите возможную причину бесплодия:A. хроническая ановуляцияB. субфертильность спермы мужаC. нарушение проходимости маточных трубD. недостаточность первой фазы менструального циклаE. недостаточность лютеиновой фазы менструального цикла6 Больная 32 лет предъявляет жалобы на нерегулярные месячные, бесплодие в течение 2 лет, наличие выделений молочного цвета из сосков. За последние два года прибавила в весе. В анамнезе две беременности: срочные роды без осложнений, медицинский аборт. Месячных нет в течение 7 мес. При осмотре: рост 160 см, масса 70 кг. Из сосков молочных желез при легком надавливании выделяется молоко. РV: шейка цилиндричекой формы, зев закрыт. Матка нормальной величины, придатки не определяются. При каком из перечисленных диагнозов отмечается подобная клиническая картина?A. вторичное бесплодие, ожирение 1 степениB. аменорея, первичное бесплодие, ожирениеC. аменорея, вторичное бесплодие, гиперпролактинемияD. нарушение менструального цикла, вторичное бесплодиеE. вторичное бесплодие, галакторрея, ожирение 1 степени7 Пациентка 30 лет, жалобы на бесплодие во втором браке. Имела в первом браке нормальные роды, и три больничных аборта. Менструальная функция не нарушена. Бимануальное исследование не обнаружило каких либо отклонений от нормы. Муж обследован, спермограмма не изменена. Какая продолжительность обследования супружеской пары (выяснение причины бесплодия) по протоколу ВОЗ?A. 4 месяцаB. 2 годаC. 3 годаD. 4 годаE. 5 лет8 Больная Н. 27 лет обратилась к врачу с жалобами на бесплодие. Из анамнеза: менструальная функция не нарушена. Акушерский анамнез: I беременность закончилась искусственным абортом в сроке 9-10 недель без осложнений, II-III - неразвивающиеся беременности в сроке 6-7 недель. В последние 4 года не предохранялась, не беременеет. Пациентка спортивного типа телосложения, масса 64 кг, рост 169 см. Отмечается рост единичных волос на верхней губе, вокруг сосков, по белой линии живота и на бедрах. Кожа лица пористая, жирная. Молочные железы несколько уменьшены в размерах, выделений из сосков нет. Гинекологическое исследование: отмечается незначительная гипертрофия малых половых губ и пигментация наружных органов. На зеркалах: слизистая влагалища и шейки матки чистая, выделения скудные, слизистые. PV: шейка матки субконической формы, матка не увеличена, придатки не определяются, своды свободны. Выделения светлые. Дополнительные методы исследования: ТФД- двухфазная базальная температура с укорочением II фазы до 3-4 дней. УЗИ - матка и яичники без изменений. Гормональные исследования -17-КС 20 мг/сут. Предполагаемый диагноз?A. синдром Киари-ФромеляB. посткастрационный синдромC. адреногенитальный синдромD. синдром поликистозных яичниковE. нейро-обменно-эндокринный синдром

Page 5: postgraduate.kaznmu.kz€¦ · Web view58 Больная И, 28 лет, доставлена с жалобами на внезапно возникшую боль внизу живота

С.Ж.АСФЕНДИЯРОВАТЫНДАҒЫҚАЗАҚ ҰЛТТЫҚ МЕДИЦИНА

УНИВЕРСИТЕТІ

КАЗАХСКИЙНАЦИОНАЛЬНЫЙМЕДИЦИНСКИЙ УНИВЕРСИТЕТ ИМЕНИ С.Д.АСФЕНДИЯРОВА

КАФЕДРА ИНТЕРНАТУРЫ И РЕЗИДЕНТУРЫ ПО АКУШЕРСТВУ И ГИНЕКОЛОГИИТЕСТОВЫЕ ЗАДАНИЯ ДЛЯ ПРОВЕДЕНИЯ ИТОГОВОЙ ГОСУДАРСТВЕННОЙ АТТЕСТАЦИИ РЕЗИДЕНТОВ

ВЫПУСКА 2012-2013 ГГ. ПО СПЕЦИАЛЬНОСТИ «АКУШЕРСТВО И ГИНЕКОЛОГИЯ»

9 В анамнезе у больной 32 лет бесплодие, двухсторонний аднексит с частыми обострениями. При поступлении высокая температура, озноб, боли по всему животу.Объективно: температура 39,2º С, пульс 128 в 1 мин., АД 110 и 70 мм рт. ст., ЧД 22 в 1 мин. Язык сухой, живот вздут, напряженный во всех отделах, болезненный с выраженным симптомом Щеткина по всему животу.Гинекологическое исследование: наружные половые органы развито правильно, оволосение по женскому типу. В зеркалах: слизистая влагалища не изменена, шейка матки с нарушением эпителиального покрова в области наружного зева. Бели слизистые, мутноватые, умеренные, без зловонного запаха. PV: смещения шейки матки резко болезненные, придатки и матку определить не удается из-за болезненности. Задний свод влагалища уплощен, болезненный. Предполагаемый диагноз?A. пельвиоперитонитB. разлитой перитонит C. эндометриоз придатковD. абсцесс Дугласова пространства.E. хронический двусторонний аднексит10 Больная 26 лет в детстве перенесла туберкулез. На «Д» учете в туберкулезном диспансере не состоит. Периодически беспокоили боли в животе. В браке 3 года, беременность не наступает. Менархе с 14 лет, последние 5 лет менструации стали короткими и скудными, через 27-28 дней. При специальном гинекологическом исследовании патологических изменений не выявлено. Предполагаемый диагноз:A. генитальный и экстрагенитальный эндометриозB. синдром поликистозных яичниковC. туберкулез гениталийD. хронический аднекситE. хламидиоз гениталий11 Больная В. 28 лет обратилась к врачу по поводу отсутствия беременности в течение 3 лет. Менархе с 13 лет, менструальный цикл не нарушен. I беременность - роды 5 лет назад. Через 3 месяца после родов установили ВМС. Через 1,5 года была госпитализирована в гинекологическое отделение по поводу острого двухстороннего сальпингита, удалена ВМС, проведено консервативное лечение. OZ: без патологии. PV: Матка, не увеличена, подвижная, безболезненная. Слева пальпируются тяжистые придатки, справа – безболезненное образование размером 5х6 см, округлой формы, туго - эластической консистенции. Наиболее вероятная причина бесплодия?A. СКПЯ B. эндометрит C. пельвиоперитоитD. хронический аднекситE. тубоовариальное образование12 Больная И. 31 года обратилась к врачу по поводу бесплодия. Из анамнеза - менструации с 14 лет, цикл регулярный, менструации по 3-4 дня, через 28 дней, болезненные. В браке с 20 лет, половая жизнь регулярная, беременность не наступает. Объективно – женщина низкого роста, черты лица – инфантильные. OZ: без патологии. PV: Влагалище узкое. Шейка матки коническая, подвижная, безболезненная. Матка в антефлексио, подвижная, безболезненная, маленькая. Придатки слева не пальпируются, справа – пальпируется округлое образование диаметром 6х6 см, туго - эластической консистенции, безболезненное. На УЗИ: Придатки слева не лоцируются, справа: яичник размерами 4,5х5,5 см, между маткой и правым яичником лоцируется жидкостное образование диаметром 5,8х6,5 см. Ваш диагноз?A. СКПЯ

Page 6: postgraduate.kaznmu.kz€¦ · Web view58 Больная И, 28 лет, доставлена с жалобами на внезапно возникшую боль внизу живота

С.Ж.АСФЕНДИЯРОВАТЫНДАҒЫҚАЗАҚ ҰЛТТЫҚ МЕДИЦИНА

УНИВЕРСИТЕТІ

КАЗАХСКИЙНАЦИОНАЛЬНЫЙМЕДИЦИНСКИЙ УНИВЕРСИТЕТ ИМЕНИ С.Д.АСФЕНДИЯРОВА

КАФЕДРА ИНТЕРНАТУРЫ И РЕЗИДЕНТУРЫ ПО АКУШЕРСТВУ И ГИНЕКОЛОГИИТЕСТОВЫЕ ЗАДАНИЯ ДЛЯ ПРОВЕДЕНИЯ ИТОГОВОЙ ГОСУДАРСТВЕННОЙ АТТЕСТАЦИИ РЕЗИДЕНТОВ

ВЫПУСКА 2012-2013 ГГ. ПО СПЕЦИАЛЬНОСТИ «АКУШЕРСТВО И ГИНЕКОЛОГИЯ»

B. эндометриозC. аплазия обоих придатковD. адреногенитальный синдромE. аплазия левых придатковсактосальпинкс справа13 Больная Л.29 лет поступила в гинекологическое отделение больницы с жалобами на отсутствие беременности в течение 4 лет, общую слабость, потливость, временами повышение температуры до 37,2 – 37,5 оС, периодически отмечает ноющие боли внизу живота. Из анамнеза: без особенностей. 3 года назад имела контакт с туберкулезным больным.Месячные с 13 лет, регулярные, без патологии. Последние 3 года менструации стали иногда задерживаться на 5-10 дней. Половая жизнь с 25 лет без применения контрацептивных средств. Через 2 года после начала половой жизни обследована: гистеросальпингография – трубы не проходимы, «четкообразной» формы. Наиболее вероятный диагноз:A. хламидиоз гениталийB. хронический аднекситC. туберкулез гениталийD. синдром поликистозных яичниковE. генитальный и экстрагенитальный эндометриоз14 С чего начинается диагностика женского бесплодия: A. кульдоскопии B. гистероскопии C. лапароскопииD. теста Шуварского-Симса-ХунераE. тестов функциональной диагностики15 При обследовании бесплодной пары в первую очередь необходимо провести следующее обследование: A. исследование спермы B. гистеросальпингографию C. цитологию влагалищных мазковD. определение базальной температуры E. гистологическое исследования эндометрия 16 Больная Н. 27 лет обратилась к врачу с жалобами на бесплодие. Из анамнеза: менструальная функция не нарушена. Акушерский анамнез: I беременность закончилась искусственным абортом в сроке 9-10 недель без осложнений, 2-3 неразвивающиеся беременности в сроке 6-7 недель. В последние 4 года не предохранялась, не беременеет. Пациентка спортивного типа телосложения, масса 64 кг, рост 169 см. Отмечается рост единичных волос на верхней губе, вокруг сосков, по белой линии живота и на бедрах. Кожа лица пористая, жирная. Молочные железы несколько уменьшены в размерах, выделений из сосков нет. Гинекологическое исследование: отмечается незначительная гипертрофия малых половых губ и пигментация наружных органов. OZ: без патологии. PV: шейка матки субконической формы, матка не увеличена, придатки не определяются, своды свободны. Дополнительные методы исследования: ТФД- двухфазная базальная температура с укорочением II фазы до 3-4 дней; УЗИ-матка и яичники без изменений; гормональные исследования-17-КС 20 мг/сут. Дифференциальную диагностику необходимо проводить с: A. миомой матки B. эндометриозом C. доброкачественными опухолями яичников D. воспалительными заболеваниями придатков матки E. гормонально-активными опухолями яичников и надпочечников

Page 7: postgraduate.kaznmu.kz€¦ · Web view58 Больная И, 28 лет, доставлена с жалобами на внезапно возникшую боль внизу живота

С.Ж.АСФЕНДИЯРОВАТЫНДАҒЫҚАЗАҚ ҰЛТТЫҚ МЕДИЦИНА

УНИВЕРСИТЕТІ

КАЗАХСКИЙНАЦИОНАЛЬНЫЙМЕДИЦИНСКИЙ УНИВЕРСИТЕТ ИМЕНИ С.Д.АСФЕНДИЯРОВА

КАФЕДРА ИНТЕРНАТУРЫ И РЕЗИДЕНТУРЫ ПО АКУШЕРСТВУ И ГИНЕКОЛОГИИТЕСТОВЫЕ ЗАДАНИЯ ДЛЯ ПРОВЕДЕНИЯ ИТОГОВОЙ ГОСУДАРСТВЕННОЙ АТТЕСТАЦИИ РЕЗИДЕНТОВ

ВЫПУСКА 2012-2013 ГГ. ПО СПЕЦИАЛЬНОСТИ «АКУШЕРСТВО И ГИНЕКОЛОГИЯ»

17 В анамнезе у больной 32 лет бесплодие, двухсторонний аднексит с частыми обострениями. При поступлении высокая температура, озноб, боли по всему животу.Объективно: температура 39,2ºС, пульс 128 в 1 мин., АД 110 и 70 мм рт. ст., ЧД 22 в 1 мин. Язык сухой, живот вздут, напряженный во всех отделах, болезненный с выраженным симптомом Щеткина по всему животу.OZ: слизистая влагалища не изменена, шейка матки с нарушением эпителиального покрова в области наружного зева. Бели слизистые, мутноватые, умеренные, без зловонного запаха. PV: смещения шейки матки резко болезненные, придатки и матку определить не удается из-за болезненности. Задний свод влагалища уплощен, болезненный. Наиболее вероятная тактика: A. гормональная терапия B. оперативное лечение C. консервативная терапия D. антибактериальная терапия E. пункция через задний свод влагалища18 Больная 26 лет в детстве перенесла туберкулез. На «Д» учете в туберкулезном диспансере не состоит. Периодически беспокоили боли в животе. В браке 3 года, беременность не наступает. Менархе с 14 лет, последние 5 лет менструации стали короткими и скудными, регулярными через 27-28 дней. При специальном гинекологическом исследовании патологических изменений не выявлено. Какое исследование подтвердит диагноз у данной пациентки? A. УЗИ B. ГСГ C. кольпоскопия D. гистероскопияE. мазок на микроскопию.19. Больная 26 лет в детстве перенесла туберкулез. На «Д» учете в туберкулезном диспансере не состоит. Периодически беспокоили боли в животе. В браке 3 года, беременность не наступает. Менархе с 14 лет, последние 5 лет менструации стали короткими и скудными, регулярными через 27-28 дней. При специальном гинекологическом исследовании патологических изменений не выявлено. Наиболее вероятный диагностический признак для выявления этиологии бесплодия у данной пациентки:A. обнаружение эрозированного эктропионаB. обнаружение тубоовариального образования C. обнаружение гонококков и другой патогенной флорыD. на рентгенограмме - маточные трубы «четкообразной» формыE. обнаружение синехий в полости матки, эндометриоидных «глазков» 20. Больная В. 28 лет обратилась к врачу по поводу отсутствия беременности в течение 3 лет. Менархе с 13 лет, без патологии. Первая беременность закончилась родами в 23-летнем возрасте. До первой беременности контрацептивами не пользовалась. Через 3 месяца после родов установили ВМС. Месячный цикл не нарушился. Через 1,5 года была госпитализирована в гинекологическое отделение по поводу острого двухстороннего сальпингита, удалена ВМС, проведено консервативное лечение. OZ: без патологии. PV: Матка в антефлексио -верзио, не увеличена, подвижная, безболезненная. Слева пальпируются тяжистые, плотные придатки, справа – безболезненное образование размером 5х6 см, округлой формы туго-эластической консистенции.Наиболее информативный метод обследования: A. УЗИ B. ГСГ C. диагностическая лапароскопия

Page 8: postgraduate.kaznmu.kz€¦ · Web view58 Больная И, 28 лет, доставлена с жалобами на внезапно возникшую боль внизу живота

С.Ж.АСФЕНДИЯРОВАТЫНДАҒЫҚАЗАҚ ҰЛТТЫҚ МЕДИЦИНА

УНИВЕРСИТЕТІ

КАЗАХСКИЙНАЦИОНАЛЬНЫЙМЕДИЦИНСКИЙ УНИВЕРСИТЕТ ИМЕНИ С.Д.АСФЕНДИЯРОВА

КАФЕДРА ИНТЕРНАТУРЫ И РЕЗИДЕНТУРЫ ПО АКУШЕРСТВУ И ГИНЕКОЛОГИИТЕСТОВЫЕ ЗАДАНИЯ ДЛЯ ПРОВЕДЕНИЯ ИТОГОВОЙ ГОСУДАРСТВЕННОЙ АТТЕСТАЦИИ РЕЗИДЕНТОВ

ВЫПУСКА 2012-2013 ГГ. ПО СПЕЦИАЛЬНОСТИ «АКУШЕРСТВО И ГИНЕКОЛОГИЯ»

D. гистерскопия E. пункционная биопсия 21 При обследовании Т. 22 лет по поводу первичной аменореи обнаружено: половой хроматин 0, хромосомный набор 44 ХУ. При каком из перечисленный диагнозов отмечается подобная клиническая картина:A. синдром Шерешевского – ТернераB. синдром ШиханаC. синдром Лоренса- Муна – БидляD. синдром Морриса (тестикуляроной феминизации)E. синдром Бабинского – Фрелиха22 Какое обследование наиболее приемлемо в первую очередь при аменорее надпочечникового генеза?A. определение прегнандиола в мочеB. определение прогестерона в кровиC. определение 17-кетостероидов в мочеD. определение фолликулостимулирующего гормона в кровиE. определение эстрогенов в крови23 К. 25 лет обратилась к гинекологу с жалобами на отсутствие месячных, слабость, зябкость. Год тому назад родила ребёнка, в родах было массивное кровотечение, проводилась гемотрансфузия. После родов гипогалактия, вскоре лактация полностью прекратилась. Объективно: женщина пониженного питания, молочные железы дряблые, скудное оволосение в подмышечных впадинах, на лобке. Наружные половые органы атрофичны, депигментированы. PV: влагалище ёмкое, шейка матки короткая, тело матки маленькое, придатки не определяются. При каком из перечисленных диагнозов отмечается подобная клиническая картина:A. синдром Морриса B. синдром ШиханаC. синдром Бабинского – ФрелихаD. синдром Лоренса- Муна – БидляE. синдром Шерешевского – Тернера24 Г. 22 лет обратилась в связи с отсутвием месячных и беременности. Из анамнеза: росла здоровым ребёнком, в возрасте 13-15 лет оперирована по поводу паховых грыж. Замуж вышла в 20 лет, мужу 20 лет – здоров. У родной сестры нет месячных. При осмотре рост 156 см, масса 55 кг. Вторичные половые признаки развиты хорошо. Наружные половые органы сформированы правильно. OZ: влагалище оканчивается слепо, шейки матки нет. PV: влагалище не рожавщей, матка и придатки не определяются. При каком из перечисленный диагнозов отмечается подобная клиническая картина:A. первичная аменореяB. вторичная аменореяC. атрезия влагалищаD. маточная аменореяE. дисфункции яичников25 Симптомокомплекс, включающий: аменорею, гирсутизм, бесплодие, ожирение и двусторонее увеличение яичников. При каком из перечисленных диагнозов отмечается подобная клиническая картина:A. синдром Иценко – КушинаB. синдром Киари – ФроммеляC. адреногенитальный синдромD. синдром Штейна – Левенталя

Page 9: postgraduate.kaznmu.kz€¦ · Web view58 Больная И, 28 лет, доставлена с жалобами на внезапно возникшую боль внизу живота

С.Ж.АСФЕНДИЯРОВАТЫНДАҒЫҚАЗАҚ ҰЛТТЫҚ МЕДИЦИНА

УНИВЕРСИТЕТІ

КАЗАХСКИЙНАЦИОНАЛЬНЫЙМЕДИЦИНСКИЙ УНИВЕРСИТЕТ ИМЕНИ С.Д.АСФЕНДИЯРОВА

КАФЕДРА ИНТЕРНАТУРЫ И РЕЗИДЕНТУРЫ ПО АКУШЕРСТВУ И ГИНЕКОЛОГИИТЕСТОВЫЕ ЗАДАНИЯ ДЛЯ ПРОВЕДЕНИЯ ИТОГОВОЙ ГОСУДАРСТВЕННОЙ АТТЕСТАЦИИ РЕЗИДЕНТОВ

ВЫПУСКА 2012-2013 ГГ. ПО СПЕЦИАЛЬНОСТИ «АКУШЕРСТВО И ГИНЕКОЛОГИЯ»

E. синдром Лоренса- Муна – Бидля26 Наиболее вероятный метод лечения при маточной форме аменореи (синдроме Аширмана): почему?A. заместительная гормонотерапия B. рассечение внутриматочных синехии кюреткой под контролем гистероскопии C. лечение гонодотропными препаратами D. хирургическое вмешательство – для E.чистые гестагены и эстрогены – гестагенными препаратами в постоянном или контрацептивном режиме – для снижения синтеза ЛГ гипофизом, а также тестостерона и андостендиона тека- клетками.27 Функциональные гормональные пробы для обследования больных с аменореей в первную очередь проводятся с целью: диагностики аномалии развития гениталий определения уровня поражения в репродуктивной системе диагностики органических поражений ЦНС выяснения чувствительности рецепторов матки уточнения структуры турецкого седла28 Наиболее вероятный метод диагностики гипергонадотропной аменореи: A. обзорная лапароскопия B. определение кариотипаC. гистеросальпингография D. ультрозвуковое исследование E. лапароскопия с биопсией гонад 29 Больная В. 25 лет обратилась к гинекологу с жалобами на отсутствие месячных, общую слабость. Год назад родила ребенка, в послеродовом периоде было массивное кровотечение, получила интенсивное лечение, проводили гемотрансфузию. После родов отмечала скудное количество молока, вскоре лактация полностью прекратилась. Объективно: дефицит веса, молочные железы дряблые, волосистость в подмышечных впадинах и на лобке скудная. Наружные половые органы атрофичные, депигментированы. PV: влагалище емкое, шейка матки укорочена, тело матки меньше нормы, область придатков без особенностей. Выберите правильный диагноз: Ведущий симптом данного диагноза?A. синдром Морриса B. синдром Шихана C. синдром Иценко-Кушинга D. синдром Бабинского-Фейлиха E. синдром Шерешевского-Тернера 31 Больная Ф. 34 лет предъявляет жалобы на быстро прогрессирующий гирсутизм, прекращение месячных. Считает себя больной в течение 6 месяцев, когда впервые стали отмечаться задержки месячных, появился рост волос на лице (борода, усы). Последние месячные 3 месяца тому назад. При осмотре: рост – 152 см, масса 57 кг. Выраженный гирсутизм. Молочные железы атрофичны. Выделений из сосков нет. PV: шейка матки цилиндрической формы, зев закрыт. Матка не увеличена. Область левых придатков без особенностей. Справа в области придатков пальпируется плотное образование размером 5,5х4,5х5,0 см. Поставьте правильный диагноз: A. опухоль надпочечников B. болезнь Иценко-Кушинга C. адреногенитальный синдром D. синдром поликистозных яичников E. гормонопродуцирующая опухоль яичника

Page 10: postgraduate.kaznmu.kz€¦ · Web view58 Больная И, 28 лет, доставлена с жалобами на внезапно возникшую боль внизу живота

С.Ж.АСФЕНДИЯРОВАТЫНДАҒЫҚАЗАҚ ҰЛТТЫҚ МЕДИЦИНА

УНИВЕРСИТЕТІ

КАЗАХСКИЙНАЦИОНАЛЬНЫЙМЕДИЦИНСКИЙ УНИВЕРСИТЕТ ИМЕНИ С.Д.АСФЕНДИЯРОВА

КАФЕДРА ИНТЕРНАТУРЫ И РЕЗИДЕНТУРЫ ПО АКУШЕРСТВУ И ГИНЕКОЛОГИИТЕСТОВЫЕ ЗАДАНИЯ ДЛЯ ПРОВЕДЕНИЯ ИТОГОВОЙ ГОСУДАРСТВЕННОЙ АТТЕСТАЦИИ РЕЗИДЕНТОВ

ВЫПУСКА 2012-2013 ГГ. ПО СПЕЦИАЛЬНОСТИ «АКУШЕРСТВО И ГИНЕКОЛОГИЯ»

32 Больная Ф. 34 лет предъявляет жалобы на быстро прогрессирующий гирсутизм, прекращение месячных. Считает себя больной в течение 6 месяцев, когда впервые стали отмечаться задержки месячных, появился рост волос на лице (борода, усы). Последние месячные 3 месяца тому назад. При осмотре: рост – 152 см, масса 57кг. Выраженный гирсутизм. Молочные железы атрофичны. Выделений из сосков нет. PV: шейка матки цилиндрической формы, зев закрыт. Матка не увеличена. Область левых придатков без особенностей. Справа в области придатков пальпируется плотное образование размером 5,5х4,5х5,0 см. Наиболее информативный метод диагностики данного заболевания: A. ГСГ B. УЗИ C. гистероскопияD. мазок на микроскопиюE. анализ крови и мочи на гормоны 33 Больная А.18 лет обратилась в женскую консультацию с жалобами на боли внизу живота, повышение температуры тела до 37,5°С, гнойные выделения из половых путей, рези при мочеиспускании. Половая жизнь с 17 лет, вне брака, от беременности предохраняется ритм - методом, беременностей не было. Заболела остро на 7 день менструального цикла, когда появились указанные выше симптомы. Объективно: состояние удовлетворительное, пульс 96 ударов в мин., ритмичный. Живот не вздут, при пальпации мягкий, болезненный в нижних отделах. Гинекологический статус: уретра инфильтрирована, шейка матки гиперемирована, отечна, обширная эрозия, из шеечного канала обильные слизисто-гнойные выделения. Матка не увеличена, болезненная при пальпации, придатки с обеих сторон утолщены, болезненны, своды глубокие. При бактериоскопии мазков из уретры и канала шейки матки обнаружены внутриклеточные диплокки. Критерии излеченности гонореи:A. отсутствие жалобB. отсутствие выделенийC. отсутствие анатомических изменений внутренних половых органовD. отсутствие гонококков в мазках в начале 3-х менструальных циклов E. отсутствие в мазках гонококков после окончания курса антибактериальной терапии34 К воспалительным процессам нижнего отдела половых органов относится: A. аднексит B. кольпит C. эндометритD. параметрит E. пельвиоперитонит35 У больной отмечается гиперемия, отек уретры, обильные гнойные выделения, рези при мочеиспускании, гиперемия влагалищной части шейки матки, из цервикального канала обильные гнойные выделения. При каком из перечисленных диагнозов отмечается подобная картина:A. кандидозB. гонореяC. хламидиозD. трихомонозE. гарднереллез36 В гинекологическое отделение поступила больная с жалобами на боли внизу живота, гнойно-кровянистые выделения из половых путей, подъем температуры до 38 °С. Заболела на 8 день менструального цикла, через 3 дня после введения ВМС. В зеркалах: шейка чистая матки. В цервикальном канале проводные нити ВМС, гнойно-кровянистые выделения. Матка несколько

Page 11: postgraduate.kaznmu.kz€¦ · Web view58 Больная И, 28 лет, доставлена с жалобами на внезапно возникшую боль внизу живота

С.Ж.АСФЕНДИЯРОВАТЫНДАҒЫҚАЗАҚ ҰЛТТЫҚ МЕДИЦИНА

УНИВЕРСИТЕТІ

КАЗАХСКИЙНАЦИОНАЛЬНЫЙМЕДИЦИНСКИЙ УНИВЕРСИТЕТ ИМЕНИ С.Д.АСФЕНДИЯРОВА

КАФЕДРА ИНТЕРНАТУРЫ И РЕЗИДЕНТУРЫ ПО АКУШЕРСТВУ И ГИНЕКОЛОГИИТЕСТОВЫЕ ЗАДАНИЯ ДЛЯ ПРОВЕДЕНИЯ ИТОГОВОЙ ГОСУДАРСТВЕННОЙ АТТЕСТАЦИИ РЕЗИДЕНТОВ

ВЫПУСКА 2012-2013 ГГ. ПО СПЕЦИАЛЬНОСТИ «АКУШЕРСТВО И ГИНЕКОЛОГИЯ»

увеличена, резко болезненна, ограничена в подвижности. Придатки не определяются, своды глубокие, болезненные. При каком из перечисленных диагнозов отмечается подобная картина?A. параметритB. эндометритC. сальпингоофоритD. пельвиоперитонит E. перфорация матки при введении ВМС37 Больная Б. 30 лет обратилась с жалобами на резкую боль, заставляющая принять вынужденное положение, повышение температуры тела до 37,8°С-38,5°С и выше, озноб. PV: при смещении шейки матки отмечается болезненность в области придатков, двухсторонняя болезненность при пальпации. В анализах крови лейкоцитоз, повышение СОЭ. При каком из перечисленных диагнозов отмечается подобная картина:A. параметритB. пиосальпинксC. пельвиоперитонитD. острый эндометритE. острый сальпингоофорит38 Боли внизу живота, иррадиирущие в поясницу, крестец, повышение температуры тела до 38°С-39°С, тахикардия. PV: при пальпации резкая болезненность матки, отклонение матки в здоровую сторону, отделить ее от инфильтрата невозможно, образование плотное, неподвижное. Клиническая картина характерна для:A. параметритаB. острого эндометритаC. пельвиоперитонитаD. острого сальпингоофоритаE. тубоовариального образования 39 Зуд, гиперемия ,отек ,пенистые выделения с неприятным запахом. Эта клиническая картина характерна для:A. кандидозаB. хламидиозаC. вирусной инфекцииD. герпетической инфекции E. трихомонадной инфекции40 Воспаление большой железы преддверия влагалища это:A. вульвитB. кольпит C. эндометритD. бартолинитE. эндоцервицит 41 Больная, 12 лет поступает в стационар с жалобами на резкие боли внизу живота, озноб, повышение температуры тела до 38,5°С, слабость. Из анамнеза: 10 дней назад произведен медицинский аборт в сроке 8 недель. PV: резкая болезненность в нижнем отделе живота, матка размягчена, чуть больше нормы, резко болезненная при пальпации, область придатков болезненна, выделения сукровичные. Определяющим в комплексной терапии данной патологии в первую очередь является: А. иммунотерапия В. симтоматическое С. антикоагулянтная терапия

Page 12: postgraduate.kaznmu.kz€¦ · Web view58 Больная И, 28 лет, доставлена с жалобами на внезапно возникшую боль внизу живота

С.Ж.АСФЕНДИЯРОВАТЫНДАҒЫҚАЗАҚ ҰЛТТЫҚ МЕДИЦИНА

УНИВЕРСИТЕТІ

КАЗАХСКИЙНАЦИОНАЛЬНЫЙМЕДИЦИНСКИЙ УНИВЕРСИТЕТ ИМЕНИ С.Д.АСФЕНДИЯРОВА

КАФЕДРА ИНТЕРНАТУРЫ И РЕЗИДЕНТУРЫ ПО АКУШЕРСТВУ И ГИНЕКОЛОГИИТЕСТОВЫЕ ЗАДАНИЯ ДЛЯ ПРОВЕДЕНИЯ ИТОГОВОЙ ГОСУДАРСТВЕННОЙ АТТЕСТАЦИИ РЕЗИДЕНТОВ

ВЫПУСКА 2012-2013 ГГ. ПО СПЕЦИАЛЬНОСТИ «АКУШЕРСТВО И ГИНЕКОЛОГИЯ»

D. дезинтоксикационная терапияE. . массивная антибактериальная терапия42 Больная 25 лет поступила в гинекологическое отделение с жалобами на повышение температуры тела до 38-39 °С, рвоту, тошноту, сильные боли внизу живота, жидкий стул. При объективном осмотре: положительный симптом Щеткина-Блюмберга, напряжение передней брюшной стенки. PV: Размеры матки и придатков из-за напряжения передней брюшной стенки не определяются, выпячивание заднего свода. Поставлен диагноз: Пельвиоперитонит. Для постановки заключительного диагноза в первую очередь необходимо произвести дополнительный метод исследования: A. УЗИ B. лапароскопия C. гистероскопия D. бактериологическийE. пункцию заднего свода 43 В диагностике эндоцервицита наиболее вероятный метод исследования:А. скрининговый цитологический В. мазок по Паппаниколау С. бактериоскопический D. бактериологический E. кольпоскопический44 Больную 35 лет беспокоят боли в области правой половой губы, подъем температуры до 37 °С. При осмотре половых органов обнаружена отечность и гиперемия правой половой губы. Пальпаторно определяется образование в толще правой половой губы размером 3×4 см. Что в первую очередь необходимо назначить, оказывая помощь: A. иммунотерапию B. вскрытие абсцесса C. симптоматическое лечение D. антибактериальную терапию E. дезинтоксикационную терапию 45 У больной С.22 лет при обследовании выявлены обильные слизисто-гнойные выделения из цервикального канала, эрозия шейки матки. В микроскопическом анализе найдены «ключевые клетки». Определяющим в комплексной терапии данной патологии в первую очередь является: А. иммунотерапия В. антикоагулянтная С. противомикробнаяD. дезинтоксикационнаяE. массивная антибактериальная 46 На приеме больная Д. 31года: через 5 дней после медицинского аборта появились боли в нижнем отделе живота, озноб, повышение температуры тела до 38 °С. При объективном исследовании обнаружено: живот мягкий, болезненный в нижнем отделе. Симптомов раздражения брюшины нет. PV: матка увеличена до 5-6 недель беременности, болезненная, мягковатой консистенции, влагалищные своды свободные. Придатки матки не определяются. Выделения коричневатые с неприятным запахом. Наиболее вероятный метод при контроле эффективности лечения данного заболевания: A. бактериоскопический B. бактериологический C. кольпоскопический D. лапароскопия

Page 13: postgraduate.kaznmu.kz€¦ · Web view58 Больная И, 28 лет, доставлена с жалобами на внезапно возникшую боль внизу живота

С.Ж.АСФЕНДИЯРОВАТЫНДАҒЫҚАЗАҚ ҰЛТТЫҚ МЕДИЦИНА

УНИВЕРСИТЕТІ

КАЗАХСКИЙНАЦИОНАЛЬНЫЙМЕДИЦИНСКИЙ УНИВЕРСИТЕТ ИМЕНИ С.Д.АСФЕНДИЯРОВА

КАФЕДРА ИНТЕРНАТУРЫ И РЕЗИДЕНТУРЫ ПО АКУШЕРСТВУ И ГИНЕКОЛОГИИТЕСТОВЫЕ ЗАДАНИЯ ДЛЯ ПРОВЕДЕНИЯ ИТОГОВОЙ ГОСУДАРСТВЕННОЙ АТТЕСТАЦИИ РЕЗИДЕНТОВ

ВЫПУСКА 2012-2013 ГГ. ПО СПЕЦИАЛЬНОСТИ «АКУШЕРСТВО И ГИНЕКОЛОГИЯ»

E. УЗИ 47 Больная С 25 лет обратилась к врачу женской консультации с жалобами на бели и контактные кровянистые выделения из половых путей в течение 3 месяцев. В зеркалах: шейка матки коническая, поверхность слизистой оболочки вокруг наружного зева на площади 2,0*2,0 см ярко красного цвета, зернистая, покрыта гноевидно-слизистыми выделениями, при дотрагивании легко кровоточит. Бимануальное исследование: шейка матки обычной консистенции, наружный зев закрыт, матка в retroflexio, нормальных размеров, плотная, подвижная, безболезненная, придатки с обеих сторон без особенностей. Своды свободны. Наиболее вероятный метод диагностики заболевания: A. скрининговый цитологический B. бактериоскопический C. бактериологический D. кольпоскопический E. мазок по Паппаниколау 48 При сформировавшемся гнойном воспалительном образовании придатков матки в первую очередь показано:A. терапия пирогеналом B. терапия гоновакциной C. хирургическое лечение D. пункция образования через влагалищный сводE. электрофорез цинка по брюшно-крестцовой методике 49 Наиболее характерные изменения в эндометрии при внематочной беременности:A. атрофия.B. пролиферация.C. железисто-кистозная гиперплазия.D. децидуальная трансформация.E. эндометриальный полип.50 Иррадиация болей в подключичную область при гинекологических заболеваниях является основным симптомом:A. миомы маткиB. кисты яичникаC. начавшегося абортаD. угрожающего выкидышаE. прервавшейся трубной беременности51 Больная 24 лет поступила в гинекологическое отделение с жалобами на мажущие кровянистые выделения из половых путей. Замужем второй год. Последняя менструация 2 месяца назад, считает себя беременной. Пульс 70 уд/в мин. АД 110/70 мм рт.ст. Живот мягкий, безболезненный. OZ: слизистая влагалища и шейки матки синюшна, мажущие кровянистые выделения темного цвета. PV: тело матки мягковатой консистенции, увеличено до 7-8 недель беременности, отклонено влево, справа от матки определяется слегка болезненное мягко-эластическое образование, величиной с женский кулак, интимно прилегает к матке. Ваш диагноз:A. правосторонняя трубная беременность, нарушенная по типу трубного выкидышаB. левосторонняя трубная беременность, нарушенная по типу трубного аборта C. правосторонняя трубная беременность, нарушенная по типу разрыва трубыD. угрожающий ранний самопроизвольный аборт в 7-8 недельE. начавшийся ранний самопроизвольный аборт в 7-8 недель52 Больная 27 лет доставлена в клинику машиной скорой помощи. Считала себя беременной, обращалась в женскую консультацию, но при осмотре беременность не была установлена. В

Page 14: postgraduate.kaznmu.kz€¦ · Web view58 Больная И, 28 лет, доставлена с жалобами на внезапно возникшую боль внизу живота

С.Ж.АСФЕНДИЯРОВАТЫНДАҒЫҚАЗАҚ ҰЛТТЫҚ МЕДИЦИНА

УНИВЕРСИТЕТІ

КАЗАХСКИЙНАЦИОНАЛЬНЫЙМЕДИЦИНСКИЙ УНИВЕРСИТЕТ ИМЕНИ С.Д.АСФЕНДИЯРОВА

КАФЕДРА ИНТЕРНАТУРЫ И РЕЗИДЕНТУРЫ ПО АКУШЕРСТВУ И ГИНЕКОЛОГИИТЕСТОВЫЕ ЗАДАНИЯ ДЛЯ ПРОВЕДЕНИЯ ИТОГОВОЙ ГОСУДАРСТВЕННОЙ АТТЕСТАЦИИ РЕЗИДЕНТОВ

ВЫПУСКА 2012-2013 ГГ. ПО СПЕЦИАЛЬНОСТИ «АКУШЕРСТВО И ГИНЕКОЛОГИЯ»

сознании, заторможена. Кожные покровы и видимые слизистые бледные, на лбу холодный пот. Пульс 130 ударов в 1 минуту, слабого наполнения, частый, плохо сосчитывается. АД 70/50 мм рт.ст. Живот равномерно вздут, перистальтика отсутствует. При пальпации брюшной стенки отмечается умеренно выраженное напряжение мышц живота, усиливающееся при пальпации. Наружное кровотечение отсутствует. Нв = 51г/л, Эр 3 000 000, лейкоциты 5 600. Наиболее вероятный диагноз:A. апоплексия яичника, геморрагический шок 3 ст.B. разрыв кисты яичника, геморрагический шок 3 ст. C. «острый живот», кровотечение, геморрагический шок 3 ст.D. внематочная беременность, нарушенная по типу трубного аборта, геморрагический шок 3 ст.E. внематочная беременность, нарушенная по типу разрыва трубы. Геморрагический шок 3 ст.53 Больная 28 лет поступила с жалобами на схваткообразные боли внизу живота и темно-кровянистые выделения из половых путей при задержке менструации на 14 дней. Было 5 беременностей; 1роды, 4 медицинских аборта, последний осложнился эндометритом. Состояние удовлетворительное, пульс 80 ударов в минуту, АД 110/70 мм рт ст. на зеркалах: цианоз слизистой влагалища и шейки матки, темно-кровянистые выделения. PV: матка в правильном положении, несколько больше нормы, подвижная, безболезненная. Слева в области придатков определяется образование тестоватой консистенции, болезненное. Справа придатки не определяются. Задний свод утолщен, болезненный. При каком из перечисленных диагнозов отмечается подобная клиническая картина.A. неполный абортB. внематочная беременностьC. кистома яичникаD. эндометритE. острый левосторонний аднексит54 Больная К, 29 лет, поступила с жалобами на схваткообразные боли внизу живота, иррадиирущие в прямую кишку, мажущие кровянистые выделения из половых путей. Заболела на 2 день после мини-аборта. Роды – 1, медицинских абортов – 2. Состояние удовлетворительное, пульс 78 ударов в минуту, АД 115/70 мм рт ст. OZ: цианоз слизистой влагалища и шейки матки, выделения кровянистые, скудные. PV: матка чуть больше нормы, подвижная, безболезненная. Справа в области придатков пальпируется образование без четких контуров, тестоватой консистенции, болезненное. Задний свод утолщен, болезненный. Назовите диагноз.A. острый метроэндометритB. обострение хронического воспаления придатков маткиC. внематочная беременностьD. остатки плодного яйцаE. острый правосторонний параметрит55 Больная С, 28 лет, поступила с жалобами на боли приступообразного характера, мажущие кровянистые выделения из половых путей, головокружение. Больная апатична, заторможена. Кожные покровы и видимые слизистые бледные, покрыта холодным потом. Задержка менструации 3 недели. Из анамнеза: хронический сальпингоофорит, 2 медицинских аборта. Живот равномерно вздут, перистальтика отсутствует. При пальпации брюшной стенки отмечается умеренно выраженное напряжение мышц живота, усиливающееся при пальпации. OZ: цианоз слизистой влагалища и шейки матки, выделения кровянистые, скудные. PV: матка чуть больше нормы, шейка матки резко болезненна при смещении, нависание заднего свода влагалища и резкая болезненность. Какой симптом описан?A. Мюссе

Page 15: postgraduate.kaznmu.kz€¦ · Web view58 Больная И, 28 лет, доставлена с жалобами на внезапно возникшую боль внизу живота

С.Ж.АСФЕНДИЯРОВАТЫНДАҒЫҚАЗАҚ ҰЛТТЫҚ МЕДИЦИНА

УНИВЕРСИТЕТІ

КАЗАХСКИЙНАЦИОНАЛЬНЫЙМЕДИЦИНСКИЙ УНИВЕРСИТЕТ ИМЕНИ С.Д.АСФЕНДИЯРОВА

КАФЕДРА ИНТЕРНАТУРЫ И РЕЗИДЕНТУРЫ ПО АКУШЕРСТВУ И ГИНЕКОЛОГИИТЕСТОВЫЕ ЗАДАНИЯ ДЛЯ ПРОВЕДЕНИЯ ИТОГОВОЙ ГОСУДАРСТВЕННОЙ АТТЕСТАЦИИ РЕЗИДЕНТОВ

ВЫПУСКА 2012-2013 ГГ. ПО СПЕЦИАЛЬНОСТИ «АКУШЕРСТВО И ГИНЕКОЛОГИЯ»

B. «Френикус-симптом»C. ВасилевскогоD. КерраE. Крик Дугласа56 Пациентка, 22 года, обратилась с жалобами на кровянистые выделения из половых путей в течение 2 дней. Последняя менструация 6 недель тому назад. Тест на беременность положительный. В анамнезе: хроническое воспаление придатков матки, 1 медицинский аборт. OZ: слизистая шейки матки цианотична, незначительные кровянистые выделения из цервикального канала, шейка матки закрыта. PV: матка при пальпации несколько размягчена, чуть больше нормы. В области придатков матки без особенностей. Какие лечебно-диагностические мероприятия наиболее целесообразно включить в план ведения?A. кульдоцентезB. вакуум-кюретаж маткиC. трансвагинальное УЗИD. прекратить половые сношения на 3 неделиE. назначить медроксипрогестерон по 10 мг в день57 Больная, 27 лет, поступила в гинекологическое отделение с жалобами на внезапные постоянные боли внизу живота, иррадиирущие в прямую кишку. В анамнезе срочные роды, длительное ношение ВМС. Задержка менструации на 2 недели. Отмечает синдром «ваньки-встаньки», двусторонний «френикус-симптом», головокружение, потерю сознания. OZ: шейка цианотична, положительный симптом Промптова. PV: матка меньше предполагаемого срока беременности, одностороннее увеличение и болезненность придатков матки. Нависание заднего свода влагалища. Крик Дугласа. Какова причина данной картины? A. нарушение гормональной функции яичниковB. длительное ношение ВМС C. срочные родыD. инфантилизмE. эндометриоз58 Больная И, 28 лет, доставлена с жалобами на внезапно возникшую боль внизу живота с иррадиацией в копчик, скудные кровянистые выделения, кратковременную потерю сознания, тошноту, слабость. Заболела при задержке менструации на 3 недели. Из анамнеза: частые воспалительные заболевания половых путей, аборт с осложнениями, длительное ношение ВМС. Состояние тяжелое. Пульс 100 ударов в минуту, АД 90/60 мм рт ст. живот умеренно вздут, участвует в акте дыхания, напряжен, болезнен, симптом раздражения брюшины положителен, стул был трижды, жидкий. PV: матка и придатки четко не определяются из-за болезненности и напряжения брюшной стенки, своды нависают, болезненные. Каков генез внематочной беременности в данном случае?A. длительное ношение ВМСB. рубцово-спаечные изменения органов малого таза C. нарушение гормональной функции яичниковD. инфантилизмE. эндометриоз59 Пациентка Л, 34 года, машиной скорой помощи доставлена в гинекологическое отделение с жалобами на приступоообразные боли в нижних отделах живота, темно-кровянистые выделения из половых путей. Последняя менструация 5 недель назад. В анамнезе: беременностей- 3: из них родов – 1, медицинский аборт – 2. Тест на беременность положительный. При УЗИ: плодное яйцо в матке не лоцируется. OZ: слизистая влагалища и шейки матки цианотичные, из матки –

Page 16: postgraduate.kaznmu.kz€¦ · Web view58 Больная И, 28 лет, доставлена с жалобами на внезапно возникшую боль внизу живота

С.Ж.АСФЕНДИЯРОВАТЫНДАҒЫҚАЗАҚ ҰЛТТЫҚ МЕДИЦИНА

УНИВЕРСИТЕТІ

КАЗАХСКИЙНАЦИОНАЛЬНЫЙМЕДИЦИНСКИЙ УНИВЕРСИТЕТ ИМЕНИ С.Д.АСФЕНДИЯРОВА

КАФЕДРА ИНТЕРНАТУРЫ И РЕЗИДЕНТУРЫ ПО АКУШЕРСТВУ И ГИНЕКОЛОГИИТЕСТОВЫЕ ЗАДАНИЯ ДЛЯ ПРОВЕДЕНИЯ ИТОГОВОЙ ГОСУДАРСТВЕННОЙ АТТЕСТАЦИИ РЕЗИДЕНТОВ

ВЫПУСКА 2012-2013 ГГ. ПО СПЕЦИАЛЬНОСТИ «АКУШЕРСТВО И ГИНЕКОЛОГИЯ»

кровянистые выделения. PV: матка чуть больше нормы, мягковатой консистенции, подвижная. Симптом Промптова положительный. Какое действие наиболее приемлемо в первую очередь?A. кольпоскопия B. гистероскопия C. пункция заднего свода D. зондирование полости матки E. рентгенография брюшной полости60 Пациентка 20 лет, обратилась с жалобами на кровянистые выделения из половых путей в течение 2 дней, рождение сгустка с кусочком белесоватой ткани. Последняя менструация 5 недель тому назад. Тест на беременность положительный. В анамнезе однократно перенесла острое воспаление придатков матки. При обследовании обнаружено небольшое количество крови во влагалище, вытекаемой из цервикального канала. Шейка матки закрыта. Матка при пальпации несколько размягчена. В области придатков матки без особенностей. Определено содержание ХГЧ – 3100 мМЕ/мл. При УЗИ обнаружено образование в области левых придатков с нечеткими контурами, жидкость в дугласовом пространстве. Какие лечебно-диагностические мероприятия наиболее целесообразно включить в план ведения?А. вакуум-кюретаж матки с удалением оставшегося плодного яйцаВ. медроксипрогестерон по 10 мг в день С. пункция заднего свода D. терапия гоновакциной E. срочная лапаротомия61 Больная 20 лет, предъявляет жалобы врачу на задержку очередной менструации, однократную рвоту, тошноту, нагрубание молочных желез. Половая жизнь регулярная. От беременностей не предохраняется. OZ: слизистая влагалища и шейки матки цианотичные, из матки – кровянистые выделения. PV: матка чуть больше нормы, мягковатой консистенции, подвижная. При пункции заднего свода обнаружена темная кровь. Какое дальнейшее действие наиболее приемлемо? А. антикоагулянтная терапия В. экстренная лапароскопия С. противомикробное лечениеD. дезинтоксикационная терапия E. рентгенография брюшной полости62 Больная Н, 36лет, поступила в гинекологическое отделение с острой болью в животе, иррадиирущей в правое подреберье, межлопаточную область, ключицу. Больная в сознании, заторможена. На вопросы отвечает односложно. Кожные покровы и видимые слизистые бледные, на лбу холодный пот. Пульс 140 ударов в 1 минуту, слабого наполнения, частый, плохо сосчитывается. АД 80/50 мм рт ст. Живот равномерно вздут, перистальтика отсутствует. При пальпации брюшной стенки отмечается умеренно выраженное напряжение мышц живота, которое усиливается при пальпации. Наружное кровотечение отсутствует. Экстренная помощь при данной патологии начинается с:A. срочной лапаротомии B. дезинтоксикационной терапии C. гемотрансфузииD. переливания свежезамороженной плазмы E. переливания гемодеза 63 Больная К, 25 лет, поступила с жалобами на схваткообразные боли внизу живота, иррадиирущие в прямую кишку и мажущие кровянистые выделения из половых путей. Заболела 2 дня назад. Роды – 1, медицинских абортов – 2. Пульс 78 ударов в минуту, АД 115/70 мм рт ст. OZ: цианоз слизистой влагалища и шейки матки, выделения кровянистые, скудные. PV: матка чуть

Page 17: postgraduate.kaznmu.kz€¦ · Web view58 Больная И, 28 лет, доставлена с жалобами на внезапно возникшую боль внизу живота

С.Ж.АСФЕНДИЯРОВАТЫНДАҒЫҚАЗАҚ ҰЛТТЫҚ МЕДИЦИНА

УНИВЕРСИТЕТІ

КАЗАХСКИЙНАЦИОНАЛЬНЫЙМЕДИЦИНСКИЙ УНИВЕРСИТЕТ ИМЕНИ С.Д.АСФЕНДИЯРОВА

КАФЕДРА ИНТЕРНАТУРЫ И РЕЗИДЕНТУРЫ ПО АКУШЕРСТВУ И ГИНЕКОЛОГИИТЕСТОВЫЕ ЗАДАНИЯ ДЛЯ ПРОВЕДЕНИЯ ИТОГОВОЙ ГОСУДАРСТВЕННОЙ АТТЕСТАЦИИ РЕЗИДЕНТОВ

ВЫПУСКА 2012-2013 ГГ. ПО СПЕЦИАЛЬНОСТИ «АКУШЕРСТВО И ГИНЕКОЛОГИЯ»

больше нормы, подвижная, безболезненная. Справа в области придатков пальпируется образование без четких контуров, тестоватой консистенции, болезненное. Задний свод утолщен, болезненный. Выберите наиболее точный метод диагностики:A. кульдоцентез B. биопсия эндометрия C. лапароскопия D. удаление эктопического очага в маточных трубахE. серийное определение ХГ 64 Наиболее частой локализацией туберкулеза женских половых органов является:A. эндометрийB. шейка маткиC. фаллопиевы трубыD. яичникиE. вульва65 При бимануальном исследовании у молодой женщины определяются двухсторонние конгломераты неподвижных плотных опухолевидных образований по ребру матки. При ректальном исследовании пальпируются многочисленные плотные узелки различной величины. Беспокоят ноющие боли внизу живота. Какой из перечисленных ниже методов исследования позволит быстро уточнить диагноз?A. биопсия шейки маткиB. гистеросальпингографияC. пневморельвиография D. УЗИE. компьюторная томография66 Пациентка 23 лет обратилась к врачу по поводу первичного бесплодия. Проведена гистеросальпингография. На снимке определяется деформация полости матки, небольшое укорочение труб, наличие расширений на концах труб в виде луковиц. Какой метод диагностики в данном случае позволит уточнить степень активности патологического процесса?A. диагностическое выскабливание слизистой полости матки B. аспирационная биопсияC. проба КохаD. биологический методE. культуральный метод67 Женщина 31 года обратилась с жалобами на первичное бесплодие. Беспокоят ноющие боли внизу живота. Менструации регулярные, скудные. При бимануальном исследовании в области придатков определяется тяжистость с обеих сторон. Проведена проба с 50 ТЕ туберкулина. Температура тела после пробы повысилась до 37,2 гр., в анализе крови – лейкоцитоз, моноцитоз, лимфопения. Несколько усилились боли внизу живота, на месте подкожной инъекции обнаружена папула в диаметре 15 мм. Какой тип реакции на вакцину наиболее выражен в данном случае?A. общая реакция и уколочная пробаB. уколочная проба и очаговая реакция C. общая и очаговая реакцияD. отрицательная уколочная проба E. общая, очаговая реакция, уколочная проба68 Пациентка 24 лет обратилась к гинекологу по поводу первичного бесплодия. На гистеросальпингограмме выявлено двухстороннее поражение ампулярно-фимбриальных отделов маточных труб. Имеется симптом “ барабанных “ палочек. Какая стадия

Page 18: postgraduate.kaznmu.kz€¦ · Web view58 Больная И, 28 лет, доставлена с жалобами на внезапно возникшую боль внизу живота

С.Ж.АСФЕНДИЯРОВАТЫНДАҒЫҚАЗАҚ ҰЛТТЫҚ МЕДИЦИНА

УНИВЕРСИТЕТІ

КАЗАХСКИЙНАЦИОНАЛЬНЫЙМЕДИЦИНСКИЙ УНИВЕРСИТЕТ ИМЕНИ С.Д.АСФЕНДИЯРОВА

КАФЕДРА ИНТЕРНАТУРЫ И РЕЗИДЕНТУРЫ ПО АКУШЕРСТВУ И ГИНЕКОЛОГИИТЕСТОВЫЕ ЗАДАНИЯ ДЛЯ ПРОВЕДЕНИЯ ИТОГОВОЙ ГОСУДАРСТВЕННОЙ АТТЕСТАЦИИ РЕЗИДЕНТОВ

ВЫПУСКА 2012-2013 ГГ. ПО СПЕЦИАЛЬНОСТИ «АКУШЕРСТВО И ГИНЕКОЛОГИЯ»

патологического процесса имеет место в данном случае в соответствии с классификацией Э. Н. Беллендира ? A. первоначальный очаг в пораженном органеB. начальное распространение процессаC. умеренное распространение процессаD. выраженное распространение процесса с поражением матки E. субтотальное или тотальное поражение матки и придатков69 При проведении профилактического осмотра у девушки 16 лет, не живущей половой жизнью, обнаружено двухстороннее увеличение придатков матки. Жалоб не предъявляет. Из анамнеза выяснено - страдает хроническим тонзиллитом с частыми обострениями, хроническим пиелонефритом. Последнее обострение было 4 месяца тому назад. Признаков воспаления нижнего отдела полового тракта не было.Каков патогенез поражения маточных труб в данном случае?A. лимфогенный путь инфицирования из аппендикса B. гематогенный путь инфицирования из миндалинC. гематогенный путь инфицирования из первичного очага в легкихD. гематогенный путь инфицирования из первичного очага в почкахE. каникулярный путь инфицирования гонореей в детстве70 На прием к гинекологу обратилась женщина 32 лет с жалобами на первичное бесплодие. Острые и хронические заболевания полового тракта отрицает. Живет в браке 8 лет. На гистеросальпингограмме, представленной пациентко, имеется симптом “четок“. Что способствовало в данном случае таким изменениям?A. скудная капиллярная сеть ампулярных отделов труб B. отсутствие анастомозов между маточной и яичниковой артериями C. отсутствие складчатости слизистой оболочки труб D. замедление кровотока в ампулярных отделах трубE. ускорение кровотока в ампулярных отделах труб71 На прием к гинекологу обратилась женщина 32 лет с жалобами на общую слабость, постоянные ноющие боли внизу живота, скудные менструации. Прошла повторные курсы антибактериальной тепапии ( римфапицином, ПАСК-ом , тубазидом) у фтизио-гинеколога. Заметного улучшения состояния не наступило. При гинекологическом осмотре в области придатков матки определяются конгломераты 6*5см, безболезненные. Выберите дальнейшую тактику ведения: A. повторный курс антибактериальной терапии B. провести инфузионную терапию C. провести двухстороннюю аднексэтомию так как определяются опухолевидные образования в области придатков матки.D. провести экстирпацию матки с придатками E. провести двухстороннюю аднексэтомию с последующей антибактериальной терапией в течении 2 лет 72 Женщина 23 лет обратилась к гинекологу с жалобами на отсутствие менструации в течение года, бесплодие. Проведена проба Коха - резко положительна. Проба с прогестероном отрицательна. Планируется гистеросальпингография. Чем характеризуется R- логическая картина матки при данной патологии:A. наличием большого, округлого дефекта B. наличием множества небольших полостей, разделенных перегородками C. наличием законтурных тенейD. полным отсутствием контрастного вещества в матке

Page 19: postgraduate.kaznmu.kz€¦ · Web view58 Больная И, 28 лет, доставлена с жалобами на внезапно возникшую боль внизу живота

С.Ж.АСФЕНДИЯРОВАТЫНДАҒЫҚАЗАҚ ҰЛТТЫҚ МЕДИЦИНА

УНИВЕРСИТЕТІ

КАЗАХСКИЙНАЦИОНАЛЬНЫЙМЕДИЦИНСКИЙ УНИВЕРСИТЕТ ИМЕНИ С.Д.АСФЕНДИЯРОВА

КАФЕДРА ИНТЕРНАТУРЫ И РЕЗИДЕНТУРЫ ПО АКУШЕРСТВУ И ГИНЕКОЛОГИИТЕСТОВЫЕ ЗАДАНИЯ ДЛЯ ПРОВЕДЕНИЯ ИТОГОВОЙ ГОСУДАРСТВЕННОЙ АТТЕСТАЦИИ РЕЗИДЕНТОВ

ВЫПУСКА 2012-2013 ГГ. ПО СПЕЦИАЛЬНОСТИ «АКУШЕРСТВО И ГИНЕКОЛОГИЯ»

E. наличием четких контуров матки 73 Пациентке 24 лет произведено раздельное выскабливание слизистой полости матки по поводу длительных кровянистых выделений из половых путей. Состоит на учете у фтизио- гинеколога. Какая особенность гистологической картины будет найдена в данном случае?A. железистая гиперплазия эндометрияB. железисто-кистозная гиперплазия эндометрияC. аденоматоз слизистой полости маткиD. наличие гигантских клеток в эндометрииE. выраженная мелкоклеточная инфильтрация 74 Неосложненный инфицированый аборт – это:A. инфекция в полости маткиB. метротролебофлебитC. сальпингоофоритD. пельвиоперитонитE. сепсис75 Осложненный инфицированый аборт – это:A. инфекция в полости маткиB. инфекция локализована в малом тазуC. перитонитD. разлитой перитонитE. сепсис76 Симптомокомплекс: повышение температуры тела до 38° С, озноб, кровянисто-гнойные выделения, болезненность матки, отсутствие инфильтрации околоматочной клетчатки, элементы плодного яйца в состоянии гнойного расплавления, характерен для:A. плацентарного полипаB. неполного самопроизвольного абортаC. неполного неосложненного инфицированного абортаD. неполного осложненного инфицированного абортаE. полного аборта77 Симптомокомплекс: повышение температуры тела до 39° С, озноб, гнойные выделения из половых путей, элементы плодного яйца в состоянии гнилостного расплавления, тромбофлебит вен голени, характерен для:A. плацентарного полипаB. неполного самопроизвольного абортаC. неполного неосложненного инфицированного абортаD. неполного осложненного инфицированного абортаE. полного аборта78 Симптомокомплекс: субфебрилитет, кровянистые мажущие выделения из половых путей, в анамнезе был аборт две недели назад, характерен для:A. плацентарного полипаB. неполного самопроизвольного абортаC. неполного неосложненного инфицированного абортаD. неполного осложненного инфицированного абортаE. полного аборта79 Признаком внебольничного инфицированного аборта является:A. боли внизу живота и в пояснице, кровянистые выделения из половых путей, повышение температуры тела свыше 37° С, указание на отхождение плодного яйца или кусочков ткани

Page 20: postgraduate.kaznmu.kz€¦ · Web view58 Больная И, 28 лет, доставлена с жалобами на внезапно возникшую боль внизу живота

С.Ж.АСФЕНДИЯРОВАТЫНДАҒЫҚАЗАҚ ҰЛТТЫҚ МЕДИЦИНА

УНИВЕРСИТЕТІ

КАЗАХСКИЙНАЦИОНАЛЬНЫЙМЕДИЦИНСКИЙ УНИВЕРСИТЕТ ИМЕНИ С.Д.АСФЕНДИЯРОВА

КАФЕДРА ИНТЕРНАТУРЫ И РЕЗИДЕНТУРЫ ПО АКУШЕРСТВУ И ГИНЕКОЛОГИИТЕСТОВЫЕ ЗАДАНИЯ ДЛЯ ПРОВЕДЕНИЯ ИТОГОВОЙ ГОСУДАРСТВЕННОЙ АТТЕСТАЦИИ РЕЗИДЕНТОВ

ВЫПУСКА 2012-2013 ГГ. ПО СПЕЦИАЛЬНОСТИ «АКУШЕРСТВО И ГИНЕКОЛОГИЯ»

B. боли внизу живота и в пояснице, гнойные выделения из половых путей, повышение температуры тела до 39° СC. боли внизу живота, кровянистые выделения из половых путей, указание на отхождение кусочков ткани, медабортD. боли внизу живота, повышение температуры тела до 38° С, кровянисто-гнойные выделения из половых путей, инфильтрация околоматочной клетчаткиE. боли внизу живота и в области поясницы, кровянисто-мажущие выделения из половых путей, снижение артериального давления, лихорадка, олигоурия или анурия80 Симптомокомплекс: повышение температуры тела до 38° С, озноб, гнойные выделения, болезненность матки, отсутствие инфильтрации околоматочной клетчатки, элементы плодного яйца в состоянии гнойного расплавления. Какие лечебные мероприятия наиболее целесообразно включить:A. длительная антибактериальная терапияB. выскабливание полости матки с последующей антибактериальной терапиейC. выскабливание полости матки, холод на низ животаD. ампутация маткиE. экстирпация матки81 У больной после внебольничного аборта нарушение гемодинамики, дыхания, почек и печени, нарушения метаболизма, нарушения центральной нервной системы, симптомы инфекции. Какие данные наиболее вероятно предполагают диагноз септического шока?A. отек легких, внутрисосудистый гемолиз, острая почечная недостаточность, развитие ДВС синдромаB. повышение температуры тела, боли внизу живота, вынужденное положение с согнутой в колене ногойC. внезапная боль внизу живота, холодный пот, снижение артериального давления, слабый частый пульс, выраженная бледностьD. повышение температуры тела 39-40°С, многократное повторение озноба, поражение отдельных органов E. выраженная желтуха с бронзовым оттенком, олигоурия или анурия, характерная бурая окраска мочи 82 Послеабортный перитонит чаще развивается при внебольничном аборте. Какая из следующих причин вероятнее всего является ведущей: A. остатки части хориона и децидуальной оболочкиB. отсутствие правил асептики и антисептикиC. распространение инфекции за пределы матки и малого тазаD. возникновение плацентарных полиповE. неправильная техника операции83 Больная поступила в стационар по поводу внебольничного аборта с подозрением на перфорацию матки. Наиболее вероятно, выбор врачебной тактики зависит от:A. размера кровопотери, наличие инфекции и перитонеальных явленийB. данных влагалищного исследования, зондирования, кровотеченияC. ограничения ушиванием перфорационного отверстияD. желания сохранения генеративной функцииE. ограничения ревизией органов малого таза84 Хирургическое удаление источника инфекции при сепсисе после внебольничного аборта производится через 6-8 часов. Чем объясняется данная тактика? A. значительным улучшением общего состояния больнойB. нарастанием симптомов интоксикации

Page 21: postgraduate.kaznmu.kz€¦ · Web view58 Больная И, 28 лет, доставлена с жалобами на внезапно возникшую боль внизу живота

С.Ж.АСФЕНДИЯРОВАТЫНДАҒЫҚАЗАҚ ҰЛТТЫҚ МЕДИЦИНА

УНИВЕРСИТЕТІ

КАЗАХСКИЙНАЦИОНАЛЬНЫЙМЕДИЦИНСКИЙ УНИВЕРСИТЕТ ИМЕНИ С.Д.АСФЕНДИЯРОВА

КАФЕДРА ИНТЕРНАТУРЫ И РЕЗИДЕНТУРЫ ПО АКУШЕРСТВУ И ГИНЕКОЛОГИИТЕСТОВЫЕ ЗАДАНИЯ ДЛЯ ПРОВЕДЕНИЯ ИТОГОВОЙ ГОСУДАРСТВЕННОЙ АТТЕСТАЦИИ РЕЗИДЕНТОВ

ВЫПУСКА 2012-2013 ГГ. ПО СПЕЦИАЛЬНОСТИ «АКУШЕРСТВО И ГИНЕКОЛОГИЯ»

C. стиханием явлений воспаленияD. нормализацией картины кровиE. необходимостью комплексной консервативной терапии85 Триада Нюринберга встречается при анаэробном сепсисе, потому что в триаду входят следующие клинические признаки: ГДЕ 3-Й СИМПТОМ?A. бронзовая окраска кожи, снижение палочко-ядерных лейкоцитовB. темная плазма, темная мочаC. бактериурия, критическое снижение температуры телаD. олигоурия или анурия, снижение уровня биллирубинаE. острая почечная недостаточность в результате гемолиза, повышение уровня гемоглобина86 Женщине 30 лет, в анамнезе 3 родов, 2 аборта, последняя беременность 3 месяца назад – срочные роды с тяжелой преэклампсией. В настоящее время кормит ребенка грудью. Оптимальным методом контрацепции в данном случае является:A. метод лактационной аменореи B. комбинированные оральные контрацептивыC. внутриматочная контрацепцияD. метод естественного планирования семьиE. механические методы контрацепции87 Женщине 30 лет, в анамнезе 3 родов, 2 аборта, последняя беременность 3 месяца назад – срочные роды с тяжелой преэклампсией. В настоящее время кормит ребенка грудью. Срок использования данного метода контрацепции:A. 3 месяцаB. 5 месяцевC. 6 месяцев D. 8 месяцевE. 12 месяцев88 У молодой женщины во время использования противозачаточного средства менструации не было. Выберите препараты, прием которых дает подобное осложнение:A. пероральные комбинированные контрацептивы B. шеечные колпачкиC. чистые гестагеныD. презервативыE. спермициды89 Назовите гормональный контрацептив, рекомендуемый женщинам с гирсутизмом, акне или себореей:A. Ярина B. Менданет 20C. Диане 35D. ТриквиларE. Жанин90 Женщине 40 лет, обратилась к врачу с жалобами на отек, гиперемию в области ягодиц. 3 дня назад была введена инъекция Депо-Провера с контрацептивной целью. Ваши действия при подготовке к инъекции:A. протереть место инъекции начиная от центра круговыми перекрывающими движениями В. протереть место инъекции 96°спиртом круговыми перекрывающими движениями С. протереть место инъекции антисептиком начиная от центра D. вымыть кожу, дать высохнуть перед инъекцией E. протереть место инъекции 96°спиртом

Page 22: postgraduate.kaznmu.kz€¦ · Web view58 Больная И, 28 лет, доставлена с жалобами на внезапно возникшую боль внизу живота

С.Ж.АСФЕНДИЯРОВАТЫНДАҒЫҚАЗАҚ ҰЛТТЫҚ МЕДИЦИНА

УНИВЕРСИТЕТІ

КАЗАХСКИЙНАЦИОНАЛЬНЫЙМЕДИЦИНСКИЙ УНИВЕРСИТЕТ ИМЕНИ С.Д.АСФЕНДИЯРОВА

КАФЕДРА ИНТЕРНАТУРЫ И РЕЗИДЕНТУРЫ ПО АКУШЕРСТВУ И ГИНЕКОЛОГИИТЕСТОВЫЕ ЗАДАНИЯ ДЛЯ ПРОВЕДЕНИЯ ИТОГОВОЙ ГОСУДАРСТВЕННОЙ АТТЕСТАЦИИ РЕЗИДЕНТОВ

ВЫПУСКА 2012-2013 ГГ. ПО СПЕЦИАЛЬНОСТИ «АКУШЕРСТВО И ГИНЕКОЛОГИЯ»

91 Женщине 40 лет, обратилась к врачу с жалобами на отек, гиперемию в области ягодиц. 3 дня назад была введена инъекция Депо-Провера с контрацептивной целью. Возможные осложнения после инъекций:A. аллергические реакцииB. кровотечениеC. инфильтратD. абсцессE. жжение92 Женщине 29 лет, в анамнезе 2 родов, 1 аборт, последняя беременность 4 месяца назад – срочные роды с тяжелой преэклампсией. Оптимальным методом контрацепции в данном случае является ВМС, потому что – этоA. обратимый долгосрочный методB. метод, не предохраняющий от ИПППC. снижает риск развития рака эндометрия D. предотвращает железодефицитную анемию E. снижает риск развития внематочной беременности93 У повторно-беременной первородящей женщины произошли преждевременные роды мертвым плодом. Перед выпиской из стационара проведено обследование: Hb 98г/л, эритроцитов 3х10 12/л, лейкоцитов 6,8х10 9/л, СОЭ 18 мм/час. Что является оптимальным средством контрацепции для пациентки:A. ВМС B. КОК C. ЧПКD. ДХСE. барьерные средства94 К врачу гинекологу обратилась девушка 18 лет по поводу выбора метода контрацепции. Не замужем. Половые партнеры разные. Какой метод контрацепции показан и почему:A. ВМС - долгосрочный и надежный метод B. КОК применяется в любом возрасте, предотвращает развитие ЖДА или уменьшает ее тяжестьC. ЧПК - эффективны в период грудного вскармливанияD. ДХС - надежный и необратимый метод контрацепцииE. барьерные средства – недорогие, немедленное восстановление фертильности, защищают от ЗППП95 Женщине 35 лет, она мать троих детей, больше рожать категорически не хочет. Какой метод контрацепции ей наиболее показан и почему? A. ВМС - долгосрочный и надежный метод B. КОК применяется в любом возрасте, предотвращает развитие ЖДА или уменьшает ее тяжестьC. ЧПК - эффективны в период грудного вскармливанияD. ДХС - надежный и необратимый метод контрацепцииE. барьерные средства, потому что недорогие и дают немедленное восстановление фертильности защищая от ЗППП96 При гинекологическом исследовании пациентки 32 лет найдено увеличение матки до 5-6 недель беременности. Отмечается увеличение матки в переднезаднем размере. Тест на беременность отрицателен. Женщина настаивает на курортном лечении. Какая разновидность водолечения показана в данном случае?A. сульфидные ванныB. хлоридные ванныC. мышьяксодержащие ванны

Page 23: postgraduate.kaznmu.kz€¦ · Web view58 Больная И, 28 лет, доставлена с жалобами на внезапно возникшую боль внизу живота

С.Ж.АСФЕНДИЯРОВАТЫНДАҒЫҚАЗАҚ ҰЛТТЫҚ МЕДИЦИНА

УНИВЕРСИТЕТІ

КАЗАХСКИЙНАЦИОНАЛЬНЫЙМЕДИЦИНСКИЙ УНИВЕРСИТЕТ ИМЕНИ С.Д.АСФЕНДИЯРОВА

КАФЕДРА ИНТЕРНАТУРЫ И РЕЗИДЕНТУРЫ ПО АКУШЕРСТВУ И ГИНЕКОЛОГИИТЕСТОВЫЕ ЗАДАНИЯ ДЛЯ ПРОВЕДЕНИЯ ИТОГОВОЙ ГОСУДАРСТВЕННОЙ АТТЕСТАЦИИ РЕЗИДЕНТОВ

ВЫПУСКА 2012-2013 ГГ. ПО СПЕЦИАЛЬНОСТИ «АКУШЕРСТВО И ГИНЕКОЛОГИЯ»

D. радоновые ванныE. углекислые ванны97 Пациентка 35 лет обратилась к гинекологу с жалобами на постоянные боли внизу живота. В анамнезе - хроническое воспаление придатков матки справа. Прошла курс антибиотикотерапии с учетом чувствительности микрофлоры. Боли уменьшились незначительно. Какой из методов физиолечения наиболее целесообразен в данном случае?A. УВЧ - терапияB. СВЧ - терапияC. местная дарсонвализация D. магнитотерапияE. гидроаэротерапия98 Пациента 28 лет обратилась к гинекологу с жалобами на ноющие боли внизу живота, скудные, редкие менструации. Неоднократно лечилась по поводу хронического аднексита. Планирует продолжить лечение на курорте.Какой из перечисленных ниже методов лечения необходимо выбрать для данной пациентки? A. водолечение (радоновые ванны)B. аэротерапиюC. гелеотерапиюD. талассотерапиюE. пелоидолечение99 Девушка 16 лет обратилась к гинекологу с жалобами на редкие, скудные менструации со времени menarche. Получает гормональную терапию, эндоназальный йонофорез с витамином В1. Эффект от лечения незначительный. Какой физиотерапевтический метод лечения необходимо добавить в данном случае?A. электрофорез йода синусоидальными модулированными токами B. электрофорез меди синусоидальными модулированными токамиC. электрофорез магния синусоидальными модулированными токамиD. электрофорез цинка синусоидальными модулированными токамиE. низкочастотную магнитотерапию100 Пациентка 47 лет обратилась к гинекологу с жалобами на ноющие боли внизу живота, нерегулярные менструации, плохой сон, раздражительность,”приливы”. В анамнезе - хронический левосторонний аднексит. Получает таблетки фемостона. Настаивает на проведении физиолечения. Какой метод физиолечения необходимо рекомендовать в данном случае?A. электрофорез с новокаином на область шеи B. электрофорез меди синусоидальными модулированными токамиC. электрофорез йода синусоидальными модулированными токамиD. ультразвуковую терапиюE. СВЧ-терапию дециметровыми волнами 101 Девочка 14 лет обратилась к гинекологу с жалобами на резко болезненные менструации. Получает медикаментозное лечение. Эффект от лечения частичный. Какой метод физиотерапии дополнит медикаментозное лечение?A. диадинамотерапияB. электрофорез кальция эндоназальныйC. эндоназальная гальванизация D. ультразвуковая терапияE. гальванизация зоны “воротника"

Page 24: postgraduate.kaznmu.kz€¦ · Web view58 Больная И, 28 лет, доставлена с жалобами на внезапно возникшую боль внизу живота

С.Ж.АСФЕНДИЯРОВАТЫНДАҒЫҚАЗАҚ ҰЛТТЫҚ МЕДИЦИНА

УНИВЕРСИТЕТІ

КАЗАХСКИЙНАЦИОНАЛЬНЫЙМЕДИЦИНСКИЙ УНИВЕРСИТЕТ ИМЕНИ С.Д.АСФЕНДИЯРОВА

КАФЕДРА ИНТЕРНАТУРЫ И РЕЗИДЕНТУРЫ ПО АКУШЕРСТВУ И ГИНЕКОЛОГИИТЕСТОВЫЕ ЗАДАНИЯ ДЛЯ ПРОВЕДЕНИЯ ИТОГОВОЙ ГОСУДАРСТВЕННОЙ АТТЕСТАЦИИ РЕЗИДЕНТОВ

ВЫПУСКА 2012-2013 ГГ. ПО СПЕЦИАЛЬНОСТИ «АКУШЕРСТВО И ГИНЕКОЛОГИЯ»

102 У пациентки 26 лет диагностирован ретроцервикальный эндометриоз I степени распространения 1, 5 года тому назад. Получает комплексную терапию, в том числе и лечение норколутом. Отмечает улучшение состояния, но боли а животе еще беспокоят. Какой метод физиотерапии можно рекомендовать в данном случае?A. ультразвуковую терапию B. магнитотерапиюC. дарсонвализациюD. ультравысокочастотную терапиюE. ультрафиолетовое облучение103 Пациентка 29 лет обратилась к врачу с жалобами на интенсивные боли внизу живота, больше справа, повышение температуры тела до субфебрильных цифр. При гинекологическом исследовании в области придатков определяется пастозность, болезненность. Назначено комплексное лечение. Какой вид физиотерапии необходимо назначить для уменьшения болевого синдрома? A. ультравысокочастотную терапиюB. ультрафиолетовое облучение “трусиковой “ зоныC. электрофорез йода синусоидальными модулированными токамиD. электрофорез лидазы и пелоидинаE. фонофорез лидазы и пелоидина104 Женщина 32 лет находится на диспансерном учете у гинеколога по поводу хронического аднексита в течение 3 лет. При гинекологическом исследовании в области правых придатков определяется тяжистость, незначительная болезненность. Какой из методов физиотерапии предпочтителен в данном случае?A. ультразвуковая терапия в импульсном режимеB. ультрафиолетовое облучение “трусиковой “ зоныC. терапия микроволнами сантиметрового диапазонаD. магнитотерапия курсами E. электрофорез кальция синусоидальными токами105 У женщины 32 лет на 10 день после лапаротомии по поводу тубоовариального образования воспалительного генеза имеется рана в области послеоперационного шва. Получает комплексную антибактериальную терапию, местное лечение. Какой вид физиотерапии предпочтителен в данном случае? A. электрофорез цинка синусоидальными модулированными токами B. электрофорез йода синусоидальными модулированными токами C. электрофорез кальция синусоидальными модулированными токами D. низкочастотное лазерное излучение E. электрофорез лидазы и пеллоидина 106 Женщине 23 произведено прерывание беременности путем медицинского аборта. В анамнезе хронический аднексит. Какой метод физиотерапии предпочтителен со дня операции?A. ультрафиолетовое облучение трусиковой зоны B. электрофорез лидазы и пеллоидинаC. низкочастотное лазерное излучениеD. терапия микроволнами сантиметрового диапазонаE. электрофорез меди синусоидальными модулированными токами107 Больная А. 40 лет жалуется на периодически возникающий зуд кожи, отеки лица и конечностей, олигурию за неделю до менструации. Менструальная функция не нарушена. В анамнезе 1 криминальный аборт. Гинекологическое исследование патологии не выявило.

Page 25: postgraduate.kaznmu.kz€¦ · Web view58 Больная И, 28 лет, доставлена с жалобами на внезапно возникшую боль внизу живота

С.Ж.АСФЕНДИЯРОВАТЫНДАҒЫҚАЗАҚ ҰЛТТЫҚ МЕДИЦИНА

УНИВЕРСИТЕТІ

КАЗАХСКИЙНАЦИОНАЛЬНЫЙМЕДИЦИНСКИЙ УНИВЕРСИТЕТ ИМЕНИ С.Д.АСФЕНДИЯРОВА

КАФЕДРА ИНТЕРНАТУРЫ И РЕЗИДЕНТУРЫ ПО АКУШЕРСТВУ И ГИНЕКОЛОГИИТЕСТОВЫЕ ЗАДАНИЯ ДЛЯ ПРОВЕДЕНИЯ ИТОГОВОЙ ГОСУДАРСТВЕННОЙ АТТЕСТАЦИИ РЕЗИДЕНТОВ

ВЫПУСКА 2012-2013 ГГ. ПО СПЕЦИАЛЬНОСТИ «АКУШЕРСТВО И ГИНЕКОЛОГИЯ»

Какой из перечисленных методов исследования наименее целесообразен?A. тесты функциональной диагностики B. Гормональные исследованияC. ЭЭГD. консультация терапевтаE. УЗИ органов малого таза108 У больной 29 лет жалобы на головные боли, приливы, сердцебиения, периодически ..….повышение АД, плаксивость. В анамнезе 8 месяцев назад было удаление обоих яичников по поводу кистомы. Повышенного питания. АД - 140/80, 150/90 мм рт.ст. При гинекологическом исследовании: сухость влагалища, матка в размерах несколько уменьшена, придатки не определяются. Такие данные предполагают, что заболевание является результатом какого из следующих синдромов?A.предменструального C. нейроциркуляторной дистония D. посткастрационного E. Гипертонической болезни F. климактерического 109 У больной 29 лет жалобы на головные боли, приливы, сердцебиения, периодически…. повышение АД, плаксивость. В анамнезе 8 месяцев назад было удаление обоих яичников по поводу кистом. Повышенного питания. АД - 140/80, 150/90мм.рт.ст. При гинекологическом исследовании: сухость влагалища, матка в размерах несколько уменьшена, придатки не определяются. Гипотеза наиболее характерного и постоянного симптома данной патологии:A. гипертонияB. приливыC. головная больD. сердцебиениеE. плаксивость110 У больной 29 лет жалобы на головные боли, приливы, сердцебиение, периодически… повышение АД, плаксивость. В анамнезе 8 месяцев назад было удаление обоих яичников по поводу кистом. Повышенного питания. АД - 140/80, 150/90 (мм.рт.ст). При гинекологическом исследовании: сухость влагалища, матка в размерах несколько уменьшена, придатки не определяются. Какое лечение наиболее целесообразно при данном заболевании?A.общеукрепляющая терапияB. физиотерапияC. гормонотерапияD. седативная терапияE. фитотерапия111 У больной 29 лет жалобы на головные боли, приливы, сердцебиения, периодически повышение АД, плаксивость. В анамнезе 8 месяцев назад было удаление обоих яичников но поводу кистом. Повышенного питания. АД - 140/80, 150/90 мм.рт.ст. При гинекологическом исследовании: сухость влагалища, матка в размерах несколько уменьшена, придатки не определяются. Наиболее целесообразна гормонотерапия:A. андрогенамиB. гестагенамиC. эстрогенами

Page 26: postgraduate.kaznmu.kz€¦ · Web view58 Больная И, 28 лет, доставлена с жалобами на внезапно возникшую боль внизу живота

С.Ж.АСФЕНДИЯРОВАТЫНДАҒЫҚАЗАҚ ҰЛТТЫҚ МЕДИЦИНА

УНИВЕРСИТЕТІ

КАЗАХСКИЙНАЦИОНАЛЬНЫЙМЕДИЦИНСКИЙ УНИВЕРСИТЕТ ИМЕНИ С.Д.АСФЕНДИЯРОВА

КАФЕДРА ИНТЕРНАТУРЫ И РЕЗИДЕНТУРЫ ПО АКУШЕРСТВУ И ГИНЕКОЛОГИИТЕСТОВЫЕ ЗАДАНИЯ ДЛЯ ПРОВЕДЕНИЯ ИТОГОВОЙ ГОСУДАРСТВЕННОЙ АТТЕСТАЦИИ РЕЗИДЕНТОВ

ВЫПУСКА 2012-2013 ГГ. ПО СПЕЦИАЛЬНОСТИ «АКУШЕРСТВО И ГИНЕКОЛОГИЯ»

D. эстрогенами и гестагенами …E. глюокортикоидами112 У больной 29 лет жалобы на раздражительность, плаксивость, головною боль, головокружение, тошноту, иногда рвоту, боли в области сердца, нагрубание молочных желез, снижение памяти, метеоризм. Все эти симптомы появляются за 6-14 дней до менструации и исчезают накануне неё. При специальном гинекологическом обследовании изменений со стороны гениталий не обнаружено, по тестам функциональной диагностики – овуляторный менструальный цикл.Наиболее вероятная тактика ведения:A. стационарное лечениеB. назначить низкодозированные КОК-иC. назначить физиолечение - "воротник» по ЩербакуD. ограничение повареной соли и жидкости во II фазу менструального циклаЕ. назначить глюокортикоиды113 У больной 29 лет жалобы на раздражительность, плаксивость, головною боль, головокружение, тошноту, иногда рвоту, боли в области сердца, нагрубание молочных желез, снижение памяти, метеоризм. Все эти симптомы появляются за 6-14 дней до менструации и исчезают накануне ее. При специальном гинекологическом обследовании изменений со стороны гениталий не обнаружено, по тестам функциональной диагностики овуляторный цикл.Наиболее вероятный диагноз:A. тиреотоксикоз.B. предменструальный синдром.C. ранний климаксD. эндометриозE. миома матки114 Пациентка 26 лет жалуется на ухудшение состояния за 5 дней до менструации: отеки, прибавку в весе, ощущение напряжения и болезненности молочных желез, раздражительность, плохое настроение, головную боль. С наступлением очередной менструации жалобы исчезают. При гинекологическом осмотре патологических изменений не выявлено.Выберите лечебную тактику:A. седативные (грандаксин) по 2 таблетке утром и в обедB. витаминотерапия (магний В6)C. диуретикиD. группа нитроимидазолаE. физиолечение115 Пациентка 32 лет обратилась к врачу-гинекологу с жалобами на боли внизу живота, которые появляются за 7 дней перед менструацией и усиливаются в 1 день менструации. Заболела через некоторое время после диатермокоагуляции шейки матки, выполненной по поводу псевдоэрозии 2 года назад. PV: матка несколько увеличена в размерах, шаровидной формы, плотной консистенции, придатки без особенностей. Шейка матки без патологических изменений.Наиболее вероятный диагноз:A. экстрагенитальный эндометриозB. миома матки субмукозная формаC. альгодисменореяD. преклимактерический периодE. нейроциркуляторная дистония ….116 Больная 19 лет обратилась в поликлинику с жалобами на раздражительность, плаксивость, головною боль, головокружение, тошноту, иногда рвоту, боли в области сердца, нагрубание

Page 27: postgraduate.kaznmu.kz€¦ · Web view58 Больная И, 28 лет, доставлена с жалобами на внезапно возникшую боль внизу живота

С.Ж.АСФЕНДИЯРОВАТЫНДАҒЫҚАЗАҚ ҰЛТТЫҚ МЕДИЦИНА

УНИВЕРСИТЕТІ

КАЗАХСКИЙНАЦИОНАЛЬНЫЙМЕДИЦИНСКИЙ УНИВЕРСИТЕТ ИМЕНИ С.Д.АСФЕНДИЯРОВА

КАФЕДРА ИНТЕРНАТУРЫ И РЕЗИДЕНТУРЫ ПО АКУШЕРСТВУ И ГИНЕКОЛОГИИТЕСТОВЫЕ ЗАДАНИЯ ДЛЯ ПРОВЕДЕНИЯ ИТОГОВОЙ ГОСУДАРСТВЕННОЙ АТТЕСТАЦИИ РЕЗИДЕНТОВ

ВЫПУСКА 2012-2013 ГГ. ПО СПЕЦИАЛЬНОСТИ «АКУШЕРСТВО И ГИНЕКОЛОГИЯ»

молочных желез, снижение памяти, метеоризм. Все эти симптомы появляются за 6-14 дней до менструации и исчезают накануне ее. При специальном гинекологическом обследовании изменений со стороны гениталий не обнаружено. Диагноз Предменструальный синдром. Наименее вероятная теория возникновения предменструального синдрома:A. гиперпролактинемияB. снижение секреция андростендионаC. увеличение содержания простагландиновD. нарушение функции системы гипоталамус-гипофиз-надпочечникиE. снижение уровня эндогенных опиоидных пептидов117 У больной 39 лет жалобы на раздражительность, плаксивость, головную боль, головокружение, тошноту, иногда рвоту, боли в области сердца, нагрубание молочных желез, снижение памяти, метеоризм. Все эти симптомы появляются за 6-14 дней до менструации и исчезают накануне ее. При специальном гинекологическом обследовании изменений со стороны гениталий не обнаружено. Диагноз Предменструальный синдром.Наименее вероятная форма данного заболевания:A. психовегетативная B. цефалгическая C. кардиогеннаяD. кризовая E. отечная118 Больная Н., 29 лет обратилась к врачу женской консультации с жалобами на головные боли, приливы, сердцебиения, периодически отмечает повышение АД, плаксивость. В анамнезе 8 месяцев назад было удаление обоих яичников по поводу кистом с обеих сторон. Объективно: повышенного питания. АД - 140/80, 150/90 мм.рт.ст. При гинекологическом исследовании: сухость влагалища, матка в размерах несколько уменьшена, придатки не определяются. Наиболее вероятная причина данной патологии:A. снижение секреции паратиреоидного гормонаB. резкое снижение выработки стероидных гормоновC. недостаток секреции лютеинизирующего гормонаD. повышение гипоталамо-гипофизарной активностиE. избирательное повышение секреции ФСГ119 Пациентка 26 лет жалуется на ухудшение состояния за 5 дней до менструации: отеки, прибавка в весе, ощущение напряжения и болезненности молочных желез, раздражительность, плохое настроение, головная боль. С наступлением очередной менструации жалобы исчезают. При гинекологическом осмотре патологических изменений не выявлено. Диагноз: ПМС, сочетанная форма.Наиболее вероятная причина этого состояния:A. снижение уровня серотонинаB. повышение уровня серотонинаC. увеличение секреция простагландиновD. снижение уровня эндорфиновE. увеличение секреция андростендиона120 Больная Е., 31 лет, поступила с жалобами на периодические резкие нарушения самочувствия, выражающиеся в депрессии, утомляемости, боли в суставах. Схваткообразные боли в животе, отеки лица, рук и ног, частые позывы на мочеиспускание. Год назад операция - тотальная овариэктомия.Причиной этого состояния может быть:A. гиперэстрогения

Page 28: postgraduate.kaznmu.kz€¦ · Web view58 Больная И, 28 лет, доставлена с жалобами на внезапно возникшую боль внизу живота

С.Ж.АСФЕНДИЯРОВАТЫНДАҒЫҚАЗАҚ ҰЛТТЫҚ МЕДИЦИНА

УНИВЕРСИТЕТІ

КАЗАХСКИЙНАЦИОНАЛЬНЫЙМЕДИЦИНСКИЙ УНИВЕРСИТЕТ ИМЕНИ С.Д.АСФЕНДИЯРОВА

КАФЕДРА ИНТЕРНАТУРЫ И РЕЗИДЕНТУРЫ ПО АКУШЕРСТВУ И ГИНЕКОЛОГИИТЕСТОВЫЕ ЗАДАНИЯ ДЛЯ ПРОВЕДЕНИЯ ИТОГОВОЙ ГОСУДАРСТВЕННОЙ АТТЕСТАЦИИ РЕЗИДЕНТОВ

ВЫПУСКА 2012-2013 ГГ. ПО СПЕЦИАЛЬНОСТИ «АКУШЕРСТВО И ГИНЕКОЛОГИЯ»

B. гипоэстрогенияC. увеличение секреция простагландиновD. снижение уровня эндорфиновE. увеличение секреция андростендиона121 Больная Е., 37 лет, поступила с жалобами на периодические резкие нарушения самочувствия, выражающиеся в депрессии, утомляемости, боли в суставах. Схваткообразные боли в животе, отеки лица, рук и ног, частые позывы на мочеиспускание. Год назад операция - тотальная овариэктомия. Лечебная тактика: A. КОК в режиме контрацепции B. прогестагены (производные 19-норстерона, 19-нортестостерона)C. заместительная гормонотерапияD. производные 17 –α этинилтестостеронаE. аналоги Гн-РГ 122 Больная Е., 39 лет, поступила с жалобами на периодические резкие нарушения самочувствия, выражающиеся в депрессии, утомляемости, боли в суставах. Схваткообразные боли в животе, отеки лица, рук и ног, частые позывы на мочеиспускание. Год назад операция - тотальная овариэктомия. В возникновении данного заболевания существенное значение имеет:A. увеличение хориального гонадотропина B. гиперэстрогеннияC. уменьшение прогестеронаD. снижение синтеза нейротрансмиттеровE. уменьшение гормона обладающих свойствами андрогенов123 Больная Е., 30 лет, поступила с жалобами на периодические резкие нарушения самочувствия, выражающиеся в депрессии, утомляемости, боли в суставах. Схваткообразные боли в животе, отеки лица, рук и ног, частые позывы на мочеиспускание. Год назад операция экстирпации матки без придатковВ возникновении данного заболевания существенное значение имеет:A. увеличение хориального гонадотропина B. гиперэстрогеннияC. уменьшение прогестеронаD. снижение синтеза нейротрансмиттеровE. уменьшение гормона обладающих свойствами андрогенов124 Больная Е., 33 лет, поступила с жалобами на периодические резкие нарушения самочувствия, выражающиеся в депрессии, утомляемости, повышение потливости, ознобы, сердцебиение в покое. Из анамнеза год назад операция экстирпации матки без придатков.В чем будет заключаться причина данного заболевания: A. увеличение хориального гонадотропина B. гиперэстрогеннияC. уменьшение прогестеронаD. снижение синтеза нейротрансмиттеровE. уменьшение гормона обладающих свойствами андрогенов125 Больная Е., 23 лет, поступила с жалобами на периодические резкие нарушения самочувствия, выражающиеся в депрессии, утомляемости, повышение потливости, ознобы, сердцебиение в покое, чувство онемения и ползания мурашек. Из анамнеза год назад операция экстирпации матки без придатков. Причиной этого состояния может быть:A. гиперэстрогения

Page 29: postgraduate.kaznmu.kz€¦ · Web view58 Больная И, 28 лет, доставлена с жалобами на внезапно возникшую боль внизу живота

С.Ж.АСФЕНДИЯРОВАТЫНДАҒЫҚАЗАҚ ҰЛТТЫҚ МЕДИЦИНА

УНИВЕРСИТЕТІ

КАЗАХСКИЙНАЦИОНАЛЬНЫЙМЕДИЦИНСКИЙ УНИВЕРСИТЕТ ИМЕНИ С.Д.АСФЕНДИЯРОВА

КАФЕДРА ИНТЕРНАТУРЫ И РЕЗИДЕНТУРЫ ПО АКУШЕРСТВУ И ГИНЕКОЛОГИИТЕСТОВЫЕ ЗАДАНИЯ ДЛЯ ПРОВЕДЕНИЯ ИТОГОВОЙ ГОСУДАРСТВЕННОЙ АТТЕСТАЦИИ РЕЗИДЕНТОВ

ВЫПУСКА 2012-2013 ГГ. ПО СПЕЦИАЛЬНОСТИ «АКУШЕРСТВО И ГИНЕКОЛОГИЯ»

B. снижение уровня простациклиновC. увеличение секреция простагландиновD. снижение уровня эндорфиновE. увеличение секреция андростендиона126 Больная Е., 33 лет, поступила с жалобами на периодические резкие нарушения самочувствия, выражающиеся в депрессии, утомляемости, повышение потливости, ознобы, сердцебиение в покое, чувство онемения и ползания мурашек. Из анамнеза год назад операция экстирпации матки без придатков. Какая из перечисленных причин наименее вероятна в возникновении данного заболевания?A. гиперэстрогенияB. снижение уровня простациклиновC. увеличение секреция простагландиновD. снижение уровня эндорфиновE. увеличение секреция андростендиона127 Пациентка 32 лет обратилась к врачу-гинекологу с жалобой на боли внизу живота, которые появляются за 7 дней перед менструацией и усиливаются в 1 день менструации. Заболела через некоторое время после диатермокоагуляции шейки матки, выполненной по поводу псевдоэрозии 2 года назад. Шейка матки без патологических изменений. PV: матка нормальных размеров, плотной консистенции, придатки без особенностей. Наиболее вероятное лечение?A. КОК в режиме контрацепции B. прогестагены (производные 19-норстерона, 19-нортестостерона)C. заместительная гормона терапияD. производные 17 –α этинилтестостеронаE. аналоги Гн-РГ 128 Больная 52 лет обратилась к гинекологу с жалобами на приливы жара до 10 раз в сутки, боли в области сердца, транзиторное повышение АД до 150/90 мм рт.ст. Постменопауза 2,5 года. При гинекологическом исследовании: наружные половые органы и влагалище в состоянии вторичной инволюции, матка уменьшена в размерах, придатки не пальпируются. Какое лечение следует рекомендовать больной:A. психотерапию, рациональную физио- и бальнеотерапиюB. органические нитраты (нитроглицерин, сустак и т.д.)C. естественные эстрогены в сочетании с прогестагенами D. синтетические прогестиныE. антигонадотропины129 Больная 19 лет обратилась с жалобами на плохое самочувствие, ожирение и отсутствие менструации. Объективно: больная повышенного питания, на плечевом поясе гипертрихоз. PV: шейка коническая, матка увеличена до 5-6 недель беременности. В области придатков с обеих сторон определяются тугоэластичные образования. В крови повышение уровня пролактина. При R-графии черепа: детское турецкое седло. Для какого синдрома характерна описанная клиника. A. СиммондсаB. Шерешевского-Тернера C. Фромеля-КиариD. «пустого» турецкого седла E. Шихана130 При проведении пункции заднего свода у женщины с внематочной беременностью, пунктат представляет собой:A. серозно-геморрагическую жидкостьB. алую кровь со сгустками

Page 30: postgraduate.kaznmu.kz€¦ · Web view58 Больная И, 28 лет, доставлена с жалобами на внезапно возникшую боль внизу живота

С.Ж.АСФЕНДИЯРОВАТЫНДАҒЫҚАЗАҚ ҰЛТТЫҚ МЕДИЦИНА

УНИВЕРСИТЕТІ

КАЗАХСКИЙНАЦИОНАЛЬНЫЙМЕДИЦИНСКИЙ УНИВЕРСИТЕТ ИМЕНИ С.Д.АСФЕНДИЯРОВА

КАФЕДРА ИНТЕРНАТУРЫ И РЕЗИДЕНТУРЫ ПО АКУШЕРСТВУ И ГИНЕКОЛОГИИТЕСТОВЫЕ ЗАДАНИЯ ДЛЯ ПРОВЕДЕНИЯ ИТОГОВОЙ ГОСУДАРСТВЕННОЙ АТТЕСТАЦИИ РЕЗИДЕНТОВ

ВЫПУСКА 2012-2013 ГГ. ПО СПЕЦИАЛЬНОСТИ «АКУШЕРСТВО И ГИНЕКОЛОГИЯ»

C. темную гемолизированную кровь D. гнойный экссудат с запахомE. в виде мясных помоев 131 Подострое течение воспаления гениталий – это:A. впервые возникший процесс с повышением температуры тела до 38-39 градусовB. впервые возникший процесс с менее выраженными клиническими проявлениямиC. длительно текущий процесс без интоксикации с субфебрильной температуройD. длительность процесса до 2 недель с повышением температуры до 39-40 градусов E. длительность процесса более 2 месяцев с субфебрильной температурой132 Клинические симптомы, характерные для интралигаментарной локализации миомы матки:A. запоры, схваткообразные болиB. менометроррагииC. затрудненное мочеиспускание, хроническая задержка мочи D. ранняя анемизация E. метроррагии, боли постоянного характера 133 Объем оперативного вмешательства при тубоовариальном образовании в пре- и постменопаузе:A. операция ВертгеймаB. одно- или двухстороннее удаление труб и яичников в пределах деструктивно измененных тканейC. экстирпация матки с придаткамиD. консервативная антибактериальная терапияE. надвлагалищная ампутация матки с придатками и резекцией большого сальника 134 Осложнения гнойных тубоовариальных образований:A. малигнизация, перекрут образованияB. кровоизлияние, асцит C. перфорация тубоовариального образования, перитонитD. перитонит, анемияE. нарушение менструального цикла135 При атипической гиперплазии эндометрия в возрасте менопаузы, у женщин без соматических и гинекологических заболеваний лечение заключается в:A. экстирпации матки с придаткамиB. гормонотерапия гестагенами и антиэстрогенамиC. сочетанная лучевая терапияD. химиотерапия и гормонотерапия E. надвлагалищная ампутация матки с придатками136 Тактика врача при перекруте ножки опухоли яичника:A. наложить зажим выше места перекрута и отсечь образованиеB. раскрутить перекрученную ножку, выделить образование яичника и удалить егоC. произвести надвлагалищную ампутацию матки с опухолью яичникаD. наложить зажим ниже места перекрута и отсечь образованиеE. раскрутить перекрученную ножку, консервативная терапия в послеоперационном периоде137 Пациентка 42 лет перенесла инфаркт миокарда, какой метод контрацепции наиболее приемлем:A. КОКB. КИК C. ЧПК D. КОК и ВМС

Page 31: postgraduate.kaznmu.kz€¦ · Web view58 Больная И, 28 лет, доставлена с жалобами на внезапно возникшую боль внизу живота

С.Ж.АСФЕНДИЯРОВАТЫНДАҒЫҚАЗАҚ ҰЛТТЫҚ МЕДИЦИНА

УНИВЕРСИТЕТІ

КАЗАХСКИЙНАЦИОНАЛЬНЫЙМЕДИЦИНСКИЙ УНИВЕРСИТЕТ ИМЕНИ С.Д.АСФЕНДИЯРОВА

КАФЕДРА ИНТЕРНАТУРЫ И РЕЗИДЕНТУРЫ ПО АКУШЕРСТВУ И ГИНЕКОЛОГИИТЕСТОВЫЕ ЗАДАНИЯ ДЛЯ ПРОВЕДЕНИЯ ИТОГОВОЙ ГОСУДАРСТВЕННОЙ АТТЕСТАЦИИ РЕЗИДЕНТОВ

ВЫПУСКА 2012-2013 ГГ. ПО СПЕЦИАЛЬНОСТИ «АКУШЕРСТВО И ГИНЕКОЛОГИЯ»

E. КИК и презерватив138 Наиболее частым механизмом развития дисфункциональных маточных кровотечений в климактерическом периоде является:A. гиполютеинизмB. персистенция фолликула C. атрезия фолликулаD. гиперпролактинемияE. нарушение в свертывающей системе крови.139 Гипопитуитаризм – это:A. снижение выработки всех гормонов гипофизаB. снижение выработки только соматотропного гормонаC. снижение выработки гонадотропного гормонаD. нарушения хромосомного набораE. снижение выработки гормонов коры надпочечников 140 Аменорея, ожирение, гирсутизм, бесплодие и поликистозные яичники характерны для синдрома:A. Киари –ФроммеляB. СиммондсаC. ШиханаD. Штейна – ЛевенталяE. Тернера –Шерешевского141 Для дисфункциональных маточных кровотечений по типу персистенции фолликула характерно:A. кариопикнотический индекс ниже 30%B. задержка менструации на 3 месяца и болееC. монофазная базальная температура, гиперплазия эндометрияD. нормальная базальная температура E. симптом зрачка ()142 Кариопикнотический индекс – это процентное отношение:A. поверхностных клеток влагалищного эпителия к общему числу клеток в мазкеB. базальных клеток влагалищного эпителия к общему числу клеток в мазкеC. поверхностных клеток к эозинофильным поверхностным клеткамD. гонококков к стафилококкам E. лейкоцитов к слущенному эпителию143 О какой форме аменореи свидетельствует отрицательный результат функциональной пробы с комбинированным эстроген-гестагенными препаратами:A. надпочечниковойB. маточной C. яичниковойD. гипофизарнойE. центральной 144 Патологическая аменорея характерна для:A. препубертатного периода B. постменопаузы C. периода лактацииD. беременностиE. бесплодия

Page 32: postgraduate.kaznmu.kz€¦ · Web view58 Больная И, 28 лет, доставлена с жалобами на внезапно возникшую боль внизу живота

С.Ж.АСФЕНДИЯРОВАТЫНДАҒЫҚАЗАҚ ҰЛТТЫҚ МЕДИЦИНА

УНИВЕРСИТЕТІ

КАЗАХСКИЙНАЦИОНАЛЬНЫЙМЕДИЦИНСКИЙ УНИВЕРСИТЕТ ИМЕНИ С.Д.АСФЕНДИЯРОВА

КАФЕДРА ИНТЕРНАТУРЫ И РЕЗИДЕНТУРЫ ПО АКУШЕРСТВУ И ГИНЕКОЛОГИИТЕСТОВЫЕ ЗАДАНИЯ ДЛЯ ПРОВЕДЕНИЯ ИТОГОВОЙ ГОСУДАРСТВЕННОЙ АТТЕСТАЦИИ РЕЗИДЕНТОВ

ВЫПУСКА 2012-2013 ГГ. ПО СПЕЦИАЛЬНОСТИ «АКУШЕРСТВО И ГИНЕКОЛОГИЯ»

145 У больных с аменореей при болезни Иценко-Кушинга имеет место гиперпродукция гормона аденогипофиза:A. соматотропногоB. тиреотропногоC. адренокортикотропногоD. фолликулостимулирующего E. лютеинизирующего146 При генитальном инфантилизме соотношение шейки и тела матки соответствуют обычноA. 1:3B. 1:2C. 1:1D. 3:1E. 2:1147 В периоде полового созревания наиболее частой причиной белей являютсяA. гонореяB. трихомониазC. экстрагенитальные заболеванияD. психогенные факторыE. несоблюдение личной гигиены148 На первом году после менархе у большинства девушек отмечаются ановуляторные маточные кровотечения, что обусловлено: A. ановуляциейB. наследственной неполноценностью гипоталамуса, гипофиза C. хроническими инфекциямиD. стрессовыми ситуациямиE. воспалительными заболеваниями гениталий149 Поступила 14 летняя девочка с жалобой на постоянные ежемесячные схваткообразные боли внизу живота и в области поясницы. Менархе отсутствует. Половая формула Ма3Р3Ах3Ме0. Причиной абдоминальных болей при аменорее в периоде пубертата может быть:A. атрезия влагалищаB. атрезия цервикального каналаC. атрезия геминаD. аплазия маткиE. воспалительный процесс гениталий150 На консультацию пришла 14 летняя девочка с жалобой на аменорею. Половая формула Ма0Р0Ах0Ме0. При обследовании был выставлен диагноз: тестикулярная феминизация. Выберите правильное утверждение, характеризующее данный синдром и тактику врача.A. сыворотка крови содержит повышенное количество тестостерона (как и в мужской сыворотке крови)B. гонады не удаляют до полного окончания полового созревания C. риск возникновения дисгерминомы в гонадах до 20 летD. задержка полового созревания из-за отсутствия эндогенных гормоновE. половое созревание отсутствует из-за избытки мужских половых гормонов151 Определите границы подросткового возраста по критериям ВОЗ:A. 15-18 летB. 14-19 летC. 12-20 летD. 10-19 лет

Page 33: postgraduate.kaznmu.kz€¦ · Web view58 Больная И, 28 лет, доставлена с жалобами на внезапно возникшую боль внизу живота

С.Ж.АСФЕНДИЯРОВАТЫНДАҒЫҚАЗАҚ ҰЛТТЫҚ МЕДИЦИНА

УНИВЕРСИТЕТІ

КАЗАХСКИЙНАЦИОНАЛЬНЫЙМЕДИЦИНСКИЙ УНИВЕРСИТЕТ ИМЕНИ С.Д.АСФЕНДИЯРОВА

КАФЕДРА ИНТЕРНАТУРЫ И РЕЗИДЕНТУРЫ ПО АКУШЕРСТВУ И ГИНЕКОЛОГИИТЕСТОВЫЕ ЗАДАНИЯ ДЛЯ ПРОВЕДЕНИЯ ИТОГОВОЙ ГОСУДАРСТВЕННОЙ АТТЕСТАЦИИ РЕЗИДЕНТОВ

ВЫПУСКА 2012-2013 ГГ. ПО СПЕЦИАЛЬНОСТИ «АКУШЕРСТВО И ГИНЕКОЛОГИЯ»

E. 15-21 год152 Закладка и формирование гонад плода детерминированы:A. хромосомным наборомB. гормональным фоном материC. влиянием мюллеровых производныхD. влиянием вольфовых производныхE. уровнем эстрогенов153 Производными мюллеровых протоков являются:A. наружные женские половые органыB. маточные трубы, матка, шейка маткиC. семявыносящие протокиD. наружные мужские половые органыE. влагалище 154 Яичники плода являются гормонально активными и вырабатывают:A. эстрогены, прогестеронB. эстрогены, андрогеныC. активин, релаксинD. ингибин, релаксинE. пролиферацию клеток мезенхимы155 Для развития паромезонефральных протоков необходимо влияние гормона:A. этинилэстрадиолаB. эстронаC. эстрадиолаD. тестостеронаE. независим от гормонов156 При кариотипе 45ХО внутренние половые органы будут развиваться по:A. женскому типуB. мужскому типуC. смешанному типуD. неопределенному типуE. характерно полное отсутствие внутренних половых органов157 Для простой вирильной формы адреногенитального синдрома у девочки наименее вероятно: A. кариотип 46ХХB. наличие маткиC. урогенитальный синусD. гипертрофия клитораE. отсутствие яичников158 Что из перечисленного характерно для полной формы тестикулярной феминизации: A. кариотип 46 ХУ, отсутствие маткиB. кариотип 46 ХХ, отсутствие маткиC. кариотип 46ХУ, наличие маткиD. кариотип 46ХХ, наличие маткиE. кариотип 46ХУ/46ХХ, отсутствие матки 159 Типичная форма дисгенезии гонад возникает при:A. кариотипе 46ХУB. кариотипе 46ХХC. кариотипе 45 ХОD. кариотипе 47 ХХУ

Page 34: postgraduate.kaznmu.kz€¦ · Web view58 Больная И, 28 лет, доставлена с жалобами на внезапно возникшую боль внизу живота

С.Ж.АСФЕНДИЯРОВАТЫНДАҒЫҚАЗАҚ ҰЛТТЫҚ МЕДИЦИНА

УНИВЕРСИТЕТІ

КАЗАХСКИЙНАЦИОНАЛЬНЫЙМЕДИЦИНСКИЙ УНИВЕРСИТЕТ ИМЕНИ С.Д.АСФЕНДИЯРОВА

КАФЕДРА ИНТЕРНАТУРЫ И РЕЗИДЕНТУРЫ ПО АКУШЕРСТВУ И ГИНЕКОЛОГИИТЕСТОВЫЕ ЗАДАНИЯ ДЛЯ ПРОВЕДЕНИЯ ИТОГОВОЙ ГОСУДАРСТВЕННОЙ АТТЕСТАЦИИ РЕЗИДЕНТОВ

ВЫПУСКА 2012-2013 ГГ. ПО СПЕЦИАЛЬНОСТИ «АКУШЕРСТВО И ГИНЕКОЛОГИЯ»

E. кариотипе 46ХХ/46ХУ160 До какого возраста у новорожденной девочки увеличение молочных желез и появление кровянистых выделений из половых путей считается гормональным кризом и не требует лечения:A. 10 дней B. 2 неделиC. 3 неделиD. 1 месяцE. 3 месяца161 О раннем половом созревании говорят при появлении вторичных половых признаков у девочек до:A. 15 летB. 13 летC. 10 летD. 8 летE. 6 лет 162 О задержке полового развития девочки говорят при:A. отсутствии роста молочных желез в 14 и менструаций в 16 летB. отсутствии роста молочных желез в 13 и менструаций в 15 летC. отсутствии роста молочных желез в 12 и менструаций в 14 летD. отсутствии роста молочных желез в 11 и менструаций в 13 летE. отсутствии роста молочных желез в 10 и менструаций в 12 лет163 Степень полового развития девочки выражается формулой:A. Ма, Ах, Р, МеB. Ма, Р, МеC. Ма, Ах, МеD. Ах, Р, МеE. Ма, Ах, Р164 Средний возраст менархе принято считать: A. 10-11 летB. 10-12 летC. 11-12 летD. 11-14 летE. 12-15 лет165 В возрасте 16 лет для пациенток с полной формой тестикулярной феминизации характерна формула полового развития:A. Ма3, Ах3,Р3, МеB. Ма2, Ах2,Р2, МеC. Ма2, Ах0,Р0, МеD. Ма2, Ах0,Р0, Ме-E. Ма0, Ах3,Р3, Ме-166 В 16 лет для пациенток с синдромом Рокитанского–Кюснера характерна формула полового развития:A. Ма3, Ах3,Р3, МеB. Ма3, Ах3,Р3, Ме-C. Ма2, Ах0,Р0, МеD. Ма2, Ах0,Р0, Ме-E. Ма0, Ах3,Р3, Ме-167 В пубертатном возрасте для девочек с синдромом Шерешевского – Тернера характерно:

Page 35: postgraduate.kaznmu.kz€¦ · Web view58 Больная И, 28 лет, доставлена с жалобами на внезапно возникшую боль внизу живота

С.Ж.АСФЕНДИЯРОВАТЫНДАҒЫҚАЗАҚ ҰЛТТЫҚ МЕДИЦИНА

УНИВЕРСИТЕТІ

КАЗАХСКИЙНАЦИОНАЛЬНЫЙМЕДИЦИНСКИЙ УНИВЕРСИТЕТ ИМЕНИ С.Д.АСФЕНДИЯРОВА

КАФЕДРА ИНТЕРНАТУРЫ И РЕЗИДЕНТУРЫ ПО АКУШЕРСТВУ И ГИНЕКОЛОГИИТЕСТОВЫЕ ЗАДАНИЯ ДЛЯ ПРОВЕДЕНИЯ ИТОГОВОЙ ГОСУДАРСТВЕННОЙ АТТЕСТАЦИИ РЕЗИДЕНТОВ

ВЫПУСКА 2012-2013 ГГ. ПО СПЕЦИАЛЬНОСТИ «АКУШЕРСТВО И ГИНЕКОЛОГИЯ»

A. отсутствие полового развития, низкий ростB. отсутствие полового развития, высокий ростC. раннее половое развитие, низкий ростD. раннее половое развитие, высокий ростE. отсутствие полового развития, нормальный рост168 Внутриутробная вирилизация наружных половых органов девочки наименее связана с:A. адреногенитальным синдромомB. смешанной формой дисгенезии гонад C. неполной формой тестикулярной феминизации D. приемом матерью препаратов с андрогенным эффектомE. приемом матерью инсулина при сахарном диабете169 Для ложной формы первичной аменореи характерно:A. повышение уровня андрогеновB. снижение уровня эстрогеновC. нормальные показатели гормоновD. снижение соотношения ЛГ/ФСГE. тоническое повышение ЛГ170 При синдроме Рокитанского-Кюснера следует ожидать: A. высокие уровни эстрогеновB. высокие уровни андрогеновC. высокие уровни пролактинаD. низкие уровни гонадотропных гормоновE. нормальные уровни гормонов171 При всех формах дисгенезии гонад характерно:A. высокие уровни эстрогеновB. высокие уровни андрогеновC. высокие уровни пролактинаD. низкие уровни гонадотропных гормоновE. высокие уровни гонадотропных гормонов172 Рост и развитие молочных желез в пубертатном периоде определяется в основном: A. эстрогенамиB. андрогенамиC. пролактиномD. соматотропным гормономE. гормонами щитовидной железы173 Для полной формы преждевременного полового развитие наименее всего характерно:A. ускорение линейного ростаB. опережение костного возрастаC. наличие изменений в кариотипе D. наличие фолликулов в яичниках и увеличение размеров маткиE. повышение уровней гонадотропных гормонов174 Положительная проба с гестагенами при аменореи позволяет исключить уровень поражения репродуктивной системы: A. корковый B. гипоталамическийC. гипофизарныйD. яичниковыйE. маточный

Page 36: postgraduate.kaznmu.kz€¦ · Web view58 Больная И, 28 лет, доставлена с жалобами на внезапно возникшую боль внизу живота

С.Ж.АСФЕНДИЯРОВАТЫНДАҒЫҚАЗАҚ ҰЛТТЫҚ МЕДИЦИНА

УНИВЕРСИТЕТІ

КАЗАХСКИЙНАЦИОНАЛЬНЫЙМЕДИЦИНСКИЙ УНИВЕРСИТЕТ ИМЕНИ С.Д.АСФЕНДИЯРОВА

КАФЕДРА ИНТЕРНАТУРЫ И РЕЗИДЕНТУРЫ ПО АКУШЕРСТВУ И ГИНЕКОЛОГИИТЕСТОВЫЕ ЗАДАНИЯ ДЛЯ ПРОВЕДЕНИЯ ИТОГОВОЙ ГОСУДАРСТВЕННОЙ АТТЕСТАЦИИ РЕЗИДЕНТОВ

ВЫПУСКА 2012-2013 ГГ. ПО СПЕЦИАЛЬНОСТИ «АКУШЕРСТВО И ГИНЕКОЛОГИЯ»

175 Современным и предпочтительным методом терапии полной формы ППС является применение: A. низкодозированных комбинированных оральных контрацептивовB. агонистов ГнРгC. андрокураD. фитотерапииE. физиотерапии176 Термин «сочная вульва» у девочек до пубертатного возраста указывает на: A. избыточное влияние андрогеновB. избыточное влияние гормона ростаC. воспалительные измененияD. избыточное влияние эстрогеновE. аномалию развития наружных половых органов177 При простой вирильной форме врожденной дисфункции коры надпочечников у пациентов имеет место ферментный дефект в цепи выработки гормонов надпочечников так, что нарушается выработка: A. эстрогеновB. кортизолаC. альдостеронаD. тестостеронаE. прогестерона178 Гнойно-кровянистые выделения из половых путей у девочек до пубертатного возраста наиболее вероятно связаны с: A. саркомой влагалища, травмой B. инородным телом во влагалищеC. саркомой влагалищаD. вульвовагинитом, травмойE. вульвовагинитом, инородным телом 179 При установленном диагнозе «полная форма ППС» у девочек терапию агонистами ГнРг (диферелин) рекомендуется проводить до возраста: A. 5 летB. 6 летC. 7 летD. 8 летE. 9 лет180 В отсутствии адекватной терапии полной формы ППС у девочек в последующем следует ожидать: A. высокий ростB. низкий ростC. ранний климаксD. бесплодиеE. риск рака матки и молочных желез181 Причиной альгоменореи у девочек наименее вероятно может быть:A. хронический сальпингитB. гиперпростагландинемияC. генитальный эндометриозD. психо-эмоциональные причиныE. потеря массы тела

Page 37: postgraduate.kaznmu.kz€¦ · Web view58 Больная И, 28 лет, доставлена с жалобами на внезапно возникшую боль внизу живота

С.Ж.АСФЕНДИЯРОВАТЫНДАҒЫҚАЗАҚ ҰЛТТЫҚ МЕДИЦИНА

УНИВЕРСИТЕТІ

КАЗАХСКИЙНАЦИОНАЛЬНЫЙМЕДИЦИНСКИЙ УНИВЕРСИТЕТ ИМЕНИ С.Д.АСФЕНДИЯРОВА

КАФЕДРА ИНТЕРНАТУРЫ И РЕЗИДЕНТУРЫ ПО АКУШЕРСТВУ И ГИНЕКОЛОГИИТЕСТОВЫЕ ЗАДАНИЯ ДЛЯ ПРОВЕДЕНИЯ ИТОГОВОЙ ГОСУДАРСТВЕННОЙ АТТЕСТАЦИИ РЕЗИДЕНТОВ

ВЫПУСКА 2012-2013 ГГ. ПО СПЕЦИАЛЬНОСТИ «АКУШЕРСТВО И ГИНЕКОЛОГИЯ»

182 Для дифференциальной диагностики причин альгоменореи у подростков применяют пробу с: A. гестагенамиB. КОКамиC. НПВП ( диклофенак)D. дексаметазономE. эстрогенами183 Полное отсутствие эффекта от применения НПВП (индометацин, диклофенак) при альгоменорее позволяет предположить: A. генитальный эндометриозB. воспалительный процесс гениталийC. гиперпростагландинемиюD. психоэмоциональный вариантE. симулирование симптомов альгоменореи184 В подростковом возрасте препаратами первого выбора для лечения генитального эндометриоза являются: A. агонисты ГнРгB. даназолC. КОКиD. гестагеныE. мини-пили185 При проявлении ювенильных кровотечений с периода менархе необходимо в первую очередь исключить: A. заболевания кровиB. нарушения диетыC. воспалительные заболевания гениталийD. синдром поликистозных яичниковE. тяжелый хронический стресс186 В клинической картине ДМК по типу атрезии фолликула имеют место:A. кратковременные до 2-3 недель задержки менструацииB. длительные до 3-6 месяцев задержки менструации C. регулярные обильные и длительные менструацииD. регулярные скудные менструацииE. регулярные болезненные менструации 187 Гиперплазия эндометрия при атрезии фолликулов развивается вследствие: A. повышенных уровней эндогенных эстрогеновB. конкурирующего влияния андрогеновC. длительного воздействия малых уровней эстрогеновD. присоединения воспалительных изменений эндометрияE. при атрезии фолликулов не характерна гиперплазия эндометрия188 Для подростков наиболее удобным и эффективным методом контрацепции является: A. воздержаниеB. симпто-термальный методC. КОКиD. ВМСE. инъекционные препараты189 В комплексной терапии синехий малых половых губ местно применяют:A. андрогеныB. эстрогены

Page 38: postgraduate.kaznmu.kz€¦ · Web view58 Больная И, 28 лет, доставлена с жалобами на внезапно возникшую боль внизу живота

С.Ж.АСФЕНДИЯРОВАТЫНДАҒЫҚАЗАҚ ҰЛТТЫҚ МЕДИЦИНА

УНИВЕРСИТЕТІ

КАЗАХСКИЙНАЦИОНАЛЬНЫЙМЕДИЦИНСКИЙ УНИВЕРСИТЕТ ИМЕНИ С.Д.АСФЕНДИЯРОВА

КАФЕДРА ИНТЕРНАТУРЫ И РЕЗИДЕНТУРЫ ПО АКУШЕРСТВУ И ГИНЕКОЛОГИИТЕСТОВЫЕ ЗАДАНИЯ ДЛЯ ПРОВЕДЕНИЯ ИТОГОВОЙ ГОСУДАРСТВЕННОЙ АТТЕСТАЦИИ РЕЗИДЕНТОВ

ВЫПУСКА 2012-2013 ГГ. ПО СПЕЦИАЛЬНОСТИ «АКУШЕРСТВО И ГИНЕКОЛОГИЯ»

C. кортикостероидыD. антибиотикиE. гестагены190 Уровни какого гормона позволяют дифференцировать яичниковую и центральную формы первичной аменореи:A. пролактинB. ТТГC. ТестостеронD. ЛГE. ФСГ191 Какой показатель может служить объективным контролем для начала терапии эстрогенами при синдроме Шерешевского – Тернера: A. линейный рост B. паспортный возрастC. уровень эстрогенов в кровиD. костный возрастE. уровень ФСГ 192 Гиперпролатинемия в подростковом возрасте характеризуется: A. низким ростомB. ДМКC. гипертрофией молочных железD. высокорослостьюE. аменореей193 Проба с дексаметазоном позволяет дифференцировать: A. только источник гиперандрогенииB. только наличие гормонопродуцирующей опухоли C. только необходимую лечебную дозу дексаметазонаD. источник гиперандрогении и наличие гормонопродуцирующей опухоли E. только необходимую поддерживающую дозу дексаметазона194 Паспортный пол ребенка устанавливается: A. врачом генетиком B. детским гинекологомC. неонатологомD. акушером-гинекологом/акушеркойE. педиатром 195 К внешним факторам, влияющим на течение пубертатного периода наименее вероятно относятся: A. освещенностьB. высота над уровнем моряC. климатD. полноценность питанияE. успеваемость в школе196 В какие сроки гестации формируются пороки развития матки:A. 4-6 недельB. 6-8 недельC. 8-10 недельD. 10-11 недель E. 11-12 недель

Page 39: postgraduate.kaznmu.kz€¦ · Web view58 Больная И, 28 лет, доставлена с жалобами на внезапно возникшую боль внизу живота

С.Ж.АСФЕНДИЯРОВАТЫНДАҒЫҚАЗАҚ ҰЛТТЫҚ МЕДИЦИНА

УНИВЕРСИТЕТІ

КАЗАХСКИЙНАЦИОНАЛЬНЫЙМЕДИЦИНСКИЙ УНИВЕРСИТЕТ ИМЕНИ С.Д.АСФЕНДИЯРОВА

КАФЕДРА ИНТЕРНАТУРЫ И РЕЗИДЕНТУРЫ ПО АКУШЕРСТВУ И ГИНЕКОЛОГИИТЕСТОВЫЕ ЗАДАНИЯ ДЛЯ ПРОВЕДЕНИЯ ИТОГОВОЙ ГОСУДАРСТВЕННОЙ АТТЕСТАЦИИ РЕЗИДЕНТОВ

ВЫПУСКА 2012-2013 ГГ. ПО СПЕЦИАЛЬНОСТИ «АКУШЕРСТВО И ГИНЕКОЛОГИЯ»

197 Пороки развития матки и влагалища чаще всего сочетаются с пороками развития: A. мочевыделительной системыB. сердечно-сосудистойC. нервной D. скелетнойE. пищеварительного тракта 198 Наиболее частой причиной полной формы ППС является:A. черепно-мозговая травма B. нейроинфекцииC. осложненное течение беременностиD. асфиксия в родахE. внутренняя гидроцефалия III желудочка199 Какой признак является манифестирующим у девочек с ложной формой ППС:A. рост молочных железB. пубархеC. появление менструацийD. ускорение ростаE. опережение костного возраста200 Какие УЗ признаки наименее вероятно указывают на незавершенное развитие репродуктивных органов A. размеры матки B. отсутствие угла между телом и шейкой маткиC. соотношение размеров шейки и тела маткиD. синистропозиция маткиE. высокое у стенок таза расположение яичников201 Какое дополнительное показание к выскабливанию полости матки и проведению гистероскопии у подростков рекомендуется в настоящее время: A. обильное маточное кровотечение B. подозрение на беременностьC. неэффективность гормонального гемостазаD. рецидивирующие мажущие кровянистые выделенияE. гиперплазия эндометрия по УЗИ 202 При гипогонадотропном гонадизме уровень поражения можно уточнить с помощью пробы с: A. гестагенамиB. КОКамиC. гонадотропинамиD. гонадотропин-релизин гормономE. чистыми эстрогенами203 Первичный гипотиреоз часто сопровождается повышением уровней: A. ФСГB. пролактинаC. прогестеронаD. кортизолаE. альдостерона204 Пациенты с АГС принимают дексаметазон( или его аналоги):A. до формирования окончательного ростаB. до менархеC. до наступления беременности

Page 40: postgraduate.kaznmu.kz€¦ · Web view58 Больная И, 28 лет, доставлена с жалобами на внезапно возникшую боль внизу живота

С.Ж.АСФЕНДИЯРОВАТЫНДАҒЫҚАЗАҚ ҰЛТТЫҚ МЕДИЦИНА

УНИВЕРСИТЕТІ

КАЗАХСКИЙНАЦИОНАЛЬНЫЙМЕДИЦИНСКИЙ УНИВЕРСИТЕТ ИМЕНИ С.Д.АСФЕНДИЯРОВА

КАФЕДРА ИНТЕРНАТУРЫ И РЕЗИДЕНТУРЫ ПО АКУШЕРСТВУ И ГИНЕКОЛОГИИТЕСТОВЫЕ ЗАДАНИЯ ДЛЯ ПРОВЕДЕНИЯ ИТОГОВОЙ ГОСУДАРСТВЕННОЙ АТТЕСТАЦИИ РЕЗИДЕНТОВ

ВЫПУСКА 2012-2013 ГГ. ПО СПЕЦИАЛЬНОСТИ «АКУШЕРСТВО И ГИНЕКОЛОГИЯ»

D. до менопаузыE. пожизненно205 При инсулин зависимом сахарном диабете темпы полового созревания, как правило снижены. Для их коррекции применяют: A. ЗГТB. КОКиC. гестагеныD. коррекцию уровня глюкозы в крови и дозы инсулинаE. сахароснижающие препараты206 Базовая терапия СПКЯ в пубертатном возрасте начинается с: A. снижения массы тела (диета, физическая нагрузка)B. применения Диане 35C. применения ЯриныD. применения ЖанинаE. применения гестагенов207 Гипоталамический синдром периода полового созревания наименее вероятно характеризуется: A. ожирениемB. повышением АДC. появлением стрий на кожеD. нарушением менструального цикла по типу олиго-аменореиE. признаками гипертиреоза208 Преимуществом сигмоидального кольпопоэза в подростковом возрасте является:A. относительная простота выполненияB. отсутствие стенозирования даже при отсутствии половой жизни C. благоприятное соотношение размеров созданного влагалищаD. быстрое достижение нормальной микрофлорыE. преимущества в настоящее время не доказаны 209 Выполнение брюшинного кольпопоэза ограничено в подростковом возрасте из-за: A. анатомических соотношений тканей промежностиB. малой растяжимостью брюшиныC. необходимостью регулярной половой жизни ( или использование протектора) D. частым развитием воспалительных изменений в неовлагалищеE. развитием стеноза входа в неовлагалище. 210 Во всех случаях первичной аменореи наиболее целесообразно провести в первую очередь: A. ультразвуковое исследование органов малого тазаB. кариотипC. ФСГ, пролактин, ТТГD. гинекологическое исследованиеE. КТ головного мозга211 Элементы «детского онанизма» являются следствием: A. воспалительного процесса наружных гениталийB. глистной инвазииC. пороков развития мочевыделительной системыD. сахарного диабетаE. наличия очага возбуждения в центральной нервной системе212 ППС при наличии кисты яичника предполагает следующую врачебную тактику: A. применение КОКов в контрацептивном режимеB. применение депо-диферелина в течение 2-3 месяцев

Page 41: postgraduate.kaznmu.kz€¦ · Web view58 Больная И, 28 лет, доставлена с жалобами на внезапно возникшую боль внизу живота

С.Ж.АСФЕНДИЯРОВАТЫНДАҒЫҚАЗАҚ ҰЛТТЫҚ МЕДИЦИНА

УНИВЕРСИТЕТІ

КАЗАХСКИЙНАЦИОНАЛЬНЫЙМЕДИЦИНСКИЙ УНИВЕРСИТЕТ ИМЕНИ С.Д.АСФЕНДИЯРОВА

КАФЕДРА ИНТЕРНАТУРЫ И РЕЗИДЕНТУРЫ ПО АКУШЕРСТВУ И ГИНЕКОЛОГИИТЕСТОВЫЕ ЗАДАНИЯ ДЛЯ ПРОВЕДЕНИЯ ИТОГОВОЙ ГОСУДАРСТВЕННОЙ АТТЕСТАЦИИ РЕЗИДЕНТОВ

ВЫПУСКА 2012-2013 ГГ. ПО СПЕЦИАЛЬНОСТИ «АКУШЕРСТВО И ГИНЕКОЛОГИЯ»

C. применения гестагенов в течение 8-10 днейD. наблюдение в течение 2-3 месяцевE. немедленное оперативное вмешательство213 Наиболее информативным и доступным методом диагностики порока развития – полное удвоение матки и влагалища в подростковом возрасте является: A. лапароскопияB. гистероскопияC. УЗИD. ректоабдоминальное исследованиеE. КТ214 Тактика при изолированном телархе:A. наблюдениеB. применение агонистов ГнРгC. применение парлоделаD. применение гестагеновE. применение высоких доз аскорбиновой кислоты215 Почему ППС при синдроме Мак-Кьюна-Олбрайта не лечат агонистами ГнРг:A. повышен только ТТГB. повышен только ФСГC. активация яичников без активации гонадотропиновD. внегонадная секреция эстрогеновE. эктопическая выработка ФСГ и ЛГ216 У девочек с высокорослостью для достижения окончательного роста применяют: A. тестостеронB. андрокурC. КОКиD. эстрогеныE. поливитамины217 При пробе с гонадолиберинами показателем функциональной зрелости гипоталамо-гипофизарно-гонадной системы является:A. резкое повышение уровня ФСГB. повышение уровня пролактинаC. резкое повышение уровня ЛГD. парадоксальное снижение уровня ФСГE. отсутствие значимых изменений уровней гонадотропных гормонов218 Девочки с первичной альгоменорееей обычно характеризуются:A. астеническим телосложениемB. повышенным весомC. увеличенным плевым поясомD. подчеркнутой феминизацией фигурыE. удлиненным туловищем 219 При изолированном телархе у девочек до пубертатного возраста лучшим методом терапии является: A. КОКиB. гестагеныC. агонисты ГнРгD. антиандрогеныE. наблюдение

Page 42: postgraduate.kaznmu.kz€¦ · Web view58 Больная И, 28 лет, доставлена с жалобами на внезапно возникшую боль внизу живота

С.Ж.АСФЕНДИЯРОВАТЫНДАҒЫҚАЗАҚ ҰЛТТЫҚ МЕДИЦИНА

УНИВЕРСИТЕТІ

КАЗАХСКИЙНАЦИОНАЛЬНЫЙМЕДИЦИНСКИЙ УНИВЕРСИТЕТ ИМЕНИ С.Д.АСФЕНДИЯРОВА

КАФЕДРА ИНТЕРНАТУРЫ И РЕЗИДЕНТУРЫ ПО АКУШЕРСТВУ И ГИНЕКОЛОГИИТЕСТОВЫЕ ЗАДАНИЯ ДЛЯ ПРОВЕДЕНИЯ ИТОГОВОЙ ГОСУДАРСТВЕННОЙ АТТЕСТАЦИИ РЕЗИДЕНТОВ

ВЫПУСКА 2012-2013 ГГ. ПО СПЕЦИАЛЬНОСТИ «АКУШЕРСТВО И ГИНЕКОЛОГИЯ»

220 Контролем адекватности ЗГТ при различных формах дисгенезии гонад служит: A. развитие вторичных половых признаковB. уровень ФСГC. уровень эстрогеновD. ректальная температураE. интеллектуальный уровень221 У девочек подросткового возраста при проведении реабилитационной терапии рационально использование гестагенов с: A. 5 по 25 день циклаB. 12 по 25 день циклаС. 14 по 25 день циклаС. 16 по 25 день цикла C. 20 по 25 день цикла222 Для поддержания менструального цикла девочкам с гипогонадотропным гипогонадизмом рационально рекомендовать:A. ЦВТB. КОКиC. Мини-пилиD. препараты ЗГТ в циклическом режимеE. препараты ЗГТ в непрерывном режиме223 Применение КОКов с лечебной и контрацептивной целью следует ограничить у девочек с: A. пороками развития мочевыделительной системыB. повышенным ИМТC. наличием судорожной готовности по данным ЭЭГD. малой массой телаE. хроническим тонзиллитом224 Комплекс циклической витаминотерапии (ЦВТ) включает витамины: A. Д САB. АВСC. СЕфолиевая кислотаD. ЕСглютаминовая кислотаE. ЕСглютаминовая кислотафолиевая кислота225 С учетом современных вспомогательных репродуктивных технологий об абсолютном бесплодии говорят в отношении пациенток с:A. синдромом Рокитанского-КюснераB. синдромом тестикулярной феминизацииC. синдрома СвайераD. синдромом Шерешевского–ТернераE. гипогонадотропным гипогонадизмом226 Отличительным признаком атрезии гимена от аплазии нижней трети влагалища является: A. ложная аменореяB. образование гематокольпосаC. выбухание и «синюшность» гименаD. возраст пациенткиE. деформация ампулы прямой кишки227 Наблюдаемая в подростковом возрасте ассимметрия развития молочных желез является признаком: A. вариантом нормы

Page 43: postgraduate.kaznmu.kz€¦ · Web view58 Больная И, 28 лет, доставлена с жалобами на внезапно возникшую боль внизу живота

С.Ж.АСФЕНДИЯРОВАТЫНДАҒЫҚАЗАҚ ҰЛТТЫҚ МЕДИЦИНА

УНИВЕРСИТЕТІ

КАЗАХСКИЙНАЦИОНАЛЬНЫЙМЕДИЦИНСКИЙ УНИВЕРСИТЕТ ИМЕНИ С.Д.АСФЕНДИЯРОВА

КАФЕДРА ИНТЕРНАТУРЫ И РЕЗИДЕНТУРЫ ПО АКУШЕРСТВУ И ГИНЕКОЛОГИИТЕСТОВЫЕ ЗАДАНИЯ ДЛЯ ПРОВЕДЕНИЯ ИТОГОВОЙ ГОСУДАРСТВЕННОЙ АТТЕСТАЦИИ РЕЗИДЕНТОВ

ВЫПУСКА 2012-2013 ГГ. ПО СПЕЦИАЛЬНОСТИ «АКУШЕРСТВО И ГИНЕКОЛОГИЯ»

B. мастопатииC. кисты молочной железыD. нарушения нормальных темпов полового развития E. гиперэстрогении228 Какой критерий позволяет прогнозировать конечный линейный рост подростка:A. уровень тестостерона в плазме кровиB. рост родителейC. степень полового развития к 15 годамD. костный возрастE. возраст Ме229 Аменорея на фоне потери массы тела в некоторых случаях может быть предшественником: A. булемииB. нервной анорексииC. СПКЯD. ВДКНE. дисгенезии гонад230 Признаками, указывающими на гиперандрогению, наименее всего является: A. жирная себореяB. акнеC. пористая кожаD. гирсутизмE. высокий рост231 Возраст менопаузы определяют после последней менструации через:A. 1 месяц B. 6 месяцевC. 9 месяцевD. 12 месяцевE. 24 месяца232 Самыми ранними симптомами снижения функции яичников являются: A. изменения продолжительности менструального циклаB. появление вазомоторных симптомовC. появление мочеполовых проблемD. отсутствие беременностиE. повышение массы тела233 О климактерическом снижении функции яичников можно судить по динамическому повышению уровня гормона:A. ТТГB. ФСГC. тестостеронаD. пролактинаE. прогестерона234 окситоцин - это гормон: A. яичниковB. гипоталамусаC. коры надпочечниковD. передней доли гипофизаE. задней доли гипофиза

Page 44: postgraduate.kaznmu.kz€¦ · Web view58 Больная И, 28 лет, доставлена с жалобами на внезапно возникшую боль внизу живота

С.Ж.АСФЕНДИЯРОВАТЫНДАҒЫҚАЗАҚ ҰЛТТЫҚ МЕДИЦИНА

УНИВЕРСИТЕТІ

КАЗАХСКИЙНАЦИОНАЛЬНЫЙМЕДИЦИНСКИЙ УНИВЕРСИТЕТ ИМЕНИ С.Д.АСФЕНДИЯРОВА

КАФЕДРА ИНТЕРНАТУРЫ И РЕЗИДЕНТУРЫ ПО АКУШЕРСТВУ И ГИНЕКОЛОГИИТЕСТОВЫЕ ЗАДАНИЯ ДЛЯ ПРОВЕДЕНИЯ ИТОГОВОЙ ГОСУДАРСТВЕННОЙ АТТЕСТАЦИИ РЕЗИДЕНТОВ

ВЫПУСКА 2012-2013 ГГ. ПО СПЕЦИАЛЬНОСТИ «АКУШЕРСТВО И ГИНЕКОЛОГИЯ»

235 Климактерические симптомы начинают проявляться при снижении количества фолликулов в яичниках менее:A. 100 000B. 70 000 C. 50 000D. 30 000E. 10 000236 менопауза наступает при числе фолликулов в яичниках не более:A. 50 000B. 30 000C. 1000D. 100E. 0237 Хирургическая менопауза наименее вероятно наступает после: A. экстирпации матки с придаткамиB. экстирпация матки без придатковC. ампутации матки с придаткамиD. ампутации матки без придатковE. консервативной миомэктомии238 При удаленной матке и придатках по поводу острого гнойного воспалительного процесса в качестве ЗГТ показаны: A. комбинированные препараты в непрерывном режимеB. комбинированные препараты в циклическом режимеC. Ччистые натуральные эстрогеныD. КОКи с высоким содержанием эстрогеновE. синтетические эстрогены239 Инсулинорезистентность – это: A. гибель островков Ланганса в поджелудочной железеB. абсолютный дефицит инсулинаC. невосприимчивость собственного инсулинаD. аллергическая реакция к экзогенному инсулинуE. восприимчивость собственного инсулина240 Индекс массы тела вычисляется путем деления: A. массы (кг)/рост (см)B. массы (кг) /рост ( кв.см)C. массы (г)/ рост (м)D. массы (г) /рост (кв.м) E. массы (кг)/рост (кв.м)241 Об абдоминальном ожирении у женщин говорят при соотношении ОТ/ОБ большем, чем:A. 0,6B. 0,7C. 0,8D. 0,85E. 0,9242 В постменопаузе выработка слабых эстрогенов происходит в:A. печениB. поджелудочной железеC. эндотелии сосудов

Page 45: postgraduate.kaznmu.kz€¦ · Web view58 Больная И, 28 лет, доставлена с жалобами на внезапно возникшую боль внизу живота

С.Ж.АСФЕНДИЯРОВАТЫНДАҒЫҚАЗАҚ ҰЛТТЫҚ МЕДИЦИНА

УНИВЕРСИТЕТІ

КАЗАХСКИЙНАЦИОНАЛЬНЫЙМЕДИЦИНСКИЙ УНИВЕРСИТЕТ ИМЕНИ С.Д.АСФЕНДИЯРОВА

КАФЕДРА ИНТЕРНАТУРЫ И РЕЗИДЕНТУРЫ ПО АКУШЕРСТВУ И ГИНЕКОЛОГИИТЕСТОВЫЕ ЗАДАНИЯ ДЛЯ ПРОВЕДЕНИЯ ИТОГОВОЙ ГОСУДАРСТВЕННОЙ АТТЕСТАЦИИ РЕЗИДЕНТОВ

ВЫПУСКА 2012-2013 ГГ. ПО СПЕЦИАЛЬНОСТИ «АКУШЕРСТВО И ГИНЕКОЛОГИЯ»

D. жировой тканиE. яичниках243 Ожирение в постменопаузе является фактором риска:A. остеопорозаB. ИБСC. АГD. деменцииE. ИБС и АГ244 Какая клиническая форма воспалительного процесса чаще всего встречается у девочек?A. кольпитB. вульвитC. эндометритD. сальпингоофоритE. пельвиоперитонит245 Наиболее частые пути распространения туберкулезной инфекции у девочек:A. воздушно-капельныйB. восходящий путьC. лимфогенноD. гематогенноE. нисходящий путь246 К анатомическим особенностям матки у новорожденной девочки относятся: A. тело и шейка матки по длине и толщине равны между собой B. тело матки маленькое, шейка почти не выражена C. матка небольшая, длина шейки почти в 3 раза больше длины тела матки D. матка имеет двурогую форму E. ширина больше длины247 Характер выделения гонадотропных гормонов, присущий нормальному менструальному циклу, устанавливается в:A. 16-17 лет B. 15-14 лет C. 13-12 лет D. 11-10 лет E. 9 лет 248 Пигментация сосков и увеличение молочных желез происходит обычно в:A. 8-9 лет B. 10-11 лет C. 12-13 лет D. 14-15 лет E. 16-18 лет 249 Наименее вероятная анатомо-физиологическая особенность гениталий у девочек: A. избыточная складчатость слизистых B. низкая эстрогенная насыщенность C. недостаточная выработка гликогена D. истонщенность и замедленная пролиферация E. высокая эстрогенная насыщенность250 Для внешнего вида при задержке полового развития наименее вероятно характерно: A. евнухоидное телосложение B. соответствие костного возраста календарному

Page 46: postgraduate.kaznmu.kz€¦ · Web view58 Больная И, 28 лет, доставлена с жалобами на внезапно возникшую боль внизу живота

С.Ж.АСФЕНДИЯРОВАТЫНДАҒЫҚАЗАҚ ҰЛТТЫҚ МЕДИЦИНА

УНИВЕРСИТЕТІ

КАЗАХСКИЙНАЦИОНАЛЬНЫЙМЕДИЦИНСКИЙ УНИВЕРСИТЕТ ИМЕНИ С.Д.АСФЕНДИЯРОВА

КАФЕДРА ИНТЕРНАТУРЫ И РЕЗИДЕНТУРЫ ПО АКУШЕРСТВУ И ГИНЕКОЛОГИИТЕСТОВЫЕ ЗАДАНИЯ ДЛЯ ПРОВЕДЕНИЯ ИТОГОВОЙ ГОСУДАРСТВЕННОЙ АТТЕСТАЦИИ РЕЗИДЕНТОВ

ВЫПУСКА 2012-2013 ГГ. ПО СПЕЦИАЛЬНОСТИ «АКУШЕРСТВО И ГИНЕКОЛОГИЯ»

C. недоразвитие вторичных половых признаков D. отсутствие развития молочных желез E. несоответствие костного возраста календарному251. На приеме в женской консультации пациентка жалуется на жжение, зуд, покраснение в области наружных половых органов, общее недомогание. При осмотре гиперемия и отек наружных половых органов. Какой предварительный диагноз НАИБОЛЕЕ вероятен:A) вульвитB) вагинитC) кольпитD) эндометритE) вульвовагинит252.На 3 день менструального цикла пациентка обратилась в женскую консультацию с жалобами на недомогание, повышение температуры, боли внизу живота, иррадиирующие в крестцовые и паховые области, гнойно-кровянистые выделения из матки. В крови лейкоцитоз, сдвиг лейкоцитарной формулы, ускорение скорости оседания эритроцитов. PV: тело матки слегка увеличено, болезненно. Какой предварительный диагноз НАИБОЛЕЕ вероятен:A) сальпингоофаритB) кольпитC) вульвитD) эндометритE) пельвиоперитонит253 НАИБОЛЕЕ целесообразная тактика ведения пациенток с кистой бартолиниевой железы:A) УФО в стадию ремиссииB) противовоспалительная терапияC) вылущивание кисты в стадию ремиссииD) УВЧ в стадию ремиссииE) вылущивание кисты в стадию воспаления254. НАИБОЛЕЕ целесообразная тактика ведения пациенток с бартолинитом в фазе инфильтрации:A) вскрытие бартолинитаB) повязка с мазью ВишневскогоC) обработка спиртовым растворомD) физиолечениеE) сухая повязка255. Пациентке в женской консультации после получения результата мазка на микрофлору выставлен предварительный диагноз – Гарднереллез. Назначение какого лекарственного препарата из перечисленных является НАИБОЛЕЕ целесообразным?A) доксициклинB) метронидазолC) ампициллинD) азитромицинE) тинидазол256. При влагалищном исследовании у пациентки выявлены следующие признаки: матка слегка увеличена, мягкой консистенции. Наружный зев закрыт. Справа в области придатков определяется мягковатое, тестоватой консистенции, болезненное образование, болезненность при движении за шейку матки. В анамнезе задержка месячных на 2 месяца. Тест на беременность (). Какой предварительный диагноз НАИБОЛЕЕ вероятен?A) прогрессирующая трубная беременность

Page 47: postgraduate.kaznmu.kz€¦ · Web view58 Больная И, 28 лет, доставлена с жалобами на внезапно возникшую боль внизу живота

С.Ж.АСФЕНДИЯРОВАТЫНДАҒЫҚАЗАҚ ҰЛТТЫҚ МЕДИЦИНА

УНИВЕРСИТЕТІ

КАЗАХСКИЙНАЦИОНАЛЬНЫЙМЕДИЦИНСКИЙ УНИВЕРСИТЕТ ИМЕНИ С.Д.АСФЕНДИЯРОВА

КАФЕДРА ИНТЕРНАТУРЫ И РЕЗИДЕНТУРЫ ПО АКУШЕРСТВУ И ГИНЕКОЛОГИИТЕСТОВЫЕ ЗАДАНИЯ ДЛЯ ПРОВЕДЕНИЯ ИТОГОВОЙ ГОСУДАРСТВЕННОЙ АТТЕСТАЦИИ РЕЗИДЕНТОВ

ВЫПУСКА 2012-2013 ГГ. ПО СПЕЦИАЛЬНОСТИ «АКУШЕРСТВО И ГИНЕКОЛОГИЯ»

B) апоплексия правого яичникаC) обострение хронического воспалительного процесса правых придатковD) киста правого яичникаE) прерванная внематочная беременность по типу трубного аборта257. Какой из перечисленных методов обследования является НАИБОЛЕЕ важным при прогрессирующей внематочной беременности:A) иммунологическая реакцияB) УЗИ органов малого тазаC) пункция заднего сводаD) лапароскопия E) диагностическое выскабливание полости матки 258. Какой из перечисленных преимуществ лапароскопии при внематочной беременности является НАИБОЛЕЕ важным:A) косметический эффектB) минимальное пребывание в стационареC) минимальная травматизация передней брюшной стенкиD) минимальное время операцииE) возможность сохранения маточной трубы259.Пациентка 28 лет, предъявляет жалобы на зуд, жжение, творожистые выделения из половых путей без запаха. Какой предварительный диагноз наиболее вероятен:A) трихомониазB) гонореяC) хламидиозD) простой серозный кольпитE) кандидоз влагалища260. При поступлении в стационар пациентка 32 лет предъявляет жалобы на высокую температуру, озноб, боли по всему животу. В анамнезе бесплодие, двухсторонний сальпингоофарит с частыми обострениями. Объективно: температура 39,2ºС, пульс 128 ударов/минуту, АД 110/70 мм рт. ст., ЧД 22 в минуту. Язык сухой, живот вздут, напряженный во всех отделах, болезненный, выраженный симптом Щеткина-Блюмберга по всему животу. OZ: слизистая влагалища не изменена, шейка матки эрозирована. Выделения из половых путей гнойные, умеренные. PV: При влагалищном исследовании смещения шейки матки резко болезненные, придатки и матку определить не удается из-за болезненности. Задний свод влагалища уплощен, болезненный. Наиболее вероятный диагноз?A) эндометриоз придатковB) хронический двусторонний сальпингоофаритC) пельвиоперитонитD) разлитой перитонитE) абсцесс Дугласова пространства261. При поступлении в стационар пациентка 32 лет предъявляет жалобы на высокую температуру, озноб, боли по всему животу. В анамнезе бесплодие, двухсторонний сальпингоофарит с частыми обострениями. Объективно: температура 39,2ºС, пульс 128 ударов/минуту, АД 110/70 мм рт. ст., ЧД 22 в минуту. Язык сухой, живот вздут, напряженный во всех отделах, болезненный, выраженный симптом Щеткина-Блюмберга по всему животу. OZ: слизистая влагалища не изменена, шейка матки эрозирована. Выделения из половых путей гнойные, умеренные. PV: При влагалищном исследовании смещения шейки матки резко болезненные, придатки и матку определить не удается из-за болезненности. Задний свод

Page 48: postgraduate.kaznmu.kz€¦ · Web view58 Больная И, 28 лет, доставлена с жалобами на внезапно возникшую боль внизу живота

С.Ж.АСФЕНДИЯРОВАТЫНДАҒЫҚАЗАҚ ҰЛТТЫҚ МЕДИЦИНА

УНИВЕРСИТЕТІ

КАЗАХСКИЙНАЦИОНАЛЬНЫЙМЕДИЦИНСКИЙ УНИВЕРСИТЕТ ИМЕНИ С.Д.АСФЕНДИЯРОВА

КАФЕДРА ИНТЕРНАТУРЫ И РЕЗИДЕНТУРЫ ПО АКУШЕРСТВУ И ГИНЕКОЛОГИИТЕСТОВЫЕ ЗАДАНИЯ ДЛЯ ПРОВЕДЕНИЯ ИТОГОВОЙ ГОСУДАРСТВЕННОЙ АТТЕСТАЦИИ РЕЗИДЕНТОВ

ВЫПУСКА 2012-2013 ГГ. ПО СПЕЦИАЛЬНОСТИ «АКУШЕРСТВО И ГИНЕКОЛОГИЯ»

влагалища уплощен, болезненный. Какой план лечебных мероприятий является НАИБОЛЕЕ целесообразным:A) консервативная терапияB) пункция через задний сводC) оперативное лечение D) антибактериальная терапияE) гормональная терапия262. У пациентки отмечается гиперемия, отек уретры, обильные гнойные выделения, рези при мочеиспускании, гиперемия влагалищной части шейки матки, из цервикального канала обильные гнойные выделения. Какой диагноз НАИБОЛЕЕ вероятен?A) гонореяB) хламидиозC) гарднереллезD) трихомонозE) кандидоз263. Пациентка Г., 25 лет, впервые обратилась с жалобами на боли в области правой большой половой губы, повышение температуры тела до 39 С, боли при ходьбе. Объективно: большая половая губа справа отечна, болезненная, гиперемирована, при пальпации имеется участок размягчения. Какой диагноз НАИБОЛЕЕ вероятен? A) киста бартолиниевой железыB) острый бартолинит в стадии инфильтрацииC) абсцесс бартолиниевой железыD) вульвовагинитE) нагноение кисты бартолиниевой железы264. Пациентка Б., обратилась к врачу женской консультации с жалобами на обильные выделения из половых органов серого цвета, часто с неприятным «рыбным» запахом, особенно после полового акта или во время менструации, зуд и жжение в области наружных половых органов, дизурические расстройства. OZ: слизистая шейки матки и влагалища слегка гиперемированы, выделения серого цвета с неприятным запахом. PV: матка в правильном положении, подвижная, безболезненная. Придатки с обеих сторон не определяются. Своды свободные, безболезненные. Какой диагноз НАИБОЛЕЕ вероятен?A) вульвовагинитB) вульвитC) эндоцервицитD) бактериальный вагинозE) кольпит265.В гинекологическое отделение поступила пациентка с жалобами на боли внизу живота, гнойно-кровянистые выделения из половых путей, подъем температуры до 38 °С. Заболела на 8 день менструального цикла, через 3 дня после введения ВМС. В зеркалах: шейка чистая матки. В цервикальном канале проводные нити ВМС, гнойно-кровянистые выделения. Матка несколько увеличена, резко болезненна, ограничена в подвижности. Придатки не определяются, своды глубокие, болезненные. Какой диагноз НАИБОЛЕЕ вероятен?A) эндометрит на фоне ВМС B) перфорация матки при введении ВМС C) параметрит на фоне ВМСD) сальпингоофорит на фоне ВМСE) пельвиоперитонит на фоне ВМС

Page 49: postgraduate.kaznmu.kz€¦ · Web view58 Больная И, 28 лет, доставлена с жалобами на внезапно возникшую боль внизу живота

С.Ж.АСФЕНДИЯРОВАТЫНДАҒЫҚАЗАҚ ҰЛТТЫҚ МЕДИЦИНА

УНИВЕРСИТЕТІ

КАЗАХСКИЙНАЦИОНАЛЬНЫЙМЕДИЦИНСКИЙ УНИВЕРСИТЕТ ИМЕНИ С.Д.АСФЕНДИЯРОВА

КАФЕДРА ИНТЕРНАТУРЫ И РЕЗИДЕНТУРЫ ПО АКУШЕРСТВУ И ГИНЕКОЛОГИИТЕСТОВЫЕ ЗАДАНИЯ ДЛЯ ПРОВЕДЕНИЯ ИТОГОВОЙ ГОСУДАРСТВЕННОЙ АТТЕСТАЦИИ РЕЗИДЕНТОВ

ВЫПУСКА 2012-2013 ГГ. ПО СПЕЦИАЛЬНОСТИ «АКУШЕРСТВО И ГИНЕКОЛОГИЯ»

266.Пациентке 28 лет по поводу первичного бесплодия назначена метросальпингография. Заключение рентгенограммы: полость матки Т-образной формы, маточные трубы укорочены, ригидные, с булавовидными расширениями в ампулярных отделах; выхода контрастного вещества в брюшную полость не наблюдается. Какой диагноз НАИБОЛЕЕ вероятен?A) хронический сальпингитB) туберкулез гениталийC) хронический сальпингоофаритD) рак маточных труб E) эндометриоз маточных труб267.У пациентки 28 лет после обследования по поводу первичного бесплодия обнаружена непроходимость маточных труб в ампулярных отделах. В анамнезе: в детстве перенесла туберкулез. На «Д» учете в туберкулезном диспансере не состоит. Какой план лечебных мероприятий является НАИБОЛЕЕ целесообразным?A) физиотерапевтическоеB) гидротубацияC) хирургическоеD) провокация гоновакцинойE) специфическое лечение268.Пациентка 33 лет с бесплодием во втором браке. Менструальная функция без патологии. Муж обследован, спермограмма не изменена. В зеркалах: шейка матки без изменений. РV: без патологии. Наиболее вероятное показание для искусственной инсеминации спермой мужа:A) хромосомные патологии в яйцеклеткеB) эректильная дисфункцияC) гиперпролактинемияD) окклюзия маточных трубE) варикоцеле269.Пациентка 29 лет обратилась в женскую консультацию с жалобами на первичное бесплодие в течение 3 лет. Менструальная функция не нарушена. При измерении базальной температуры - двухфазный цикл. Половая жизнь регулярная, в браке, от беременности не предохранялась. Результат гистеросальпингографии - маточные трубы проходимы. При исследовании спермограммы мужа выявлены астено- и олигозооспермия II степени. После лечения мужа андрологом сохраняется олигозооспермия II степени. Какой диагноз НАИБОЛЕЕ вероятен?A) субфертильность спермы мужаB) тератоозоспермияC) недостаточность лютеиновой фазы менструального циклаD) хроническая ановуляцияE) недостаточность первой фазы менструального цикла270.Пациентка, 32 лет обратилась к врачу по поводу отсутствия беременности в течение 3 лет. Менархе с 13 лет, менструальный цикл регулярный. Первая беременность закончилась родами. Через 6 месяцев после родов установили ВМС. Через 4 года была госпитализирована в гинекологическое отделение по поводу острого двухстороннего сальпингита, была удалена ВМС. В последующие три года не предохранялась. При гинекологическом осмотре PV: Влагалище рожавшей. Шейка матки цилиндрической формы. Матка не увеличена, подвижная, безболезненная. Слева пальпируются тяжистые придатки, справа – образование размером 5,0х6,0 см, округлой формы тугоэластической консистенции, болезненное. Какой диагноз НАИБОЛЕЕ вероятен?A) хронический двухсторонний аднекситB) киста яичника

Page 50: postgraduate.kaznmu.kz€¦ · Web view58 Больная И, 28 лет, доставлена с жалобами на внезапно возникшую боль внизу живота

С.Ж.АСФЕНДИЯРОВАТЫНДАҒЫҚАЗАҚ ҰЛТТЫҚ МЕДИЦИНА

УНИВЕРСИТЕТІ

КАЗАХСКИЙНАЦИОНАЛЬНЫЙМЕДИЦИНСКИЙ УНИВЕРСИТЕТ ИМЕНИ С.Д.АСФЕНДИЯРОВА

КАФЕДРА ИНТЕРНАТУРЫ И РЕЗИДЕНТУРЫ ПО АКУШЕРСТВУ И ГИНЕКОЛОГИИТЕСТОВЫЕ ЗАДАНИЯ ДЛЯ ПРОВЕДЕНИЯ ИТОГОВОЙ ГОСУДАРСТВЕННОЙ АТТЕСТАЦИИ РЕЗИДЕНТОВ

ВЫПУСКА 2012-2013 ГГ. ПО СПЕЦИАЛЬНОСТИ «АКУШЕРСТВО И ГИНЕКОЛОГИЯ»

C) синдром поликистозных яичников (СКПЯ)D) тубоовариальное образованиеE) параметрит271.Пациентка 30 лет, жалобы на бесплодие во втором браке. Имела в первом браке нормальные роды, 2 аборта. Менструальная функция не нарушена. Муж обследован, спермограмма не изменена. Какая наиболее вероятная продолжительность обследования супружеской пары?A) 4 годаB) 2 годаC) 3 годаD) 1 годE) 5 лет272.Больная Н., 27 лет, в детстве перенесла туберкулез. На «Д» учете в туберкулезном диспансере не состоит. Периодически беспокоили боли в животе. В браке 3 года, беременность не наступает. Менархе с 14 лет, последние 5 лет менструации стали короткими и скудными. При гинекологическом исследовании патологических изменений не выявлено. Наиболее вероятный диагноз:A) генитальный эндометриозB) туберкулез гениталийC) хронический аднекситD) синдром поликистозных яичниковE) хламидиоз гениталий273.Пациентка 31 год, обратилась к врачу по поводу бесплодия. Из анамнеза - менструации с 13 лет, болезненные, цикл регулярный. В браке, половая жизнь регулярная, беременность не наступает. PV: Влагалище узкое. Шейка матки коническая, подвижная, безболезненная. Матка в нормальном положении, подвижная, безболезненная, обычных размеров. Придатки слева не пальпируются, справа между маткой и яичником обнаружено округлое образование диаметром 6,0х6,0 см, тугоэластической консистенции, безболезненное. Какой диагноз НАИБОЛЕЕ вероятен?A) СКПЯB) сактосальпинкс справаC) киста яичника справаD) эндометриозE) сальпингоофорит справа274.Пациентка, 25 лет, обратилась в женскую консультацию с жалобами на отсутствие беременности в течение 2 лет, общую слабость, потливость, временами повышение температуры до 37,2 – 37,5°С, периодически отмечает боли внизу живота ноющего характера. Последние 3 года менструации стали скудными. Через 1,5 года после начала половой жизни прошла гистеросальпингографию – трубы непроходимы, «четкообразной» формы. Какой диагноз НАИБОЛЕЕ вероятен?A) хламидиоз гениталий.B) туберкулез гениталий.C) синдром поликистозных яичников.D) хронический аднексит.E) генитальный эндометриоз 275.С чего НАИБОЛЕЕ вероятно начинается диагностика женского бесплодия, и с какой целью? A) теста Шуварского-Симса-Хунера – для определения числа и подвижности сперматозоидов в цервикальной слизиB) тестов функциональной диагностики – для оценки полноценности менструального цикла

Page 51: postgraduate.kaznmu.kz€¦ · Web view58 Больная И, 28 лет, доставлена с жалобами на внезапно возникшую боль внизу живота

С.Ж.АСФЕНДИЯРОВАТЫНДАҒЫҚАЗАҚ ҰЛТТЫҚ МЕДИЦИНА

УНИВЕРСИТЕТІ

КАЗАХСКИЙНАЦИОНАЛЬНЫЙМЕДИЦИНСКИЙ УНИВЕРСИТЕТ ИМЕНИ С.Д.АСФЕНДИЯРОВА

КАФЕДРА ИНТЕРНАТУРЫ И РЕЗИДЕНТУРЫ ПО АКУШЕРСТВУ И ГИНЕКОЛОГИИТЕСТОВЫЕ ЗАДАНИЯ ДЛЯ ПРОВЕДЕНИЯ ИТОГОВОЙ ГОСУДАРСТВЕННОЙ АТТЕСТАЦИИ РЕЗИДЕНТОВ

ВЫПУСКА 2012-2013 ГГ. ПО СПЕЦИАЛЬНОСТИ «АКУШЕРСТВО И ГИНЕКОЛОГИЯ»

C) гистероскопии – для выявления внутриматочной патологии D) кульдоскопии – для выявления патологии в Дугласовом пространстве E) лапароскопии – для выявления патологии органов малого таза и брюшной полости 276Пациентка, 32 лет, с анамнезом вторичное бесплодие, двухсторонний аднексит с частыми обострениями, поступила в стационар с высокой температурой, озноб, боли по всему животу. Объективно: температура 40ºС, пульс 120 ударов/минуту, АД 90/70 мм рт. ст. Язык сухой, живот вздут, напряженный во всех отделах, болезненный с выраженными перитонеальными симптомами. OZ: шейка матки эрозирована. Выделения гноевидные, обильные. PV: При влагалищном исследовании смещения шейки матки резко болезненные, придатки и матку определить не удается из-за болезненности. Задний свод влагалища уплощен, болезненный. Выставлен разлитой перитонит. Какая тактика НАИБОЛЕЕ оправдана в данном случае?A) пункция заднего свода B) оперативное лечениеC) антибактериальная терапияD) гормональная терапияE) кульдоскопия277.Пациентка, 25 лет, в детстве перенесла туберкулез. На «Д» учете в туберкулезном диспансере не состоит. Периодически беспокоили боли в животе. В браке 2 года, беременность не наступает. Менархе с 14 лет, последние 5 лет менструации стали короткими и скудными. При специальном гинекологическом исследовании патологических изменений не выявлено. Какой из перечисленных методов обследования является НАИБОЛЕЕ важным? A) гистероскопия B) УЗИ органов малого тазаC) микроскопия мазкаD) гистеросальпингографияE) кольпоскопия 278.В стационар обратилась пациентка через 5 дней после медицинского аборта, когда появились боли в нижнем отделе живота, озноб, повышение температуры тела до 38 °С. При осмотре: живот мягкий, болезненный в нижнем отделе. Симптомов раздражения брюшины нет. При влагалищном исследовании: матка увеличена до 6-7 недель беременности, болезненная, мягковатой консистенции. Наружный зев шейки матки пропускает 1 поперечный палец. Придатки матки не определяются. Выделения кровянистые с неприятным запахом. Какой из перечисленных методов обследования является НАИБОЛЕЕ важным?A) бактериоскопическийB) бактериологический C) УЗИ органов малого таза D) кольпоскопический E) лапароскопия279.Какой план лечебных мероприятий из перечисленных является НАИБОЛЕЕ целесообразным при сформировавшемся гнойном воспалительном образовании придатков матки?A) хирургическое лечение B) пункция образования через влагалищный сводC) терапия пирогеналом D) терапия гоновакциномE) физиолечение280.Пациентка, 24 лет, поступила в гинекологическое отделение клиники с жалобами на мажущие кровянистые выделения из половых путей. Замужем, в течение 2 лет страдает бесплодием. Общее состояние удовлетворительное. Кожные покровы и видимые слизистые нормальной окраски. Язык

Page 52: postgraduate.kaznmu.kz€¦ · Web view58 Больная И, 28 лет, доставлена с жалобами на внезапно возникшую боль внизу живота

С.Ж.АСФЕНДИЯРОВАТЫНДАҒЫҚАЗАҚ ҰЛТТЫҚ МЕДИЦИНА

УНИВЕРСИТЕТІ

КАЗАХСКИЙНАЦИОНАЛЬНЫЙМЕДИЦИНСКИЙ УНИВЕРСИТЕТ ИМЕНИ С.Д.АСФЕНДИЯРОВА

КАФЕДРА ИНТЕРНАТУРЫ И РЕЗИДЕНТУРЫ ПО АКУШЕРСТВУ И ГИНЕКОЛОГИИТЕСТОВЫЕ ЗАДАНИЯ ДЛЯ ПРОВЕДЕНИЯ ИТОГОВОЙ ГОСУДАРСТВЕННОЙ АТТЕСТАЦИИ РЕЗИДЕНТОВ

ВЫПУСКА 2012-2013 ГГ. ПО СПЕЦИАЛЬНОСТИ «АКУШЕРСТВО И ГИНЕКОЛОГИЯ»

чистый, влажный. Температура 36,6°С. Пульс 70 ударов/минуту, удовлетворительного напряжения и наполнения. Живот мягкий, безболезненный. При осмотре в зеркалах – слизистая влагалища и шейки матки синюшна, выделения темные, мажущие кровянистые. Матка в anteflexio-versio, подвижная. Тело матки мягковатой консистенции, увеличено соответственно 7-8 недель беременности. Слева от матки определяется образование «тестоватой» консистенции, болезненное. Какой предварительный диагноз НАИБОЛЕЕ вероятен?A) левосторонняя трубная беременность, нарушенная по типу трубного аборта B) начавшийся ранний самопроизвольный аборт в 7-8 недельC) угрожающий ранний самопроизвольный аборт в 7-8 недельD) левосторонняя трубная беременность, нарушенная по типу разрыва трубыE) левосторонняя трубная беременность, прогрессирующая 281.Пациентка 33 лет, доставлена в стационар машиной скорой помощи. Больная в сознании, но слабо реагирует на окружающее. На вопросы отвечает односложно, вяло. В анамнезе задержка менструации на 6 недель. Кожные покровы и видимые слизистые бледные, на лбу холодный пот. Пульс 120 ударов/минуту, слабого наполнения, частый, плохо сосчитывается. АД 70/40 мм рт.ст. Живот равномерно вздут, перистальтика отсутствует. При пальпации брюшной стенки отмечается умеренно выраженное напряжение мышц живота, усиливающееся при пальпации. Наружное кровотечение отсутствует. Нв 66г/л. Какой предварительный диагноз НАИБОЛЕЕ вероятен?A) внематочная беременность, нарушенная по типу разрыва трубы, геморрагический шок 3 ст.B) внематочная беременность, нарушенная по типу разрыва трубы, геморрагический шок 2 ст.C) апоплексия яичника, геморрагическая форма, геморрагический шок 3 ст.D) разрыв кисты яичника, геморрагический шок геморрагический шок 3 ст.E) начинающийся самопроизвольный выкидыш, геморрагический шок 2 ст.282У пациентки во время использования противозачаточного средства менструации не было. Выберите НАИБОЛЕЕ вероятный метод, применение которых дает подобное осложнение:A) пероральные комбинированные контрацептивы B) шеечные колпачкиC) чистые гестагеныD) презервативыE) спермициды283.К гинекологу обратилась пациентка 18 лет по поводу выбора метода контрацепции. Не замужем. Половые партнеры разные. Какой метод контрацепции наиболее оправдан в данном случае?A) внутриматочные контрацептивыB) пероральные комбинированные контрацептивыC) чистые гестагеныD) хирургическая стерилизацияE) барьерные средства284.Пациентка 35 лет, мать пятерых детей, больше не планирует рожать. Какой метод контрацепции ей НАИБОЛЕЕ показан? A) внутриматочные контрацептивыB) пероральные комбинированные контрацептивыC) чистые гестагеныD) хирургическая стерилизацияE) барьерные средства285.НАИБОЛЕЕ вероятный объем лечебного вмешательства при тубоовариальном образовании в позднем репродуктивном периоде:A) надвлагалищная ампутация матки с придатками

Page 53: postgraduate.kaznmu.kz€¦ · Web view58 Больная И, 28 лет, доставлена с жалобами на внезапно возникшую боль внизу живота

С.Ж.АСФЕНДИЯРОВАТЫНДАҒЫҚАЗАҚ ҰЛТТЫҚ МЕДИЦИНА

УНИВЕРСИТЕТІ

КАЗАХСКИЙНАЦИОНАЛЬНЫЙМЕДИЦИНСКИЙ УНИВЕРСИТЕТ ИМЕНИ С.Д.АСФЕНДИЯРОВА

КАФЕДРА ИНТЕРНАТУРЫ И РЕЗИДЕНТУРЫ ПО АКУШЕРСТВУ И ГИНЕКОЛОГИИТЕСТОВЫЕ ЗАДАНИЯ ДЛЯ ПРОВЕДЕНИЯ ИТОГОВОЙ ГОСУДАРСТВЕННОЙ АТТЕСТАЦИИ РЕЗИДЕНТОВ

ВЫПУСКА 2012-2013 ГГ. ПО СПЕЦИАЛЬНОСТИ «АКУШЕРСТВО И ГИНЕКОЛОГИЯ»

B) надвлагалищная ампутация матки с придатками и резекцией большого сальника C) экстирпация матки с придаткамиD) консервативная антибактериальная терапияE) одно- или двухстороннее удаление труб и яичников 286.НАИБОЛЕЕ вероятное осложнение гнойных тубоовариальных образований:A) малигнизацияB) кровоизлияние C) перфорация D) перитонитE) перекрут 287.Что такое редукция эмбрионов?A) рассечение блестящей оболочки эмбриона перед имплантацией в матку B) инкубация яйцеклетки и сперматозоида в среде с целью достижения оплодотворенияC) операция по уменьшению количества живых эмбрионов под контролем эхографииD) инъекция сперматозоида в цитоплазму клеткиE) криоконсервация ооцитов и эмбрионов288.НАИБОЛЕЕ вероятное условие для редукции эмбриона:A) срок беременности 5 – 11 недельB) отсутствие сердцебиения C) четвертая степень чистоты влагалищного мазкаD) наличие визуально идентифицируемых отклонений в развитии эмбрионаE) срок беременности 3- 6 недель289.НАИБОЛЕЕ вероятное показание к проведению ЭКО:A) отсутствие маточных труб или непроходимость трубB) синдром поликистозных яичников (СПКЯ)C) миома маткиD) опухоли яичниковE) аденомиоз290.Пациентка 26 лет, обратилась в женскую консультацию по поводу задержки очередной менструации. Жалобы на тошноту, рвоту по утрам, утомляемость, раздражительность. В анамнезе - три медицинских аборта. При влагалищном исследовании: шейка матки цианотичная, гипертрофирована, бочкообразной формы, наружный зев закрыт, эксцентрично расположен. Матка мягкой консистенции, несколько больше нормы. Придатки не пальпируются. Своды свободные. Движения за шейку безболезненные. Выделения мажущие, кровянистые. Наиболее вероятный диагноз? A) аборт в ходу. ОААB) начавшийся самопроизвольный аборт. ОААC) шеечная беременность. ОААD) маточная беременность. ОААE) маточная беременность. Низкая плацентация. ОАА291.Пациентка 20 лет, предъявляет жалобы на задержку очередной менструации, тошноту. Половая жизнь регулярная. От беременностей не предохраняется. OZ: слизистая влагалища и шейки матки цианотичные, выделения – темные, кровянистые, мажущие. PV: матка чуть больше нормы, мягковатой консистенции, подвижная. В области придатков с одной стороны образование «тестоватой» консистенции. НАИБОЛЕЕ вероятный план лечебных мероприятий?A) антикоагулянтная терапияB) дезинтоксикационная терапияC) противомикробное лечение

Page 54: postgraduate.kaznmu.kz€¦ · Web view58 Больная И, 28 лет, доставлена с жалобами на внезапно возникшую боль внизу живота

С.Ж.АСФЕНДИЯРОВАТЫНДАҒЫҚАЗАҚ ҰЛТТЫҚ МЕДИЦИНА

УНИВЕРСИТЕТІ

КАЗАХСКИЙНАЦИОНАЛЬНЫЙМЕДИЦИНСКИЙ УНИВЕРСИТЕТ ИМЕНИ С.Д.АСФЕНДИЯРОВА

КАФЕДРА ИНТЕРНАТУРЫ И РЕЗИДЕНТУРЫ ПО АКУШЕРСТВУ И ГИНЕКОЛОГИИТЕСТОВЫЕ ЗАДАНИЯ ДЛЯ ПРОВЕДЕНИЯ ИТОГОВОЙ ГОСУДАРСТВЕННОЙ АТТЕСТАЦИИ РЕЗИДЕНТОВ

ВЫПУСКА 2012-2013 ГГ. ПО СПЕЦИАЛЬНОСТИ «АКУШЕРСТВО И ГИНЕКОЛОГИЯ»

D) рентгенография брюшной полостиE) экстренная лапароскопия292.Пациентка Н, 40 лет, поступает в гинекологическое отделение с острой болью в животе, иррадиирущую в правое подреберье, межлопаточную область, ключицу. Больная временами теряет сознание. На вопросы отвечает односложно. Кожные покровы и видимые слизистые бледные, на лбу холодный липкий пот. Пульс 140 ударов/минуту, слабого наполнения, частый, плохо сосчитывается. АД 70/40 мм рт. ст. Живот равномерно вздут. При пальпации брюшной стенки отмечается выраженное напряжение мышц живота. Наружное кровотечение отсутствует. НАИБОЛЕЕ вероятный план лечебных мероприятий?A) срочная лапаротомияB) дезинтоксикационная терапияC) гемотрансфузияD) переливание свежезамороженной плазмыE) лапароскопия293.Пациентка, 25 лет, поступила с жалобами на схваткообразные боли внизу живота, иррадиирущие в прямую кишку и мажущие кровянистые выделения из половых путей. Заболела 2 дня назад. В анамнезе: роды – 1, медицинские аборты – 2. Состояние удовлетворительное, пульс 78 ударов/минуту, АД 115/70 мм рт. ст. На зеркалах: цианоз слизистой влагалища и шейки матки, выделения кровянистые, скудные. PV: матка чуть больше нормы, подвижная, безболезненная. Справа в области придатков пальпируется образование без четких контуров, тестоватой консистенции, болезненное. Задний свод уплощен, болезненный. НАИБОЛЕЕ вероятный план лечебных мероприятий?A) кульдоцентез B) биопсия эндометрияC) лапароскопияD) определение ХГE) рентгенография брюшной полости294.Пациентка обратилась с жалобами на боли внизу живота, повышение температуры тела до 37,5ºС, гнойные выделения из половых путей, болезненное мочеиспускание. При осмотре: уретра инфильтрирована, в зеркалах – шейка матки гиперемирована, эрозирована, выделения обильные слизисто-гнойные. При влагалищном исследовании: матка не увеличена, болезненна при пальпации, придатки с обеих сторон утолщены, болезненны. При бактериоскопии мазков – гонококки, располагающиеся вне- и внутриклеточно. НАИБОЛЕЕ вероятный диагноз?A) свежая острая гонорея нижнего отдела половых органовB) свежая острая восходящая гонореяC) подострая восходящая гонореяD) хроническая гонорея нижнего отдела половых органовE) торпидная гонорея295.Пациентке 25 лет произведена гистеросальпингография по поводу первичного бесплодия. На снимке: полость матки Т-образной формы, маточные трубы укорочены, ригидные, с булавовидными расширениями в ампулярных отделах, выхода контрастного вещества в брюшную полость не наблюдается. НАИБОЛЕЕ вероятный диагноз?A) хронический сальпингит B) эндометриоз маточных трубC) рак маточных трубD) туберкулез половых органовE) острый сальпингоофорит296.Выберите наиболее частую локализацию туберкулеза женских половых органов:

Page 55: postgraduate.kaznmu.kz€¦ · Web view58 Больная И, 28 лет, доставлена с жалобами на внезапно возникшую боль внизу живота

С.Ж.АСФЕНДИЯРОВАТЫНДАҒЫҚАЗАҚ ҰЛТТЫҚ МЕДИЦИНА

УНИВЕРСИТЕТІ

КАЗАХСКИЙНАЦИОНАЛЬНЫЙМЕДИЦИНСКИЙ УНИВЕРСИТЕТ ИМЕНИ С.Д.АСФЕНДИЯРОВА

КАФЕДРА ИНТЕРНАТУРЫ И РЕЗИДЕНТУРЫ ПО АКУШЕРСТВУ И ГИНЕКОЛОГИИТЕСТОВЫЕ ЗАДАНИЯ ДЛЯ ПРОВЕДЕНИЯ ИТОГОВОЙ ГОСУДАРСТВЕННОЙ АТТЕСТАЦИИ РЕЗИДЕНТОВ

ВЫПУСКА 2012-2013 ГГ. ПО СПЕЦИАЛЬНОСТИ «АКУШЕРСТВО И ГИНЕКОЛОГИЯ»

A) эндометрийB) яичникиC) шейка маткиD) фаллопиевы трубыE) вульва297.При бимануальном исследовании у пациентки 28 лет определяются двухсторонние неподвижные плотные образования по ребру матки. При ректальном исследовании пальпируются многочисленные плотные узелки различной величины. Беспокоят ноющие боли внизу живота. Какой из перечисленных методов обследования является НАИБОЛЕЕ важным?A) биопсия шейки маткиB) метросальпингографияC) пневморельвиография D) УЗИ органов малого тазаE) компьюторная томография298.На приеме у гинеколога пациентка 23 лет обратилась по поводу первичного бесплодия. Проведена гистеросальпингография. На снимке определяется деформация полости матки, небольшое укорочение труб, наличие расширений на концах труб в виде булав. Какой НАИБОЛЕЕ вероятный метод диагностики применяется для уточнения степени активности патологического процесса?A) диагностическое выскабливание слизистой полости матки B) аспирационная биопсияC) проба КохаD) биологический методE) культуральный метод299.Пациентка 31 года обратилась с жалобами на ноющие боли внизу живота. В детстве перенесла туберкулез. Менструации регулярные, но скудные. В анамнезе бесплодие. При бимануальном исследовании в области придатков определяется тяжистость с обеих сторон. НАИБОЛЕЕ вероятный диагноз?A) хронический сальпингит B) эндометриоз маточных трубC) рак маточных трубD) туберкулез половых органовE) острый сальпингоофорит300.Пациентка 31 года обратилась с жалобами на ноющие боли внизу живота. В детстве перенесла туберкулез. Менструации регулярные, но скудные. В анамнезе бесплодие. При бимануальном исследовании в области придатков определяется тяжистость с обеих сторон. НАИБОЛЕЕ вероятный метод диагностики?A) взятие мазка на микрофлору B) проведение метросальпингографииC) определение онкомаркеровD) проведение компьютерной томографииE) взятие отделяемого на бакпосев301.НАИБОЛЕЕ вероятный основной клинический симптом туберкулезного поражения придатков матки:A) хроническая тазовая больB) аменореяC) менометроррагия

Page 56: postgraduate.kaznmu.kz€¦ · Web view58 Больная И, 28 лет, доставлена с жалобами на внезапно возникшую боль внизу живота

С.Ж.АСФЕНДИЯРОВАТЫНДАҒЫҚАЗАҚ ҰЛТТЫҚ МЕДИЦИНА

УНИВЕРСИТЕТІ

КАЗАХСКИЙНАЦИОНАЛЬНЫЙМЕДИЦИНСКИЙ УНИВЕРСИТЕТ ИМЕНИ С.Д.АСФЕНДИЯРОВА

КАФЕДРА ИНТЕРНАТУРЫ И РЕЗИДЕНТУРЫ ПО АКУШЕРСТВУ И ГИНЕКОЛОГИИТЕСТОВЫЕ ЗАДАНИЯ ДЛЯ ПРОВЕДЕНИЯ ИТОГОВОЙ ГОСУДАРСТВЕННОЙ АТТЕСТАЦИИ РЕЗИДЕНТОВ

ВЫПУСКА 2012-2013 ГГ. ПО СПЕЦИАЛЬНОСТИ «АКУШЕРСТВО И ГИНЕКОЛОГИЯ»

D) первичное бесплодиеE) вторичное бесплодие302.При проведении профилактического осмотра у девушки 16 лет, обнаружено двухстороннее увеличение придатков матки. Жалоб не предъявляет. В анамнезе хронический тонзиллит, с частыми обострениями. Последнее обострение было 4 месяца тому назад. Признаков воспаления нижнего отдела полового тракта не было. Каков НАИБОЛЕЕ вероятный патогенез поражения маточных труб в данном случае?A) лимфогенный путь инфицирования из аппендикса B) гематогенный путь инфицирования из миндалинC) гематогенный путь инфицирования из первичного очага в легкихD) гематогенный путь инфицирования из первичного очага в почкахE) каникулярный путь инфицирования гонореей в детстве303.На прием к гинекологу обратилась пациентка 33 лет с жалобами на отсутствие беременности. В браке 8 лет. Острые и хронические заболевания половых органов отрицает. На гистеросальпингограмме – симптом “четок“. Что НАИБОЛЕЕ вероятно способствовало в данном случае таким изменениям?A) замедление кровотока в ампулярных отделах трубB) отсутствие складчатости слизистой оболочки маточных трубC) скудная капиллярная сеть всех отделов маточных труб D) отсутствие анастомозов между маточной и яичниковой артериями E) ускорение кровотока в ампулярных отделах труб304.Пациентка 23 лет обратилась к гинекологу с жалобами на отсутствие менструации в течение года, бесплодие. Проведена проба Коха - резко положительна. Проба с прогестероном отрицательна. Планируется гистеросальпингография. Какую НАИБОЛЕЕ вероятную R- картину матки ожидаете увидеть при данной патологии?A) наличие большого, округлого дефекта наполнения полости матки B) наличие множества небольших полостей внутри матки, разделенных перегородками C) наличие законтурных за пределами эндометрия тенейD) полное отсутствие контрастного вещества в маткеE) наличие четких контуров матки305.НАИБОЛЕЕ информативный метод определения половой инфекции:A) иммуноферментный анализ (ИФА)B) полимеразная цепная реакция (ПЦР)C) посев микрофлоры на чувствительность к антибиотикамD) анализы кровиE) мазок на степень чистоты влагалища306.Назначение, какого средства контрацепции с минимальным риском развития воспалительных заболеваний органов малого таза из перечисленных является НАИБОЛЕЕ целесообразным?A) презерватив B) химические контрацептивыC) внутриматочные контрацептивыD) диафрагмаE) оральные контрацептивы307.Пациента 35 лет, в анамнезе 3 родов, последняя беременность 6 месяцев назад – срочные роды с кровотечением. В настоящее время кормит ребенка грудью. НАИБОЛЕЕ целесообразным методом контрацепции в данном случае является:A) метод лактационной аменореи B) комбинированные оральные контрацептивы, не содержащие гестагены

Page 57: postgraduate.kaznmu.kz€¦ · Web view58 Больная И, 28 лет, доставлена с жалобами на внезапно возникшую боль внизу живота

С.Ж.АСФЕНДИЯРОВАТЫНДАҒЫҚАЗАҚ ҰЛТТЫҚ МЕДИЦИНА

УНИВЕРСИТЕТІ

КАЗАХСКИЙНАЦИОНАЛЬНЫЙМЕДИЦИНСКИЙ УНИВЕРСИТЕТ ИМЕНИ С.Д.АСФЕНДИЯРОВА

КАФЕДРА ИНТЕРНАТУРЫ И РЕЗИДЕНТУРЫ ПО АКУШЕРСТВУ И ГИНЕКОЛОГИИТЕСТОВЫЕ ЗАДАНИЯ ДЛЯ ПРОВЕДЕНИЯ ИТОГОВОЙ ГОСУДАРСТВЕННОЙ АТТЕСТАЦИИ РЕЗИДЕНТОВ

ВЫПУСКА 2012-2013 ГГ. ПО СПЕЦИАЛЬНОСТИ «АКУШЕРСТВО И ГИНЕКОЛОГИЯ»

C) внутриматочные контрацептивыD) чистые оральные контрацептивы, содержащие гестагеныE) механические методы контрацепции308.У молодой пациентки во время использования ВМК «Мирены» менструации не было. Назначение, какого метода из перечисленных является НАИБОЛЕЕ целесообразным и не дает осложнение?A) оральные комбинированные контрацептивы B) шеечные колпачкиC) чистые гестагеныD) презервативыE) спермициды309.Назовите НАИБОЛЕЕ вероятный гормональный контрацептив, рекомендуемый женщинам с гирсутизмом, АКНЕ или себореей:A) ярина B) линданет 20C) диане 35D) триквилорE) жанин310.Абсолютным ПРОТИВОПОКАЗАНИЕМ к использованию комбинированных контрацептивов являются:A) эндометриоз шейки матки B) гирсутизм, АКНЕ или себореяC) острый тромбофлебит, гипертоническая болезнь 2-3 степениD) состояние после операции на маткеE) миома матки311.Пациентка 40 лет, обратилась к врачу с жалобами на отек, гиперемию, боли в области вен на нижних конечностях. Страдает заболеваниями печени и желчного пузыря. Предложите НАИБОЛЕЕ вероятный метод планирования семьи:A) симптотермальный B) миниабортC) лактационная аменореяD) барьерныйE) химический312.Женщина 32 лет находится на диспансерном учете у гинеколога по поводу хронического аднексита в течение 3 лет. При гинекологическом исследовании - в области правых придатков определяется тяжистость, незначительная болезненность. Какой из методов физиотерапии НАИБОЛЕЕ предпочтителен в данном случае?A) ультразвуковая терапия в импульсном режимеB) ультрафиолетовое облучение “ трусиковой “ зоныC) терапия микроволнами сантиметрового диапазонаD) магнитотерапия курсами E) электрофорез кальция синусоидальными токами313.У пациентки 32 лет после лапаротомии по поводу тубоовариального образования воспалительного генеза имеется частичное рассхождение послеоперационного шва. Получает комплексную антибактериальную терапию, местное лечение. Какой план лечебных мероприятий является НАИБОЛЕЕ целесообразным?A) электрофорез цинка синусоидальными модулированными токамиB) электрофорез йода синусоидальными модулированными токами

Page 58: postgraduate.kaznmu.kz€¦ · Web view58 Больная И, 28 лет, доставлена с жалобами на внезапно возникшую боль внизу живота

С.Ж.АСФЕНДИЯРОВАТЫНДАҒЫҚАЗАҚ ҰЛТТЫҚ МЕДИЦИНА

УНИВЕРСИТЕТІ

КАЗАХСКИЙНАЦИОНАЛЬНЫЙМЕДИЦИНСКИЙ УНИВЕРСИТЕТ ИМЕНИ С.Д.АСФЕНДИЯРОВА

КАФЕДРА ИНТЕРНАТУРЫ И РЕЗИДЕНТУРЫ ПО АКУШЕРСТВУ И ГИНЕКОЛОГИИТЕСТОВЫЕ ЗАДАНИЯ ДЛЯ ПРОВЕДЕНИЯ ИТОГОВОЙ ГОСУДАРСТВЕННОЙ АТТЕСТАЦИИ РЕЗИДЕНТОВ

ВЫПУСКА 2012-2013 ГГ. ПО СПЕЦИАЛЬНОСТИ «АКУШЕРСТВО И ГИНЕКОЛОГИЯ»

C) электрофорез кальция синусоидальными модулированными токамиD) низкочастотное лазерное излучение E) электрофорез лидазы и пеллоидина 314.В стационар поступила пациентка 24 лет с диагнозом: Неполный инфицированный аборт. Какой план лечебных мероприятий является НАИБОЛЕЕ целесообразным?A) экстирпация матки B) инфузионная терапияC) надвлагалищная ампутация матки D) раздельное диагностическое выскабливаниеE) удаление остатков плодного яйца315.Пациентка 24 лет поступила в гинекологический стационар с жалобами на высокую температуру, слабость, кровянистые выделения из половых путей. Из анамнеза: 3 дня тому назад с целью прерывания беременности в полость матки ввела железный катетер. Последние месячные не помнит. На учете по поводу беременности не состоит. Общее состояние при поступлении средней тяжести. Температура тела 39°С. Живот напряжен, при пальпации болезнен в области пупка. Симптом Щеткина-Блюмберга положительный. На зеркалах – имеются следы уколов на передней губе шейки матки. Из цервикального канала – кровянистые выделения с неприятным запахом в умеренном количестве. При вагинальном исследовании – шейка матки конической формы, зев пропускает кончик пальца. Матку отдельно пальпировать не удается. В малом тазу пальпируется образование без четких контуров, малоподвижное, болезненное при пальпации. Придатки не пальпируются. Какой НАИБОЛЕЕ вероятный диагноз?A) криминальный аборт. СепсисB) криминальный аборт. Септическое состояниеC) криминальный аборт. Перфорация маткиD) криминальный аборт. Септический шокE) криминальный аборт. Пельвиоперитонит316.Пациентка 24 лет поступила в гинекологический стационар с жалобами на высокую температуру, слабость, кровянистые выделения из половых путей. Из анамнеза: 3 дня тому назад с целью прерывания беременности в полость матки ввела железный катетер. Последние месячные не помнит. На учете по поводу беременности не состоит. Общее состояние при поступлении средней тяжести. Температура тела 39°С. Живот напряжен, при пальпации болезнен в области пупка. Симптом Щеткина-Блюмберга положительный. На зеркалах – имеются следы уколов на передней губе шейки матки. Из цервикального канала – кровянистые выделения с неприятным запахом в умеренном количестве. При вагинальном исследовании – шейка матки конической формы, зев пропускает кончик пальца. Матку отдельно пальпировать не удается. В малом тазу пальпируется образование без четких контуров, малоподвижное, болезненное при пальпации. Придатки не пальпируются. Какова НАИБОЛЕЕ целесообразная тактика дальнейшего ведения больного?A) экстирпация матки в экстренном порядкеB) динамическое наблюдениеC) надвалгалищная ампутация матки в экстренном порядкеD) выскабливание полости маткиE) антибактериальная терапия317.Пациентке 33 лет произведено прерывание беременности путем медицинского аборта. В анамнезе хронический сальпингооофарит. Какой метод физиотерапии является НАИБОЛЕЕ целесообразным?A) ультрафиолетовое облучение трусиковой зоны; B) электрофорез лидазы и пеллоидина;

Page 59: postgraduate.kaznmu.kz€¦ · Web view58 Больная И, 28 лет, доставлена с жалобами на внезапно возникшую боль внизу живота

С.Ж.АСФЕНДИЯРОВАТЫНДАҒЫҚАЗАҚ ҰЛТТЫҚ МЕДИЦИНА

УНИВЕРСИТЕТІ

КАЗАХСКИЙНАЦИОНАЛЬНЫЙМЕДИЦИНСКИЙ УНИВЕРСИТЕТ ИМЕНИ С.Д.АСФЕНДИЯРОВА

КАФЕДРА ИНТЕРНАТУРЫ И РЕЗИДЕНТУРЫ ПО АКУШЕРСТВУ И ГИНЕКОЛОГИИТЕСТОВЫЕ ЗАДАНИЯ ДЛЯ ПРОВЕДЕНИЯ ИТОГОВОЙ ГОСУДАРСТВЕННОЙ АТТЕСТАЦИИ РЕЗИДЕНТОВ

ВЫПУСКА 2012-2013 ГГ. ПО СПЕЦИАЛЬНОСТИ «АКУШЕРСТВО И ГИНЕКОЛОГИЯ»

C) низкочастотное лазерное излучение;D) терапия микроволнами сантиметрового диапазона;E) электрофорез меди синусоидальными модулированными токами318.Пациентка 37 лет обратилась к гинекологу с жалобами на постоянные боли внизу живота. В анамнезе - хроническое воспаление придатков матки. Прошла курс антибиотикотерапии с учетом чувствительности микрофлоры. Боли уменьшились незначительно. Какой из методов физиолечения НАИБОЛЕЕ целесообразен в данном случае?A) УВЧ- терапияB) СВЧ - терапияC) местная дарсонвализация D) магнитотерапияE) электрофорез319. Пациентка 20 лет поступила в гинекологическое отделение через 10 дней после криминального аборта. Объективно: одышка, усиливающаяся в горизонтальном положении, кожа теплая на ощупь, тахикардия. Температура тела 39,0 – 37,50С, сопровождается ознобом. Почасовой диурез – 30,0мл. На зеркалах слизистая влагалища гиперемирована, выделения из цервикального канала гнойные с неприятным запахом. PV: шейка матки цилиндрическая, цервикальный канал пропускает кончик пальца. Тело матки увеличено до 8-9 недельного срока беременности, мягковатой консистенции, болезненно при пальпации. Придатки не пальпируются. Своды свободны. В данном случае имеет место клинический вариант сепсиса:А)септицемияБ) септикопиемияВ) септический шокГ) хрониосепсисД) тромбический вариант сепсиса320. Женщина 24 лет поступила в гинекологический стационар с жалобами на высокую температуру, слабость, кровянистые выделения из половых путей. Из анамнеза: 5 дней назад с целью прерывания беременности в полость матки ввела железный катетер. После обследования установлен диагноз: Неполный инфицированный аборт. Сепсис? Перфорация матки? Ваша тактикаА) экстирпация матки в экстренном порядкеБ)АБ, дезинтоксикационная терапия в течение 2-4 часов, экстирпация матки В) надвлагалищная ампутация маткиГ) АБ, дезинтоксикационная терапия в течение 2-4 часов, надвлагалищная ампутация маткиД) АБ, дезинтоксикационная терапия в течение 2-4 часов, выскабливание полости матки321. Пациентка 15 лет выписана из гинекологического стационара с диагнозом: ЮМК. С целью остановки кровотечения использован гормональный гемостаз новинетом по схеме. В стационаре находилась 7 дней, выписана после прекращения кровянистых выделений. Дальнейшая тактика?А) продолжить лечение новинетом в режиме КОК до 21 дня, затем гестагены по схеме – 3 месяцаБ) продолжить лечение новинетом в режиме КОК до 6 месяцевВ) продолжить лечение новинетом в режиме КОК до 3 месяцев, затем гестагены по схеме до 6 месяцевГ) прекратить лечение новинетом, начать лечение гестагенами Д) продолжить лечение новинетом в режиме КОК до 21 дня, далее –

Page 60: postgraduate.kaznmu.kz€¦ · Web view58 Больная И, 28 лет, доставлена с жалобами на внезапно возникшую боль внизу живота

С.Ж.АСФЕНДИЯРОВАТЫНДАҒЫҚАЗАҚ ҰЛТТЫҚ МЕДИЦИНА

УНИВЕРСИТЕТІ

КАЗАХСКИЙНАЦИОНАЛЬНЫЙМЕДИЦИНСКИЙ УНИВЕРСИТЕТ ИМЕНИ С.Д.АСФЕНДИЯРОВА

КАФЕДРА ИНТЕРНАТУРЫ И РЕЗИДЕНТУРЫ ПО АКУШЕРСТВУ И ГИНЕКОЛОГИИТЕСТОВЫЕ ЗАДАНИЯ ДЛЯ ПРОВЕДЕНИЯ ИТОГОВОЙ ГОСУДАРСТВЕННОЙ АТТЕСТАЦИИ РЕЗИДЕНТОВ

ВЫПУСКА 2012-2013 ГГ. ПО СПЕЦИАЛЬНОСТИ «АКУШЕРСТВО И ГИНЕКОЛОГИЯ»

симптоматическое лечение.322. Подберите препараты для лечения трихомонадного вульвовагинита:А) клотримазолБ) метронидазолВ) гино-травоген, гино-певарилГ) фазижинД) дифлюкан323. Подберите препарат для лечения кандидозного вагинита:А) клотримазолБ) метронидазолВ) гино-травоген, гино-певарилГ) фазижинД) дифлюкан324. Больная 32 года, результат гистологического исследования эндометрия по поводу ДМК – железисто-кистозная гиперплазия. Тактика врача:А) назначение гормональной терапииБ) направление на оперативное лечениеВ) наблюдениеГ) иммунокорригирующая терапияД) сан-курортное лечение325. Выберите гормональный препарат для лечения железисто-кистозной гиперплазии эндометрия у больной 47 лет:А) синтетические прогестиныБ) эстрогеныВ) гестагеныГ) андрогеныд) кортикостероиды326. При обострении хронического сальпингоофорита по типу невралгии тазовых нервов наименее эффективно:А) антибиотикотерапияБ) электрофорез амидопирина В) диадинамические токи Г) ультрафиолетовая эритемотерапия Д) амплипульстерапия327. Неосложненный инфицированный аборт – это гнойно-септическое заболевание, при котором:А) инфекция ограничена плодным яйцом и децидуальной оболочкой маткиБ) инфекция вышла за пределы матки, но осталась в малом тазуВ) инфекция распространилась за пределы малого таза и стала генерализованнойГ) развивается гнойно-резорбтивная лихорадкаД) развивается тромбофлебит маточных сосудов328. Осложненный инфицированный аборт – это гнойно-септическое заболевание, при котором:А) инфекция ограничена плодным яйцом и децидуальной оболочкой маткиБ) инфекция ограничена плодным яйцомВ) инфекция ограничена децидуальной оболочкой маткиГ) инфекция вышла за пределы матки, но осталась в малом тазуД) инфекция распространилась за пределы малого таза и стала генерализованной329. Больная В., 30 лет, обратилась к врачу женской консультации с жалобами на первичное бесплодие, редкие менструации. Объективно: повышенного питания, ИМТ – 32, полосы

Page 61: postgraduate.kaznmu.kz€¦ · Web view58 Больная И, 28 лет, доставлена с жалобами на внезапно возникшую боль внизу живота

С.Ж.АСФЕНДИЯРОВАТЫНДАҒЫҚАЗАҚ ҰЛТТЫҚ МЕДИЦИНА

УНИВЕРСИТЕТІ

КАЗАХСКИЙНАЦИОНАЛЬНЫЙМЕДИЦИНСКИЙ УНИВЕРСИТЕТ ИМЕНИ С.Д.АСФЕНДИЯРОВА

КАФЕДРА ИНТЕРНАТУРЫ И РЕЗИДЕНТУРЫ ПО АКУШЕРСТВУ И ГИНЕКОЛОГИИТЕСТОВЫЕ ЗАДАНИЯ ДЛЯ ПРОВЕДЕНИЯ ИТОГОВОЙ ГОСУДАРСТВЕННОЙ АТТЕСТАЦИИ РЕЗИДЕНТОВ

ВЫПУСКА 2012-2013 ГГ. ПО СПЕЦИАЛЬНОСТИ «АКУШЕРСТВО И ГИНЕКОЛОГИЯ»

растяжения на животе, бедрах. Имеется рост волос на верхней губе, вокруг околососковых кружков, по средней линии живота. Вагинальное исследование: матка нормальных размеров, яичники увеличены с обеих сторон, плотной консистенции, подвижные. Наиболее вероятен диагноз:А) адреногенитальный синдромБ) синдром Шершевского - ТернераВ) синдром Штейна-Левенталя Г) синдром ШиханаД) СПКЯ центрального генеза330. Пациентке 20 лет. Обратилась к гинекологу в связи с редкими месячными, ростом волос на верхней губе, подбородке с 15 лет. В течение года регулярной половой жизни без предохранения не беременеет. В анамнезе у сестры также нерегулярный менструальный цикл. Менархе с 16 лет, месячные через 2-3 месяца. Рост 162 см, масса тела – 52 кг. Молочные железы слабо развиты, выделений из сосков нет. Вокруг ореол – стержневые волосы, по белой линии живота – рост волос. PV: матка маленькая, правые придатки не определяются; слева – пальпируется яичник размером 4,5 х 3,0 х 2,5см, безболезненный. Поставьте предварительный диагноз:А) синдром поликистозных яичников на фоне надпочечниковой дисфункцииБ) гормонопродуцирующая опухоль яичникаВ) синдром Штейна-ЛевенталяГ) синдром поликистозных яичников центрального генезаД) нарушение менструального цикла, гирсутизм неясного генеза331. При синдроме поликистоза яичников на фоне надпочечниковой дисфункции назначается:А)гормонотерапия дексаметазономБ) клиновидная резекция яичниковВ) КОК в режиме контрацепцииГ) дюфастонД) АКТГ332. Пациентка С., 38 лет, обратилась в связи с прекращением месячных. Последняя менструация 1,5 года назад. Наряду с отсутвием месячных беспокоят приливы до 15 раз в сутки, плохой сон, потливость. Месячные с 13 лет были регулярными, умеренными, безболезненными до 36 лет. Замужем, имела 5 беременностей, из них 2 родов в срок, 3 медицинских аборта. Беременности и роды протекали без особенностей. Объективно: соматически здорова, рост 160см, масса – 62кг. Молочные железы мягкие, без патологических выделений из сосков. При гинекологическом осмотре патологии не выявлено. Поставьте предварительный диагноз: А) синдром истощенных яичниковБ) ранняя менопаузаВ) синдром резистентных яичниковГ) ранний климаксД) синдром Шихана333. Наиболее вероятные клинические признаки истинного абсцесса бартолиновой железы: а) температура тела до 39-40°С, ознобы, флюктуация, увеличенная, болезненная железа б) температура тела до 39-40°С, озноб, болезненное образование в толще верхней трети большой половой губы в) флюктуация в области увеличенной и болезненной бартолиновой железы г) болезненное образование в толще верхней трети большой половой губы д) температура тела до 39-40°С, безболезненное образование в толще верхней трети большой половой губы

Page 62: postgraduate.kaznmu.kz€¦ · Web view58 Больная И, 28 лет, доставлена с жалобами на внезапно возникшую боль внизу живота

С.Ж.АСФЕНДИЯРОВАТЫНДАҒЫҚАЗАҚ ҰЛТТЫҚ МЕДИЦИНА

УНИВЕРСИТЕТІ

КАЗАХСКИЙНАЦИОНАЛЬНЫЙМЕДИЦИНСКИЙ УНИВЕРСИТЕТ ИМЕНИ С.Д.АСФЕНДИЯРОВА

КАФЕДРА ИНТЕРНАТУРЫ И РЕЗИДЕНТУРЫ ПО АКУШЕРСТВУ И ГИНЕКОЛОГИИТЕСТОВЫЕ ЗАДАНИЯ ДЛЯ ПРОВЕДЕНИЯ ИТОГОВОЙ ГОСУДАРСТВЕННОЙ АТТЕСТАЦИИ РЕЗИДЕНТОВ

ВЫПУСКА 2012-2013 ГГ. ПО СПЕЦИАЛЬНОСТИ «АКУШЕРСТВО И ГИНЕКОЛОГИЯ»

334. К моменту окончания антибиотикотерапии у больных с обострением хронического сальпингоофорита наименее вероятно отмечается: а) нормализация температуры тела б) улучшение субъективного состояния в) углубление иммунодефицита г) снижение гемоглобинад) снижение лейкоцитов335. У больных с хламидийным цервицитом (вне беременности) не используется:а) доксициклин б) эритромицинв) сумамед г) ампициллин д) тетрациклин336. К какой степени дисплазии относятся следующие изменения: маловыраженные нарушения дифференцировки эпителия, умеренная пролиферация базального слоя, сохранена структура и полярность расположения клеток:А) дисплазия легкой степениБ) дисплазия умеренной степениВ) дисплазия тяжелой степениГ) преинвазивный ракД) аденоакантома337. У больной 45 лет имеются рецидивирующие кровотечения в течение одного года. Ей следует назначить:А) раздельное диагностическое выскабливание полости маткиБ) оперативное лечениеВ) гормональное лечениеГ) симптоматическое лечениеД) аспирационная биопсия338. Больная 34 лет, менструальный цикл, овуляторный, за 1-2-3 дня до менструации ржаво-коричневые выделения. Тип нарушения менструального цикла:А) метроррагияБ) меноррагияВ) нормальный циклГ) гипоменореяД) дисменорея339. При обследовании больных выявлены: рН вагинального отделяемого более 4,5, гиперемия слизистой влагалища, положительный аминный тест, "ключевые клетки" при микроскопическом исследовании вагинального отделяемого. Наиболее вероятный диагноз:А) бактериальный вагинозБ) неспецифический вагинитВ) кандидозный вагинитГ) трихомонадный кольпитД) вагинит, вызванный инородным телом во влагалище340. Больная 20 лет заболела остро с повышением температуры до 38,4 ºС . Начало заболевания совпало с началом месячных. Беспокоят резкие боли внизу живота. При специальном осмотре обращает на себя внимание резкая болезненность придатков с обеих сторон, матка нормальной величины, инфильтратов нет. Наиболее вероятно, что у больной:А) острое воспаление придатков матки

Page 63: postgraduate.kaznmu.kz€¦ · Web view58 Больная И, 28 лет, доставлена с жалобами на внезапно возникшую боль внизу живота

С.Ж.АСФЕНДИЯРОВАТЫНДАҒЫҚАЗАҚ ҰЛТТЫҚ МЕДИЦИНА

УНИВЕРСИТЕТІ

КАЗАХСКИЙНАЦИОНАЛЬНЫЙМЕДИЦИНСКИЙ УНИВЕРСИТЕТ ИМЕНИ С.Д.АСФЕНДИЯРОВА

КАФЕДРА ИНТЕРНАТУРЫ И РЕЗИДЕНТУРЫ ПО АКУШЕРСТВУ И ГИНЕКОЛОГИИТЕСТОВЫЕ ЗАДАНИЯ ДЛЯ ПРОВЕДЕНИЯ ИТОГОВОЙ ГОСУДАРСТВЕННОЙ АТТЕСТАЦИИ РЕЗИДЕНТОВ

ВЫПУСКА 2012-2013 ГГ. ПО СПЕЦИАЛЬНОСТИ «АКУШЕРСТВО И ГИНЕКОЛОГИЯ»

Б) острый аппендицитВ) разрыв кисты яичникаГ) острое воспаление придатков гонорейной этиологииД) туберкулезное воспаление придатков матки341. В яичниках наименее вероятно продуцируются:А) эстрогеныБ) андрогеныВ) прогестеронГ) ингибинД) пролактин342. При проведении у больной с аменореей гормональной пробы с эстрогенами и гестагенами отрицательный результат её (отсутствие менструальной реакции) свидетельствует: а) о наличии аменореи центрального генеза б) о яичниковой форме аменореи в) о маточной форме аменореи г) о наличии синдрома склерокистозных яичников д) об аменореи надпочечникового генеза343. Истинная (патологическая) аменорея наименее вероятно обусловлена:А) гипотиреозомБ) дисфункцией надпочечниковВ) гиперпролактинемиейГ) атрезией девственной плевыД) синдром Шерешевского- Тернера344. Больная 28 лет обратилась с жалобами на увеличение массы тела на 18кг, редкие месячные. Данные симптомы развились после родов. Менструальная функция до родов носила нормальный характер. Роды 2 года тому назад осложнились послеродовым эндометритом. Объективно: ИМТ- 40, ожирение по центральному типу- жировая клетчатка располагается в верхней части туловища, на животе. Стрии на молочных железах, животе, конечности худые, незначительный гирсутизм, АД-150/90 мм.рт.ст. при PV: патология не выявлена из- за ожирения матка и придатки не определяются. Представленные данные позволяют предположить диагноз:А)послеродовый эндокринный синдромБ) синдром ШиханаВ) синдром Рокитанского-КюстнераГ) синдром Киари-ФроммеляД) синдром резистентных яичников345. Больная 28 лет обратилась с жалобами на общую слабость, легкую утомляемость, похудание, отсутствие месячных в течение года. Роды 2 года назад с кровопотерей более 2 л. Объективно: астеническое телосложение, АД-90/60 мм рм.ст. При гинекологическом исследовании выявлена: выраженная сухость слизистой влагалища, матка меньше нормы, придатки не определяются. Перечисленные данные позволяют предположить диагноз:А) синдром Иценко-КушингаБ) синдром ШиханаВ) синдром Рокитанского-Кюстнера Г) синдром Киари-ФроммеляД) дизгенезия гонад346. Яичниковая аменорея является первичной при:А) дизгенезии гонадБ) истощенных яичников

Page 64: postgraduate.kaznmu.kz€¦ · Web view58 Больная И, 28 лет, доставлена с жалобами на внезапно возникшую боль внизу живота

С.Ж.АСФЕНДИЯРОВАТЫНДАҒЫҚАЗАҚ ҰЛТТЫҚ МЕДИЦИНА

УНИВЕРСИТЕТІ

КАЗАХСКИЙНАЦИОНАЛЬНЫЙМЕДИЦИНСКИЙ УНИВЕРСИТЕТ ИМЕНИ С.Д.АСФЕНДИЯРОВА

КАФЕДРА ИНТЕРНАТУРЫ И РЕЗИДЕНТУРЫ ПО АКУШЕРСТВУ И ГИНЕКОЛОГИИТЕСТОВЫЕ ЗАДАНИЯ ДЛЯ ПРОВЕДЕНИЯ ИТОГОВОЙ ГОСУДАРСТВЕННОЙ АТТЕСТАЦИИ РЕЗИДЕНТОВ

ВЫПУСКА 2012-2013 ГГ. ПО СПЕЦИАЛЬНОСТИ «АКУШЕРСТВО И ГИНЕКОЛОГИЯ»

В) нарушении функции гипоталамических структур под влиянием тяжелых инфекцийГ) после выскабливания слизистой полости маткиД) после удаления двусторонних цистаденом347.Для уточнения генеза вторичной яичниковой аменореи в первую очередь необходимо произвести:А)УЗИ органов малого тазаБ) определение гонадотропных гормоновдиолаВ) метросальпингографиюГ) определение уровня андрогеновД) кольпоскопию348.При синдроме Киари-Фроммеля отмечена повышенная продукция:А) ФСГ и ЛГБ) эстрогеновВ) тестостеронаГ) пролактинаД) прогестерона349. Для первичных поликистозных яичников характерны следующие клинические симптомы:А) начало месячных своевременно, олигоаменорея, первичное бесплодие, умеренный гирсутизмБ) позднее начало месячных, олигоаменорея, выраженный гирсутизм, вторичное бесплодиеВ) начало заболевания после инфекций, стресса, вторичное бесплодие, гирсутизм, ожирениеГ) нарушение месячных после потери веса, вторичная аменореяД) галакторея, ожирение, головные боли, аменорея350. При трансвагинальном УЗИ выявляется двустороннее увеличение яичников, объем их более 9*3 см, наличие мелких кист по периферии диаметром от 3 до 8 мм. Перечисленные особенности структуры яичников характерны для:А) двусторонних функциональных кист яичниковБ) поликистоза яичниковВ) мелкокистозной дегенерации яичниковГ) хронического оофоритаД) эндометриоза яичников

351. При эндометриозе гистеросальпингографию проводят:во время овуляциина 10-11 день менструального цикла на 5-7 день менструального цикла во II фазу менструального цикланезависимо от фазы менструального цикла

352. Оптимальная диагностическая тактика при подозрении на внутренний эндометриозгистероскопия с диагностическим выскабливанием эндометрия гистеросальпингография во II фазу менструального циклалапароскопиякомпьютерная томографиядиагностическое выскабливание эндометрия

353. Показания к хирургическому лечению больных внутренним эндометриозомальгодисменореядиспареунияотсутствие положительного эффекта от консервативной терапии в течение 2 месяцев

Page 65: postgraduate.kaznmu.kz€¦ · Web view58 Больная И, 28 лет, доставлена с жалобами на внезапно возникшую боль внизу живота

С.Ж.АСФЕНДИЯРОВАТЫНДАҒЫҚАЗАҚ ҰЛТТЫҚ МЕДИЦИНА

УНИВЕРСИТЕТІ

КАЗАХСКИЙНАЦИОНАЛЬНЫЙМЕДИЦИНСКИЙ УНИВЕРСИТЕТ ИМЕНИ С.Д.АСФЕНДИЯРОВА

КАФЕДРА ИНТЕРНАТУРЫ И РЕЗИДЕНТУРЫ ПО АКУШЕРСТВУ И ГИНЕКОЛОГИИТЕСТОВЫЕ ЗАДАНИЯ ДЛЯ ПРОВЕДЕНИЯ ИТОГОВОЙ ГОСУДАРСТВЕННОЙ АТТЕСТАЦИИ РЕЗИДЕНТОВ

ВЫПУСКА 2012-2013 ГГ. ПО СПЕЦИАЛЬНОСТИ «АКУШЕРСТВО И ГИНЕКОЛОГИЯ»

внутреннийэндометриоз в сочетании с предраком эндометрияретроцервикальныйэндометриоз

354. Малые формы эндометриоза диагностируются при проведении гистеросальпингографиидиагностического выскабливаниялапароскопиигистероскопииУЗИ

355. Характерным признаком внутреннего эндометриоза, выявляемый при метросальпингографии, наиболее вероятно является:

увеличение полости маткидеформация полости маткиналичие "дефектов наполнения"наличие «законтурных теней»ригидность маточных труб

356. Наименее характерный симптом перитонеального эндометриоза нарушения менструальной функциидиспареунияболи в нижних отделах живота бесплодиеневынашивание

357. Выбор метода лечения эндометриоза НАИМЕНЕЕ всего зависит от

возраста пациентокзаинтересованности в беременностилокализации эндометриоидного процессатяжести клинического течениячувствительности ткани к гормональной терапии

358. Для профилактики эндометриоза шейки матки диатермокоагуляцию с целью лечения эрозии следует проводить

на 5-7 день циклаво второй половине цикла вне зависимости от циклав середине циклапосле менструации

359. Для профилактики эндометриоза шейки матки лазерную вапоризацию или криодеструкцию с целью лечения эрозии следует проводить

на 5-7 день циклаво второй половине цикла вне зависимости от циклав середине циклапосле менструации

360. Какому заболеванию наиболее вероятно соответствует данная лапароскопическая картина?

Page 66: postgraduate.kaznmu.kz€¦ · Web view58 Больная И, 28 лет, доставлена с жалобами на внезапно возникшую боль внизу живота

С.Ж.АСФЕНДИЯРОВАТЫНДАҒЫҚАЗАҚ ҰЛТТЫҚ МЕДИЦИНА

УНИВЕРСИТЕТІ

КАЗАХСКИЙНАЦИОНАЛЬНЫЙМЕДИЦИНСКИЙ УНИВЕРСИТЕТ ИМЕНИ С.Д.АСФЕНДИЯРОВА

КАФЕДРА ИНТЕРНАТУРЫ И РЕЗИДЕНТУРЫ ПО АКУШЕРСТВУ И ГИНЕКОЛОГИИТЕСТОВЫЕ ЗАДАНИЯ ДЛЯ ПРОВЕДЕНИЯ ИТОГОВОЙ ГОСУДАРСТВЕННОЙ АТТЕСТАЦИИ РЕЗИДЕНТОВ

ВЫПУСКА 2012-2013 ГГ. ПО СПЕЦИАЛЬНОСТИ «АКУШЕРСТВО И ГИНЕКОЛОГИЯ»

Внутренний эндометриозЭкстрагенитальный эндометриозМетастазы злокачественной меланомыГемосидероз органов малого тазаНаружный эндометриоз

361. Для препарата ДАНАЗОЛ, используемого для лечения эндометриоза, наименее всего подходит следующее описание

ингибитор гонадотропиновувеличивает в крови уровень глобулина, связывающего половые стероидыувеличивает концентрацию свободного тестостерона в организметормозит стероидогенез в яичникахподавляет пролиферативную и секреторную активность эндометриальных желез

362. Непременное условие эффективности хирургического лечения эндометриоза – применение после операции

импульсного магнитного полярадоновых водферментных препаратовгормонотерапии электрофореза йода и цинка

363. Какие средства контрацепции следует избегать женщинам с эндометриозоминъекционные контрацептивы внутриматочная гормональная система Миренакомбинированные оральные контрацептивычистые гестагенывнутриматочные спирали

364. Оптимальные средства контрацепции для женщин с эндометриозомритмические спермициды барьерныегормональныевнутриматочные

365. При сочетании эндометриоза и гиперпластических процессов в молочных железах наименее всего показано

совместное с маммологом ведение больных применение тамоксифенаисключение препаратов, содержащих эстрогеныкомплексное лечениехирургическое лечение

366. Для лечения наружного эндометриоза более эффективно использоватьгормональные препараты

Page 67: postgraduate.kaznmu.kz€¦ · Web view58 Больная И, 28 лет, доставлена с жалобами на внезапно возникшую боль внизу живота

С.Ж.АСФЕНДИЯРОВАТЫНДАҒЫҚАЗАҚ ҰЛТТЫҚ МЕДИЦИНА

УНИВЕРСИТЕТІ

КАЗАХСКИЙНАЦИОНАЛЬНЫЙМЕДИЦИНСКИЙ УНИВЕРСИТЕТ ИМЕНИ С.Д.АСФЕНДИЯРОВА

КАФЕДРА ИНТЕРНАТУРЫ И РЕЗИДЕНТУРЫ ПО АКУШЕРСТВУ И ГИНЕКОЛОГИИТЕСТОВЫЕ ЗАДАНИЯ ДЛЯ ПРОВЕДЕНИЯ ИТОГОВОЙ ГОСУДАРСТВЕННОЙ АТТЕСТАЦИИ РЕЗИДЕНТОВ

ВЫПУСКА 2012-2013 ГГ. ПО СПЕЦИАЛЬНОСТИ «АКУШЕРСТВО И ГИНЕКОЛОГИЯ»

хирургическое лечение - лапаротомиюхирургическое лечение - лапароскопиюкомбинированное лечение – сочетание лапароскопии и гормонотерапиикомбинированное лечение – сочетание лапаротомии и гормонотерапии

367. У женщин, страдающих бесплодием, обусловленным эндометриозом, первым этапом лечения является

гормонотерапия хирургическое лечение – лапаротомия с удалением очагов эндометриозахирургическое лечение – лапароскопия с лазеровапоризацией очагов эндометриозафизиолечениевведение внутриматочного гормонального контрацептива Мирена

368. Термин «аденомиоз» правомочен только при узловой форме внутреннего эндометриозапри наличии гиперплазии и гипертрофии мышечных волоконпри внутреннем эндометриозе 2-3 степенипри атипической гиперплазии эндометрияпри наличии гипер/полименореи

369. Для морфологической картины аденомиоза наименее характерностромальный компонент превалирует над железистым эпителием эпителий характеризуется низкой рецепторной активностьюв эпителии отсутствуют секреторные преобразования железистый эпителий превалирует над стромальным компонентомжелезистый эпителий аденомиоза развивается из базальной мембраны

370. Эндометриоз, сопровождающийся гиперплазией и гипертрофией мышечных волокон матки

внутренний эндометриозузловая форма внутреннего эндометриозааденомиозаденоматозаденомиома

371. Наименее подходящее описание для 1-ой степени распространения внутреннего эндометриоза на современном этапе

является гистологической находкой при изучении микропрепаратов удаленной маткиклинические проявления не характерныэхографические признаки не характернынеобходимо назначение гормональной терапиигистологическая особенность матки – ретракция мышечных пучков после родов и/или внутриматочных вмешательств

372. В каком возрастном периоде чаще всего выявляется аденомиозв ювенильномв репродуктивномв позднем репродуктивном и пременопаузев перименопаузев менопаузе

373. Наименее характерный признак для аденомиозагипер/полименореявторичная железодефицитная анемияболи в нижних отделах живота накануне и в первые дни менструации

Page 68: postgraduate.kaznmu.kz€¦ · Web view58 Больная И, 28 лет, доставлена с жалобами на внезапно возникшую боль внизу живота

С.Ж.АСФЕНДИЯРОВАТЫНДАҒЫҚАЗАҚ ҰЛТТЫҚ МЕДИЦИНА

УНИВЕРСИТЕТІ

КАЗАХСКИЙНАЦИОНАЛЬНЫЙМЕДИЦИНСКИЙ УНИВЕРСИТЕТ ИМЕНИ С.Д.АСФЕНДИЯРОВА

КАФЕДРА ИНТЕРНАТУРЫ И РЕЗИДЕНТУРЫ ПО АКУШЕРСТВУ И ГИНЕКОЛОГИИТЕСТОВЫЕ ЗАДАНИЯ ДЛЯ ПРОВЕДЕНИЯ ИТОГОВОЙ ГОСУДАРСТВЕННОЙ АТТЕСТАЦИИ РЕЗИДЕНТОВ

ВЫПУСКА 2012-2013 ГГ. ПО СПЕЦИАЛЬНОСТИ «АКУШЕРСТВО И ГИНЕКОЛОГИЯ»

увеличение размеров маткинормальные размеры матки

374. Ведущий акустический критерий аденомиоза при УЗИаномальные кистозные полости в миометрии при нормальных размерах маткиокруглость формы увеличенной матки«мелкоячеистость» миометрия при нормальных размерах маткиуменьшение передне - заднего размера маткиизменение формы матки до шаровидной при неизмененных размерах матки

375. НАИМЕНЕЕ информативный эндоскопический критерий аденомиоза при проведении гистероскопии

деформация увеличенной полости маткинарушение рельефа стенок полости матки (грубый складчатый характер базальной мембраны) – симптом «булыжной мостовой», или «феномен волнообразования». деформация полости матки узлами с желтым или бледно-желтым оттенком, без четких границналичие поверхностных эндометриоидных «глазков»«пчелиные соты» - устья эндометриоидных ходов, открывающиеся в просвет полости матки

376. Какая из теорий патогенеза эндометриоза признана несостоятельнойимплантационная теорияметапластическая теориятеория эмбрионального происхожденияконцепция принадлежности гетеротопий к опухолямряд иммунологических концепций

377. «Малые» формы эндометриозаединичные гетеротопии на тазовой брюшине и яичниках без наличия спаечных и рубцовых процессовгетеротопии на поверхности одного или обоих яичников с образованием мелких кистналичие периовариального спаечного процессаналичие перитубарного спаечного процессагетеротопии на брюшине прямокишечно-маточного пространства со смещением матки

378. Наименее характерная причина бесплодия при эндометриозеановуляциянеполноценная секреторная фазаспаечный процесс в малом тазу фагоцитоз сперматозоидовистощение оогенеза

379. Наименее характерное нарушение иммунного равновесия при эндометриозеТ – клеточный иммунодефицитугнетение функции Т - супрессоровактивация В – лимфоцитарной системыувеличение функции естественных киллеровснижение функции естественных киллеров

380. Какой гестаген находится в капсуле внутриматочной гормональной системы «Мирена»

диеногестдезогестреллевоноргестрелгестоденципротерон ацетат

Page 69: postgraduate.kaznmu.kz€¦ · Web view58 Больная И, 28 лет, доставлена с жалобами на внезапно возникшую боль внизу живота

С.Ж.АСФЕНДИЯРОВАТЫНДАҒЫҚАЗАҚ ҰЛТТЫҚ МЕДИЦИНА

УНИВЕРСИТЕТІ

КАЗАХСКИЙНАЦИОНАЛЬНЫЙМЕДИЦИНСКИЙ УНИВЕРСИТЕТ ИМЕНИ С.Д.АСФЕНДИЯРОВА

КАФЕДРА ИНТЕРНАТУРЫ И РЕЗИДЕНТУРЫ ПО АКУШЕРСТВУ И ГИНЕКОЛОГИИТЕСТОВЫЕ ЗАДАНИЯ ДЛЯ ПРОВЕДЕНИЯ ИТОГОВОЙ ГОСУДАРСТВЕННОЙ АТТЕСТАЦИИ РЕЗИДЕНТОВ

ВЫПУСКА 2012-2013 ГГ. ПО СПЕЦИАЛЬНОСТИ «АКУШЕРСТВО И ГИНЕКОЛОГИЯ»

381. СА-125, как маркер эндометриоза, определяется в плазме кровив перитонеальной жидкостив плазме крови и перитонеальной жидкостив вагинальном секрете в аспирате из полости матки

382. Наименее характерные иммунные изменения при эндометриозеприсутствие антиспермальных антител в плазме крови и/или различных секретах репродуктивного трактаповышение концентрации макрофагов с высокой антиспермальной активностью в перитонеальной жидкостиаутоиммунные процессы за счет резорбции продуктов метаболизма функционирующих эндометриоидных гетеротопийувеличение содержания тромбоксана в перитонеальной жидкостиуменьшение содержания простагландинов в перитонеальной жидкости

383. Единственным клиническим проявлением «малых форм» эндометриоза часто является

альгодисменореягиперполименореябесплодиедиспареуниядисхезия

384. Эндометриоз шейки матки наименее характерен после: абортов диатермокоагуляции шейки матки гистеросальпингографии родов, осложненных разрывом шейки матки физиологических родов385. Наименее характерное изменение для аденомиоза

гиперплазия мышечной ткани матки равномерное увеличение размеров матки накануне менструации неравномерное увеличение размеров матки накануне менструации образование в миометрии плотных узлов, окруженных капсулой распространение эндометриоидной ткани на всю толщу миометрия

386. Для эндометриоидных кист яичника наименее характерны следующие клинические симптомы

резкие боли в низу живота прогрессирующая альгоменореянарушение функции кишечника нарушение функции мочевого пузыря гиперполименорея

387. Условия, обеспечивающие информативность метросальпингографии в целях диагностики внутреннего зндометриоза тела матки

применение только жирорастворимого контрастного раствора"тугое" заполнение полости матки контрастным раствором проведение исследования во вторую фазу менструального цикла проведение исследования в первую фазу менструального циклапроведение исследования в любой день менструального цикла

Page 70: postgraduate.kaznmu.kz€¦ · Web view58 Больная И, 28 лет, доставлена с жалобами на внезапно возникшую боль внизу живота

С.Ж.АСФЕНДИЯРОВАТЫНДАҒЫҚАЗАҚ ҰЛТТЫҚ МЕДИЦИНА

УНИВЕРСИТЕТІ

КАЗАХСКИЙНАЦИОНАЛЬНЫЙМЕДИЦИНСКИЙ УНИВЕРСИТЕТ ИМЕНИ С.Д.АСФЕНДИЯРОВА

КАФЕДРА ИНТЕРНАТУРЫ И РЕЗИДЕНТУРЫ ПО АКУШЕРСТВУ И ГИНЕКОЛОГИИТЕСТОВЫЕ ЗАДАНИЯ ДЛЯ ПРОВЕДЕНИЯ ИТОГОВОЙ ГОСУДАРСТВЕННОЙ АТТЕСТАЦИИ РЕЗИДЕНТОВ

ВЫПУСКА 2012-2013 ГГ. ПО СПЕЦИАЛЬНОСТИ «АКУШЕРСТВО И ГИНЕКОЛОГИЯ»

388. Для аденомиоза наименее характерногиперплазия мышечной ткани маткиравномерное увеличение размеров матки накануне менструациинеравномерное увеличение размеров матки накануне менструациираспространение эндометриоидной ткани на всю толщу миометрияобразование в миометрии плотных узлов, окруженных капсулой

389. Эндометриоидную кисту яичника менее всего необходимо дифференцироватьс воспалительным образованием придатков матки с кистомой яичника с субсерозной миомой матки с метастазом Крукенбергас внутренним эндометриозом

390. Наименее оправданное показание к хирургическому лечению у больных эндометриозом

неэффективность консервативного лечения при ретроцервикальном эндометриозеналичие эндометриоидной кисты яичника эндометриоз послеоперационного рубца передней брюшной внутренний эндометриоз тела матки II степени подозрение на истинный бластоматозный процесс в яичнике

391. Термин аденомиоз применяетсяво всех случаях выявления эндометриоза независимо от локализации только при очаговых разрастаниях эндометриоидной ткани во внутреннем слое матки при эндометриозе, который сопровождается образованием кист только в тех случаях, когда прорастание миометрия сопровождается гиперплазией мышечной ткани только при ретроцервикальном эндометриозе

392. Для диагностики внутреннего эндометриоза методом гистеросальпингографии наиболее благоприятными являются следующие дни менструального цикла

за 1-2 дня до начала менструации сразу после окончания менструации на 12-14-й день на 16-18-й денвна 20-22-й день

393. Для профилактики развития эндометриоза шейки матки диатермокоагуляцию псевдоэрозии проводят в следующие дни менструального цикла

за 1-2 дня до начала менструации сразу после окончания менструации на 12-14-й день на 16-18-й день на 20-22-й день

394. Для профилактики развития эндометриоза шейки матки криодеструкцию проводят в следующие дни менструального цикла

за 1-2 дня до начала менструации сразу после менструации на 12-14-й день на 16-18-й день на 20-22-й день

395. При эндометриозе гистеросальпингографию проводят:

Page 71: postgraduate.kaznmu.kz€¦ · Web view58 Больная И, 28 лет, доставлена с жалобами на внезапно возникшую боль внизу живота

С.Ж.АСФЕНДИЯРОВАТЫНДАҒЫҚАЗАҚ ҰЛТТЫҚ МЕДИЦИНА

УНИВЕРСИТЕТІ

КАЗАХСКИЙНАЦИОНАЛЬНЫЙМЕДИЦИНСКИЙ УНИВЕРСИТЕТ ИМЕНИ С.Д.АСФЕНДИЯРОВА

КАФЕДРА ИНТЕРНАТУРЫ И РЕЗИДЕНТУРЫ ПО АКУШЕРСТВУ И ГИНЕКОЛОГИИТЕСТОВЫЕ ЗАДАНИЯ ДЛЯ ПРОВЕДЕНИЯ ИТОГОВОЙ ГОСУДАРСТВЕННОЙ АТТЕСТАЦИИ РЕЗИДЕНТОВ

ВЫПУСКА 2012-2013 ГГ. ПО СПЕЦИАЛЬНОСТИ «АКУШЕРСТВО И ГИНЕКОЛОГИЯ»

на 7-8 день менструального цикла на 10-11 день менструального цикла во время овуляцииво вторую фазу менструального циклане зависимо от фазы менструального цикла

396. При "малых" формах эндометриоза размер эндометриоидных гетеротопий не превышает

0.4 см 0.5 см 0.6 см 0.7 см 0.8 см

397. Влияние беременности на развитие эндометриозавызывает увеличение эндометриоидных гетеротопий в первые 2-3 месяца, а затем наступает обратное развитие, особенно заметное в послеродовом периоде у лактирующих женщин вызывает увеличение эндометриоидных гетеротопий в послеродовом периоде у лактирующих женщин активизируется развитие эндометриоза на протяжении всей беременности вызывает увеличение эндометриоидных гетеротопий начиная со второго триместра беременностивызывает увеличение эндометриоидных гетеротопий начиная с третьего триместра беременности

398. О сочетанной локализации внутреннего эндометриоза тела матки можно говорить, если имеется его локализация

корпоральная и истмическаяистмическая и яичники яичники и корпоральнаякорпоральная и шейка матки истмическая и шейка матки

399. Выраженность альгоменореи у больных с внутренним эндометриозом тела матки находится в прямой зависимости

от распространения эндометриозаот возраста женщины от наличия экстрагенитальной патологии от массы женщины от роста женщины

400. Болевой симптом наименее характерен при следующей локализации эндометриозатело матки яичники шейка матки маточные трубы ретроцервикальная область

401. Тип нарушения менструального цикла при эндометриозе метроррагия опсоменорея олигоменорея гипоменорея менометроррагия402. Для больных с эндометриозом яичников наименее характерно

Page 72: postgraduate.kaznmu.kz€¦ · Web view58 Больная И, 28 лет, доставлена с жалобами на внезапно возникшую боль внизу живота

С.Ж.АСФЕНДИЯРОВАТЫНДАҒЫҚАЗАҚ ҰЛТТЫҚ МЕДИЦИНА

УНИВЕРСИТЕТІ

КАЗАХСКИЙНАЦИОНАЛЬНЫЙМЕДИЦИНСКИЙ УНИВЕРСИТЕТ ИМЕНИ С.Д.АСФЕНДИЯРОВА

КАФЕДРА ИНТЕРНАТУРЫ И РЕЗИДЕНТУРЫ ПО АКУШЕРСТВУ И ГИНЕКОЛОГИИТЕСТОВЫЕ ЗАДАНИЯ ДЛЯ ПРОВЕДЕНИЯ ИТОГОВОЙ ГОСУДАРСТВЕННОЙ АТТЕСТАЦИИ РЕЗИДЕНТОВ

ВЫПУСКА 2012-2013 ГГ. ПО СПЕЦИАЛЬНОСТИ «АКУШЕРСТВО И ГИНЕКОЛОГИЯ»

сохраненный двухфазный менструальный цикл боли внизу живота накануне менструации бесплодие альгодисменореяметроррагия

403. Образование спаечного процесса при эндометриоидных кистах связано с: сопутствующим воспалительным процессом микроперфорацией эндометриоидных очагов во время менструации нарушением функции соседних органов расположением очагов между листками широкой связки матки за счет быстрого накопления менструальной крови в очагах404. Больная 28 лет предъявляет жалобы на тянущие боли внизу живота, усиливающиеся

перед и во время менструации, мажущие темные кровянистые выделения до и после менструации, бесплодие в течение 5 лет. В анамнезе один самопроизвольный выкидыш при сроке беременности 5-6 нед. В возрасте 20 лет оперирована по поводу разрыва кисты яичника, произведена резекция правого яичника. РV: шейка матки цилиндрическая, матка в anteflexio, ограничено в подвижности, нормальных размеров, безболезненная, справа и кзади от матки пальпируется опухолевидное образование размером 8,0 х 8,0 см, тугоэластической консистенции, малоподвижное, спаянное с заднебоковой поверхностью матки, умеренно болезненное. Левые придатки не увеличены. Выделения из половых путей слизистые. При лапаротомии опухоль шоколадного цвета, спаяна с окружающими тканями. Диагноз:

трубная беременность справа миома матки с субсерозным расположением узла спаечная болезнь эндометриоидная киста яичника справа эндометриоз яичников405. У больных с эндометриоидными кистами яичников нецелесообразно проведение

следующего дополнительного метода исследованияэкскреторной урографииирригоскопииректороманоскопиикольпоскопиигистероскопии

406. Наиболее характерный признак внутреннего эндометриоза, выявляемый при метросальпингографии

увеличение полости матки деформация полости матки наличие "дефектов наполнения" наличие "законтурных теней" ригидность маточных труб407. Оптимальная длительность лечения больных агонистами гонадолиберина

составляет: 3 месяца 5 месяцев 6 месяцев 9 месяцев 12 месяцев

Page 73: postgraduate.kaznmu.kz€¦ · Web view58 Больная И, 28 лет, доставлена с жалобами на внезапно возникшую боль внизу живота

С.Ж.АСФЕНДИЯРОВАТЫНДАҒЫҚАЗАҚ ҰЛТТЫҚ МЕДИЦИНА

УНИВЕРСИТЕТІ

КАЗАХСКИЙНАЦИОНАЛЬНЫЙМЕДИЦИНСКИЙ УНИВЕРСИТЕТ ИМЕНИ С.Д.АСФЕНДИЯРОВА

КАФЕДРА ИНТЕРНАТУРЫ И РЕЗИДЕНТУРЫ ПО АКУШЕРСТВУ И ГИНЕКОЛОГИИТЕСТОВЫЕ ЗАДАНИЯ ДЛЯ ПРОВЕДЕНИЯ ИТОГОВОЙ ГОСУДАРСТВЕННОЙ АТТЕСТАЦИИ РЕЗИДЕНТОВ

ВЫПУСКА 2012-2013 ГГ. ПО СПЕЦИАЛЬНОСТИ «АКУШЕРСТВО И ГИНЕКОЛОГИЯ»

408. Наименее подходящая локализация для внутреннего генитального эндометриоза

яичники трубы серозный покров прямой и сигмовидной кишки наружные половые органы матка409. Наименее характерные клинические симптомы для внутреннего эндометриоза тела

матки бесплодие мажущие кровянистые выделения до и после менструации гиперполименорея вторичная анемия боли в глубине таза, отдающие в прямую кишку410. Наименее информативные методы исследования для диагностики

ретроцервикального эндометриоза лапароскопию ректороманоскопию биопсию патологических участков в заднем своде влагалища о6зорную рентгенографию брюшной полости УЗИ411. Наименее информативный метод диагностики внутреннего эндометриоза гистеросальпингография ультразвуковое исследование гистероскопия лапароскопия магнитно-резонансная томография412. К спаечному процессу в малом тазу при эндометриоидных кистах яичников не

приводит перфорации эндометриоидных кист с излитием содержимого в брюшную полость асептическое реактивное воспаление в малом тазу нарушения менструального цикла "прорастание" эндометриоза в близлежащие органы сопутствующее хроническое воспаление придатков матки413. Наименее характерные клинические признаки наружного эндометриоза увеличение размеров эндометриоидных образований во вторую фазу менструального цикла повышение температуры тела накануне менструации альгоменорея гиперменорея тазовые боли414. Для перфорации зндометриоидного образования яичников, как правило, характерно боли в животе, признаки раздражения брюшины повышение температуры тела накануне менструации кровянистые выделения из половых путей лейкопения низкая СОЭ415. Для диагностики эндометриоза шейки матки применяют, как правило, следующие

методы исследования

Page 74: postgraduate.kaznmu.kz€¦ · Web view58 Больная И, 28 лет, доставлена с жалобами на внезапно возникшую боль внизу живота

С.Ж.АСФЕНДИЯРОВАТЫНДАҒЫҚАЗАҚ ҰЛТТЫҚ МЕДИЦИНА

УНИВЕРСИТЕТІ

КАЗАХСКИЙНАЦИОНАЛЬНЫЙМЕДИЦИНСКИЙ УНИВЕРСИТЕТ ИМЕНИ С.Д.АСФЕНДИЯРОВА

КАФЕДРА ИНТЕРНАТУРЫ И РЕЗИДЕНТУРЫ ПО АКУШЕРСТВУ И ГИНЕКОЛОГИИТЕСТОВЫЕ ЗАДАНИЯ ДЛЯ ПРОВЕДЕНИЯ ИТОГОВОЙ ГОСУДАРСТВЕННОЙ АТТЕСТАЦИИ РЕЗИДЕНТОВ

ВЫПУСКА 2012-2013 ГГ. ПО СПЕЦИАЛЬНОСТИ «АКУШЕРСТВО И ГИНЕКОЛОГИЯ»

кольпоскопию, прицельную биопсию шейки матки ректовагинальное исследование гистероскопию диагностическое выскабливание цервикального канала гистеросальпингографию416. Наименее всего эндометриоз шейки матки необходимо дифференцировать с раком шейки матки с эндоцервицитом с эритроплакией с лейкоплакией с псевдозрозией417. Наименее всего эндометриоз тела матки необходимо дифференцировать с подслизистой миомой матки с полипозом эндометрия с маточной беременностью с раком эндометрия с хроническим эндометритом418. Следующие данные гистероскопии свидетельствуют о наличии внутреннего

эндометриоза увеличение полости матки выраженная деформация полости матки наличие полиповидных разрастаний на фоне бледно-розовой слизистой точечные отверстия, из которых выделяется жидкая

кровь бледность стенок полости матки419. Характерными признаками внутреннего эндометриоза, выявляемыми при

метросальпингографии, являются значительное увеличение полости матки выраженная деформация полости матки наличие "дефектов наполнения" наличие "законтурных теней" деформация и нарушение проходимости маточных труб420. Больная,35лет, обратилась в женскую консультацию с жалобами на тупые ноющие

боли внизу живота с иррадиацией в прямую кишку и промежность. Боли усиливаются во время менструации. Вагинально: кзади от матки с двух сторон имеется образование тугоэластической консистенции d=3 см, спаяно с маткой ограничивает ее подвижность. Матка болезненна при пальпации.

Каков предварительный диагноз? пиовар пиосальпинкс апоплексия яичников параовариальная киста яичников эндометриоз яичников421. К наружному генитальному эндометриозу не относится: Аденомиоз Эндометриоз яичников Эндометриоз влагалища Эндометриоз шейки матки

Page 75: postgraduate.kaznmu.kz€¦ · Web view58 Больная И, 28 лет, доставлена с жалобами на внезапно возникшую боль внизу живота

С.Ж.АСФЕНДИЯРОВАТЫНДАҒЫҚАЗАҚ ҰЛТТЫҚ МЕДИЦИНА

УНИВЕРСИТЕТІ

КАЗАХСКИЙНАЦИОНАЛЬНЫЙМЕДИЦИНСКИЙ УНИВЕРСИТЕТ ИМЕНИ С.Д.АСФЕНДИЯРОВА

КАФЕДРА ИНТЕРНАТУРЫ И РЕЗИДЕНТУРЫ ПО АКУШЕРСТВУ И ГИНЕКОЛОГИИТЕСТОВЫЕ ЗАДАНИЯ ДЛЯ ПРОВЕДЕНИЯ ИТОГОВОЙ ГОСУДАРСТВЕННОЙ АТТЕСТАЦИИ РЕЗИДЕНТОВ

ВЫПУСКА 2012-2013 ГГ. ПО СПЕЦИАЛЬНОСТИ «АКУШЕРСТВО И ГИНЕКОЛОГИЯ»

Эндометриоз связок422. Наиболее информативный метод диагностики эндометриоза шейки матки: Кульдоскопия Гистероскопия Лапароскопия Кольпоскопия Метросальпингография423. Наиболее характерный симптом для аденомиоза: Дизурия Метеоризм Френикус симптом Дисменорея Метроррагия424. Часто встречающийся симптом при эндометриозе яичников: Дизурия Метеоризм Френикус симптом Симптом «Острого живота» Метроррагия425. Наиболее информативный метод диагностики внутреннего эндометриоза: Кольпоскопия Пельвиография Пневмоперитонеум Гистероскопия Лапароскопия426. Наиболее оправданное показание для хирургического лечения эндометриоза: Внутрений эндометриоз Эндометриоз яичников Эндометриоз шейки матки Эндометриоз влагалища Ретроцервикальный Эндометриоз427. Малые формы эндометриоза: аденомиоз эндометриоз шейки матки эндометриоидные гетеротопии на тазовой брюшине эндометриоз пупка эндометриоз яичников428. При внутреннем эндометриозе поражается: маточная труба сигмовидная кишка матка шейка матки яичник429. Диагноз внутреннего эндометриоза устанавливается на основании данных: УЗИ ГСГ после менструации лапароскопии пневмопельвиографии

Page 76: postgraduate.kaznmu.kz€¦ · Web view58 Больная И, 28 лет, доставлена с жалобами на внезапно возникшую боль внизу живота

С.Ж.АСФЕНДИЯРОВАТЫНДАҒЫҚАЗАҚ ҰЛТТЫҚ МЕДИЦИНА

УНИВЕРСИТЕТІ

КАЗАХСКИЙНАЦИОНАЛЬНЫЙМЕДИЦИНСКИЙ УНИВЕРСИТЕТ ИМЕНИ С.Д.АСФЕНДИЯРОВА

КАФЕДРА ИНТЕРНАТУРЫ И РЕЗИДЕНТУРЫ ПО АКУШЕРСТВУ И ГИНЕКОЛОГИИТЕСТОВЫЕ ЗАДАНИЯ ДЛЯ ПРОВЕДЕНИЯ ИТОГОВОЙ ГОСУДАРСТВЕННОЙ АТТЕСТАЦИИ РЕЗИДЕНТОВ

ВЫПУСКА 2012-2013 ГГ. ПО СПЕЦИАЛЬНОСТИ «АКУШЕРСТВО И ГИНЕКОЛОГИЯ»

компьютерной томографии430. Для диагностики внутреннего эндометриоза тела матки методом

гистеросальпингографии наиболее благоприятными являются следующие дни менструального цикла:

за 1-2 дня до начала менструации сразу после окончания менструации на 12-14-й день на 16-18-й день на 20-22-й день 431. Для профилактики развития эндометриоза шейки матки электроэксцизию

псевдоэрозии органа проводят в следующие дни менструального цикла: за 1-2 дня до начала менструации сразу после окончания менструации на 12-14-й день на 16-18-й день на 20-22-й день 432. При "малых" формах эндометриоза размер эндометриоидных гетеротопий не

превышает: 0,4 см 0,5 см 0,6 см 0,7 см 0,8 см 433. Характерные особенности, присущие внутреннему генитальному эндометриозу: развитие массивного спаечного процесса в малом тазу образование крупных кист (до 5-6 см в диаметре) атрофия мышечных волокон, окружающих очаги эндометриоза нарушение менструального цикла уменьшение матки434. Для диагностики ретроцервикального эндометриоза используют следующие

дополнительные методы исследования: лапароскопию, ректороманоскопию ректороманоскопию, гистроскопию, кольпоскопию лапароскопию, ректороманоскопию, кольпоскопию обзорную рентгенографию брюшной полости ректороманоскопию, обзорную рентгенографию брюшной полости435. Для аденомиоза наименее характерно: гиперплазия мышечной ткани матки равномерное увеличение размеров матки накануне менструации неравномерное увеличение размеров матки накануне менструации образование в миометрии плотных узлов, окруженных капсулой непропорциональное увеличение передней и задней стенки436. Женщина 38 лет обратилась с жалобами на мажущие кровянистые выделения до и

после очень болезненной менструации в течение последних 6 месяцев. В анамнезе 2 родов и 2 медицинских аборта без осложнений, последний - год назад. Размеры матки до 4 недель беременности. Вероятный диагноз:

миома матки маточная беременность

Page 77: postgraduate.kaznmu.kz€¦ · Web view58 Больная И, 28 лет, доставлена с жалобами на внезапно возникшую боль внизу живота

С.Ж.АСФЕНДИЯРОВАТЫНДАҒЫҚАЗАҚ ҰЛТТЫҚ МЕДИЦИНА

УНИВЕРСИТЕТІ

КАЗАХСКИЙНАЦИОНАЛЬНЫЙМЕДИЦИНСКИЙ УНИВЕРСИТЕТ ИМЕНИ С.Д.АСФЕНДИЯРОВА

КАФЕДРА ИНТЕРНАТУРЫ И РЕЗИДЕНТУРЫ ПО АКУШЕРСТВУ И ГИНЕКОЛОГИИТЕСТОВЫЕ ЗАДАНИЯ ДЛЯ ПРОВЕДЕНИЯ ИТОГОВОЙ ГОСУДАРСТВЕННОЙ АТТЕСТАЦИИ РЕЗИДЕНТОВ

ВЫПУСКА 2012-2013 ГГ. ПО СПЕЦИАЛЬНОСТИ «АКУШЕРСТВО И ГИНЕКОЛОГИЯ»

полипоз эндометрия аденомиоз внематочная беременность437. Наименее оправданная рекомендация для снижения частоты эндометриоза матки: профилактика инструментальных абортов уменьшение частоты диагностических выскабливаний, уменьшение частоты гистеросальпингографий уменьшение частоты инвазивных внутриматочных манипуляций уменьшение частоты родов438. Наименее опасная группа риска по развитию генитального эндометриоза –

пациентки с: климактерическим синдромом воспалением органов малого таза бесплодием рецидивирующими кистами яичников дисменореей439. Наименее характерный фактор риска развития эндометриоза: нереализованная репродуктивная функция короткий интервал между родами нарушения менструальной функции у подростков генетические и семейные факторы «отложенная первая беременность»440. Маркер для диагностики эндометриоза: ВПЧ СА 125 ВПГ ЦМВ 17-КС441. В пременопаузе предпочтительно: назначение монофазных КОКов экстирпация матки ампутация матки физиолечение антигонадотропины442. Для профилактики рецидива эндометриоза не назначают: трехфазные КОКи антигонадотропины агонисты ГнРГ. монофазные КОКи гестагены443. Механизм действия антигонадотропинов: сгущение слизи в цервикальном канале блокада гонадотропной секреции гипофиза, медикаментозная «гонадэктомия» торможение пролиферации децидуализация и атрофия эндометрия торможение пролиферации эндометрия и овуляторного пика гонадотропинов444. Побочное действие антигонадотропинов: андрогензависимая дерматопатия, гиперлипидемия, гипертензия

Page 78: postgraduate.kaznmu.kz€¦ · Web view58 Больная И, 28 лет, доставлена с жалобами на внезапно возникшую боль внизу живота

С.Ж.АСФЕНДИЯРОВАТЫНДАҒЫҚАЗАҚ ҰЛТТЫҚ МЕДИЦИНА

УНИВЕРСИТЕТІ

КАЗАХСКИЙНАЦИОНАЛЬНЫЙМЕДИЦИНСКИЙ УНИВЕРСИТЕТ ИМЕНИ С.Д.АСФЕНДИЯРОВА

КАФЕДРА ИНТЕРНАТУРЫ И РЕЗИДЕНТУРЫ ПО АКУШЕРСТВУ И ГИНЕКОЛОГИИТЕСТОВЫЕ ЗАДАНИЯ ДЛЯ ПРОВЕДЕНИЯ ИТОГОВОЙ ГОСУДАРСТВЕННОЙ АТТЕСТАЦИИ РЕЗИДЕНТОВ

ВЫПУСКА 2012-2013 ГГ. ПО СПЕЦИАЛЬНОСТИ «АКУШЕРСТВО И ГИНЕКОЛОГИЯ»

не обнаружено гиперкоагуляция, задержка жидкости вегетососудистые симптомы увеличение массы тела, гиперлипидемия, задержка жидкости445. К побочным эффектам и осложнениям при применении Мирены не относится: изменение настроения, головные боли, масталгии, тошнота, акне образование функциональных кист яичников, которые, как правило, регрессируют

самостоятельно и не требуют удаления ВМК возможны нарушения менструального цикла циклические маточные кровотечения олиго и аменорея 446. Мирену рекомендуют вводить: с 1го по 3й день менструального цикла с 4го по 6й день менструального цикла с 1 го по 5й день менструального цикла с 5го по 6й день менструального цикла с 2го по 3й день менструального цикла

447. Механизм действия лечебного эффекта Мирены при эндометриозе основан на: подавляется функциональная активность эндометрия: тормозится пролиферация

эндометрия, развивается атрофия эндометриальных желёз, псевдодецидуальная трансформация стромы и сосудистые изменения, что препятствует

имплантации; происходит изменение физикохимических свойств цервикальной слизи что затрудняет

пенетрацию сперматозоидов; уменьшается подвижность сперматозоидов в полости матки и маточных трубах повышается вязкость цервикальной слизи

448. К способам введения спермы при искусственном осеменении не относитсявлагалищныйинтрацервикальныйматочный трубныйтрансабдоминальный в воронку маточной трубы

449. Что не характерно для маточного способа введения спермы при искусственном осеменении

инфицирование полости матки развитие иммунологического конфликта, обусловленного наличием антиспермальных

антителболезненные сокращения маткиболее высокие шансы наступления беременностипростота выполнения450. Сколько осеменений в матку максимум можно провести в течение одного

менструального цикла1 раз 2 разанеограниченное количество раз

Page 79: postgraduate.kaznmu.kz€¦ · Web view58 Больная И, 28 лет, доставлена с жалобами на внезапно возникшую боль внизу живота

С.Ж.АСФЕНДИЯРОВАТЫНДАҒЫҚАЗАҚ ҰЛТТЫҚ МЕДИЦИНА

УНИВЕРСИТЕТІ

КАЗАХСКИЙНАЦИОНАЛЬНЫЙМЕДИЦИНСКИЙ УНИВЕРСИТЕТ ИМЕНИ С.Д.АСФЕНДИЯРОВА

КАФЕДРА ИНТЕРНАТУРЫ И РЕЗИДЕНТУРЫ ПО АКУШЕРСТВУ И ГИНЕКОЛОГИИТЕСТОВЫЕ ЗАДАНИЯ ДЛЯ ПРОВЕДЕНИЯ ИТОГОВОЙ ГОСУДАРСТВЕННОЙ АТТЕСТАЦИИ РЕЗИДЕНТОВ

ВЫПУСКА 2012-2013 ГГ. ПО СПЕЦИАЛЬНОСТИ «АКУШЕРСТВО И ГИНЕКОЛОГИЯ»

3-5 разпо желанию пациентки451. К показаниям для искусственного осеменения спермой мужа (ИОМ) не относитсягипоспадия уретрыэректильная дисфункция вагинизманомалии влагалища азооспермия452. К показаниям для искусственного осеменения спермой донора (ИОД) не относитсялюбые тяжелые нарушения сперматогенеза, не поддающиеся лечению эректильная дисфункция несовместимость супругов по резус-факторунаследственные заболевания у мужа, при которых использование спермы не желательноазооспермия453. Условия для выбора доноров спермы не включают следующие критериивозраст моложе 36 летфизическое и психическое здоровье отсутствие наследственных заболеваний выраженные фенотипические проявлениягруппу крови454. Современный метод лечения тяжелых форм мужского бесплодия, позволяющий

иметь генетически родного ребенка искусственное осеменение спермой донора (ИОД)искусственное осеменение спермой мужа (ИОМ) программа экстракорпорального оплодотворения (ЭКО)программа переноса эмбрионов (ПЭ) расширение метода ЭКО и ПЭ – интрацитоплазматическая инъекция одного сперматозоида

в яйцеклетку (ИКСИ)455. Плодовитость человека в обычных естественных условиях в популяции (%

наступления беременности на одну попытку) не более 10% не более 20% не более 30%не более 40%не более 50%.456. Какая техника применяется для получения отдельных сперматозоидов у мужчин с

некрозооспермией, аспермией, обструктивной азооспермиейиспользуется сперма донора (ИОД - искусственное осеменение спермой донора) прямая транскутанная аспирация сперматозоидов из яичка программа экстракорпорального оплодотворения (ЭКО)программа переноса эмбрионов (ПЭ)расширение метода ЭКО и ПЭ – интрацитоплазматическая инъекция одного сперматозоида в

яйцеклетку (ИКСИ)457. Прямая транскутанная аспирация сперматозоидов из яичка не применяется у мужчин

с азооспермией некрозооспермией аспермией

Page 80: postgraduate.kaznmu.kz€¦ · Web view58 Больная И, 28 лет, доставлена с жалобами на внезапно возникшую боль внизу живота

С.Ж.АСФЕНДИЯРОВАТЫНДАҒЫҚАЗАҚ ҰЛТТЫҚ МЕДИЦИНА

УНИВЕРСИТЕТІ

КАЗАХСКИЙНАЦИОНАЛЬНЫЙМЕДИЦИНСКИЙ УНИВЕРСИТЕТ ИМЕНИ С.Д.АСФЕНДИЯРОВА

КАФЕДРА ИНТЕРНАТУРЫ И РЕЗИДЕНТУРЫ ПО АКУШЕРСТВУ И ГИНЕКОЛОГИИТЕСТОВЫЕ ЗАДАНИЯ ДЛЯ ПРОВЕДЕНИЯ ИТОГОВОЙ ГОСУДАРСТВЕННОЙ АТТЕСТАЦИИ РЕЗИДЕНТОВ

ВЫПУСКА 2012-2013 ГГ. ПО СПЕЦИАЛЬНОСТИ «АКУШЕРСТВО И ГИНЕКОЛОГИЯ»

эректильной дисфункциейобструктивной азооспермией458. Метод ИКСИ (интрацитоплазматическая инъекция одного сперматозоида в

яйцеклетку) не показан приазооспермии неудачных попытках ЭКО и ПЭ без микроманипуляцийнарушениях акросомальной реакции эректильной дисфункцииобструктивной азооспермии459. Метод донации ооцитов используется при поликистозных изменениях яичников синдроме резистентных яичников или преждевременного истощения яичниковтрубном бесплодииэндометриозесиндроме Майера - Кюстнера - Рокитанского460. FISH – анализ (методика флуоресцентной in situ гибридизации) – молекулярно-

цитогенетическое исследование, не применяется для преимплантационной диагностики врожденной хромосомной патологииперинатальной диагностики врожденной хромосомной патологиипостнатальной диагностики врожденной хромосомной патологии (из пуповинной крови) диагностики внутриутробной инфекции

диагностики врожденной хромосомной патологии у взрослых (из крови)461. FISH – анализ (методика флуоресцентной in situ гибридизации) не используется для

диагностикигемофилии А и Вмиопатии Дюшена врожденной ЦМВ инфекциисиндрома Даунасиндрома Патау462. FISH – анализ (методика флуоресцентной in situ гибридизации) с целью

преимплантационной генетической диагностики проводят с тканью хориона с полярными тельцами и бластомерами эмбрионас тканью плацентыс амниотической жидкостьюс кровью плода463. Родина ЭКО СШАФранция АнглияРоссияНидерланды464. Абсолютное показание для донации ооцитов поликистоз яичниковэндометриозпременопауза истощение оогенезабесплодие неясного генеза

Page 81: postgraduate.kaznmu.kz€¦ · Web view58 Больная И, 28 лет, доставлена с жалобами на внезапно возникшую боль внизу живота

С.Ж.АСФЕНДИЯРОВАТЫНДАҒЫҚАЗАҚ ҰЛТТЫҚ МЕДИЦИНА

УНИВЕРСИТЕТІ

КАЗАХСКИЙНАЦИОНАЛЬНЫЙМЕДИЦИНСКИЙ УНИВЕРСИТЕТ ИМЕНИ С.Д.АСФЕНДИЯРОВА

КАФЕДРА ИНТЕРНАТУРЫ И РЕЗИДЕНТУРЫ ПО АКУШЕРСТВУ И ГИНЕКОЛОГИИТЕСТОВЫЕ ЗАДАНИЯ ДЛЯ ПРОВЕДЕНИЯ ИТОГОВОЙ ГОСУДАРСТВЕННОЙ АТТЕСТАЦИИ РЕЗИДЕНТОВ

ВЫПУСКА 2012-2013 ГГ. ПО СПЕЦИАЛЬНОСТИ «АКУШЕРСТВО И ГИНЕКОЛОГИЯ»

465. К осложнениям программы экстракорпорального оплодотворения (ЭКО) и переноса эмбрионов (ПЭ) не относится

внематочная беременностьсиндром гиперстимуляции яичниковмногоплодная беременностьувеличение частоты невынашивания беременности гипертензия беременных466. К абсолютным показаниям для донации ооцитов не относитсядисгенезия гонад СПКЯсиндром резистентных яичниковхирургическая кастрация (синдром постовариоэктомии)постменопауза467. Для стимуляции суперовуляции в программе ЭКО и ПЭ не используются антагонисты эстрогенов (кломифен и его аналоги)препараты человеческого менопаузального гонадотропина (чМГ)агонисты и антагонисты гонадотропин- рилизинг-гормонапрепараты ФСГ ингибиторы гонадотропинов468. К описанию синдрома гиперстимуляции яичников (СГЯ) не подходитятрогенное состояниенефизиологический ответ яичников на экзогенное введение стимуляторов овуляцииширокий спектр клинических и лабораторных проявлений спонтанное состояниепри особой тяжести может привести к летальному исходу469. Для клиники СГЯ (синдрома гиперстимуляции яичников) не характерновыраженное увеличение яичников с формированием фолликулярных и лютеиновых кистгиповолемия, гемоконцентрация, гипопротеинемия, электролитный дисбалансасцит, гидроторакс, анасарка, олигурия, ОПНтромбоэмболические осложнения, РДС взрослых артериальная гипертензия и брадикардия470. В случае наступления беременности синдром гиперстимуляции яичников (СГЯ)

средней и тяжелой степени тяжестикупируется сразу быстро прогрессирует в тяжелую форму протекает без изменений на фоне СГЯ беременность не наступаеткупируется через 10-12 дней471. К факторам риска СГЯ (синдрома гиперстимуляции яичников) не относится возраст старше 35 летСПКЯвозраст младше 35 летастеническое телосложениестимуляция суперовуляции агонистами гонадотропин- рилизинг-гормона472. Снижение риска развития СГЯ (синдрома гиперстимуляции яичников) не связано с внедрением в клинику рекомбинантных гонадотропиновс применением для стимуляция суперовуляции антагонистов гонадотропин- рилизинг-

гормона

Page 82: postgraduate.kaznmu.kz€¦ · Web view58 Больная И, 28 лет, доставлена с жалобами на внезапно возникшую боль внизу живота

С.Ж.АСФЕНДИЯРОВАТЫНДАҒЫҚАЗАҚ ҰЛТТЫҚ МЕДИЦИНА

УНИВЕРСИТЕТІ

КАЗАХСКИЙНАЦИОНАЛЬНЫЙМЕДИЦИНСКИЙ УНИВЕРСИТЕТ ИМЕНИ С.Д.АСФЕНДИЯРОВА

КАФЕДРА ИНТЕРНАТУРЫ И РЕЗИДЕНТУРЫ ПО АКУШЕРСТВУ И ГИНЕКОЛОГИИТЕСТОВЫЕ ЗАДАНИЯ ДЛЯ ПРОВЕДЕНИЯ ИТОГОВОЙ ГОСУДАРСТВЕННОЙ АТТЕСТАЦИИ РЕЗИДЕНТОВ

ВЫПУСКА 2012-2013 ГГ. ПО СПЕЦИАЛЬНОСТИ «АКУШЕРСТВО И ГИНЕКОЛОГИЯ»

с инфузией альбумина и гидроксиэтил крахмала (ГЭК) во время программы ЭКО и ПЭ с наступлением беременностис обязательным УЗИ и гормональным мониторингом процесса стимуляции суперовуляции473. Критерии прогнозирования СГЯ (синдрома гиперстимуляции яичников) не

включаютполикистозный характер ответа на введение препаратов агонистов гонадотропин- рилизинг-

гормонавысокий уровень эстрадиола в плазме крови (> 50пмоль/л на фолликул) повышенный уровень в крови онкомаркера СА-125 (>35 ЕД/мл) пониженный уровень в крови онкомаркера СА-125 (<35 ЕД/мл)многократные стимуляции суперовуляции в анамнезе474. Показания к хирургическому лечению СГЯ (синдрома гиперстимуляции яичников)

не включаютвнутрибрюшное кровотечение при разрыве кист яичникаперекрут яичникасопутствующая внематочная беременность увеличение в плазме крови уровня СА-125, маркера аденокарциномы яичниковподозрение на рак яичников475. Бесплодный брак – отсутствие беременности у супругов детородного возраста в

течение 1 года2 лет3 лет4 лет5 лет 476. Тактика базового обследования пары при бесплодии не включает

одновременное обследование мужчины и женщиныназначение инвазивных методов обследования (гистероскопия, ГСГ, лапароскопия) женщине

только после результатов спермограммы длительность полного обследования не более 3-4 месяцев первоначальное обследование мужчины с последующим обследованием жены

работу гинеколога и уролога в тесном контакте477. Инфекционный скрининг женщины при обследовании бесплодной пары не включает

мазок на степень чистотыопределение флоры влагалища и ее чувствительности к антибиотикам посевы на хламидии, уреаплазмы, микоплазмы, гонорею, трихомонады

определение вируса папилломы человекаантитела в крови к хламидиям, уреаплазме, микоплазме 478. TORCH-комплекс в инфекционном скрининге женщины при обследовании

бесплодной пары не включает определение антител IgG и IgM к краснухеопределение антител IgG и IgM к токсоплазмозу определение антител IgG и IgM к ВПГ 1 и 2 типов определение антител IgG и IgM к бруцеллезуопределение антител IgG и IgM к ЦМВ479. Наиболее распространенная форма женского бесплодияиммунологическое бесплодиеэндокринное бесплодие

Page 83: postgraduate.kaznmu.kz€¦ · Web view58 Больная И, 28 лет, доставлена с жалобами на внезапно возникшую боль внизу живота

С.Ж.АСФЕНДИЯРОВАТЫНДАҒЫҚАЗАҚ ҰЛТТЫҚ МЕДИЦИНА

УНИВЕРСИТЕТІ

КАЗАХСКИЙНАЦИОНАЛЬНЫЙМЕДИЦИНСКИЙ УНИВЕРСИТЕТ ИМЕНИ С.Д.АСФЕНДИЯРОВА

КАФЕДРА ИНТЕРНАТУРЫ И РЕЗИДЕНТУРЫ ПО АКУШЕРСТВУ И ГИНЕКОЛОГИИТЕСТОВЫЕ ЗАДАНИЯ ДЛЯ ПРОВЕДЕНИЯ ИТОГОВОЙ ГОСУДАРСТВЕННОЙ АТТЕСТАЦИИ РЕЗИДЕНТОВ

ВЫПУСКА 2012-2013 ГГ. ПО СПЕЦИАЛЬНОСТИ «АКУШЕРСТВО И ГИНЕКОЛОГИЯ»

трубно-перитонеальное бесплодиебесплодие вследствие эндометриозаматочное бесплодие480. Какие пациентки не входят в группу риска по бесплодию

женщины старше 35 летс нейроэндокринными нарушениями менструального цикла

с ИПППс генитальным эндометриозом с травмами шейки матки481. В течение какого времени с момента обращения супружеской пары рекомендуется

установить причину бесплодия (алгоритм ВОЗ, 1998)в течение 1-2 мес в течение 3-4 мес в течение 5-6 мес в течение 6-12 мес в течение 1-2 лет

482. После уточнения диагноза рекомендуемая длительность непрерывного лечения бесплодия до применения ВРТ не должна превышать

полгода1 год 1-1,5 года 2-3 года 3-х лет

483. Рентгенологические методы исследования, применяемые при обследовании женщин с бесплодием, не включают

краниограмму компьютерную томографиюгистерографиюмагнитно-резонансную томографию рентгенпельвиметрию

484. Для диагностики мужского бесплодия не применяютанализ цитологии секрета простаты и семенных пузырьковопределение антиспермальных антителисследование эякулята (развернутая спермограмма), МАР-тест уретроскопиюбактериологический анализ спермы

485. Основной показатель благоприятного исхода беременности после ЭКО число эмбрионов, имплантировавшихся в полость маткичастота невынашивания беременности после ЭКО показатель «take home baby» наступление маточной беременности по данным УЗИколичество доношенных беременностей

486. К факторам снижения фертильности, связанным с возрастом женщины, не относитсяснижение количества яйцеклеток и нарушение овуляцииухудшение качества яйцеклеток и возрастание числа хромосомных и генных мутацийдисфункция лютеиновой фазы нарушение проходимости маточных трубнекомпетентность эндометрия

Page 84: postgraduate.kaznmu.kz€¦ · Web view58 Больная И, 28 лет, доставлена с жалобами на внезапно возникшую боль внизу живота

С.Ж.АСФЕНДИЯРОВАТЫНДАҒЫҚАЗАҚ ҰЛТТЫҚ МЕДИЦИНА

УНИВЕРСИТЕТІ

КАЗАХСКИЙНАЦИОНАЛЬНЫЙМЕДИЦИНСКИЙ УНИВЕРСИТЕТ ИМЕНИ С.Д.АСФЕНДИЯРОВА

КАФЕДРА ИНТЕРНАТУРЫ И РЕЗИДЕНТУРЫ ПО АКУШЕРСТВУ И ГИНЕКОЛОГИИТЕСТОВЫЕ ЗАДАНИЯ ДЛЯ ПРОВЕДЕНИЯ ИТОГОВОЙ ГОСУДАРСТВЕННОЙ АТТЕСТАЦИИ РЕЗИДЕНТОВ

ВЫПУСКА 2012-2013 ГГ. ПО СПЕЦИАЛЬНОСТИ «АКУШЕРСТВО И ГИНЕКОЛОГИЯ»

487. Первоначальный скрининг бесплодной пары не включаетспермограмму ИПППгистеросальпингографиюгормоны крови лапароскопию

488. К лекарственным препаратам, отрицательно влияющим на фертильность женщины, не относятся

цитостатики половые стероиды антибактериальные препаратыпрепараты, используемые для лечения гастроинтестинальных расстройствнейролептики и антидепрессанты

489. Клиническое обследование женщины с бесплодием не включаетопределение ИМТопределение гирсутного числа определение степени развития молочных желез/галактореиопределение состояния щитовидной железы проведение посткоитального теста

490. Гормональный скрининг женщины с бесплодием не включаетпролактин ЛГ, ФСГТТГ АКТГтестостерон

491. Женщине старше 45 лет в программе ЭКО с целью определения состояния молочных желез необходимо провести

УЗИ молочных желез маммографиюанализ на онкомаркеры биопсию молочных железпальпацию молочных желез

492. К наиболее вероятным факторам снижения фертильности у мужчин не относятсясахарный диабеттуберкулез оперативные вмешательства на органы брюшной полостипрофессиональные вредности вредные привычки

493. Клиническое обследование мужчины с бесплодием не включаетизмерение роста и весаопределение признаков гипоандрогенииопределение признаков гинекомастии определение спермограммы андрологическое обследование гениталий, паховой области и предстательной железы

494. Основные лабораторные методы в первичном обследовании мужчин с бесплодием не включают

развернутую спермограммуМАР - тест

Page 85: postgraduate.kaznmu.kz€¦ · Web view58 Больная И, 28 лет, доставлена с жалобами на внезапно возникшую боль внизу живота

С.Ж.АСФЕНДИЯРОВАТЫНДАҒЫҚАЗАҚ ҰЛТТЫҚ МЕДИЦИНА

УНИВЕРСИТЕТІ

КАЗАХСКИЙНАЦИОНАЛЬНЫЙМЕДИЦИНСКИЙ УНИВЕРСИТЕТ ИМЕНИ С.Д.АСФЕНДИЯРОВА

КАФЕДРА ИНТЕРНАТУРЫ И РЕЗИДЕНТУРЫ ПО АКУШЕРСТВУ И ГИНЕКОЛОГИИТЕСТОВЫЕ ЗАДАНИЯ ДЛЯ ПРОВЕДЕНИЯ ИТОГОВОЙ ГОСУДАРСТВЕННОЙ АТТЕСТАЦИИ РЕЗИДЕНТОВ

ВЫПУСКА 2012-2013 ГГ. ПО СПЕЦИАЛЬНОСТИ «АКУШЕРСТВО И ГИНЕКОЛОГИЯ»

анализ секрета простатыклинические анализы крови и мочи по показаниям тестикулярную биопсию

495. К дополнительным методам исследования мужчины с бесплодием не относится краниограмма и компьютерная томография (при подозрении на аденому гипофиза)определение ФСГ, пролактина, тестостерона спермограмматестикулярная биопсиятермография мошонки

496. К наименее вероятным факторам снижения фертильности у мужчин относятся оперативные вмешательства на органы брюшной полостиИПППнервные и психические заболеваниясексуальная и эякуляторная дисфункциикурение

497. По результатам комплексного обследования статистически наиболее достоверная частота мужского бесплодия в последнее время

10 -20% 25 -30% 35-40%55-60%более 60%.498. В каких случаях может быть поставлен диагноз «бесплодие неясного генеза»

при проходимости маточных трубпри нормальных показателях ФСГ, пролактина, тестостеронапри неизмененной спермограмметолько после проведения тестикулярной биопсии только после проведения лапароскопии

499. Наиболее вероятная частота бесплодия неясного генеза среди бесплодных пар1 - 2% 4 - 5%5 - 10% 10 - 15%20 - 30%.500. При какой ситуации женщине в бесплодном браке рекомендуется ЭКО как

первоначальный выбор леченияопухоли яичников миома маткиэндокринное бесплодие

возраст женщины старше 35 летгенитальный эндометриоз 501. При какой ситуации бесплодным парам менее всего показано ЭКО как

первоначальный выбор леченияиммунологическое бесплодие эндокринное бесплодиетрубное бесплодие с пластикой труб в анамнезевозраст женщины старше 35 летмужское бесплодие

Page 86: postgraduate.kaznmu.kz€¦ · Web view58 Больная И, 28 лет, доставлена с жалобами на внезапно возникшую боль внизу живота

С.Ж.АСФЕНДИЯРОВАТЫНДАҒЫҚАЗАҚ ҰЛТТЫҚ МЕДИЦИНА

УНИВЕРСИТЕТІ

КАЗАХСКИЙНАЦИОНАЛЬНЫЙМЕДИЦИНСКИЙ УНИВЕРСИТЕТ ИМЕНИ С.Д.АСФЕНДИЯРОВА

КАФЕДРА ИНТЕРНАТУРЫ И РЕЗИДЕНТУРЫ ПО АКУШЕРСТВУ И ГИНЕКОЛОГИИТЕСТОВЫЕ ЗАДАНИЯ ДЛЯ ПРОВЕДЕНИЯ ИТОГОВОЙ ГОСУДАРСТВЕННОЙ АТТЕСТАЦИИ РЕЗИДЕНТОВ

ВЫПУСКА 2012-2013 ГГ. ПО СПЕЦИАЛЬНОСТИ «АКУШЕРСТВО И ГИНЕКОЛОГИЯ»

502. К абсолютным показаниям для донации ооцитов не относится старший репродуктивный возраст и/или перименопауза аутоиммунное повреждение яичников

синдром преждевременного истощения яичниковповреждение фолликулярного аппарата после радио- и/или химиотерапии

аплазия яичников503. К относительным показаниям для донации ооцитов не относитсянеоднократное получение эмбрионов низкого качества, перенос которых не приводил к

наступлению беременности старший репродуктивный возраст и/или перименопауза синдром преждевременного истощения яичниковнедостаточный ответ яичников на индукцию суперовуляции смерть единственного ребенка

504. Абсолютное показание для донации ооцитов неоднократное получение эмбрионов низкого качества, перенос которых не приводил к

наступлению беременности старший репродуктивный возраст и/или перименопауза смерть единственного ребенка недостаточный ответ яичников на индукцию суперовуляции синдром резистентных яичников

505. Относительное показание для донации ооцитовдисгенезия гонад

аутоиммунное повреждение яичниковсиндром резистентных яичников

повреждение фолликулярного аппарата после радио- и/или химиотерапии неудачные повторные попытки ЭКО в анамнезе

506. К требованиям, предъявляемым к донорам ооцитов, не относитсявозраст от 20 до 35 лет

соматическое здоровье наличие выраженных фенотипических признаковотсутствие выраженных фенотипических проявленийналичие собственного здорового ребенка507. Этапы программы ЭКО с донацией ооцитов не включают

выбор донора ооцитов (обычно по фенотипическим данным и/или группе крови).синхронизацию менструальных циклов донора и реципиента.

стимуляцию донора, пункцию и забор яйцеклеток донора оплодотворение перенос эмбрионов донору508. Какое требование не предъявляется к суррогатной материвозраст от 20 до 35 лет

соматическое здоровье отсутствие выраженных фенотипических проявленийпсихическое здоровьеналичие собственного здорового ребенка509. К показаниям к суррогатному материнству не относитсяаплазия матки (синдром Рокитанского-Кюстнера)

внутриматочные синехии, не поддающиеся терапии синдром резистентных яичников

Page 87: postgraduate.kaznmu.kz€¦ · Web view58 Больная И, 28 лет, доставлена с жалобами на внезапно возникшую боль внизу живота

С.Ж.АСФЕНДИЯРОВАТЫНДАҒЫҚАЗАҚ ҰЛТТЫҚ МЕДИЦИНА

УНИВЕРСИТЕТІ

КАЗАХСКИЙНАЦИОНАЛЬНЫЙМЕДИЦИНСКИЙ УНИВЕРСИТЕТ ИМЕНИ С.Д.АСФЕНДИЯРОВА

КАФЕДРА ИНТЕРНАТУРЫ И РЕЗИДЕНТУРЫ ПО АКУШЕРСТВУ И ГИНЕКОЛОГИИТЕСТОВЫЕ ЗАДАНИЯ ДЛЯ ПРОВЕДЕНИЯ ИТОГОВОЙ ГОСУДАРСТВЕННОЙ АТТЕСТАЦИИ РЕЗИДЕНТОВ

ВЫПУСКА 2012-2013 ГГ. ПО СПЕЦИАЛЬНОСТИ «АКУШЕРСТВО И ГИНЕКОЛОГИЯ»

соматические заболевания, при которых вынашивание беременности противопоказано неудачные повторные попытки ЭКО при неоднократном получении эмбрионов высокого

качества, перенос которых не приводит к наступлению беременности 510. Этапы программы ЭКО с суррогатным материнством не включают

выбор суррогатной материсинхронизацию менструальных циклов пациентки и суррогатной матери

стимуляцию пациентки, пункцию и забор яйцеклеток пациенткиоплодотворение перенос эмбрионов пациентке511. Подготовительный этап лечения бесплодия при СПКЯ центрального генеза,

сопровождающим ожирением, не включаетдиетотерапиюназначение метаформинафизические нагрузки

назначение дексаметазонаназначение дифенина512. К противопоказаниям к стимуляции овуляции при СПКЯ не относится

противопоказания к беременности гормонально-зависимые онкологические заболевания, в т.ч. в анамнезенепереносимость препаратов ХГ, ФСГ наличие кисты яичников

гиперпролактинемия513. К особенностям стимуляции овуляции при СПКЯ не относится

высокий риск СГЯнеобходимость подбора дозы препаратаначало с минимальных доз низкий риск СГЯвысокий риск паразитарных пиков ЛГ

514. К критериям выбора стартовой дозы гонадотропинов для стимуляции овуляции не относится

возраст пациенткиколичество фолликулов d<10 мм на 2-3 день менструального цикла общий объем яичниковбазальный уровень ФСГ базальный уровень ЛГ

515. УЗИ мониторинг при зарегистрированном росте фолликулов на фоне стимуляции овуляции проводится

каждые 1-2 днякаждые 3 дняв день 2 разакаждые 4-5 днейв неделю 1 раз

516. УЗИ мониторинг при зарегистрированном росте фолликулов на фоне стимуляции овуляции НЕ проводится с целью

оценки скорости роста фолликулов, толщины эндометрияоценки адекватности и коррекции дозы гонадотропиновконтроля преждевременной лютеинизации введения ХГ по достижению критериев зрелости

Page 88: postgraduate.kaznmu.kz€¦ · Web view58 Больная И, 28 лет, доставлена с жалобами на внезапно возникшую боль внизу живота

С.Ж.АСФЕНДИЯРОВАТЫНДАҒЫҚАЗАҚ ҰЛТТЫҚ МЕДИЦИНА

УНИВЕРСИТЕТІ

КАЗАХСКИЙНАЦИОНАЛЬНЫЙМЕДИЦИНСКИЙ УНИВЕРСИТЕТ ИМЕНИ С.Д.АСФЕНДИЯРОВА

КАФЕДРА ИНТЕРНАТУРЫ И РЕЗИДЕНТУРЫ ПО АКУШЕРСТВУ И ГИНЕКОЛОГИИТЕСТОВЫЕ ЗАДАНИЯ ДЛЯ ПРОВЕДЕНИЯ ИТОГОВОЙ ГОСУДАРСТВЕННОЙ АТТЕСТАЦИИ РЕЗИДЕНТОВ

ВЫПУСКА 2012-2013 ГГ. ПО СПЕЦИАЛЬНОСТИ «АКУШЕРСТВО И ГИНЕКОЛОГИЯ»

регистрации овуляции517. К критериям овуляции не относится

уменьшение размера фолликуланечеткость контуров фолликулажидкость в дугласовом пространстве снижение ЛГ ниже 10 МЕ/д исчезновение фолликула

518. С целью профилактики СГЯ (синдрома гиперстимуляции яичников) у женщины с СПКЯ при индукции овуляции не применяют

выбор дозы препаратапонижающий протокол увеличение дозы ХГотказ от введения ХГантагонисты ГнРГ

519. Почему на современном этапе лечения бесплодия при СПКЯ не проводят клиновидную резекцию яичников

из-за технических трудностей проведения операциииз-за наличия в арсенале современных препаратовиз-за отсутствия эффективности операции из-за снижения фолликулярного резерва яичниковиз-за риска спаечного процесса

520. К преимуществам лапароскопии при лечении бесплодия у женщин с СПКЯ не относится

удаление или разрушение мозгового слоя яичников как источника андрогеноввозможность выявления сочетанной патологии ОМТ, которая может быть причиной бесплодия высокий процент излечения пациенток от СПКЯминимизация риска повреждения фолликулярного аппарата яичниковминимизация риска спаечного процесса

521. Диагностические критерии легкой степени СГЯ (синдрома гиперстимуляции яичников) не включают следующие признаки

яичники не более 6 см в диаметре анасаркаколичество фолликулов более 8дискомфорт, гастроинтестинальные проявленияуровень эстрадиола >2000 пг/мл, но <4000 пг/мл

522. Диагностические критерии средней степени СГЯ (синдрома гиперстимуляции яичников) не включают следующие признаки

яичники не более 8 см в диаметреасцит, гидротораксгиповолемия, гемоконцентрация тахикардия, снижение АД, повышение Ht, гиперкоагуляция,уровень эстрадиола >4000 пг/мл

523. Диагностические критерии тяжелой степени СГЯ (синдрома гиперстимуляции яичников) не включают следующие признаки

яичники более 10 см в диаметреасцит, гидроторакс, полиорганные нарушениягиповолемия, гемоконцентрация, гиперкоагуляция гипотензия, тахикардия

Page 89: postgraduate.kaznmu.kz€¦ · Web view58 Больная И, 28 лет, доставлена с жалобами на внезапно возникшую боль внизу живота

С.Ж.АСФЕНДИЯРОВАТЫНДАҒЫҚАЗАҚ ҰЛТТЫҚ МЕДИЦИНА

УНИВЕРСИТЕТІ

КАЗАХСКИЙНАЦИОНАЛЬНЫЙМЕДИЦИНСКИЙ УНИВЕРСИТЕТ ИМЕНИ С.Д.АСФЕНДИЯРОВА

КАФЕДРА ИНТЕРНАТУРЫ И РЕЗИДЕНТУРЫ ПО АКУШЕРСТВУ И ГИНЕКОЛОГИИТЕСТОВЫЕ ЗАДАНИЯ ДЛЯ ПРОВЕДЕНИЯ ИТОГОВОЙ ГОСУДАРСТВЕННОЙ АТТЕСТАЦИИ РЕЗИДЕНТОВ

ВЫПУСКА 2012-2013 ГГ. ПО СПЕЦИАЛЬНОСТИ «АКУШЕРСТВО И ГИНЕКОЛОГИЯ»

РДС взрослых 524. Диагностические критерии критической степени СГЯ (синдрома гиперстимуляции

яичников) не включают следующие признакиОПН, тромбоэмболические осложнения асцит, двусторонний гидроторакс, гидроперикард, анасарка, полиорганные нарушениягиповолемия, гемоконцентрация Нt 55% и выше уровень эстрадиола < 4000 пг/млE) РДС взрослых

525. Частота тяжелых форм СГЯ (синдрома гиперстимуляции яичников) при проведении стимуляции суперовуляции в программе ЭКО

менее 1%1 - 2%2 - 3%3 - 4%4 - 5%

526. К факторам риска СГЯ (синдрома гиперстимуляции яичников) не относитсямультифолликулярные яичникивведение неадекватных доз гонадотропиновразвитие 10 и более фолликулов в каждом яичнике размером ≥12 мм

гиперстеническое телосложениеприменение ХГ в поддерживающей терапии

527. Тактика ведения СГЯ (синдрома гиперстимуляции яичников) легкой степени наблюдение в амбулаторных условиях, профилактическая терапиягоспитализация в гинекологическое отделение, комплексное лечениеинтенсивное наблюдение в стационаре, посиндромная терапияпрерывание беременности профилактические мероприятия

528. Тактика ведения СГЯ (синдрома гиперстимуляции яичников) средней степени тяжести

наблюдение в амбулаторных условиях, профилактическая терапия госпитализация в гинекологическое отделение, комплексное лечениеинтенсивное наблюдение в стационаре, посиндромная терапияпрерывание беременности профилактические мероприятия

529. Тактика ведения СГЯ (синдрома гиперстимуляции яичников) тяжелой степенинаблюдение в амбулаторных условиях, профилактическая терапиягоспитализация в гинекологическое отделение, комплексное лечение интенсивное наблюдение в стационаре, посиндромная терапияпрерывание беременности профилактические мероприятия

530. Тактика ведения критической формы СГЯ (синдрома гиперстимуляции яичников) наблюдение в амбулаторных условиях, профилактическая терапиягоспитализация в гинекологическое отделение, комплексное лечениеинтенсивное наблюдение в стационаре, посиндромная терапия прерывание беременности профилактические мероприятия

531. Принципы терапии СГЯ (синдрома гиперстимуляции яичников) не включаютустранение гиповолемии и гемоконцентрации

Page 90: postgraduate.kaznmu.kz€¦ · Web view58 Больная И, 28 лет, доставлена с жалобами на внезапно возникшую боль внизу живота

С.Ж.АСФЕНДИЯРОВАТЫНДАҒЫҚАЗАҚ ҰЛТТЫҚ МЕДИЦИНА

УНИВЕРСИТЕТІ

КАЗАХСКИЙНАЦИОНАЛЬНЫЙМЕДИЦИНСКИЙ УНИВЕРСИТЕТ ИМЕНИ С.Д.АСФЕНДИЯРОВА

КАФЕДРА ИНТЕРНАТУРЫ И РЕЗИДЕНТУРЫ ПО АКУШЕРСТВУ И ГИНЕКОЛОГИИТЕСТОВЫЕ ЗАДАНИЯ ДЛЯ ПРОВЕДЕНИЯ ИТОГОВОЙ ГОСУДАРСТВЕННОЙ АТТЕСТАЦИИ РЕЗИДЕНТОВ

ВЫПУСКА 2012-2013 ГГ. ПО СПЕЦИАЛЬНОСТИ «АКУШЕРСТВО И ГИНЕКОЛОГИЯ»

устранение порозности эндотелия сосудов и поддержание адекватного ОЦПобеспечение адекватного венозного возврата к сердцу за счет эвакуации асцитической жидкостикоррекцию гипопротеинемии и электролитного дисбаланса, улучшение перфузии почек коррекцию параметров гемостаза, профилактику кровотечения, применение коагулянтов

532. Эвакуация жидкости из брюшной полости при тяжелой степени СГЯ (синдрома гиперстимуляции яичников)

не проводится, происходит самостоятельная резорбцияпроводится при лапароскопиипроводится при лапаротомии проводится при абдоминальном или трансвагинальном лапароцентезе под УЗ контролемне проводится, применяется консервативная терапия асцита

533. Профилактика СГЯ (синдрома гиперстимуляции яичников) не включает следующие мероприятия

выявление пациенток групп риска и выбор для них щадящих протоколов стимуляцииприменение низкодозовых протоколов со стартовой дозой 50 МЕ и минимальным увеличением

дозы (25 МЕ) применение ЛГ-содержащих гонадотропиновпервый УЗ контроль ответа яичника не позднее 5 дня стимуляциикоррекцию гиперинсулинемии при СПКЯ с инсулинорезистентностью

534. Во время стимуляции при угрозе развития СГЯ не осуществляютотказ от введения овуляторной дозы ХГснижение овуляторной дозы ХГаспирацию всех фолликулов при пункцииотмену ПЭ, их криоконсервацию и перенос в полость матки в другом менструальном цикле поддержку лютеиновой фазы препаратами ХГ

535. Оптимальный срок редукции эмбрионов трансабдоминальным способом при многоплодной беременности

в 5-6 недель в 7-8 недель в 9-10 недель в 11-12 недель в 12-14 недель

536. За одну процедуру редуцируют не более одного эмбриона не более двух эмбрионовне более трех эмбрионовна усмотрение врачапо желанию женщины

537. К условиям, усложняющим проведение редукции эмбрионов трансабдоминальным способом при многоплодной беременности, не относится

ожирение у женщиныположение матки в retroflexio инфекционное поражение кожи на передней брюшной стенке отсутствие необходимости общего обезболиванияотносительно большие размеры редуцированного эмбриона, увеличивающие сроки его

резорбции 538. К вспомогательным репродуктивным технологиям не относится

искусственная инсеминация

Page 91: postgraduate.kaznmu.kz€¦ · Web view58 Больная И, 28 лет, доставлена с жалобами на внезапно возникшую боль внизу живота

С.Ж.АСФЕНДИЯРОВАТЫНДАҒЫҚАЗАҚ ҰЛТТЫҚ МЕДИЦИНА

УНИВЕРСИТЕТІ

КАЗАХСКИЙНАЦИОНАЛЬНЫЙМЕДИЦИНСКИЙ УНИВЕРСИТЕТ ИМЕНИ С.Д.АСФЕНДИЯРОВА

КАФЕДРА ИНТЕРНАТУРЫ И РЕЗИДЕНТУРЫ ПО АКУШЕРСТВУ И ГИНЕКОЛОГИИТЕСТОВЫЕ ЗАДАНИЯ ДЛЯ ПРОВЕДЕНИЯ ИТОГОВОЙ ГОСУДАРСТВЕННОЙ АТТЕСТАЦИИ РЕЗИДЕНТОВ

ВЫПУСКА 2012-2013 ГГ. ПО СПЕЦИАЛЬНОСТИ «АКУШЕРСТВО И ГИНЕКОЛОГИЯ»

преимплантационная генетическая диагностика вапооризация яичников при лапароскопииЭКОвспомогательный хэтчинг

539. Вспомогательные репродуктивные технологии (ВРТ)-современные методики проведения стимуляции овуляции система методик лечения бесплодия, при которых отдельные или все этапы оплодотворения и

раннего развития эмбрионов осуществляются вне организмавапооризация яичников при лапароскопииопределение проходимости маточных труб во время лапароскопиисальпингоовариолизис

540. К противопоказаниям к применению ЭКО не относятсясоматические, психические заболевания, при которых вынашивание беременности

противопоказаноопухоли яичниковзлокачественные заболеванияострые воспалительные заболевания миоматозные узлы с общим объемом до 4 см

541. Наиболее вероятное показание к суррогатному материнству седловидная матка аденомиоз хронический эндометрит, атрофическая форма

хронический эндометрит, гипертрофическая форманеудачные повторные попытки ЭКО при неоднократном получении эмбрионов низкого качества,

перенос которых не приводит к наступлению беременности542. Что прежде всего можно порекомендовать пациентке с аномалией развития матки с

проблемой бесплодия хирургическую коррекцию порока развития с последующей консервативной терапиейдонацию ооцитовсуррогатное материнствоЭКОвспомогательный хетчинг

543. Что не относится к показаниям для проведения гистерорезектоскопии при маточном факторе бесплодия

полипы эндометрия перегородка в полости матки аденомиозвнутриматочные синехии

субмукозная форма миомы матки544. Что такое «вспомогательный хэтчинг»

предимплантационная генетическая диагностикаинтрацитоплазматическая инъекция сперматозоида в яйцеклеткукриоконсервация эмбрионов процедура подготовки эмбриона к имплантации до его переноса отбор сперматозоида по морфологическим критериям при большом увеличении перед

проведением ИКСИ 545. Лауреат Нобелевской премии в 2010 году в области медицины за заслуги в

разработке метода ЭКО

Page 92: postgraduate.kaznmu.kz€¦ · Web view58 Больная И, 28 лет, доставлена с жалобами на внезапно возникшую боль внизу живота

С.Ж.АСФЕНДИЯРОВАТЫНДАҒЫҚАЗАҚ ҰЛТТЫҚ МЕДИЦИНА

УНИВЕРСИТЕТІ

КАЗАХСКИЙНАЦИОНАЛЬНЫЙМЕДИЦИНСКИЙ УНИВЕРСИТЕТ ИМЕНИ С.Д.АСФЕНДИЯРОВА

КАФЕДРА ИНТЕРНАТУРЫ И РЕЗИДЕНТУРЫ ПО АКУШЕРСТВУ И ГИНЕКОЛОГИИТЕСТОВЫЕ ЗАДАНИЯ ДЛЯ ПРОВЕДЕНИЯ ИТОГОВОЙ ГОСУДАРСТВЕННОЙ АТТЕСТАЦИИ РЕЗИДЕНТОВ

ВЫПУСКА 2012-2013 ГГ. ПО СПЕЦИАЛЬНОСТИ «АКУШЕРСТВО И ГИНЕКОЛОГИЯ»

Роберт ЭдвардсПатрик СтептойГовард ДжонсДжорджина ДжонсПоль Деврой

546. Кто предложил метод ИКСИ для лечения бесплодияРоберт ЭдвардсПатрик СтептойГовард ДжонсДжорджина Джонс Поль Деврой

547. Процедура «вспомогательного хэтчинга» - этовыявление функционально зрелых сперматозоидов при связывании их с гиалуроновой кислотойселекция эмбрионов проведение отверстия в оболочке эмбриона до его переноса с целью подготовки эмбриона к

имплантации криоконсервация эмбрионовотбор сперматозоидов по морфологическим критериям при большом увеличении

548. Первый казахстанский ребенок «из пробирки» родился в 1992 годув 1994 году в 1996 годув 1998 годув 1999 году

549. Какая из ниже перечисленных методик ВРТ наименее популярна на современном этапе

ИМСИИКСИТЕЗА ГИФТ/ЗИФТ ПИКСИ

550. Методика ИМСИвыявление функционально зрелых сперматозоидов при связывании их с гиалуроновой кислотойселекция эмбрионовпроведение отверстия в оболочке эмбриона до его переноса с целью подготовки эмбриона к

имплантации криоконсервация эмбрионов отбор сперматозоида по морфологическим критериям при большом увеличении и введение в

яйцеклетку

Акушерство

1. Оплодотворенная яйцеклетка пребывает в маточной трубе:A. 10-12 часовB. 1-2 дняC. 3-4 дняD. 5-6 дней

Page 93: postgraduate.kaznmu.kz€¦ · Web view58 Больная И, 28 лет, доставлена с жалобами на внезапно возникшую боль внизу живота

С.Ж.АСФЕНДИЯРОВАТЫНДАҒЫҚАЗАҚ ҰЛТТЫҚ МЕДИЦИНА

УНИВЕРСИТЕТІ

КАЗАХСКИЙНАЦИОНАЛЬНЫЙМЕДИЦИНСКИЙ УНИВЕРСИТЕТ ИМЕНИ С.Д.АСФЕНДИЯРОВА

КАФЕДРА ИНТЕРНАТУРЫ И РЕЗИДЕНТУРЫ ПО АКУШЕРСТВУ И ГИНЕКОЛОГИИТЕСТОВЫЕ ЗАДАНИЯ ДЛЯ ПРОВЕДЕНИЯ ИТОГОВОЙ ГОСУДАРСТВЕННОЙ АТТЕСТАЦИИ РЕЗИДЕНТОВ

ВЫПУСКА 2012-2013 ГГ. ПО СПЕЦИАЛЬНОСТИ «АКУШЕРСТВО И ГИНЕКОЛОГИЯ»

E. 7-8 дней2. Сколько времени проходит с момента попадания эмбриона в полость матки до имплантации:A. 24 часаB. 36 часовC. 48 часовD. 60 часовE. 72 часа3. На какие сутки с момента оплодотворения начинается имплантация:A. 1-еB. 3-иC. 5-еD. 7-еE. 9-е 4. Эмбриональный период человека начинается с появления: A. яйцеклеткиB. зиготыC. морулыD. бластоцистыE. эмбрионального диска5. Эмбриональный диск появляется на:A. 1-ой неделеB. 2-ой неделеC. 3-ей неделеD. 4-ой неделеE. 5-ой неделе6. На какой неделе беременности заканчивается эмбриональный период человека:A. 5-6-ой B. 7-8-ой C. 8-9-ой D. 9-10ой E. 10-11-ой 7. Больная 22 лет жалоб не предъявляет, явилась на консультацию по контрацепции. Менструальная функция без особенностей, половая жизнь с 22 лет. Объективно без особенностей. При вагинальном исследовании: матка нормальной величины, справа от нее определяется образование 5,5*5,5 см, подвижное, безболезненное, своды свободные.Ваш диагноз:A. Миома маткиB. Тубоовариальное образованиеC. Перекрут ножки опухоли яичникаD. Киста яичникаE. Внематочная беременность8. При длительности менструального цикла 28 дней его следует считать:A. НормопонирующимB. АнтепонирующимC. ПостпонирующимD. ГиперпонирующимE. Гипопонирующим9. Антенатальная гибель плода - это гибель плода с:

Page 94: postgraduate.kaznmu.kz€¦ · Web view58 Больная И, 28 лет, доставлена с жалобами на внезапно возникшую боль внизу живота

С.Ж.АСФЕНДИЯРОВАТЫНДАҒЫҚАЗАҚ ҰЛТТЫҚ МЕДИЦИНА

УНИВЕРСИТЕТІ

КАЗАХСКИЙНАЦИОНАЛЬНЫЙМЕДИЦИНСКИЙ УНИВЕРСИТЕТ ИМЕНИ С.Д.АСФЕНДИЯРОВА

КАФЕДРА ИНТЕРНАТУРЫ И РЕЗИДЕНТУРЫ ПО АКУШЕРСТВУ И ГИНЕКОЛОГИИТЕСТОВЫЕ ЗАДАНИЯ ДЛЯ ПРОВЕДЕНИЯ ИТОГОВОЙ ГОСУДАРСТВЕННОЙ АТТЕСТАЦИИ РЕЗИДЕНТОВ

ВЫПУСКА 2012-2013 ГГ. ПО СПЕЦИАЛЬНОСТИ «АКУШЕРСТВО И ГИНЕКОЛОГИЯ»

A. 22 недель беременности до начала родовой деятельностиB. 28 недель беременности до начала родовой деятельности C. 32 недель беременности до начала родовой деятельностиD. 38 недель беременности до начала родовой деятельности E. С беременности ранних сроков до 7 дней жизни 10. В структуре материнской смертности в Республике Казахстан основнаядоля приходится на:A. Гипертензивные состояния во время беременностиB. Экстрагенитальную патологию C. КровотеченияD. Гнойно-септическую инфекциюE. Разрыв матки 11. Перинатальная смертность рассчитывается по следующей формуле:A. Количество мертворожденных/ количество родов *1000B. Количество умерших в первые 168 часов/ количество новорожденных *1000C. Количество мертворожденных и умерших новорожденных в первые 168 часов/ количество родившихся живыми и мертвыми *1000D. Количество мертворожденных и умерших новорожденных в первые 168 часов/ количество родившихся живыми и мертвыми *100E. Количество мертворожденных и умерших новорожденных в первые 168 часов/ количество родившихся живыми и мертвыми *1000012. Наиболее важным мероприятием по снижению перинатальной смертности является:

A. Своевременное решение вопроса о возможности сохранения беременностиB. Проведение анализа причин перинатальной смертностиC. Полноценная диета беременнойD. Своевременная диагностика угрожаемых состояний плодаE. Улучшение санитарно-просветительной работы с беременными13.Сколько существует уровней родовспомогательных учреждений в РеспубликеКазахстан для госпитализации беременных, рожениц и родильниц:A. 1B. 2C. 3D. 4E. 514. Механизм действия гормональной контрацепции:A. Блокада овуляции путем торможения секреции рилизинг-факторов ЛГ и ФСГB. Блокада биохимических и биофизических процессов в эндометрииC. Сгущение цервикальной слизиD. Усиление перистальтики маточных трубE. Повышение кислотности влагалища15. Перинатальный центр организовывается на количество населения:A. 300 тысячB. 500 тысячC. 600 тысячD. 800 тысячE. 1млн.16. Кариопикнотический индекс - это отношение:

Page 95: postgraduate.kaznmu.kz€¦ · Web view58 Больная И, 28 лет, доставлена с жалобами на внезапно возникшую боль внизу живота

С.Ж.АСФЕНДИЯРОВАТЫНДАҒЫҚАЗАҚ ҰЛТТЫҚ МЕДИЦИНА

УНИВЕРСИТЕТІ

КАЗАХСКИЙНАЦИОНАЛЬНЫЙМЕДИЦИНСКИЙ УНИВЕРСИТЕТ ИМЕНИ С.Д.АСФЕНДИЯРОВА

КАФЕДРА ИНТЕРНАТУРЫ И РЕЗИДЕНТУРЫ ПО АКУШЕРСТВУ И ГИНЕКОЛОГИИТЕСТОВЫЕ ЗАДАНИЯ ДЛЯ ПРОВЕДЕНИЯ ИТОГОВОЙ ГОСУДАРСТВЕННОЙ АТТЕСТАЦИИ РЕЗИДЕНТОВ

ВЫПУСКА 2012-2013 ГГ. ПО СПЕЦИАЛЬНОСТИ «АКУШЕРСТВО И ГИНЕКОЛОГИЯ»

A. количества эозинофильных клеток к общему количеству поверхностных клеток в мазке B. количества клеток с пикнотическими ядрами к общему количествуповерхностных клеток в мазкеC. количества базальных и парабазальных клеток к общему количествуповерхностных клеток в мазкеD. количества клеток с пикнотическими ядрами к количеству эозинофильных клеток в мазкеE. количества эозинофильных клеток к количеству поверхностных клеток в мазке17. Измерение ректальной температуры у пациентки 23 лет с нарушением менструального цикла показало, что подъем температуры до 37,3С произошел на 14 день менструального цикла и сохранялся в течение 7 дней. Указанные данные характерны для:A. Нормального двухфазного циклаB. Ановуляторного циклаC. Недостаточности лютеиновой фазыD. Недостаточности фолликулиновой фазыE. Удлинения лютеиновой фазы18. Как влияют спермициды на течение ВИЧ инфекции:A. Повышают риск трансмиссии ВИЧ инфекции при половых контактахB. Не влияют на риск трансмиссии ВИЧ инфекции при половых контактахC. Снижают риск трансмиссии ВИЧ инфекции при половых контактахD. Ухудшают течение инфекции ВИЧ инфекцииE. Надежный вид контрацепции19. Консультативная помощь - это:A. Вид психологической и медицинской поддержкиB. Вид правового влиянияC. Директивный вид отношений с пациентомD. Информирование пациента без объяснения последствийE. Оказание медицинской помощи20. Для ВИЧ инфицированной пациентки метод лактационной аменореи (МЛА)является методом контрацепции:A. НеправильнымB. НадежнымC. Предпочтительным в экономически неразвитых странахD. Эффективным в течении одного месяца после родовE. Эффективным при применении со спермицидами21. В анализах крови у ВИЧ инфицированных пациенток наиболее характерны следующиеизменения:A. Анемия (ниже 80 г/л ), нейтропия (менее 100000 мм3) в течение 30 днейB. Полицитемия, гиперкоагуляцияC. Лейкоцитоз со сдвигом формулы влево, повышение СОЭ D. Анемия, появление бластных клетокE. Тромбоцитопения, лейкоцитоз или лейкопения22.Что такое политика охраны репродуктивного здоровья:A. …Безопасное материнство, планирование семьиB. Увеличение количества школы мамC. Планирование семьи и повышение количества абортов в целях снижения количества нежелательных беременностейD. Планирование семьиE. Экономический рост рождаемости

Page 96: postgraduate.kaznmu.kz€¦ · Web view58 Больная И, 28 лет, доставлена с жалобами на внезапно возникшую боль внизу живота

С.Ж.АСФЕНДИЯРОВАТЫНДАҒЫҚАЗАҚ ҰЛТТЫҚ МЕДИЦИНА

УНИВЕРСИТЕТІ

КАЗАХСКИЙНАЦИОНАЛЬНЫЙМЕДИЦИНСКИЙ УНИВЕРСИТЕТ ИМЕНИ С.Д.АСФЕНДИЯРОВА

КАФЕДРА ИНТЕРНАТУРЫ И РЕЗИДЕНТУРЫ ПО АКУШЕРСТВУ И ГИНЕКОЛОГИИТЕСТОВЫЕ ЗАДАНИЯ ДЛЯ ПРОВЕДЕНИЯ ИТОГОВОЙ ГОСУДАРСТВЕННОЙ АТТЕСТАЦИИ РЕЗИДЕНТОВ

ВЫПУСКА 2012-2013 ГГ. ПО СПЕЦИАЛЬНОСТИ «АКУШЕРСТВО И ГИНЕКОЛОГИЯ»

23. По определению ВОЗ репродуктивное здоровье - это:A. Состояние полного физического здоровьяB. Отсутствие болезней репродуктивной системыC. Состояние полного психического здоровьяD. Состояние социального благополучияE. Счастливый брак24. Репродуктивное право - это:A. Право регулировать количество детейB. Интервал между родами 1 годC. Бесплатная медицинская помощьD. Дополнительная заработная платаE. Обеспечение жильем25. Перечислите основные функции женской консультации:A. Обеспечить ранний охват беременных, проведение психопрофилактической подготовки к родамB. Обследование и лечение беременных и гинекологических больных гинекологических заболеваний, планирование семьиC. Профилактика осложнений во время беременности, родов, после родов, планирование семьиD. Диспансеризация, совместная работа с родильными домамиE. Пропаганда здорового образа жизни26. Показатель перинатальной смертности (ПС) рассчитывается по следующей формуле: число мертворожденных1. ПС = -------------------------------------------- х1000 число родов

число умерших интранатально2. ПС = -------------------------------------------- х1000 число родившихся мертвыми

число мертворожденных число умерших в первые 168 часов после рождения3. ПС = ------------------------------------------------------- х1000 число родившихся живыми и мертвыми

число умерших антенатально4. ПС = ------------------------------------------ х1000 число родившихся мертвыми

число мертворожденных число умерших в первые 168 часов после рождения5. ПС = ------------------------------------------------------- х1000 число родившихся живыми. 27. Показатель интранатальной смертности рассчитывается по следующей формуле: число мертворожденных 1. ИС = ------------------------------------- х1000 число родов

число умерших интранатально

Page 97: postgraduate.kaznmu.kz€¦ · Web view58 Больная И, 28 лет, доставлена с жалобами на внезапно возникшую боль внизу живота

С.Ж.АСФЕНДИЯРОВАТЫНДАҒЫҚАЗАҚ ҰЛТТЫҚ МЕДИЦИНА

УНИВЕРСИТЕТІ

КАЗАХСКИЙНАЦИОНАЛЬНЫЙМЕДИЦИНСКИЙ УНИВЕРСИТЕТ ИМЕНИ С.Д.АСФЕНДИЯРОВА

КАФЕДРА ИНТЕРНАТУРЫ И РЕЗИДЕНТУРЫ ПО АКУШЕРСТВУ И ГИНЕКОЛОГИИТЕСТОВЫЕ ЗАДАНИЯ ДЛЯ ПРОВЕДЕНИЯ ИТОГОВОЙ ГОСУДАРСТВЕННОЙ АТТЕСТАЦИИ РЕЗИДЕНТОВ

ВЫПУСКА 2012-2013 ГГ. ПО СПЕЦИАЛЬНОСТИ «АКУШЕРСТВО И ГИНЕКОЛОГИЯ»

2. ИС = ------------------------------------------------------- х1000 число родившихся живыми и мертвыми

число мертворожденных число умерших в первые 168 часов после рождения3. ИС = ------------------------------------------------------- х1000 число родившихся живыми и мертвыми

число умерших антенатально4. ИС = ------------------------------------------ х1000 число родившихся мертвыми

число мертворожденных число умерших в первые 168 часов после рождения 5. ИС = ------------------------------------------------------- х1000 число родившихся живыми. 28. Показатель антенатальной смертности рассчитывается по следующей формуле: число мертворожденных 1. АС = ------------------------------------- х1000 число родов

число умерших интранатально2. АС = -------------------------------------------- х1000 число родившихся мертвыми

число мертворожденных число умерших в первые 168 часов после рождения3. АС = ------------------------------------------------------- х1000 число родившихся живыми и мертвыми

число умерших антенатально4. АС = ------------------------------------------------------- х1000 число родившихся живыми и мертвыми

число мертворожденных число умерших в первые 168 часов после рождения 5. АС = ------------------------------------------------------- х1000 число родившихся живыми. 29. Понятие «перинатальная смертность» означает:A. Смертность детей в течение первого месяца жизниB. Мертворождаемость C. постнатальная смертностьD. Мертворождаемость смертность детей в течение первых 7 дней жизниE. Мертворождаемость смертность детей в течение 1 месяца жизниF. Мертворождаемость смертность детей в течение 1 года жизни30. Понятие «мертворождаемость» объединяет:A. Смертность детей в течение первых 7 дней жизни B. Смертность детей в течение первого месяца жизни

Page 98: postgraduate.kaznmu.kz€¦ · Web view58 Больная И, 28 лет, доставлена с жалобами на внезапно возникшую боль внизу живота

С.Ж.АСФЕНДИЯРОВАТЫНДАҒЫҚАЗАҚ ҰЛТТЫҚ МЕДИЦИНА

УНИВЕРСИТЕТІ

КАЗАХСКИЙНАЦИОНАЛЬНЫЙМЕДИЦИНСКИЙ УНИВЕРСИТЕТ ИМЕНИ С.Д.АСФЕНДИЯРОВА

КАФЕДРА ИНТЕРНАТУРЫ И РЕЗИДЕНТУРЫ ПО АКУШЕРСТВУ И ГИНЕКОЛОГИИТЕСТОВЫЕ ЗАДАНИЯ ДЛЯ ПРОВЕДЕНИЯ ИТОГОВОЙ ГОСУДАРСТВЕННОЙ АТТЕСТАЦИИ РЕЗИДЕНТОВ

ВЫПУСКА 2012-2013 ГГ. ПО СПЕЦИАЛЬНОСТИ «АКУШЕРСТВО И ГИНЕКОЛОГИЯ»

C. Интранатальную и раннюю неонатальную D. Антенатальную и раннюю неонатальную E. Антенатальную и интранатальную смертность 31. Наибольший удельный вес в структуре перинатальной смертности, какправило, имеет:A. Антенатальная смертностьB. Интранатальная смертностьC. Ранняя неонатальная смертностьD. Неонатальная смертностьE. Младенческая32.Осмотр каких специалистов является обязательным при физиологической беременности:A. Терапевт, окулистB. Терапевт, стоматологC. Терапевт, венерологD. Терапевт, лор -врачE. Лор-врач, стоматолог33. При физиологически протекающей беременности осмотр стоматолога должен быть в следующие сроки гестации:A. В 12 недель и 20 недель B. В 12 недель и 36 недельC. В 16 недель и 32 неделиD. При первой явке и 38 недельE. При первой явке.34. Основной показатель, характеризующий работу женской консультации: A. Ранний охват беременных врачебным наблюдениемB. Частота преждевременных родовC. Своевременная выдача декретного отпускаD. Показатель перинатальной смертностиE. Частота осмотра терапевтом. 35. Ведение беременных женщин в женской консультации проводится методом:A. Оказания медицинской помощи при обращении пациенток в женскую консультациюB. Профилактических осмотров на предприятияхC. Периодических осмотров декретированного контингентаD. Диспансерного наблюденияE. Патронажа36. Раннее выявление и профилактика рака половых органов у женщиносуществляется при:A. Обращении женщин в ж/кB. Обращении женщин в гинекологическое отделениеC. Профилактических осмотрахD. Диспансеризации беременныхE. Обращении женщин в санатории37. Предварительные профилактические осмотры проводятся при:A. ТрудоустройствеB. Увольнении с работыC. Направлении на санаторно-курортное лечениеD. Направлении на стационарное лечениеE. Смене места жительства

Page 99: postgraduate.kaznmu.kz€¦ · Web view58 Больная И, 28 лет, доставлена с жалобами на внезапно возникшую боль внизу живота

С.Ж.АСФЕНДИЯРОВАТЫНДАҒЫҚАЗАҚ ҰЛТТЫҚ МЕДИЦИНА

УНИВЕРСИТЕТІ

КАЗАХСКИЙНАЦИОНАЛЬНЫЙМЕДИЦИНСКИЙ УНИВЕРСИТЕТ ИМЕНИ С.Д.АСФЕНДИЯРОВА

КАФЕДРА ИНТЕРНАТУРЫ И РЕЗИДЕНТУРЫ ПО АКУШЕРСТВУ И ГИНЕКОЛОГИИТЕСТОВЫЕ ЗАДАНИЯ ДЛЯ ПРОВЕДЕНИЯ ИТОГОВОЙ ГОСУДАРСТВЕННОЙ АТТЕСТАЦИИ РЕЗИДЕНТОВ

ВЫПУСКА 2012-2013 ГГ. ПО СПЕЦИАЛЬНОСТИ «АКУШЕРСТВО И ГИНЕКОЛОГИЯ»

38. На одном акушерско-гинекологическом участке проживает (человек):A. 100-200B. 1000-2000C. 3000-5000D. 4000 -6000E. 5000-700039. Учетная форма (УФ) смотрового кабинета:A. Индивидуальная карта беременной (УФ № 111)B. Карта диспансерного наблюдения (УФ №30)C. Медицинская карта амбулаторного больного (УФ №25)D. История родов (УФ №113)E. Обменная карта (УФ №15)40. Обязательным методом обследования женщин при профосмотрах является:A. Онкоцитологическое исследование мазковB. Обзорная рентгенография брюшной полостиC. УЗИ органов малого тазаD. ГистероскопияE. Общий анализ крови41. Основная цель профилактических осмотров у женщин:A. Выявление пороков развития половых органовB. Ранняя диагностика нарушений менструального циклаC. Ранняя диагностика рака шейки маткиD. Ранняя диагностика хронических заболеваний гениталийE. Диагностика беременности42. В составе поликлиник для раннего выявления онкозаболеваний женских половых органов имеется:A. Эндоскопический кабинетB. Клиническая лабораторияC. Рентгенологический кабинетD. Отделение функциональной диагностикиE. Смотровой кабинет43. В мире в структуре материнской смертности первое место занимает:A. Гипертензия, вызванная беременностьюB. Экстрагенитальная патологияC. Послеродовое кровотечениеD. Послеродовый сепсисE. Разрыв матки44. Первое место в структуре ранней неонатальной смертности в мире занимает:A. Родовая травмаB. Врожденные пороки развитияC. ИнфекцияD. Незрелость легкихE. Асфиксия45. При физиологически протекающей беременности осмотр терапевта должен быть в следующие сроки гестации:A. В 12 недель и 20 недельB. В 12 недель и 30 недельC. При первой явке и 36 недель

Page 100: postgraduate.kaznmu.kz€¦ · Web view58 Больная И, 28 лет, доставлена с жалобами на внезапно возникшую боль внизу живота

С.Ж.АСФЕНДИЯРОВАТЫНДАҒЫҚАЗАҚ ҰЛТТЫҚ МЕДИЦИНА

УНИВЕРСИТЕТІ

КАЗАХСКИЙНАЦИОНАЛЬНЫЙМЕДИЦИНСКИЙ УНИВЕРСИТЕТ ИМЕНИ С.Д.АСФЕНДИЯРОВА

КАФЕДРА ИНТЕРНАТУРЫ И РЕЗИДЕНТУРЫ ПО АКУШЕРСТВУ И ГИНЕКОЛОГИИТЕСТОВЫЕ ЗАДАНИЯ ДЛЯ ПРОВЕДЕНИЯ ИТОГОВОЙ ГОСУДАРСТВЕННОЙ АТТЕСТАЦИИ РЕЗИДЕНТОВ

ВЫПУСКА 2012-2013 ГГ. ПО СПЕЦИАЛЬНОСТИ «АКУШЕРСТВО И ГИНЕКОЛОГИЯ»

D. В 16 недель и 32 неделиE. При первой явке и 30 недель46. Воздействие вредных факторов на плод в первый критический периодразвития плода приводит к:A. Порокам развития плодаB. Эмбриотоксическому эффектуC. Первичной плацентарной недостаточности D. Вторичной плацентарной недостаочностиE. Задержке внутриутробного развития плода47. Воздействие вредных факторов на плод во второй критический периодразвития плода приводит к:A. Порокам развития плодаB. Эмбриотоксическому эффектуC. Первичной плацентарной недостаточности D. Вторичной плацентарной недостаочностиE. Многоводию48. Бластопатия - это заболевание эмбриона, развившееся в период:A. ОплодотворенияB. ИмплантацииC. Развития осевых зачатковD. ПлацентацииЕ. Органогенеза49. Эмбриопатия - это заболевание плода, развившееся в сроки:A. До 10-11 недельB. 16-17 недельC. 20-22 неделиD. 26-27 недель1. 30-32 недель50. Укажите формулу среднего артериального давления, показывающую периферическое сосудистое сопротивление (САД – систолическое АД, ДАД – диастолическое АД): A. (САД2ДАД)/3 B. (САДДАД)/2 C. (2САД2ДАД)/3 D. (САД2ДАД)/2 E. (2САДДАД)/3 51. Уровень сахара в крови при сахарном диабете І стадии:A. 7.7ммоль/л B. 7.9ммоль/лC. 8.0ммоль/лD. 8.1ммоль/лE. 8.2ммоль/л52. Антидиабетические препараты, противопоказанные при беременности:A. Пероральные антидиабетические B. Простой инсуллинC. Инсуллин-цинк суспензионный аморфныйD. Инсуллин-цинк суспензионный E. Инсуллин-протамин53. В каком сроке беременности начинает функционировать щитовидная железа плода:

Page 101: postgraduate.kaznmu.kz€¦ · Web view58 Больная И, 28 лет, доставлена с жалобами на внезапно возникшую боль внизу живота

С.Ж.АСФЕНДИЯРОВАТЫНДАҒЫҚАЗАҚ ҰЛТТЫҚ МЕДИЦИНА

УНИВЕРСИТЕТІ

КАЗАХСКИЙНАЦИОНАЛЬНЫЙМЕДИЦИНСКИЙ УНИВЕРСИТЕТ ИМЕНИ С.Д.АСФЕНДИЯРОВА

КАФЕДРА ИНТЕРНАТУРЫ И РЕЗИДЕНТУРЫ ПО АКУШЕРСТВУ И ГИНЕКОЛОГИИТЕСТОВЫЕ ЗАДАНИЯ ДЛЯ ПРОВЕДЕНИЯ ИТОГОВОЙ ГОСУДАРСТВЕННОЙ АТТЕСТАЦИИ РЕЗИДЕНТОВ

ВЫПУСКА 2012-2013 ГГ. ПО СПЕЦИАЛЬНОСТИ «АКУШЕРСТВО И ГИНЕКОЛОГИЯ»

A. 5-6 недельB. 12-16 недельC. 25-28 недельD. 32-34 неделиE. 38-39 недель54.Цель инфузионной терапии при гипертензии беременных тяжелой степени:A. ДезинтоксикацияB. ГемоконцентрацияC. Устранение гиповолемииD. Нормализация адекватного диуреза E. Десенсибилизация55.Патогенетическое действие магния сульфата при лечении тяжелой преэклампсии:A. Гипотензивное B. Седативное C. Мочегонное D. Профилактика судорогE. Анальгезирующее56. Какое содержание белка в суточной моче расценивают как протеинурию:A. 0,003 г и вышеB. 0,033 г и вышеC. 0,3 г и выше D. 1,0 г и вышеE. 1,5 г и выше57. Беременная К. поступила в роддом с диагнозом: Беременность 36-37 недель, преэклампсия тяжелой степени. Тактика акушера: A. Ограничиться комплексным лечением в течение 12-24 часовB. Комплексное лечение с одновременной подготовкой организма к родам, родоразрешить в течение 72 часовC. Подготовка организма к родам пропедил-гелем в течение 72 часов D. Комплексное лечение с одновременной подготовкой организма к родам, родоразрешить в течение 24-48 часовE. Родоразрешить путем операции кесарево сечение в экстренном порядке58. У беременной в сроке гестации 35 недель на приеме в женской консультации измерено АД. Впервые обнаружено повышение АД до -140/90 мм ртст. Тактика врача:A. Немедленная госпитализация в роддомB. Произвести повторное измерение АД через 30 минут C. Консультация терапевтаD. Консультация невропатологаE. Консультация окулиста59. В гомеостазе переношенного новорожденного наблюдается:A. ГиперкоагуляцияB. ГипокоагуляцияC. Повышение АЧТВD. Активирование плазменного звенаE. Активирование эритроцитарного звена60. Гормональная функция плаценты при переношенной беременности:A. СнижаетсяB. Повышается

Page 102: postgraduate.kaznmu.kz€¦ · Web view58 Больная И, 28 лет, доставлена с жалобами на внезапно возникшую боль внизу живота

С.Ж.АСФЕНДИЯРОВАТЫНДАҒЫҚАЗАҚ ҰЛТТЫҚ МЕДИЦИНА

УНИВЕРСИТЕТІ

КАЗАХСКИЙНАЦИОНАЛЬНЫЙМЕДИЦИНСКИЙ УНИВЕРСИТЕТ ИМЕНИ С.Д.АСФЕНДИЯРОВА

КАФЕДРА ИНТЕРНАТУРЫ И РЕЗИДЕНТУРЫ ПО АКУШЕРСТВУ И ГИНЕКОЛОГИИТЕСТОВЫЕ ЗАДАНИЯ ДЛЯ ПРОВЕДЕНИЯ ИТОГОВОЙ ГОСУДАРСТВЕННОЙ АТТЕСТАЦИИ РЕЗИДЕНТОВ

ВЫПУСКА 2012-2013 ГГ. ПО СПЕЦИАЛЬНОСТИ «АКУШЕРСТВО И ГИНЕКОЛОГИЯ»

C. Повышаетя постепенноD. Повышается с последующим снижениемE. Снижается с последующим повышением61. Показанием к операции кесарево сечения при переношенной беременности является:A. Продольное положение плодаB. Возраст старше 30 летC. Юный возрастD. Тазовое предлежание плодаE. Паритет родов62. При поступлении в роддом повторнородящей Н., 26 лет, выставлен диагноз: Беременность 42 недели. Объективно: положение плода продольное, предлежит головка, сердцебиение плода ясное, ритмичное, 140 уд. в мин. Предполагаемая масса плода 3600,0. Влагалищное исследование: шейка матки «зрелая». Тактика врача: A. Подготовка организма к родам в течение 2 –х днейB. Подготовка организма к родам в течение 3 –х днейC. Амниотомия, родовозбуждениеD. Экстренная операция кесарева сеченияE. Плановая операция кесарева сечения63.При поступлении в роддом первородящей Н., 25 лет, выставлен диагноз: Беременность 42 недели. Объективно: положение плода продольное, предлежит головка, сердцебиение плода ясное, ритмичное, 140 уд/ мин. Предполагаемая масса плода 4100,0. Влагалищное исследование: шейка матки «незрелая». Тактика врача: A. Подготовка организма к родам в течение 2 –х днейB. Подготовка организма к родам в течение 3 –х днейC. Амниотомия, родовозбуждениеD. Экстренная операция кесарева сеченияE. Плановая операция кесарева сечения64. При поступлении в роддом первородящей Н., 28 лет, выставлен диагноз: Беременность 42 недели. Тазовое предлежание плода. Дородовое излитие околоплодных вод. Объективно: положение плода продольное, предлежит тазовый конец, сердцебиение плода ясное, ритмичное, 140 уд. в мин. Предполагаемая масса плода 3800,0. Влагалищное исследование: шейка матки «незрелая», околоплодные воды светлые. Тактика врача: A. Подготовка организма к родам в течение сутокB. Подготовка организма к родам в течение 3 –х днейC. Амниотомия, родовозбуждениеD. Экстренная операция кесарева сеченияE. Плановая операция кесарева сечения65. Основной признак плоско - рахитического таза:A. Крестцовый мыс смещен в полость таза B. Гребни подвздошных костей расправленыC. Гребни подвздошных костей широко расставленыD. Крестец утолщен, плоскийЕ. Увеличены размеры выхода малого таза66. Первородящей Н., 25 лет, выставлен диагноз: Беременность 42 недели. Первый период родов. Вторичная слабость родовой деятельности. Объективно: положение плода продольное, предлежит головка, сердцебиение плода ясное, ритмичное, 140 уд. в мин. Схватки через 5-6 минут по 25 сек, слабой силы. Предполагаемая масса плода 3500,0. Тактика врача: A. Амниотомия

Page 103: postgraduate.kaznmu.kz€¦ · Web view58 Больная И, 28 лет, доставлена с жалобами на внезапно возникшую боль внизу живота

С.Ж.АСФЕНДИЯРОВАТЫНДАҒЫҚАЗАҚ ҰЛТТЫҚ МЕДИЦИНА

УНИВЕРСИТЕТІ

КАЗАХСКИЙНАЦИОНАЛЬНЫЙМЕДИЦИНСКИЙ УНИВЕРСИТЕТ ИМЕНИ С.Д.АСФЕНДИЯРОВА

КАФЕДРА ИНТЕРНАТУРЫ И РЕЗИДЕНТУРЫ ПО АКУШЕРСТВУ И ГИНЕКОЛОГИИТЕСТОВЫЕ ЗАДАНИЯ ДЛЯ ПРОВЕДЕНИЯ ИТОГОВОЙ ГОСУДАРСТВЕННОЙ АТТЕСТАЦИИ РЕЗИДЕНТОВ

ВЫПУСКА 2012-2013 ГГ. ПО СПЕЦИАЛЬНОСТИ «АКУШЕРСТВО И ГИНЕКОЛОГИЯ»

B. РодостимуляцияC. Амниотомия с последующей родостимуляциейD. Экстренная операция кесарева сеченияE. Родостимуляция, при безэффективности - операция кесарево сечение67. Первородящей Н., 23 лет, выставлен диагноз: Беременность 40 недель. Первый период родов. Первичная слабость родовой деятельности. Объективно: положение плода продольное, предлежит головка, сердцебиение плода ясное, ритмичное, 140 уд/ мин. Схватки через 5-6 минут по 25 сек, слабой силы. Предполагаемая масса плода 3500,0. Тактика врача: A. АмниотомияB. РодостимуляцияC. Амниотомия с последующей родостимуляциейD. Экстренная операция кесарева сеченияE. Акушерский сон - отдых68.Первородящей Н., 25 лет, выставлен диагноз: Беременность 40 недель. Первый период родов. Вторичная слабость родовой деятельности. Объективно: роженица уставшая, хочет спать. Положение плода продольное, предлежит головка, сердцебиение плода ясное, ритмичное, 140 уд/ мин. Околоплодные воды – целы. Схватки через 5-6 минут по 25 сек, слабой силы. Предполагаемая масса плода 3300,0. Тактика врача: A. АмниотомияB. РодостимуляцияC. Амниотомия с последующей родостимуляциейD. Экстренная операция кесарева сеченияE. Акушерский сон – отдых69. У первородящей Н., 23 лет, при обследовании определены размеры таза: D.Sp- 26 см, D.Cr.- 28 см, D.Tr . – 31 см, C.ext.- 17 см. При влагалищном исследовании мыс достижим, C.diag 10 см. Ваш диагноз:A. Общеравномерно суженый таз I степениB. Простой плоский таз I степениC. Косо-суженный тазD. Поперечно-суженный таз II степениE. Плоско-рахитический таз II степени70. Максимальное сгибание головки, длительное прижатие головки ко входу в малый таз. Какому виду узкого таза соответствует данный биомеханизм родов:A. Общеравномерносуженному B. ПлоскорахитическомуC. Нормальному D. Простому плоскому E. Поперечно-суженному71. Беременность 28 недель, отеки на пердней стенке живота, голенях. АД 160/110 мм рт.ст., 17/110 мм рт.ст., белок в моче 5,0 г/л. Ваш диагноз: A. Артериальная гипертензия на фоне заболевания почекB. Преэклампсия легкой степениC. Отеки беременныхD. Преэклампсия тяжелой степениE. Гестационная гипертензия72. На приеме в женской консультации беременной в сроке гестации 28 недель выставлен диагноз: Преэклампсия тяжелой степени. До приезда скорой помощи необходимо ввести:A. Диазепам

Page 104: postgraduate.kaznmu.kz€¦ · Web view58 Больная И, 28 лет, доставлена с жалобами на внезапно возникшую боль внизу живота

С.Ж.АСФЕНДИЯРОВАТЫНДАҒЫҚАЗАҚ ҰЛТТЫҚ МЕДИЦИНА

УНИВЕРСИТЕТІ

КАЗАХСКИЙНАЦИОНАЛЬНЫЙМЕДИЦИНСКИЙ УНИВЕРСИТЕТ ИМЕНИ С.Д.АСФЕНДИЯРОВА

КАФЕДРА ИНТЕРНАТУРЫ И РЕЗИДЕНТУРЫ ПО АКУШЕРСТВУ И ГИНЕКОЛОГИИТЕСТОВЫЕ ЗАДАНИЯ ДЛЯ ПРОВЕДЕНИЯ ИТОГОВОЙ ГОСУДАРСТВЕННОЙ АТТЕСТАЦИИ РЕЗИДЕНТОВ

ВЫПУСКА 2012-2013 ГГ. ПО СПЕЦИАЛЬНОСТИ «АКУШЕРСТВО И ГИНЕКОЛОГИЯ»

B. Сульфат магния C. Диуретики D. ГанглиоблокаторыE. Эуфиллин73. Доставлена беременная без сознания в сроке гестации 31-32 недели. Дома был приступ судорог. Состояние тяжелое, без сознания. АД 180/100 мм рт.ст., АД 190/110 мм рт.ст. Генерализованные отеки. Сердцебиение плода ясное, ритмичное 140 ударов в минуту. Ваш диагноз:A. Беременность 31-32 недели. ЭпилепсияB. Беременность 31-32 недели. ГломерулонефритC. Беременность 31-32 недели. Преэклампсия тяжелой степениD. Беременность 31-32 недели. Эклампсия. Кома. E. Беременность 31-32 недели. Острая почечная недостаточность74. Беременной при сроке гестации 31-32 недели выставлен диагноз: Эклампсия. Объективно: состояние тяжелое, без сознания. АД 180/100 мм рт.ст. Генерализованные отеки. Сердцебиение плода ясное, ритмичное 140 ударов в минуту. Вагинально: шейка матки «незрелая». Врачебная тактика:A. Пролонгировать беременность до 34 недель беременностиB. Пролонгировать беременность до 36 недель беременности C. Пролонгировать беременность до 38 недель беременности D. Амниотомия, родовозбуждениеE. Стабилизация состояния. Операция кесарево сечение75. Ведущее звено в патогенезе гипертензии беременных:A. Обширная вазоконстрикция, гиповолемия, повреждение эндотелия, ДВС синдром B. Нарушение плацентарного барьера, обширная вазоконстрикция, ДВС синдромC. Нарушение водно-солевого обмена, обширная вазоконстрикция, нарушение реологииD. Нарушения в ЦНС, обширная вазоконстрикция, повреждение эндотелия, ДВС синдромE. Нарушение маточно-плацентарного кровообращения, повреждение эндотелия, ДВС синдром76. Беременная 24 лет обратилась с жалобами на боль в эпигастральной области. При осмотре: Беременность 31-32 недели. АД 160/110 мм рт.ст., отеки на животе, на нижних конечностях. Моча при кипячении мутная. Ваш диагноз:A. Беременность 31-32 недель. Гестационная гипертензияB. Беременность 31-32 недель. Преэклампсия легкой степениC. Беременность 31-32 недель. Преэклампсия средней степени.D. Беременность 31-32 недель. Преэклампсия тяжелой степени. E. Беременность 31-32 недель. Эклампсия 77. Приступ эклампсии наименее вероятно может развиться:A. Во время беременностиB. Во время родовC. В раннем послеродовом периоде.D. В позднем послеродовом периоде. E. Через 6-8 недель после родов78. Согласно клиническим протоколам родоразрешение при преэклампсии тяжелой степени должно быть произведено:A. Сразу при поступленииB. В течение 2-3 часовC. В течение 4-6 часовD. В течение 8-10 часов

Page 105: postgraduate.kaznmu.kz€¦ · Web view58 Больная И, 28 лет, доставлена с жалобами на внезапно возникшую боль внизу живота

С.Ж.АСФЕНДИЯРОВАТЫНДАҒЫҚАЗАҚ ҰЛТТЫҚ МЕДИЦИНА

УНИВЕРСИТЕТІ

КАЗАХСКИЙНАЦИОНАЛЬНЫЙМЕДИЦИНСКИЙ УНИВЕРСИТЕТ ИМЕНИ С.Д.АСФЕНДИЯРОВА

КАФЕДРА ИНТЕРНАТУРЫ И РЕЗИДЕНТУРЫ ПО АКУШЕРСТВУ И ГИНЕКОЛОГИИТЕСТОВЫЕ ЗАДАНИЯ ДЛЯ ПРОВЕДЕНИЯ ИТОГОВОЙ ГОСУДАРСТВЕННОЙ АТТЕСТАЦИИ РЕЗИДЕНТОВ

ВЫПУСКА 2012-2013 ГГ. ПО СПЕЦИАЛЬНОСТИ «АКУШЕРСТВО И ГИНЕКОЛОГИЯ»

E. В течение 24-48 часов79. Показание для ИВЛ при гипертензионном состоянии:A. Преэклампсия легкой степени.B. Преэклампсия средней степени. C. Преэклампсия тяжелой степени.D. Дыхательная недостаточность E. Артериальная гипертония80. В первую очередь при оказании неотложной помощи при приступе эклампсии необходимо:A. Уложить на левый бок, предотвратить механическую асфиксию B. Обеспечить подачу кислородаC. Уложить на спину, предотвратить ушибы D. Ввести в/в сульфат магнияE. Ввести в/в промедол81. Влияние гипертензии беременных на плод: A. Преждевременное истощение функции коры надпочечниковB. Развивитие гиперволемииC. Развивитие гиперкетонурииD. Врожденные аномалии почекE. Перенашивание беременности82. При инфузионной терапии у больных с гипертензией беременных гематокрит не должен быть ниже:A. 27-28%B. 25-26%C. 20-24%D. 29-30%E. 32-34%83. В первом периоде родов у беременных с преэклампсией необходимо предусмотреть:A. Эпидуральную анестезиюB. Спазмолитики C. Магния сульфат D. НейролептикиE. Анальгетики84. Основные клинические признаки преэклампсии:А. Артериальная гипертензияотекиВ. ОтекипротеинурияA. Артериальная гипертензия отекиB. ПротеинурияC. Артериальная гипертензия протеинурия85. Показание для досрочного родоразрешения при преэклампсии:A. Преэклампсия легкой степениB. Хроническая гипоксия плодаC. Предлежание плацентыD. Преэклампсия тяжелой степениE. Фетоплацентарная недостаточность86. Методом выбора противосудорожной терапии при гипертензиях беременных является:A. Магнезиальная терапияB. Спазмолитическая терапияC. Гипотензивная терапия

Page 106: postgraduate.kaznmu.kz€¦ · Web view58 Больная И, 28 лет, доставлена с жалобами на внезапно возникшую боль внизу живота

С.Ж.АСФЕНДИЯРОВАТЫНДАҒЫҚАЗАҚ ҰЛТТЫҚ МЕДИЦИНА

УНИВЕРСИТЕТІ

КАЗАХСКИЙНАЦИОНАЛЬНЫЙМЕДИЦИНСКИЙ УНИВЕРСИТЕТ ИМЕНИ С.Д.АСФЕНДИЯРОВА

КАФЕДРА ИНТЕРНАТУРЫ И РЕЗИДЕНТУРЫ ПО АКУШЕРСТВУ И ГИНЕКОЛОГИИТЕСТОВЫЕ ЗАДАНИЯ ДЛЯ ПРОВЕДЕНИЯ ИТОГОВОЙ ГОСУДАРСТВЕННОЙ АТТЕСТАЦИИ РЕЗИДЕНТОВ

ВЫПУСКА 2012-2013 ГГ. ПО СПЕЦИАЛЬНОСТИ «АКУШЕРСТВО И ГИНЕКОЛОГИЯ»

D. Диуретическая терапияE. Анальгетическая терапия87. Стартовая – нагрузочная доза магния сульфата составляет:A. 1 г сухого вещества сульфата магния в/в медленно в течение 10-15 минутB. 2 г сухого вещества сульфата магния в/в медленно в течение 10-15 минутC. 3 г сухого вещества сульфата магния в/в медленно в течение 10-15 минутD. 4 г сухого вещества сульфата магния в/в медленно в течение 10-15 минутE. 5 г сухого вещества сульфата магния в/в медленно в течение 10-15 минут 88. Поддерживающая доза сульфата магния составляет:A. 20 мл 25% р-ра на 120 мл физиологического р-ра в/в капельноB. 40 мл 25% р-ра на 220 мл физиологического р-ра в/в капельноC. 50 мл 25% р-ра на 320 мл физиологического р-ра в/в капельноD. 60 мл 25% р-ра на 420 мл физиологического р-ра в/в капельноE. 80 мл 25% р-ра на 320 мл физиологического р-ра в/в капельно89. При каком уровне АД начинают гипотензивную терапию при гипертензии беременных:A. 110/70 мм рт.ст.B. 120/80 мм рт.ст.C. 130/90 мм рт.ст.D. 150/100 мм рт.ст.E. 160/100 мм рт.ст.90. Какой уровень систолического АД поддерживается при проведении гипотензивной терапии у беременных с преэклампсией:A. 110/70 мм рт.ст.B. 120/80 мм рт.ст.C. 130/90 мм рт.ст.D. 150/100 мм рт.ст.E. 160/100 мм рт.ст.91.Тактика врача после классического поворота плода на ножку: A. Применение пособия по Цовьянову B. Кесарево сечение C. Классичесское ручное пособиеD. Экстракция плода за ножкуE. Плодоразрушающая операция 92. Укажите дозу дексаметазона, используемую для профилактики респираторного дистресс -синдрома у плода:A. 5 мгB. 10 мгC. 15 мгD. 24 мгE. 48 мг93. Концентрация альфа-фетопротеина при беременности повышается: A. При патологии плаценты B. При дефекте нервной трубки у плодаC. При патологии почек у плода D. При патологии матки E. При патологии внутренних органов плода 94. В каком сроке беременности определяют альфа-фетопротеин у матери:A. 5-6 недель

Page 107: postgraduate.kaznmu.kz€¦ · Web view58 Больная И, 28 лет, доставлена с жалобами на внезапно возникшую боль внизу живота

С.Ж.АСФЕНДИЯРОВАТЫНДАҒЫҚАЗАҚ ҰЛТТЫҚ МЕДИЦИНА

УНИВЕРСИТЕТІ

КАЗАХСКИЙНАЦИОНАЛЬНЫЙМЕДИЦИНСКИЙ УНИВЕРСИТЕТ ИМЕНИ С.Д.АСФЕНДИЯРОВА

КАФЕДРА ИНТЕРНАТУРЫ И РЕЗИДЕНТУРЫ ПО АКУШЕРСТВУ И ГИНЕКОЛОГИИТЕСТОВЫЕ ЗАДАНИЯ ДЛЯ ПРОВЕДЕНИЯ ИТОГОВОЙ ГОСУДАРСТВЕННОЙ АТТЕСТАЦИИ РЕЗИДЕНТОВ

ВЫПУСКА 2012-2013 ГГ. ПО СПЕЦИАЛЬНОСТИ «АКУШЕРСТВО И ГИНЕКОЛОГИЯ»

B. 10-12 недельC. 16-18 недельD. 18-20 недельE. 15-20 недель95. Децелерация это: A. Учащение сердечных сокрашений плода B. Урежение сердечных сокрашений плодаC. Аритмичное сердцебиение плодаD. Экстрасистолия E. Глухое сердцебиение96. Амниоскопия наиболее информативна для диагностики:A. Гемолитической болезни плода B. Предлежания петель пуповиныC. Предлежания плацентыD. МаловодияE. Многоводия97. На хроническую плацентарную недостаточность указывает:A. Снижение уровня эстриола на одно и более стандартных отклонений от среднего уровня B. Повышение уровня прогестерона на одно и более стандартных отклонений от среднего уровняC. Увеличение экскреции эстрадиола с мочой ниже 16 мг/сутD. Уменьшение экскреции эстрадиола с мочой ниже 26 мг/сутE. Повышение уровня прегнандиола на пять и более стандартных отклонений от среднего уровня98. Для чего определяют «биофизический профиль плода»:A. Дигностика предлежания плодаB. Диагностика положения плодаC. Диагностика угрожаемых состояний плодаD. Диагностика пороков развитияE. Диагностика гемолитической болезни плода99. Амниоцентез – это:A. Исследование оптическим прибором состояния плодного пузыряB. Исследование оптическим прибором состояния околоплодных водC. Пункция амниотической полости, взятие околоплодных вод D. Пункция пуповины плода, взятие крови из пуповины E. Измерение количества амниотической жидкости100. Кордоцентез – это:A. Пункция амниотической полостиB. Взятие крови из пуповины плодаC. Исследование оптическим прибором состояния плодного пузыряD. Исследование оптическим прибором состояния околоплодных водE. Исследование ворсин хориона101. Беременная 23-х лет со сроком гестации 35 недель обратилась с жалобами на головную боль, тошноту, рвоту. Объективно: АД 160/110 мм рт.ст., отеки на животе, на нижних конечностях, пастозность лица. Моча при кипячении мутная. Ваш диагноз:A. Беременность 35 недель. Гестационная гипертензияB. Беременность 35 недель. Преэклампсия легкой степениC. Беременность 35 недель. Преэклампсия тяжелой степени.D. Беременность 35 недель. ЭклампсияE. Беременность 35 недель. Острый гастрит

Page 108: postgraduate.kaznmu.kz€¦ · Web view58 Больная И, 28 лет, доставлена с жалобами на внезапно возникшую боль внизу живота

С.Ж.АСФЕНДИЯРОВАТЫНДАҒЫҚАЗАҚ ҰЛТТЫҚ МЕДИЦИНА

УНИВЕРСИТЕТІ

КАЗАХСКИЙНАЦИОНАЛЬНЫЙМЕДИЦИНСКИЙ УНИВЕРСИТЕТ ИМЕНИ С.Д.АСФЕНДИЯРОВА

КАФЕДРА ИНТЕРНАТУРЫ И РЕЗИДЕНТУРЫ ПО АКУШЕРСТВУ И ГИНЕКОЛОГИИТЕСТОВЫЕ ЗАДАНИЯ ДЛЯ ПРОВЕДЕНИЯ ИТОГОВОЙ ГОСУДАРСТВЕННОЙ АТТЕСТАЦИИ РЕЗИДЕНТОВ

ВЫПУСКА 2012-2013 ГГ. ПО СПЕЦИАЛЬНОСТИ «АКУШЕРСТВО И ГИНЕКОЛОГИЯ»

102. Беременная 23-х лет со сроком гестации 35 недель обратилась с жалобами на головную боль, тошноту, рвоту. Выставлен диагноз: Преэклампсия тяжелой степени. Объективно: матка соответствует 35 неделям беременности, сердцебиение плода ясное, ритмичное, 140 уд/в мин. PV: шейка матки «зрелая», предлежит головка. Акушерская тактика:A. Досрочное родоразрешение через естественные родовые пути B. Досрочное родоразрешение путем операции кесарева сеченияC. Провести профилактику СДР плода в течение 48 часов, после чего родоразрешить через естественные половые путиD. Провести профилактику СДР плода в течение 48 часов, после чего родоразрешить путем операции кесарева сечения E. На фоне проводимой терапии пролонгировать беременность до 38 недель103. Беременная 22-х лет со сроком гестации 36 недель обратилась с жалобами на тошноту, рвоту. Выставлен диагноз: Преэклампсия тяжелой степени. Объективно: матка соответствует 36 неделям беременности, сердцебиение плода ясное, ритмичное, 140 уд/ мин. PV: шейка матки "незрелая", предлежит тазовый конец. Акушерская тактика:A. Досрочное родоразрешение через естественные родовые путиB. Досрочное родоразрешение путем операции кесарева сеченияC. Провести профилактику СДР плода в течение 48 часов, после чего родоразрешитьчерез естественные половые путиD. Провести профилактику СДР плода в течение 48 часов, после чего родоразрешитьпутем операции кесарева сеченияE. На фоне проводимой терапии пролонгировать беременность до 38 недель104. Беременная 29-и лет со сроком гестации 38 недель обратилась с жалобами на боль в эпигастральной области, тошноту, рвоту. Объективно: АД 170/110 мм рт.ст., отеки на животе, на нижних конечностях, пастозность лица. В ОАМ белок 3 г/л. Ваш диагноз:A. Беременность 38 недель. Гестационная гипертензияB. Беременность 38 недель. Преэклампсия легкой степениC. Беременность 38 недель. Преэклампсия тяжелой степени.D. Беременность 38 недель. ЭклампсияE. Беременность 38 недель. Острый гастрит105. У беременной 30-и лет со сроком гестации 35 недель на приеме определено АД 140/90 мм рт.ст. Жалоб нет. Объективно: отеков нет. В ОАМ белок отсутствует. Ваш диагноз:A. Беременность 35 недель. Гестационная гипертензияB. Беременность 35 недель. Преэклампсия легкой степениC. Беременность 35 недель. Преэклампсия тяжелой степени.D. Беременность 35 недель. Хроническая гипертензияE. Беременность 35 недель. Хронический гломерулонефрит106. У беременной 30-и лет со сроком гестации 35 недель на приеме определено АД 140/90 мм рт.ст. Жалоб нет. Выставлен диагноз: Гипертензия, вызванная беременностью. Акушерская тактика:A. Досрочное родоразрешение через естественные родовые пути B. Досрочное родоразрешение путем операции кесарева сеченияC. Провести профилактику СДР плода в течение 48 часов, после чего родоразрешить через естественные половые путиD. Провести профилактику СДР плода в течение 48 часов, после чего родоразрешить путем операции кесарева сечения E. Пролонгировать беременность, при нарастании гипертензии - родоразрешить

Page 109: postgraduate.kaznmu.kz€¦ · Web view58 Больная И, 28 лет, доставлена с жалобами на внезапно возникшую боль внизу живота

С.Ж.АСФЕНДИЯРОВАТЫНДАҒЫҚАЗАҚ ҰЛТТЫҚ МЕДИЦИНА

УНИВЕРСИТЕТІ

КАЗАХСКИЙНАЦИОНАЛЬНЫЙМЕДИЦИНСКИЙ УНИВЕРСИТЕТ ИМЕНИ С.Д.АСФЕНДИЯРОВА

КАФЕДРА ИНТЕРНАТУРЫ И РЕЗИДЕНТУРЫ ПО АКУШЕРСТВУ И ГИНЕКОЛОГИИТЕСТОВЫЕ ЗАДАНИЯ ДЛЯ ПРОВЕДЕНИЯ ИТОГОВОЙ ГОСУДАРСТВЕННОЙ АТТЕСТАЦИИ РЕЗИДЕНТОВ

ВЫПУСКА 2012-2013 ГГ. ПО СПЕЦИАЛЬНОСТИ «АКУШЕРСТВО И ГИНЕКОЛОГИЯ»

107. Беременной 25-и лет со сроком гестации 34 недели выставлен диагноз: Преэклампсия легкой степени. Акушерская тактика: A. Досрочное родоразрешение через естественные родовые пути B. Досрочное родоразрешение путем операции кесарева сеченияC. Провести профилактику СДР плода в течение 48 часов, после чего родоразрешить через естественные половые путиD. Провести профилактику СДР плода в течение 48 часов, после чего родоразрешить путем операции кесарева сечения E. Пролонгировать беременность, при нарастании симптомов - родоразрешить 108. У беременной со сроком гестации 35 недель с задержкой внутриутробного развития плода установлена декомпенсированная плацентарная недостаточность (на допплерометрии- реверсный кровоток). Акушерская тактика:A. Досрочное родоразрешение через естественные родовые пути B. Досрочное родоразрешение путем операции кесарева сеченияC. Провести профилактику СДР плода, после чего родоразрешить через естественные половые путиD. Провести профилактику СДР, после чего родоразрешить путем операции кесарева сечения E. Пролонгировать беременность, при нарастании пацентарной недостаточности - родоразрешить 109. У беременной со сроком гестации 35 недель и задержкой внутриутробного развития плода излились зеленые околоплодные воды, родовой деятельности нет. Акушерская тактика:A. Досрочное родоразрешение через естественные родовые пути B. Досрочное родоразрешение путем операции кесарева сеченияC. Провести профилактику СДР, после чего родоразрешить через естественные половые путиD. Провести профилактику СДР, после чего родоразрешить путем операции кесарева сечения E. Наблюдение в течение 24 часов, при отсутствии спонтанной родовой деятельности - родовозбуждение110. У беременной со сроком гестации 38 недель с задержкой внутриутробного развития плода по данным допплерометрии нарушений маточно-плацентарного и плодово-плацентарного кровотока нет. PV: шейка матки «зрелая», предлежит головка. Акушерская тактика:A. Амниотомия, родовозбуждение B. Экстренная операция кесарева сеченияC. Плановая операция кесарева сечения D. Провести профилактику СДР, после чего родоразрешить через естественные половые путиE. На фоне проводимой терапии пролонгировать беременность до 40 недель111. Показанием для кесарева сечения при фетоплацентарной недостаточности является:A. МаловодиеB. На кардиотокографии базальная частота 120/160 ударов/мин C. Нарушение маточно-плодово –плацентарного кровотока I ст.D. Задержка внутриутробного развития I ст. E. Реверсный или нулевой кровоток по допплерометрии 112. Какие изменения происходят в желудочно-кишечном тракте во время беременности:A. Гиперхлоргидрия B. Повышение моторной функции желудка C. Повышение перистальтики кишечника D. Повышение секреции пепсина E. Гипохлоргидрия113. Какие изменения происходят в сердечно-сосудистой системе во время беременности:A. Гипертрофия левого желудочка, повышение выброса сердца на 40%

Page 110: postgraduate.kaznmu.kz€¦ · Web view58 Больная И, 28 лет, доставлена с жалобами на внезапно возникшую боль внизу живота

С.Ж.АСФЕНДИЯРОВАТЫНДАҒЫҚАЗАҚ ҰЛТТЫҚ МЕДИЦИНА

УНИВЕРСИТЕТІ

КАЗАХСКИЙНАЦИОНАЛЬНЫЙМЕДИЦИНСКИЙ УНИВЕРСИТЕТ ИМЕНИ С.Д.АСФЕНДИЯРОВА

КАФЕДРА ИНТЕРНАТУРЫ И РЕЗИДЕНТУРЫ ПО АКУШЕРСТВУ И ГИНЕКОЛОГИИТЕСТОВЫЕ ЗАДАНИЯ ДЛЯ ПРОВЕДЕНИЯ ИТОГОВОЙ ГОСУДАРСТВЕННОЙ АТТЕСТАЦИИ РЕЗИДЕНТОВ

ВЫПУСКА 2012-2013 ГГ. ПО СПЕЦИАЛЬНОСТИ «АКУШЕРСТВО И ГИНЕКОЛОГИЯ»

B. Понижение выброса сердца на 40%, повышение тонуса стенок сосудов C. Повышение тонуса стенок сосудов, повышение выброса сердца на 40%D. Тахикардия, понижение выброса сердца на 40% E. Гипертрофия правого желудочка, понижение выброса сердца на 40% 114. Факторы, приводящие к развитию холестаза: A. Длительное действие прогестеронаB. Гипохолестеринемия C. Врожденная патология поджелудочной железыD. Астеническое телосложениеE. Гиперандрогения 115. Когда чаще встречается внутрипеченочный холестаз: A. В 1 триместреB. Во 2 триместреC. В 3 триместреD. За 2-3 недели до родов E. После родов 116. Клиника внутрипечоночного холестаза: A. Зуд кожных покровов B. Повышение уровня непрямого билирубина C. Бледность кожных покровов D. Снижение уровня общего билирубинаE. Металический привкус во рту117. Основные симптомы диабетической комы A. Тремор конечностейB. Запах ацетона изо ртаC. Повышение апетита D. Тонические судороги E. Бледность кожных покровов118. Какое количество килокаллорий должен содержать рацион беременной с сахарным диабетом и ожирением:A. 1800-1900 B. 2000-2200C. 2300-3000 D. 3100-3500 E. 3600-3800 119. После обнаружения внутрипеченочного холестаза необходимо: A. Экстренное прерывание беременности в любом срокеB. Пролонгирование беременности с назначением антигистаминных препаратовC. Назначение антигистаминых препаратов D. Назначение желчегонных препаратовE. Назначение рефлексотерапии 120. Беременной 23 лет выставлен диагноз: Беременность 32 недели. Острый жировой гепатоз. Акушерская тактика: A. Подготовка к родам и комплексное лечение жирового гепатозаB. Досрочное родоразрешение в течение 24-48 часов C. Продолжать консервативное лечение жирового гепатоза до доношенного срокаD. Общеукрепляющая терапия и комплексное лечение гепатозаE. Подготовка к оперативному родоразрешению

Page 111: postgraduate.kaznmu.kz€¦ · Web view58 Больная И, 28 лет, доставлена с жалобами на внезапно возникшую боль внизу живота

С.Ж.АСФЕНДИЯРОВАТЫНДАҒЫҚАЗАҚ ҰЛТТЫҚ МЕДИЦИНА

УНИВЕРСИТЕТІ

КАЗАХСКИЙНАЦИОНАЛЬНЫЙМЕДИЦИНСКИЙ УНИВЕРСИТЕТ ИМЕНИ С.Д.АСФЕНДИЯРОВА

КАФЕДРА ИНТЕРНАТУРЫ И РЕЗИДЕНТУРЫ ПО АКУШЕРСТВУ И ГИНЕКОЛОГИИТЕСТОВЫЕ ЗАДАНИЯ ДЛЯ ПРОВЕДЕНИЯ ИТОГОВОЙ ГОСУДАРСТВЕННОЙ АТТЕСТАЦИИ РЕЗИДЕНТОВ

ВЫПУСКА 2012-2013 ГГ. ПО СПЕЦИАЛЬНОСТИ «АКУШЕРСТВО И ГИНЕКОЛОГИЯ»

121. Беременной 33 лет выставлен диагноз: Беременность 34 недели. HELLP синдром. Акушерская тактика: A. Профилактика СДР плода, подготовка к родам B. Досрочное родоразрешение в течение 24-48 часов C. Продолжать консервативное лечение до доношенного срокаD. Общеукрепляющая терапия E. Оперативное родоразрешение в доношенном сроке122. Тактика врача при остром панкреатите:A. Прерывание беременности в любом сроке при появлении клиникиB. Прерывание беременности при взятии на «Д» учетC. Комплексное лечение панкреатита, сохраняющая терапия D. Комплексное лечение панкреатита, прерывание беременностиE. Родоразрешение оперативным путем123.В патогенезе развития СПИДа, какой фермент вируса играет основную роль: A. Обратная транскриптаза B. Синтез ДНК C. Коллогеназа D. СтрептокиназаE. РНК полимераза124. К TORCH-инфекциям относятся: A. Стафиллококки B. Токсоплазмы C. Стрептококки D. Кишечная палочка E. Гарднерелла 125.Когда развиваются патологические изменения плода у беременных женщин, больных сифилисом: A. До12 недель B. После 20 недель C. В 30 недельD. В 38-40 недель E. В родах126. Наиболее частый путь передачи вируса простого герпеса плоду:A. АнтенатальныйB. Интранатальный C. ПостнатальныйD. ГематогенныйE. Лимфогенный127.В каком сроке гестаци вирус краснухи опасен для плода :A. 4-8 недель B. 20 недель C. 30 недельD. Перед родами E. Во время родов128.Осложнения для плода и новорожденного при инфицировании уреаплазмозом и микоплазмозом беременной:A. Внутриутробная пневмония B. Врожденный порок сердца

Page 112: postgraduate.kaznmu.kz€¦ · Web view58 Больная И, 28 лет, доставлена с жалобами на внезапно возникшую боль внизу живота

С.Ж.АСФЕНДИЯРОВАТЫНДАҒЫҚАЗАҚ ҰЛТТЫҚ МЕДИЦИНА

УНИВЕРСИТЕТІ

КАЗАХСКИЙНАЦИОНАЛЬНЫЙМЕДИЦИНСКИЙ УНИВЕРСИТЕТ ИМЕНИ С.Д.АСФЕНДИЯРОВА

КАФЕДРА ИНТЕРНАТУРЫ И РЕЗИДЕНТУРЫ ПО АКУШЕРСТВУ И ГИНЕКОЛОГИИТЕСТОВЫЕ ЗАДАНИЯ ДЛЯ ПРОВЕДЕНИЯ ИТОГОВОЙ ГОСУДАРСТВЕННОЙ АТТЕСТАЦИИ РЕЗИДЕНТОВ

ВЫПУСКА 2012-2013 ГГ. ПО СПЕЦИАЛЬНОСТИ «АКУШЕРСТВО И ГИНЕКОЛОГИЯ»

C. Поражение ЦНСD. Антенатальная гибель плодаE. Дисплазия тазобедренных суставов129. Какими антибактериальными препаратами лечат хламидиоз во время беременности:A. Нитрофураны B. Пенициллины C. Тетрациклины D. МакролидыE. Сульфаниламиды130.При слабости родовой деятельности у роженицы с рубцом на матке показано:A. Родостимуляция окситоциномB. Экстренное кесарево сечениеC. Родостимуляция простагландинамиD. АмниотомияE. Выжидательная тактика131. Первобеременная Н., 22 лет, поступила в роддом с жалобами на схваткообразные боли внизу живота в течение 4-х часов. Срок беременности 39-40 недель. Объективно схватки через 5 минут до 35 секунд. Вагинально: шейка матки сглажена, открытие маточного зева на 2 см, плодный пузырь цел, предлежит головка, прижата ко входу в малый таз. Диагноз:A. Беременность 39-40 недель. Прелиминарный период.B. Беременность 39-40 недель. Патологический прелиминарный период.C. Беременность 39-40 недель. Ложные схватки.D. Беременность 39-40 недель. Первый период родов, латентная фаза.E. Беременность 39-40 недель. Первый период родов, активная фаза.132. Повторнобеременная Н., 23 лет, поступила в роддом с жалобами на схваткообразные боли внизу живота в течение 4-х часов. Срок беременности 38-39 недель. Объективно схватки через 4-5 минут до 45 секунд. Вагинально: шейка матки сглажена, открытие маточного зева на 4 см, плодный пузырь цел, предлежит головка, прижата ко входу в малый таз. Диагноз:A. Беременность 38-39 недель. Прелиминарный период.B. Беременность 38-39 недель. Патологический прелиминарный период.C. Беременность 38-39 недель. Ложные схватки.D. Беременность 38-39 недель. Первый период родов, латентная фаза.E. Беременность 38-39 недель. Первый период родов, активная фаза.133. Повторнобеременная Н., 33 лет, поступила в роддом с жалобами на схваткообразные боли внизу живота в течение 6-и часов. Срок беременности 38-39 недель. Объективно схватки через 2-3 минуты до 60 секунд, потужного характера. Вагинально: шейка матки сглажена, открытие маточного зева полное, плодного пузыря нет, предлежит головка, большим сегментом во входе в малый таз. Диагноз:A. Беременность 38-39 недель. Ложные схватки.B. Беременность 38-39 недель. Патологический прелиминарный период.C. Беременность 38-39 недель. Первый период родов, латентная фаза.D. Беременность 38-39 недель. Первый период родов, активная фаза.E. Беременность 38-39 недель. Второй период родов.134. У первородящей К., 24 лет, первый период родов продолжается 8 часов. Объективно схватки через 4-5 минут до 45 секунд. Вагинально: шейка матки сглажена, открытие маточного зева на 5 см, плодный пузырь цел, предлежит головка, прижата ко входу в малый таз. Оцените темп родов:A. МонотонныйB. Быстрый

Page 113: postgraduate.kaznmu.kz€¦ · Web view58 Больная И, 28 лет, доставлена с жалобами на внезапно возникшую боль внизу живота

С.Ж.АСФЕНДИЯРОВАТЫНДАҒЫҚАЗАҚ ҰЛТТЫҚ МЕДИЦИНА

УНИВЕРСИТЕТІ

КАЗАХСКИЙНАЦИОНАЛЬНЫЙМЕДИЦИНСКИЙ УНИВЕРСИТЕТ ИМЕНИ С.Д.АСФЕНДИЯРОВА

КАФЕДРА ИНТЕРНАТУРЫ И РЕЗИДЕНТУРЫ ПО АКУШЕРСТВУ И ГИНЕКОЛОГИИТЕСТОВЫЕ ЗАДАНИЯ ДЛЯ ПРОВЕДЕНИЯ ИТОГОВОЙ ГОСУДАРСТВЕННОЙ АТТЕСТАЦИИ РЕЗИДЕНТОВ

ВЫПУСКА 2012-2013 ГГ. ПО СПЕЦИАЛЬНОСТИ «АКУШЕРСТВО И ГИНЕКОЛОГИЯ»

C. УмеренныйD. НедостаточныйE. Достаточный135. У роженицы К., 29 лет, первый период родов продолжается 8 часов. Объективно схватки через 5-6 минут до 30 секунд. Вагинально: шейка матки сглажена, открытие маточного зева на 2 см, плодный пузырь цел, предлежит головка, прижата ко входу в малый таз. Оцените темп родов:A. МонотонныйB. Быстрый C. УмеренныйD. НедостаточныйE. Достаточный136. Роженица К., 27 лет, поступила в роддом через 5 часов от начала схваток. Объективно: схватки через 3-4 минут до 40 секунд. Вагинально: шейка матки сглажена, открытие маточного зева на 3 см, плодный пузырь цел, предлежит головка, прижата ко входу в малый таз. Через какое время следует произвести повторное влагалищное исследование:A. 2 часаB. 3 часаC. 4 часаD. 5 часовE. 6 часов137. Повторнобеременная Н., 29 лет, поступила в роддом с жалобами на схваткообразные боли внизу живота в течение 3-х часов. Срок беременности 38-39 недель. Объективно: схватки нерегулярные через 20-30 минут по 15- 20 секунд. Вагинально: шейка матки «зрелая», плодный пузырь цел, предлежит головка, прижата ко входу в малый таз. Диагноз:A. Беременность 38-39 недель. Прелиминарный период.B. Беременность 38-39 недель. Патологический прелиминарный период.C. Беременность 38-39 недель. Предвестники родов.D. Беременность 38-39 недель. Первый период родов, латентная фаза.E. Беременность 38-39 недель. Первый период родов, активная фаза.138. Повторнобеременная Н., 30 лет, поступила в роддом с жалобами на схваткообразные боли внизу живота в течение 8-и часов. Срок беременности 38-39 недель. Объективно схватки нерегулярные через 20-30 минут по 10-20 секунд. Вагинально: шейка матки «зрелая», плодный пузырь цел, предлежит головка, прижата ко входу в малый таз.Диагноз:A. Беременность 38-39 недель. Прелиминарный период.B. Беременность 38-39 недель. Патологический прелиминарный период.C. Беременность 38-39 недель. Предвестники родов.D. Беременность 38-39 недель. Первый период родов, латентная фаза.E. Беременность 38-39 недель. Первый период родов, активная фаза.139. Каким символом отмечают на партограмме открытие шейки матки:A. «О»B. «Х»C. «М»D. «А»E. «I»140. Каким символом отмечают на партограмме местоположение головки:A. «О»

Page 114: postgraduate.kaznmu.kz€¦ · Web view58 Больная И, 28 лет, доставлена с жалобами на внезапно возникшую боль внизу живота

С.Ж.АСФЕНДИЯРОВАТЫНДАҒЫҚАЗАҚ ҰЛТТЫҚ МЕДИЦИНА

УНИВЕРСИТЕТІ

КАЗАХСКИЙНАЦИОНАЛЬНЫЙМЕДИЦИНСКИЙ УНИВЕРСИТЕТ ИМЕНИ С.Д.АСФЕНДИЯРОВА

КАФЕДРА ИНТЕРНАТУРЫ И РЕЗИДЕНТУРЫ ПО АКУШЕРСТВУ И ГИНЕКОЛОГИИТЕСТОВЫЕ ЗАДАНИЯ ДЛЯ ПРОВЕДЕНИЯ ИТОГОВОЙ ГОСУДАРСТВЕННОЙ АТТЕСТАЦИИ РЕЗИДЕНТОВ

ВЫПУСКА 2012-2013 ГГ. ПО СПЕЦИАЛЬНОСТИ «АКУШЕРСТВО И ГИНЕКОЛОГИЯ»

B. «Х»C. «М»D. «А»E. «I»141. Каким символом отмечают на партограмме зеленые околоплодные воды:1. «О»2. «Х»3. «М»4. «С»5. «I»142. Каким символом отмечают на партограмме целые околоплодные воды :A. «О»B. «Х»C. «М»D. «С»E. «I»143. Каким символом отмечают на партограмме прозрачные околоплодные воды:A. «О»B. «Х»C. «М»D. «А»E. «С»144. Каким символом отмечают на партограмме отсутствие околоплодных вод:A. «О»B. «Х»C. «М»D. «А»E. «С»145. Какой из указанных параметров партограммы помогает диагностировать гипоксию плода:A. Динамика открытия шейки маткиB. Характер схватокC. Цвет околоплодных водD. Конфигурация головки плодаE. Продвижение головки плода146. На партограмме в графе конфигурация головки отмечено «». Это свидетельствует о:A. Хорошей родовой деятельностиB. Начале второго периода родовC. Неэффективности схватокD. Несоответствии размеров таза матери и головки плодаE. Соответствии размеров таза матери и головки плода147. Через какой промежуток времени в партограмме отмечают местоположение головки плода:A. 2 часаB. 3 часаC. 4 часаD. 5 часовE. 6 часов148. Через какой промежуток времени в партограмме отмечают частоту сердцебиения плода:

Page 115: postgraduate.kaznmu.kz€¦ · Web view58 Больная И, 28 лет, доставлена с жалобами на внезапно возникшую боль внизу живота

С.Ж.АСФЕНДИЯРОВАТЫНДАҒЫҚАЗАҚ ҰЛТТЫҚ МЕДИЦИНА

УНИВЕРСИТЕТІ

КАЗАХСКИЙНАЦИОНАЛЬНЫЙМЕДИЦИНСКИЙ УНИВЕРСИТЕТ ИМЕНИ С.Д.АСФЕНДИЯРОВА

КАФЕДРА ИНТЕРНАТУРЫ И РЕЗИДЕНТУРЫ ПО АКУШЕРСТВУ И ГИНЕКОЛОГИИТЕСТОВЫЕ ЗАДАНИЯ ДЛЯ ПРОВЕДЕНИЯ ИТОГОВОЙ ГОСУДАРСТВЕННОЙ АТТЕСТАЦИИ РЕЗИДЕНТОВ

ВЫПУСКА 2012-2013 ГГ. ПО СПЕЦИАЛЬНОСТИ «АКУШЕРСТВО И ГИНЕКОЛОГИЯ»

A. 10 минутB. 15 минутC. 20 минутD. 25 минутE. 30 минут149. Через какой промежуток времени в партограмме отмечают частоту и характер схваток:A. 10 минутB. 20 минутC. 30 минутD. 40 минутE. 50 минут150.Какое открытие шейки матки свидетельствует о начале активной фазы первого периода родов:A. 1 смB. 2 смC. 3 смD. 4 смE. 5 см151. Через какой промежуток времени при физиологических родах в партограмме отмечают артериальное давление и пульс роженицы:A. 30 минут B. 1 часC. 2 часаD. 3 часаE. 4 часа153. Через какой промежуток времени при физиологических родах в партограмме отмечают температуру тела роженицы:A. 1 часB. 2 часаC. 3 часаD. 4 часаE. 5 часов154. Слабые схватки у роженицы в партограмме маркируют:A. ТочкамиB. Косыми штрихамиC. Сплошным закрашиваниемD. Поперечными штрихамиE. Продольными штрихами155. Схватки средней силы у роженицы в партограмме маркируют:A. ТочкамиB. Косыми штрихамиC. Сплошным закрашиваниемD. Поперечными штрихамиE. Продольными штрихами156. Схватки хорошей силы у роженицы в партограмме маркируют:A. ТочкамиB. Косыми штрихамиC. Сплошным закрашиваниемD. Поперечными штрихами

Page 116: postgraduate.kaznmu.kz€¦ · Web view58 Больная И, 28 лет, доставлена с жалобами на внезапно возникшую боль внизу живота

С.Ж.АСФЕНДИЯРОВАТЫНДАҒЫҚАЗАҚ ҰЛТТЫҚ МЕДИЦИНА

УНИВЕРСИТЕТІ

КАЗАХСКИЙНАЦИОНАЛЬНЫЙМЕДИЦИНСКИЙ УНИВЕРСИТЕТ ИМЕНИ С.Д.АСФЕНДИЯРОВА

КАФЕДРА ИНТЕРНАТУРЫ И РЕЗИДЕНТУРЫ ПО АКУШЕРСТВУ И ГИНЕКОЛОГИИТЕСТОВЫЕ ЗАДАНИЯ ДЛЯ ПРОВЕДЕНИЯ ИТОГОВОЙ ГОСУДАРСТВЕННОЙ АТТЕСТАЦИИ РЕЗИДЕНТОВ

ВЫПУСКА 2012-2013 ГГ. ПО СПЕЦИАЛЬНОСТИ «АКУШЕРСТВО И ГИНЕКОЛОГИЯ»

E. Продольными штрихами157. При влагалищном исследовании обнаружено, что швы на головке плода легко прощупываются, кости черепа не соприкасаются друг с другом. Каким символом в партограмме обозначают указанные данные:A. (-) B. (0)C. ()D. ()E. ()158. При влагалищном исследовании обнаружено, что кости черепа слегка соприкасаются друг с другом. Каким символом в партограмме обозначают указанные данные:A. (-) B. (0)C. ()D. ()E. ()159. При влагалищном исследовании обнаружено, что кости черепа плода заходят друг на друга. Каким символом в партограмме обозначают указанные данные:A. (-) B. (0)C. ()D. ()E. ()160. При влагалищном исследовании обнаружено, что кости черепа плода значительно заходят друг на друга. Каким символом в партограмме обозначают указанные данные:A. (-) B. (0)C. ()D. ()E. ()161. Укажите местоположение линии «действия» в партограмме: A. Справа от линии «бдительности» на расстоянии 3-х часовB. Слева от линии «бдительности» на расстоянии 3-х часовC. Справа от линии «бдительности» на расстоянии 4-х часовD. Слева от линии «бдительности» на расстоянии 4-х часовE. Справа от линии «бдительности» на расстоянии 5 часов162. В партограмме в графе «количество схваток за 10 минут» закрашены 2 клетки косыми штрихами. Это означает, что в данный момент у роженицы родовая деятельность протекает со следующей частотой и характером схваток:A. 2 схватки слабой силыB. 2 схватки средней силыC. 2 схватки хорошей силы D. Нерегулярные, слабые схватки E. Прелиминарные схватки 163. В партограмме в графе «количество схваток за 10 минут» закрашены 4 клетки косыми штрихами. Это означает, что в данный момент у роженицы родовая деятельность протекает со следующей частотой и характером схваток:A. 4 схватки слабой силы

Page 117: postgraduate.kaznmu.kz€¦ · Web view58 Больная И, 28 лет, доставлена с жалобами на внезапно возникшую боль внизу живота

С.Ж.АСФЕНДИЯРОВАТЫНДАҒЫҚАЗАҚ ҰЛТТЫҚ МЕДИЦИНА

УНИВЕРСИТЕТІ

КАЗАХСКИЙНАЦИОНАЛЬНЫЙМЕДИЦИНСКИЙ УНИВЕРСИТЕТ ИМЕНИ С.Д.АСФЕНДИЯРОВА

КАФЕДРА ИНТЕРНАТУРЫ И РЕЗИДЕНТУРЫ ПО АКУШЕРСТВУ И ГИНЕКОЛОГИИТЕСТОВЫЕ ЗАДАНИЯ ДЛЯ ПРОВЕДЕНИЯ ИТОГОВОЙ ГОСУДАРСТВЕННОЙ АТТЕСТАЦИИ РЕЗИДЕНТОВ

ВЫПУСКА 2012-2013 ГГ. ПО СПЕЦИАЛЬНОСТИ «АКУШЕРСТВО И ГИНЕКОЛОГИЯ»

B. 4 схватки средней силыC. 4 схватки хорошей силы D. Регулярные, слабые схватки E. Прелиминарные схватки 164. В партограмме в графе «количество схваток за 10 минут» произведено сплошное закрашивание 5-ти клеток. Это означает, что в данный момент у роженицы родовая деятельность протекает со следующей частотой и характером схваток:A. 5 схваток слабой силыB. 5 схваток средней силыC. 5 схваток хорошей силыD. Регулярные потугиE. Нерегулярные потуги 165. В партограмме в графе «количество схваток за 10 минут» произведено сплошное закрашивание 3–х клеток. Это означает, что в данный момент у роженицы родовая деятельность протекает со следующей частотой и характером схваток:A. 3 схватки по 5-10 секундB. 3 схватки по 15-20 секундC. 3 схватки по 25-30 секундD. 3 схватки по 30-35 секундE. 3 схватки по 40-45 секунд166. В партограмме в графе «количество схваток за 10 минут» закрашены 2 клетки точками. Это означает, что в данный момент у роженицы родовая деятельность протекает со следующей частотой и характером схваток:A. 2 схватки слабой силыB. 2 схватки средней силыC. 2 схватки хорошей силы D. Нерегулярные, слабые схватки E. Прелиминарные схватки 167. В партограмме в графе «количество схваток за 10 минут» закрашены 3 клетки точками. Это означает, что в данный момент у роженицы родовая деятельность протекает со следующей частотой и характером схваток:A. 3 схватки слабой силыB. 3 схватки средней силыC. 3 схватки хорошей силы D. Нерегулярные, слабые схватки E. Прелиминарные схватки 168. В партограмме первородящей линия «открытия шейки матки» расположена слева от линии «бдительности». Указанные данные свидетельствуют, что темп родов:A. НедостаточныйB. ДостаточныйC. Чрезмерный D. Дискоординированный E. Характерен для начала 2-го периода169. В партограмме роженицы в графе «количество схваток за 10 минут» закрашивание произведено точками. Какой силе схваток соответствует данная маркировка:A. Слабой B. Средней C. Хорошей

Page 118: postgraduate.kaznmu.kz€¦ · Web view58 Больная И, 28 лет, доставлена с жалобами на внезапно возникшую боль внизу живота

С.Ж.АСФЕНДИЯРОВАТЫНДАҒЫҚАЗАҚ ҰЛТТЫҚ МЕДИЦИНА

УНИВЕРСИТЕТІ

КАЗАХСКИЙНАЦИОНАЛЬНЫЙМЕДИЦИНСКИЙ УНИВЕРСИТЕТ ИМЕНИ С.Д.АСФЕНДИЯРОВА

КАФЕДРА ИНТЕРНАТУРЫ И РЕЗИДЕНТУРЫ ПО АКУШЕРСТВУ И ГИНЕКОЛОГИИТЕСТОВЫЕ ЗАДАНИЯ ДЛЯ ПРОВЕДЕНИЯ ИТОГОВОЙ ГОСУДАРСТВЕННОЙ АТТЕСТАЦИИ РЕЗИДЕНТОВ

ВЫПУСКА 2012-2013 ГГ. ПО СПЕЦИАЛЬНОСТИ «АКУШЕРСТВО И ГИНЕКОЛОГИЯ»

D. ЧрезмернойE. Потуги 170. В партограмме роженицы в графе «количество схваток за 10 минут» произведено сплошное закрашивание. Укажите какова продолжительность схваток:A. До 10 секундB. 10-15 секундC. 20-25 секундD. 30-35 секундE. 40-45 секунд171. К врачу женской консультации обратилась беременная со сроком гестации 8-9 недель. При объективном осмотре обнаружено повышение артериального давления до 140/90 мм рт.ст. Укажите наиболее вероятную причину повышения артериального давления: A. Вгетососудистая дистонияB. Хроническая гипертензияC. Гипертензия, вызванная беременностьюD. Преэклампсия легкой степениE. Преэклампсия тяжелой степени 172. Наиболее вероятная причина повышения артериального давления после 22 недель беременности:A. Хроническая артериальная гипертензияB. ПреэклампсияC. Хронический гломерулонефритD. Хронический пиелонефритE. Нейро-циркуляторная дистония 173. Для беременных с хронической артериальной гипертензией наиболее характерно: A. Повышение артериального давления в ранние и поздние сроки беременности и снижение его в середине гестацииB. Повышение артериального давления в течение всей беременностиC. Повышение артериального давления в течение всей беременности и в родахD. Повышение артериального давления в родах и послеродовом периоде E. Снижение артериального давления в ранние и поздние сроки беременности и повышение его в середине гестации174. Во втором периоде родов у роженицы с хронической артериальной гипертензией повысилось артериальное давление до 170/100 мм рт.ст, сердцебиение плода 146 ударов в 1 минуту, ясное, ритмичное, головка плода в плоскости выхода малого таза. Акушерская тактика:A. Произвести перинеотомиюB. Наложить акушерские щипцыC. Провести перидуральную анестезиюD. Ввести сульфат магнияE. Произвести кесарево сечение175. С целью управляемой нормотонии во втором периоде родов используют:A. Сульфат магнияB. β-адреноблокаторыC. ГанглиоблокаторыD. Глюкозо-новокаиновую смесьE. Перидуральную анестезию176. Наиболее частым заболеванием почек у беременных является:

Page 119: postgraduate.kaznmu.kz€¦ · Web view58 Больная И, 28 лет, доставлена с жалобами на внезапно возникшую боль внизу живота

С.Ж.АСФЕНДИЯРОВАТЫНДАҒЫҚАЗАҚ ҰЛТТЫҚ МЕДИЦИНА

УНИВЕРСИТЕТІ

КАЗАХСКИЙНАЦИОНАЛЬНЫЙМЕДИЦИНСКИЙ УНИВЕРСИТЕТ ИМЕНИ С.Д.АСФЕНДИЯРОВА

КАФЕДРА ИНТЕРНАТУРЫ И РЕЗИДЕНТУРЫ ПО АКУШЕРСТВУ И ГИНЕКОЛОГИИТЕСТОВЫЕ ЗАДАНИЯ ДЛЯ ПРОВЕДЕНИЯ ИТОГОВОЙ ГОСУДАРСТВЕННОЙ АТТЕСТАЦИИ РЕЗИДЕНТОВ

ВЫПУСКА 2012-2013 ГГ. ПО СПЕЦИАЛЬНОСТИ «АКУШЕРСТВО И ГИНЕКОЛОГИЯ»

A. Гломерулонефрит B. Гидронефроз C. ПиелонефритD. Мочекаменная болезньE. Киста почек177. Предпочтительные антибактериальные препараты для лечения пиелонефрита у беременных в первом триместре:A. Аминогликозиды B. Нитрофураны C. Макролиды D. Полусинтетичесие пенициллиныE. Сульфаниламиды178. Оптимальный метод родоразрешения у беременных с хроническим пиелонефритом:A. Кесарево сечение в плановом порядкеB. Кесарево сечение в экстренном порядкеC. Кесарево сечение в срочном порядке D. Через естественные родовые пути E. Через естественные родовые пути с наложением акушерских щипцов 179. Наиболее частая форма течения хронического гломерулонефрита у беременных:A. Латентная B. Гипертоническая C. Нефротическая D. СмешаннаяE. С выраженной симптоматикой 180. Наиболее частое осложнение беременности при гломерулонефрите:A. Преэклампсия B. ГидрокаликозC. Предлежание плацентыD. Маловодие E. Почечная колика181. Наиболее частое заболевание печени, связанное с беременностью:A. Вирусный гепатит А B. Вирусный гепатит В C. Холестатический гепатозD. Острый жировой гепатозE. Холецистит182. Наиболее частым акушерским осложнением при вирусном гепатите является:A. Острая печеночная недостаточностьB. Внутриутробное инфицирование плодаC. Интранатальная гибель плодаD. Кровотечение в последовом и раннем послеродовом периодах E. Дородовое излитие околоплодных вод183. При выявлении вирусного гепатита «А» у беременных в доношенном сроке показано:A. Наблюдение до начала спонтанной родовой деятельности B. Плановое кесарево сечение после проведения симптоматического леченияC. Экстренное кесарево сечение без проведения симптоматического леченияD. Родоразрешение через естественные родовые пути после предварительного симптоматического лечения

Page 120: postgraduate.kaznmu.kz€¦ · Web view58 Больная И, 28 лет, доставлена с жалобами на внезапно возникшую боль внизу живота

С.Ж.АСФЕНДИЯРОВАТЫНДАҒЫҚАЗАҚ ҰЛТТЫҚ МЕДИЦИНА

УНИВЕРСИТЕТІ

КАЗАХСКИЙНАЦИОНАЛЬНЫЙМЕДИЦИНСКИЙ УНИВЕРСИТЕТ ИМЕНИ С.Д.АСФЕНДИЯРОВА

КАФЕДРА ИНТЕРНАТУРЫ И РЕЗИДЕНТУРЫ ПО АКУШЕРСТВУ И ГИНЕКОЛОГИИТЕСТОВЫЕ ЗАДАНИЯ ДЛЯ ПРОВЕДЕНИЯ ИТОГОВОЙ ГОСУДАРСТВЕННОЙ АТТЕСТАЦИИ РЕЗИДЕНТОВ

ВЫПУСКА 2012-2013 ГГ. ПО СПЕЦИАЛЬНОСТИ «АКУШЕРСТВО И ГИНЕКОЛОГИЯ»

E. Экстренное родоразрешение через естественные родовые пути без предварительного симптоматического лечения184. У беременной в сроке 32-33 недели выставлен диагноз вирусного гепатита «В». Тактика врача:A. Симптоматическое лечение и пролонгирование беременности B. Плановое кесарево сечение после проведения симптоматического леченияC. Экстренное кесарево сечение без проведения симптоматического леченияD. Срочное родоразрешение через естественные родовые пути после предварительного симптоматического леченияE. Экстренное родоразрешение через естественные родовые пути без предварительного симптоматического лечения185. Беременной в сроке 34 недели выставлен диагноз острого жирового гепатоза. Тактика врача:A. Симптоматическое лечение и пролонгирование беременности B. Плановое кесарево сечение после проведения симптоматического леченияC. Экстренное кесарево сечение без проведения симптоматического леченияD. Срочное родоразрешение через естественные родовые пути после предварительного симптоматического леченияE. Досрочное родоразрешение через естественные родовые пути без предварительного симптоматического лечения186. Наиболее часто встречающаяся форма анемии у беременных:A. Мегалобластная, гиперхромная B. Железодефицитная, гипохромная C. Гипопластическая D. Гемолитическая E. Апластическая187. Развитие сердечной недостаточности у беременных в сроке 24-26 недель, как правило, связано с:A. Увеличением объема циркулирующей крови и минутного объема сердцаB. Увеличением объема циркулирующей крови и снижением минутного объема сердца C. Увеличением объема циркулирующей крови и снижением сердечного выброса D. Снижением объема циркулирующей крови и минутного объема сердцаE. Снижением объема циркулирующей крови и повышением минутного объема сердца188. Пролонгирование беременности допустимо при:A. Недостаточности митрального клапана и отсутствии гемодинамических нарушенийB. Стенозе митрального клапана и отсутствии гемодинамических нарушенийC. Аортальных пороках и отсутствии гемодинамических нарушенийD. Пороках сердца с мерцательной аритмиейE. Декомпенсированных пороках сердца189. Укажите порок сердца, при котором роды через естественные родовые пути возможны:A. Возвратный и подострый септический эндокардит без нарушения кровообращенияB. Любой порок с выраженным нарушением кровообращенияC. Сочетание порока сердца с акушерской патологиейD. Порок с длительно существующей мерцательной аритмиейE. Врожденный дефект межпредсердной перегородки без нарушения кровообращения190.Беременная с гестационным пиелонефритом получает антибиотики широкого спектра действия, гипертермия периодически повторяется. Эффективность антибактериальной терапии следует оценить через:

Page 121: postgraduate.kaznmu.kz€¦ · Web view58 Больная И, 28 лет, доставлена с жалобами на внезапно возникшую боль внизу живота

С.Ж.АСФЕНДИЯРОВАТЫНДАҒЫҚАЗАҚ ҰЛТТЫҚ МЕДИЦИНА

УНИВЕРСИТЕТІ

КАЗАХСКИЙНАЦИОНАЛЬНЫЙМЕДИЦИНСКИЙ УНИВЕРСИТЕТ ИМЕНИ С.Д.АСФЕНДИЯРОВА

КАФЕДРА ИНТЕРНАТУРЫ И РЕЗИДЕНТУРЫ ПО АКУШЕРСТВУ И ГИНЕКОЛОГИИТЕСТОВЫЕ ЗАДАНИЯ ДЛЯ ПРОВЕДЕНИЯ ИТОГОВОЙ ГОСУДАРСТВЕННОЙ АТТЕСТАЦИИ РЕЗИДЕНТОВ

ВЫПУСКА 2012-2013 ГГ. ПО СПЕЦИАЛЬНОСТИ «АКУШЕРСТВО И ГИНЕКОЛОГИЯ»

A. 24 часаB. 48 часовC. 72 часаD. 96 часовE. 120 часов191.Наиболее частой причиной прерывания беременности во втором триместре является:A. Истмико-цервикальная недостаточность B. Гипертонус маткиC. Воспалительные заболевания половой системы D. Механическая травма E. Хромосомные заболевания192. Наиболее частой причиной прерывания беременности в ранних сроках является: A. Истмико-цервикально недостаточность B. Гипертонус маткиC. Воспалительные заболевания половой системы D. Механическая травма E. Хромосомные заболевания193.Клиническая картина гипотиреоза:A. Бледность, отечность, брадикардия, сухость кожи, заторможенность речиB. Влажная кожа, тахикардия, быстрая речь, гипертензия, гипергидратацияC. Экзофтальм, тремор рук, заторможенность речи, гипотензия, гипергидратацияD. Энофтальм, повышение аппетита, похудание, гипертензия гипергидратация E. Умственное развитие адекватное, артериальная гипертензия, влажная кожа, тахикардия194.В женскую консультацию обратилась беременная с диагнозом: Беременность 4-5 недель. Ревматическая болезнь. Приобретенный порок сердца – митральный стеноз. Состояние после комиссуротомии. В анамнезе ревматизм, приобретенный порок сердца – стеноз митрального клапана, 1 год назад была проведена митральная комиссуротомия. В течение последних 5 лет ревматическая болезнь не обострялась. Ваша тактика:A. Госпитализация в стационар, прерывание беременности до 12 недельB. Пролонгирование беременности до 28 недель, родоразрешение в плановом порядке. C. Пролонгирование беременности до 32 недель, после 30 недель проведение кардиальной и антиревматической терапии D. Пролонгирование беременности до 36 недель, после 30 недель проведение кардиальной и антиревматической терапии E. Пролонгирование беременности до доношенного срока, после 30 недель проведение кардиальной и антиревматической терапии 195. У беременной в сроке гестации 32 недели отмечается желтушное окрашивание кожи и склер. Объективно: АД 140/90 мм рт.ст., отеки на нижних конечностях.В лабораторных исследованиях крови отмечен гемолиз, тромбоциты до 80 тыс.,гемоглобин 78 г/л. Предварительный диагноз:A. HELLP-синдромB. Гепатит АC. Гепатит СD. Гепатит ВE. Гепатит Д196. Клиническая картина тиреотоксикоза?A. Бледность, отечность лица, брадикардия, сухость кожи, B. Влажная кожа, тахикардия, быстрая речь, гипертензия

Page 122: postgraduate.kaznmu.kz€¦ · Web view58 Больная И, 28 лет, доставлена с жалобами на внезапно возникшую боль внизу живота

С.Ж.АСФЕНДИЯРОВАТЫНДАҒЫҚАЗАҚ ҰЛТТЫҚ МЕДИЦИНА

УНИВЕРСИТЕТІ

КАЗАХСКИЙНАЦИОНАЛЬНЫЙМЕДИЦИНСКИЙ УНИВЕРСИТЕТ ИМЕНИ С.Д.АСФЕНДИЯРОВА

КАФЕДРА ИНТЕРНАТУРЫ И РЕЗИДЕНТУРЫ ПО АКУШЕРСТВУ И ГИНЕКОЛОГИИТЕСТОВЫЕ ЗАДАНИЯ ДЛЯ ПРОВЕДЕНИЯ ИТОГОВОЙ ГОСУДАРСТВЕННОЙ АТТЕСТАЦИИ РЕЗИДЕНТОВ

ВЫПУСКА 2012-2013 ГГ. ПО СПЕЦИАЛЬНОСТИ «АКУШЕРСТВО И ГИНЕКОЛОГИЯ»

C. Экзофтальм, тремор рук, заторможенность речи, гипотензияD. Энофтальм, повышение аппетита, похудание, гипертензияE. Артериальная гипертензия, сухая кожа, тахикардия197. При каком врожденном пороке сердца допустимо вынашивание беременности:A. Тетрада ФаллоB. Коарктация аортыC. Комплекс ЭйзенменгераD. Дефект межпредсердной перегородкиE. Пентада Фалло198. Количество околоплодных вод при физиологической доношенной беременности:A. 100-200млB. 200-300млC. 300-400млD. 500-1500млE. 1600-2000мл199. Беременная со сроком беременности 9 недель жалуется на рвоту до 22-х раз, потерю в весе. Объективно: гипотония, тахикардия до 110 раз в минуту, снижение диуреза, остаточный азот и креатинин повышены, в моче ацетон . Ваш диагноз:A. Беременность 9 недель. Ранний токсикоз, рвота беременных тяжелой степени.B. Беременность 9 недель. Ранний токсикоз, рвота беременных средней степени C. Беременность 9 недель. Ранний токсикоз. Рвота беременных степени легкойD. Беременность 9 недель. Острый холецистопанкреатитE. Беременность 9 недель. Дизентерия, энтероколит.200. Беременная со сроком беременности 8-9 недель жалуется на рвоту до 22-х раз в сутки, потерю в весе. Выставлен диагноз: Беременность 8-9 недель. Ранний токсикоз. Рвота беременных тяжелой степени. Тактика врача?A. Госпитализация, пролонгирование беременности на фоне инфузионной терапииB. Госпитализация, инфузионная дезинтоксикационная, общеукрепляющая терапияC. Госпитализация, комплексное лечение в течение 2 суток, при безэффективности- прерывание беременностиD.Госпитализация в дневной стационар, при безэффективности- прерывание беременности. E.Диспансерное наблюдение, госпитализация в дневной стационар

201. Беременная со сроком беременности 9 недель жалуется на рвоту до 15-ти раз в сутки. Объективно: артериальное давление 90/60 мм рт.ст, пульс 96 раз в минуту, диурез не изменен, остаточный азот и креатинин в норме. В моче ацетон . Ваш диагноз:

A. Беременность 9 недель. Ранний токсикоз, рвота беременных тяжелой степени.B. Беременность 9 недель. Ранний токсикоз, рвота беременных средней степени C. Беременность 9 недель. Ранний токсикоз. Рвота беременных степени легкойD. Беременность 9 недель. Острый холецистопанкреатитE. Беременность 9 недель. Дизентерия, энтероколит. 202. Беременная со сроком беременности 8-9 недель жалуется на постоянное слюнотечение, потерю в весе. Объективно: гипотония, тахикардия до 110 раз в минуту, диурез снижен, остаточный азот и креатинин повышен. В моче ацетон . Ваш диагноз:A. Беременность 8-9 недель. Птиализм.B. Беременность 8-9 недель. Ранний токсикоз. Остеопатия беременных.C. Беременность 8-9 недель. Ранний токсикоз. Рвота беременных средней степени

Page 123: postgraduate.kaznmu.kz€¦ · Web view58 Больная И, 28 лет, доставлена с жалобами на внезапно возникшую боль внизу живота

С.Ж.АСФЕНДИЯРОВАТЫНДАҒЫҚАЗАҚ ҰЛТТЫҚ МЕДИЦИНА

УНИВЕРСИТЕТІ

КАЗАХСКИЙНАЦИОНАЛЬНЫЙМЕДИЦИНСКИЙ УНИВЕРСИТЕТ ИМЕНИ С.Д.АСФЕНДИЯРОВА

КАФЕДРА ИНТЕРНАТУРЫ И РЕЗИДЕНТУРЫ ПО АКУШЕРСТВУ И ГИНЕКОЛОГИИТЕСТОВЫЕ ЗАДАНИЯ ДЛЯ ПРОВЕДЕНИЯ ИТОГОВОЙ ГОСУДАРСТВЕННОЙ АТТЕСТАЦИИ РЕЗИДЕНТОВ

ВЫПУСКА 2012-2013 ГГ. ПО СПЕЦИАЛЬНОСТИ «АКУШЕРСТВО И ГИНЕКОЛОГИЯ»

D. Беременность 8-9 недель. Острый панкреатитE. Беременность 8-9 недель. Дизентерия.203. Беременная со сроком беременности 8-9 недель жалуется на постоянное слюнотечение, потерю в весе. Объективно: гипотония, тахикардия до 90 уд. в минуту, диурез снижен, остаточный азот и креатинин повышен. В моче ацетон . Выставлен диагноз: Беременность 8-9 недель. Ранний токсикоз. Птиализм. Тактика врача?A. Госпитализация, комплексная дезинтоксикационная, инфузионная, общеукрепляющая терапияB. Госпитализация, диета, общеукрепляющая терапияC. Госпитализация, прерывание беременности на фоне инфузионной терапииD. Диспансерное наблюдениеE. Госпитализация в дневной стационар204. Беременная со сроком беременности 28 недель отмечает отеки на нижних конечностях, передней стенке живота. Головной боли, головокружения нет, зрение ясное. Объективно: состояние тяжелое, АД 160/110 и 165/100 мм рт.ст, пульс 90 в минуту, анасарка. Матка увеличена до 28 недель беременности, не возбудима. Положение плода продольное, головное. Сердцебиение плода ясное, ритмичное до 136 ударов в минуту. Ваш диагноз:A. Беременность 28 недель. Хроническая артериальная гипертензия.B. Беременность 28 недель. Хронический гломерулонефрит, смешанная формаC. Беременность 28 недель. Преэклампсия тяжелой степени D. Беременность 28 недель. Преэклампсия легкой степениE. Беременность 28 недель. Обострение хронического пиелонефрита205. Беременная со сроком беременности 32 недель отмечает отеки на нижних конечностях, передней стенке живота. Объективно: состояние тяжелое, АД 160/110 мм рт.ст. 165/100 мм рт.ст. пульс 90 в минуту, анасарка. Выставлен диагноз: Беременность 32 недели. Преэклампсия тяжелой степени. Ваша тактика:A. Комплексное лечение преэклампсии в условиях ОРИТ, профилактика СДР плода,досрочное родоразрешение в течение 24-48 часовB. Лечение преэклампсии в условиях отделения патологии беременности C. Лечение преэклампсии в условиях ОРИТ с оценкой эффективности через 1 неделю D. Продолжить амбулаторное наблюдениеE. Госпитализация и лечение в дневном стационаре206. В родильный дом доставили беременную без сознания. Родственники отмечают, что она 6 дней назад жаловалась на головные боли. При осмотре: скованность, состояние тяжелое, без сознания. АД 180/100 мм рт.ст., матка увеличена сответственно 31-32 неделям беременности, положение плода продольное, головное, сердцебиение плода 140 ударов в минуту, ясное, ритмичное, моча по катетеру мутная, в количестве 40 мл. Ваш диагноз?A. Беременность 31-32 недель. Вирусный менингитB. Беременность 31-32 недель. Преэклампсия тяжелой степениC. Беременность 31-32 недель. Острое нарушение мозгового кровообращенияD. Беременность 31-32 недель. Эклампсия. КомаE. Беременность 31-32 недель. Острая почечная недостаточность.207.В родильный дом доставили беременную без сознания. Выставлен диагноз: Беременность 31-32 недель. Эклампсия. Кома. Врачебная тактика?A. 1. Интубация трахеи, искусственная вентиляция легких, экстренное кесарево сечениеB. Лечение эклампсии в условиях отделения патологии беременности C. Лечение эклампсии в условиях ОРИТ с оценкой эффективности через 1 неделю D. Диспансерное наблюдение

Page 124: postgraduate.kaznmu.kz€¦ · Web view58 Больная И, 28 лет, доставлена с жалобами на внезапно возникшую боль внизу живота

С.Ж.АСФЕНДИЯРОВАТЫНДАҒЫҚАЗАҚ ҰЛТТЫҚ МЕДИЦИНА

УНИВЕРСИТЕТІ

КАЗАХСКИЙНАЦИОНАЛЬНЫЙМЕДИЦИНСКИЙ УНИВЕРСИТЕТ ИМЕНИ С.Д.АСФЕНДИЯРОВА

КАФЕДРА ИНТЕРНАТУРЫ И РЕЗИДЕНТУРЫ ПО АКУШЕРСТВУ И ГИНЕКОЛОГИИТЕСТОВЫЕ ЗАДАНИЯ ДЛЯ ПРОВЕДЕНИЯ ИТОГОВОЙ ГОСУДАРСТВЕННОЙ АТТЕСТАЦИИ РЕЗИДЕНТОВ

ВЫПУСКА 2012-2013 ГГ. ПО СПЕЦИАЛЬНОСТИ «АКУШЕРСТВО И ГИНЕКОЛОГИЯ»

E. Госпитализация и лечение в дневном стационаре208.У беременной со сроком 9-10 недель отмечается жалобы на боли в костях, мышцах, изменение походки, парестезия в области таза и лобка. Ваш диагноз:A. Беременность 8-9 недель. Корешковый синдромB. Беременность 8-9 недель. Симфизит C. Беременность 8-9 недель. Остеопатия беременныхD. Беременность 8-9 недель. Воспаление седалищного нерваE. Беременность 8-9 недель. Угроза прерывания беременности 210.У беременной при сроке беременности 28-29 недель жалобы на отеки. В анамнезехроническая артериальная гипертензия. Объективно: АД 140/90, отеки на нижних конечностях, в моче белок - 0,33 г/л. Матка увеличена до 28 недель беременности, положение плода продольное, головное, сердцебиение плода ясное, ритмичное до 140 ударов в минуту. Ваш диагноз:A. Беременность 28 недель. Преэклампсия легкой степени. Хроническая артериальная гипертензия B. Беременность 28 недель. Хроническая артериальная гипертензия C. Беременность 28 недель. Отеки, вызванные беременностьюD. Беременность 28 недель. Острый гломерулонефрит E. Беременность 28 недель. Острый пиелонефрит211.В приемный покой обратилась первобеременная, 24 лет со сроком беременности 37-38 недель. При УЗИ определена двойня, оба плода в продольном положении, головном предлежании. Предположительный вес плодов 3000 г. План ведения родов:A. Кесарево сечение в плановом порядкеB. Консервативные роды, при развитии слабости родовой деятельности – кесарево сечение C. Консервативные роды, при развитии слабости родовой деятельности - родостимуляцияD. Консервативные роды, кесарево сечение по жизненным показаниям со стороны роженицыE. Амниотомия, родовозбуждение окситоцином212. Признаком многоплодной беременности является определение:A. Тазового предлежания плодаB. Снижение шевелений плодаC. Трех или более крупных частей плодаD. Косого и поперечного положения плодаE. Многоводия213. При преждевременных родах с целью профилактики дистресс-синдрома плода глюкокортикоиды применяются до:A. 32 недель беременностиB. 34 недель беременности C. 36 недель беременностиD. 38 недель беременностиE. 40 недель беременности214.При физиологической беремености плацента преимущественно располагается:A. В дне маткиB. В нижнем сегменте матки, C. По передней стенке маткиD. По задней стенке матки E. По боковой стенке матки.215. Одной из причин предлежания плаценты являются:A. Хромосомные аномалииB. Врожденные пороки развития плода

Page 125: postgraduate.kaznmu.kz€¦ · Web view58 Больная И, 28 лет, доставлена с жалобами на внезапно возникшую боль внизу живота

С.Ж.АСФЕНДИЯРОВАТЫНДАҒЫҚАЗАҚ ҰЛТТЫҚ МЕДИЦИНА

УНИВЕРСИТЕТІ

КАЗАХСКИЙНАЦИОНАЛЬНЫЙМЕДИЦИНСКИЙ УНИВЕРСИТЕТ ИМЕНИ С.Д.АСФЕНДИЯРОВА

КАФЕДРА ИНТЕРНАТУРЫ И РЕЗИДЕНТУРЫ ПО АКУШЕРСТВУ И ГИНЕКОЛОГИИТЕСТОВЫЕ ЗАДАНИЯ ДЛЯ ПРОВЕДЕНИЯ ИТОГОВОЙ ГОСУДАРСТВЕННОЙ АТТЕСТАЦИИ РЕЗИДЕНТОВ

ВЫПУСКА 2012-2013 ГГ. ПО СПЕЦИАЛЬНОСТИ «АКУШЕРСТВО И ГИНЕКОЛОГИЯ»

C. Воспалительные процессы эндометрияD. Маловодие, многоводиеE. Неправильные положения плода 216. В роддом поступила беременная с жалобами на слабость, кровотечение из половых путей. В обменной карте при последней явке выставлен диагноз: Беременность 35-36 недель. Полное предлежание плаценты. Тактика акушера-гинеколога:A. Родоразрешение путем операции кесарево сечение в плановом порядкеB. Сохраняющая терапия, пролонгирование беременностиC. Родоразрешение путем операции кесарево сечение в срочном порядкеD. Амниотомия и родовозбуждение в плановом порядкеE. Родоразрешение путем кесарево сечения в экстренном порядке217. В роддом поступила беременная со сроком 35-36 недель беременности с жалобами на слабость, кровотечение из половых путей. В анамнезе: 4 медицинских аборта, последний осложнился метроэндометритом, данная беременность пятая. Объективно: кожные покровы бледные, артериальное давление 90/60 мм рт.ст, пульс 96 в минуту, матка не возбудима, положение плода продольное, головка высоко над входом в малый таз, сердцебиение плода 140 ударов в минуту, отеков нет. В зеркалах: кровянистые выделения. Предварительный диагноз: A. Беременность 35-36 недель. Предлежание плаценты. Геморрагический шок 1 степениB. Беременность 35-36 недель. Преждевременная отслойка нормально расположенной плаценты.C. Беременность 35-36 недель. Кровотечение из варикозно расширенных вен влагалищаD. Беременность 35-36 недель.Начавшийся разрыв матки. Геморрагический шок 1 степениE. Беременность 35-36 недель. Предлежание плаценты. Геморрагический шок 2 степени218. Беременная в 19-20 недель обратилась с жалобами на внезапное кровотечение из половых путей. Объективно: высота дна матки - 25 см, окружность живота - 89 см. На зеркалах: шейка матки цианотичная, выделения кровянистые, имеются соединительнотканные пузырьки диаметром до 0,8 см с жидкостным содержимым. Вагинально: матка увеличена до 24-25 недель, цервикальный канал проходим для 1,5 см.Ваш предварительный диагноз:A. Беременность 19-20 недель. Аборт в ходуB. Беременность 19-20 недель. Пузырный заносC. Беременность 19-20 недель. Начавшийся самопроизвольный абортD. Беременность 19-20 недель, предлежание плацентыE. Беременность 19-20 недель, преждевременная отслойка нормально расположенной плаценты219.Беременная в 9-10 недель обратилась с жалобами на внезапное кровотечение из половых путей. При УЗ-исследовании выявлено несоотвествие размеров матки сроку гестации, эмбрион не визуализируется, в полости матки картина «снежной бури». Ваш предварительный диагноз:A. Беременность 9-10 недель. Аборт в ходуB. Беременность 9-10 недель. Пузырный заносC. Беременность 9-10 недель. Начавшийся самопроизвольный абортD. Беременность 9-10 недель. Предлежание хорионаE. Беременность 9-10 недель. Несостоявшийся аборт220.Беременная в 9-10 недель обратилась с жалобами на внезапное кровотечение из половых путей. Выставлен диагноз: Беременность 9-10 недель. Пузырный занос. Ваша тактика:A. Выскабливание полости маткиB. Пролонгирование беременности

Page 126: postgraduate.kaznmu.kz€¦ · Web view58 Больная И, 28 лет, доставлена с жалобами на внезапно возникшую боль внизу живота

С.Ж.АСФЕНДИЯРОВАТЫНДАҒЫҚАЗАҚ ҰЛТТЫҚ МЕДИЦИНА

УНИВЕРСИТЕТІ

КАЗАХСКИЙНАЦИОНАЛЬНЫЙМЕДИЦИНСКИЙ УНИВЕРСИТЕТ ИМЕНИ С.Д.АСФЕНДИЯРОВА

КАФЕДРА ИНТЕРНАТУРЫ И РЕЗИДЕНТУРЫ ПО АКУШЕРСТВУ И ГИНЕКОЛОГИИТЕСТОВЫЕ ЗАДАНИЯ ДЛЯ ПРОВЕДЕНИЯ ИТОГОВОЙ ГОСУДАРСТВЕННОЙ АТТЕСТАЦИИ РЕЗИДЕНТОВ

ВЫПУСКА 2012-2013 ГГ. ПО СПЕЦИАЛЬНОСТИ «АКУШЕРСТВО И ГИНЕКОЛОГИЯ»

C. Ампутация матки без придатковD. Экстирпация матки без придатковE. Сохраняющая терапия, химиотерапия 221.Беременной произведено выскабливание полости матки в 9-10 недель по поводу пузырного заноса, передана на диспансерное наблюдение в женскую консультацию. Гистологический диагноз: трофобластическая болезнь, пузырный занос. Определение какого гормона необходимо проводить систематически: A. Хорионического гонадотропина в мочеB. Прогестерона в кровиC. Тестостерона в крови D. 17-КС в мочеE. Фолликулостимулирующего гормона крови 222.Беременной произведено выскабливание полости матки в 9-10 недель по поводу пузырного заноса. Гистологический диагноз: трофобластическая болезнь, пузырный занос. Передана на диспансерное наблюдение в женскую консультацию.Какова длительность диспансерного наблюдения пациентки с пузырным заносом: A. 6 месяцевB. 1 годC. 2 годаD. 3 годаE. 4 года 223.Основной клинический симптом преждевременной отслойки нормально расположенной плаценты:A. Локальная болезненность и напряжение маткиB. Аномалия родовой деятельностиC. Шарообразная форма маткиD. Кровотечение из половых путейE. Дородовое излитие околоплодных вод224. При влагалищном исследовании у первородящей открытие маточного зева 4 см, определено неполное предлежание плаценты и тазовое предлежание плода. Тактика ведения родов?A. Амниотомия и родостимуляция B. Продолжать роды вести консервативно C. Кесарево сечение в экстренном порядкеD. Роды вести по ЦовьяновуE. Экстракция плода за тазовый конец225. У первородящей, не состоявшей на учете по беременности, в доношенном сроке при влагалищном исследовании открытие маточного зева 4 см, предлежит губчатая ткань на всем протяжении. Ваш диагноз:A. Беременность доношенная. I период родов. Полное предлежание плаценты.B. Беременность доношенная. I период родов. Неполное предлежание плаценты.C. Беременность доношенная. II период родов. Краевое предлежание плаценты.D. Беременность доношенная. I период родов. Низкая плацентация.E. Беременность доношенная. I период родов. Краевое предлежание плаценты.226.У первородящей, не состоявшей на учете по беременности, в доношенном сроке при влагалищном исследовании открытие маточного зева 4 см, определяется губчатая ткань на протяжении 3 см, оболочки шероховатые. Ваш диагноз:A. Беременность доношенная. I период родов. Полное предлежание плаценты.

Page 127: postgraduate.kaznmu.kz€¦ · Web view58 Больная И, 28 лет, доставлена с жалобами на внезапно возникшую боль внизу живота

С.Ж.АСФЕНДИЯРОВАТЫНДАҒЫҚАЗАҚ ҰЛТТЫҚ МЕДИЦИНА

УНИВЕРСИТЕТІ

КАЗАХСКИЙНАЦИОНАЛЬНЫЙМЕДИЦИНСКИЙ УНИВЕРСИТЕТ ИМЕНИ С.Д.АСФЕНДИЯРОВА

КАФЕДРА ИНТЕРНАТУРЫ И РЕЗИДЕНТУРЫ ПО АКУШЕРСТВУ И ГИНЕКОЛОГИИТЕСТОВЫЕ ЗАДАНИЯ ДЛЯ ПРОВЕДЕНИЯ ИТОГОВОЙ ГОСУДАРСТВЕННОЙ АТТЕСТАЦИИ РЕЗИДЕНТОВ

ВЫПУСКА 2012-2013 ГГ. ПО СПЕЦИАЛЬНОСТИ «АКУШЕРСТВО И ГИНЕКОЛОГИЯ»

B. Беременность доношенная. I период родов. Неполное предлежание плаценты.C. Беременность доношенная. II период родов. Краевое предлежание плаценты.D. Беременность доношенная. I период родов. Низкая плацентация.E. Беременность доношенная. I период родов. Краевое предлежание плаценты.227.У первородящей в доношенном сроке при ультразвуковом исследовании выявлено, что нижний край плаценты на расстоянии 4 см от внутреннего зева. Ваш диагноз:A. Беременность доношенная. Полное предлежание плаценты.B. Беременность доношенная. Неполное предлежание плаценты.C. Беременность доношенная. Краевое предлежание плаценты.D. Беременность доношенная. Низкая плацентация.E. Беременность доношенная.228. Во время операции кесарево сечение, произведенного в связи с полным предлежанием плаценты, при попытке отделения плаценты началось обильное кровотечение, плацента не отделяется на всем протяжении. Ваш диагноз:A. Плотное прикрепление плацентыB. Полное истинное приращение плацентыC. Нарушения отделения плацентыD. Неполное истинное приращение плаценты E. Преждевременная отслойка нормально расположенной плаценты229. Во время операции кесарево сечение, произведенного в связи с полным предлежанием плаценты, при попытке отделения плаценты началось обильное кровотечение, плацента отделилась по периферии и не отделяется на участке 5*6 см. Ваш диагноз: A. Плотное прикрепление плацентыB. Полное истинное приращение плацентыC. Нарушения отделения плацентыD. Неполное истинное приращение плаценты E. Преждевременная отслойка нормально расположенной плаценты230. Во время операции кесарево сечение, произведенного, в связи с полным предлежанием плаценты, при попытке отделения плаценты началось обильное кровотечение, плацента не отделяется на всем протяжении. Выставлен диагноз: полное истинное приращение плацентыВаша дальнейшая тактика:A. Введение окситоцина в миометрийB. Введение окситоцина внутривенноC. Экстирпация матки с придаткамиD. Экстирпация матки без придатковE. Ампутация матки без придатков231. Для профилактики дистресс-синдрома плода при преждевременных родах применяются:A. β-адреномиметикиB. β-адреноблокаторыC. ГестагеныD. ГлюкокортикоидыE. Эстрогены232. Основная роль в патогенезе аномалий родовой деятельности отводится:A. Снижению уровня эстрогенаB. Повышению уровня эстрогенаC. Повышению уровня прогестерона D. Снижению уровня кальция

Page 128: postgraduate.kaznmu.kz€¦ · Web view58 Больная И, 28 лет, доставлена с жалобами на внезапно возникшую боль внизу живота

С.Ж.АСФЕНДИЯРОВАТЫНДАҒЫҚАЗАҚ ҰЛТТЫҚ МЕДИЦИНА

УНИВЕРСИТЕТІ

КАЗАХСКИЙНАЦИОНАЛЬНЫЙМЕДИЦИНСКИЙ УНИВЕРСИТЕТ ИМЕНИ С.Д.АСФЕНДИЯРОВА

КАФЕДРА ИНТЕРНАТУРЫ И РЕЗИДЕНТУРЫ ПО АКУШЕРСТВУ И ГИНЕКОЛОГИИТЕСТОВЫЕ ЗАДАНИЯ ДЛЯ ПРОВЕДЕНИЯ ИТОГОВОЙ ГОСУДАРСТВЕННОЙ АТТЕСТАЦИИ РЕЗИДЕНТОВ

ВЫПУСКА 2012-2013 ГГ. ПО СПЕЦИАЛЬНОСТИ «АКУШЕРСТВО И ГИНЕКОЛОГИЯ»

E. Снижению уровня кортизола233. При проведении ультразвукового исследовании в сроке 37-38 недель выявлено, что плацента находится на расстоянии 7,5 см от внутреннего зева.Ваш диагноз:A. Беременность доношенная. Полное предлежание плаценты.B. Беременность доношенная. Неполное предлежание плаценты.C. Беременность доношенная. Краевое предлежание плаценты.D. Беременность доношенная. Низкая плацентация.E. Беременность доношенная.234. У беременной с предлежанием плаценты в сроке 26-27 недель появились кровянистые выделения из половых путей в количестве 150 мл. Тактика врача:A. Госпитализация, экстренное кесарево сечениеB. Лечение в дневном стационареC. Госпитализация, экстренное кесарево сечение, ампутация маткиD. Госпитализация, сохраняющая терапия E. Госпитализация, наблюдение до спонтанных родов в стационаре235. У беременной с предлежанием плаценты в сроке 37-38 недель появились кровянистые выделения из половых путей в количестве 150 мл. Тактика врача:A. Госпитализация, экстренное кесарево сечениеB. Лечение в дневном стационареC. Госпитализация, экстренное кесарево сечение, ампутация маткиD. Госпитализация, сохраняющая терапия E. Госпитализация, наблюдение до спонтанных родов в стационаре236. Беременная с полным предлежанием плаценты в сроке 37-38 недель поступила вроддом, родовой деятельности нет, из половых путей слизистые выделения.Тактика врача: A. Госпитализация, плановое кесарево сечение, с последующей ампутацией маткиB. Госпитализация, экстренное кесарево сечениеC. Госпитализация, плановое кесарево сечениеD. Госпитализация, экстренное кесарево сечение, ампутация маткиE. Госпитализация, сохраняющая терапия237. Беременной с преждевременной отслойкой нормально расположенной плаценты в сроке 37-38 недель показана:A. Госпитализация, экстренное кесарево сечениеB. Госпитализация, плановое кесарево сечениеC. Госпитализация, срочное кесарево сечениеD. Госпитализация, сохраняющая терапия E. Госпитализация, наблюдение до спонтанных родов в стационаре238. У роженицы диагностирована преждевременная отслойка нормально расположенной плаценты в первом периоде родов при открытии маточного зева на 2-3 см. Тактика врача:A. Амниотомия, консервативное ведение родовB. Кесарево сечение в экстренном порядкеC. Консервативное ведение родовD. Амниотомия, родостимуляцияE. Наложение акушерских щипцов

Page 129: postgraduate.kaznmu.kz€¦ · Web view58 Больная И, 28 лет, доставлена с жалобами на внезапно возникшую боль внизу живота

С.Ж.АСФЕНДИЯРОВАТЫНДАҒЫҚАЗАҚ ҰЛТТЫҚ МЕДИЦИНА

УНИВЕРСИТЕТІ

КАЗАХСКИЙНАЦИОНАЛЬНЫЙМЕДИЦИНСКИЙ УНИВЕРСИТЕТ ИМЕНИ С.Д.АСФЕНДИЯРОВА

КАФЕДРА ИНТЕРНАТУРЫ И РЕЗИДЕНТУРЫ ПО АКУШЕРСТВУ И ГИНЕКОЛОГИИТЕСТОВЫЕ ЗАДАНИЯ ДЛЯ ПРОВЕДЕНИЯ ИТОГОВОЙ ГОСУДАРСТВЕННОЙ АТТЕСТАЦИИ РЕЗИДЕНТОВ

ВЫПУСКА 2012-2013 ГГ. ПО СПЕЦИАЛЬНОСТИ «АКУШЕРСТВО И ГИНЕКОЛОГИЯ»

239. У роженицы диагностирована преждевременная отслойка нормально расположенной плаценты в первом периоде родов при открытии маточного зева на 3-4 см. Тактика врача:A. Амниотомия и консервативное ведение родовB. Кесарево сечение в экстренном порядкеC. Консервативное ведение родовD. Амниотомия, родостимуляцияE. Наложение акушерских щипцов240. У роженицы во втором периоде родов диагностирована преждевременная отслойка нормально расположенной плаценты, начавшаяся острая гипоксия плода, головка на тазовом дне. Тактика врача:A. Консервативное ведение родовB. Кесарево сечение в экстренном порядкеC. ЭпизиотомияD. Родостимуляция окситоцином внутривенно капельноE. Наложение акушерских щипцов241. У роженицы при вагинальном исследовании: открытие маточного зева полное, крестцовая впадина и лонное сочленение свободны, головка плода не отталкивается, стреловидный шов в левом косом размере, малый родничок справа и спереди. Определите позицию, вид плода и местоположение головки:A. I позиция, передний вид, головка прижата ко входу в малый таз B. I позиция, задний вид, головка большим сегментом во входе в малый таз C. II позиция, передний вид, головка малым сегментом во входе в малый таз D. II позиция, задний вид, головка над входом в малый таз E. I позиция, передний вид, головка малым сегментом во входе в малый таз 242. При каком предлежании головка прорезывается малым косым размером:A. Переднем виде затылочного B. Заднем виде затылочного C. Переднетеменном D. ЛобномE. Лицевом243. При каком предлежании головка прорезывается средним косым размером:A. Переднем виде затылочного B. Заднем виде затылочного C. Переднетеменном D. ЛобномE. Лицевом244. При переднем виде затылочного предлежания родовая опухоль расположена в области:A. Малого родничкаB. Большого родничкаC. ЛбаD. ЛицаE. Середины расстояния между малым и большим родничками245. При заднем виде затылочного предлежания родовая опухоль расположена в области:A. Малого родничкаB. Большого родничкаC. Середины расстояния между малым и большим родничками D. ЛицаE. Подбородка

Page 130: postgraduate.kaznmu.kz€¦ · Web view58 Больная И, 28 лет, доставлена с жалобами на внезапно возникшую боль внизу живота

С.Ж.АСФЕНДИЯРОВАТЫНДАҒЫҚАЗАҚ ҰЛТТЫҚ МЕДИЦИНА

УНИВЕРСИТЕТІ

КАЗАХСКИЙНАЦИОНАЛЬНЫЙМЕДИЦИНСКИЙ УНИВЕРСИТЕТ ИМЕНИ С.Д.АСФЕНДИЯРОВА

КАФЕДРА ИНТЕРНАТУРЫ И РЕЗИДЕНТУРЫ ПО АКУШЕРСТВУ И ГИНЕКОЛОГИИТЕСТОВЫЕ ЗАДАНИЯ ДЛЯ ПРОВЕДЕНИЯ ИТОГОВОЙ ГОСУДАРСТВЕННОЙ АТТЕСТАЦИИ РЕЗИДЕНТОВ

ВЫПУСКА 2012-2013 ГГ. ПО СПЕЦИАЛЬНОСТИ «АКУШЕРСТВО И ГИНЕКОЛОГИЯ»

246. У повторнородящей родовая деятельность в течение 3-х часов, схватки каждые 1-2 минуты, по 50-55 секунд. Вагинально: открытие маточного зева на 7 см, плодный пузырь цел, головка малым сегментом во входе в малый таз. Оцените темп родов: A. Стремительный B. Быстрый C. НедостаточныйD. Слабый E. Удовлетворительный247. У повторнородящей родовая деятельность в течение 2-х часов, схватки каждые 1-2 минуты, по 50-55 секунд. Вагинально: открытие маточного зева полное, плодного пузыря нет, головка большим сегментом во входе в малый таз. Оцените темп родов: A. Стремительный B. Быстрый C. НедостаточныйD. Слабый E. Удовлетворительный248. У первородящей родовая деятельность в течение 3-х часов, схватки каждые 1-2 минуты, по 50-55 секунд. Вагинально: открытие маточного зева полное, плодного пузыря нет, головка большим сегментом во входе в малый таз. Оцените темп родов: A. Стремительный B. Быстрый C. НедостаточныйD. Слабый E. Удовлетворительный249. У первородящей родовая деятельность в течение 5-ти часов, схватки каждые 1-2 минуты, по 50-55 секунд. Вагинально: открытие маточного зева полное, плодного пузыря нет, головка большим сегментом во входе в малый таз. Оцените темп родов: A. Стремительный B. Быстрый C. НедостаточныйD. Слабый E. Удовлетворительный250. У первородящей родовая деятельность в течение 15-ти часов, схватки каждые 5 минут, по 30 секунд. Вагинально: открытие маточного зева 7 см, плодного пузыря нет, головка малым сегментом во входе в малый таз. Оцените темп родов: A. Стремительный B. Быстрый C. ЧрезмерныйD. Слабый E. Удовлетворительный

251.Первородящая 23 лет доставлена в роддом после припадка эклампсии, который произошел дома. Беременность 37-38 недель, состояние тяжелое, сознание заторможенное. АД 150/100 мм

Page 131: postgraduate.kaznmu.kz€¦ · Web view58 Больная И, 28 лет, доставлена с жалобами на внезапно возникшую боль внизу живота

С.Ж.АСФЕНДИЯРОВАТЫНДАҒЫҚАЗАҚ ҰЛТТЫҚ МЕДИЦИНА

УНИВЕРСИТЕТІ

КАЗАХСКИЙНАЦИОНАЛЬНЫЙМЕДИЦИНСКИЙ УНИВЕРСИТЕТ ИМЕНИ С.Д.АСФЕНДИЯРОВА

КАФЕДРА ИНТЕРНАТУРЫ И РЕЗИДЕНТУРЫ ПО АКУШЕРСТВУ И ГИНЕКОЛОГИИТЕСТОВЫЕ ЗАДАНИЯ ДЛЯ ПРОВЕДЕНИЯ ИТОГОВОЙ ГОСУДАРСТВЕННОЙ АТТЕСТАЦИИ РЕЗИДЕНТОВ

ВЫПУСКА 2012-2013 ГГ. ПО СПЕЦИАЛЬНОСТИ «АКУШЕРСТВО И ГИНЕКОЛОГИЯ»

рт. ст., пульс 98 уд/минуту. Выраженные отеки на ногах. Определите наиболее вероятную тактику ведения беременной:A. пролонгирование беременности на фоне леченияB. проведение комплексной интенсивной терапии в течение 2-3 днейC. кесарево сечение на фоне комплексной интенсивной терапииD. родовозбуждение с последующим наложением акушерских щипцовE. подготовка родовых путей с целью индукции родов252.У роженицы, с предполагаемой массой плода 4500,0 гр., потуги через 1-2 минуты по 50-60 секунд, интенсивные. Признак Вастена положительный, нижний сегмент болезненный при пальпации вне схватки, контракционное кольцо на уровне пупка. Не мочится 3 часа. Предлежит головка, фиксирована во входе в малый таз. Сердцебиение плода до 160 уд/минуту. Влагалищное исследование: открытие полное, края шейки матки отечные. Головка фиксирована во входе в малый таз. Стреловидный шов в правом косом размере. Малый родничок справа у крестца. На головке выражена родовая опухоль. Мыс не достигается. Наиболее вероятный диагноз:

A. II период родов. Крупный плод. Чрезмерная родовая деятельность.B. II период родов. Крупный плод. Дискоординированная родовая деятельность.C. II период родов. Крупный плод. Клинически узкий таз. Угрожающий разрыв матки.D. II период родов. Клинически узкий таз. Крупный плод. Начавшийся разрыв матки.E. II период родов. Клинически узкий таз. Крупный плод. Совершившийся разрыв матки.

253.Беременная 28 лет. Срок беременности 38 недель. В анамнезе 1,5 года назад кесарево сечение. При поступлении жалобы на постоянные боли внизу живота, которые усиливаются при шевелении плода. Матка в нормотонусе. Предлежит головка. Сердцебиение плода ясное, 136 уд/минуту. При пальпации области послеоперационного рубца боли усиливаются. Влагалищное исследование: шейка мягкая, длина 1,5 см, цервикальный канал пропускает 1 поперечный палец за внутренний зев, в области которого определяется «переходной валик». Предлежит головка. Мыс не достижим. Выделения слизистые. Наиболее вероятный диагноз:

A. беременность 38 недель. Рубец на матке. Предвестники родовB. беременность 38 недель. Рубец на матке. Преждевременная отслойка нормально

расположенной плацентыC. беременность 38 недель. Рубец на матке. I период родовD. Беременность 38 недель. Угрожающий разрыв матки по рубцуE. Беременность 38 недель. Начавшийся разрыв матки по рубцу

254.У повторнобеременной, первородящей женщины 26 лет произошел полный разрыв матки в нижнем сегменте. Плод погиб, признаков инфекции нет. Наиболее вероятно пациентке показано:A. лапаротомия, ушивание разрыва матки в нижнем сегментеB. лапаротомия, надвлагалищная ампутация матки без придатковC. лапаротомия, экстирпация матки без придатковD. лапаротомия, надвлагалищная ампутация матки с трубамиE. лапаротомия, экстирпация матки с трубами255.При наружном акушерском обследовании беременной в дне матки пальпируется крупная часть мягкой консистенции, у входа в малый таз - крупная часть плотной консистенции, шаровидной формы, слева и кпереди - ровная без выступов поверхность, справа и кзади - бугристая поверхность. Наиболее вероятный диагноз?

A. тазовое предлежание, 1-я позиция, передний видB. тазовое предлежание, 2-я позиция, задний вид C. головное предлежание, 1-я позиция, задний видD. головное предлежание, 1-я позиция, передний вид

Page 132: postgraduate.kaznmu.kz€¦ · Web view58 Больная И, 28 лет, доставлена с жалобами на внезапно возникшую боль внизу живота

С.Ж.АСФЕНДИЯРОВАТЫНДАҒЫҚАЗАҚ ҰЛТТЫҚ МЕДИЦИНА

УНИВЕРСИТЕТІ

КАЗАХСКИЙНАЦИОНАЛЬНЫЙМЕДИЦИНСКИЙ УНИВЕРСИТЕТ ИМЕНИ С.Д.АСФЕНДИЯРОВА

КАФЕДРА ИНТЕРНАТУРЫ И РЕЗИДЕНТУРЫ ПО АКУШЕРСТВУ И ГИНЕКОЛОГИИТЕСТОВЫЕ ЗАДАНИЯ ДЛЯ ПРОВЕДЕНИЯ ИТОГОВОЙ ГОСУДАРСТВЕННОЙ АТТЕСТАЦИИ РЕЗИДЕНТОВ

ВЫПУСКА 2012-2013 ГГ. ПО СПЕЦИАЛЬНОСТИ «АКУШЕРСТВО И ГИНЕКОЛОГИЯ»

E. головное предлежание, 2-я позиция, передний вид256Повторнородящая, поступила в роддом с жалобами на схваткообразные боли внизу живота при беременности 30 недель. При обследовании: схватки через 15-20 минут по 20-25 секунд, слабой интенсивности. Воды не отходили. Сердцебиение плода до 136 уд/минуту. Соматически здорова. PV: шейка матки укорочена до 1 см, мягкая, цервикальный канал свободно пропускает 1 поперечный палец за внутренний зев. Плодный пузырь цел. Предлежит головка. Выделения слизистые. Поставлен диагноз: Начинающиеся преждевременные роды при беременности 30 недель. Головное предлежание. Наиболее вероятная акушерская тактика:A. усилить родовую деятельностьB. наблюдать за динамикой родовой деятельностиC. прекратить родовую деятельностьD. проводить профилактику дистресс-синдромаE. передать под строгий контроль женской консультации

257Повторнородящая, поступила в роддом с жалобами на схваткообразные боли внизу живота при беременности 30 недель. При обследовании: схватки через 15-20 минут по 20-25 секунд, слабой интенсивности. Воды не отходили. Сердцебиение плода до 136 уд/минуту. Соматически здорова. PV: шейка матки укорочена до 1 см, мягкая, цервикальный канал свободно пропускает 1 поперечный палец за внутренний зев. Плодный пузырь цел. Предлежит головка. Выделения слизистые. Наиболее вероятный диагноз:

A. Начинающиеся преждевременные роды при беременности 30 недель. B. Угрожающие преждевременные роды при беременности 30 недель. C. Беременность 30 недель. Предвестники родовD. Беременность 30 недель. I период родовE. Беременность 30 недель. ИЦН

258.В роддом доставлена повторнобеременная со сроком беременности 32 недели. Беременность третья, в анамнезе два искусственных аборта, после второго аборта был эндометрит. Среди полного благополучия из половых путей появились обильные кровянистые выделения. Кровопотеря около 300,0 мл. Из обменной карты беременной выявлено, что при УЗИ в 30 недель выявлено полное предлежание плаценты. АД 105/60 мм рт. ст, пульс 90 уд/минуту. Матка в нормотонусе, легко возбудима при пальпации. Положение плода поперечное. Сердцебиение плода не выслушивается. Наиболее вероятная акушерская тактика:

A. инфузионная терапия и кесарево сечениеB. гемотрансфузия и кесарево сечениеC. амниотомия и наружно-внутренний поворотD. амниотомия и плодоразрушающая операцияE. амниотомия и надвлагалищная ампутация матки

259.В каком случае при аномалиях родовой деятельности НАИБОЛЕЕ целесообразно проведение родостимуляции?

A. наличие сниженного базального тонуса маткиB. наличие повышенного базального тонуса маткиC. начальная степень дискоординации сокращений мышц маткиD. спастическая сегментарная дистоцияE. спастическая тотальная дистоция

260 Роды в ягодичном предлежании. Безводный период 20 минут. Во время влагалищного исследования произошло выпадение петли пуповины. Сердцебиение плода ритмичное, 160 уд/минуту. Раскрытие шейки матки 6-7 см. НАИБОЛЕЕ вероятная тактика?

A. не вынимая руки из влагалища перейти в операционную и произвести кесарево сечениеB. заправить петлю пуповины и вести роды консервативно

Page 133: postgraduate.kaznmu.kz€¦ · Web view58 Больная И, 28 лет, доставлена с жалобами на внезапно возникшую боль внизу живота

С.Ж.АСФЕНДИЯРОВАТЫНДАҒЫҚАЗАҚ ҰЛТТЫҚ МЕДИЦИНА

УНИВЕРСИТЕТІ

КАЗАХСКИЙНАЦИОНАЛЬНЫЙМЕДИЦИНСКИЙ УНИВЕРСИТЕТ ИМЕНИ С.Д.АСФЕНДИЯРОВА

КАФЕДРА ИНТЕРНАТУРЫ И РЕЗИДЕНТУРЫ ПО АКУШЕРСТВУ И ГИНЕКОЛОГИИТЕСТОВЫЕ ЗАДАНИЯ ДЛЯ ПРОВЕДЕНИЯ ИТОГОВОЙ ГОСУДАРСТВЕННОЙ АТТЕСТАЦИИ РЕЗИДЕНТОВ

ВЫПУСКА 2012-2013 ГГ. ПО СПЕЦИАЛЬНОСТИ «АКУШЕРСТВО И ГИНЕКОЛОГИЯ»

C. кесарево сечение в экстренном порядкеD. извлечение плода за тазовый конецE. извлечение плода за ножку

261.При влагалищном исследовании роженицы определяется гладкая поверхность, пересеченная швом. С одной стороны к шву примыкает надпереносье и надбровные дуги, с другой - передний угол большого родничка. НАИБОЛЕЕ вероятное предлежание плода:

A. передний вид затылочного предлежанияB. задний вид затылочного предлежанияC. переднеголовное предлежаниеD. лобное предлежаниеE. лицевое предлежание

262.Первородящая 35 лет, в родах 10 часов. Срок беременности 41-42 недели. Предполагаемая масса плода 3900 гр. Сердцебиение плода 160 уд/минуту. PV: шейка сглажена, открытие 5 см. Плодный пузырь цел. Головка прижата ко входу в малый таз. Размеры таза нормальные. НАИБОЛЕЕ вероятная акушерская тактика:

A. родостимуляция окситоциномB. амниотомияC. наблюдение в динамикеD. кесарево сечениеE. введение физиологического раствора

263.Повторнобеременная, первородящая, 28 лет. В анамнезе 2 самопроизвольных выкидыша. В родах 20 часов. Схватки редкие, короткие, слабые. Открытие маточного зева 6 см. Соразмерность головки плода и таза матери полная. Безводный промежуток 10 часов. На КТГ – угрожаемое состояние плода. НАИБОЛЕЕ вероятная акушерская тактика:

A. начать антибактериальную терапиюB. введение физиологического раствораC. начать стимуляцию родовой деятельностиD. произвести операцию кесарева сеченияE. изменить положение роженицы

264.Повторнородящая доставлена в родильное отделение по поводу срочных родов. Предлежит тазовый конец, родовая деятельность активная. В процессе исследования излились околоплодные воды, после чего сердцебиение плода стало редким, до 90 ударов/минуту. PV: открытие шейки полное, плодного пузыря нет, во влагалище прощупывается ножка плода и выпавшая пуповина, ягодицы плода во входе таза. НАИБОЛЕЕ вероятная акушерская тактика?

A. заправить пуповину, продолжить консервативное ведение родовB. внутривенное введение физиологического раствораC. произвести кесарево сечениеD. произвести экстракцию плода за тазовый конецE. изменить положение роженицы

265.Первобеременная, 26 лет, поступила в роддом с отошедшими околоплодными водами и первичной слабостью родовой деятельности, по поводу чего проводилась стимуляция окситоцином. Через 10 минут от начала потуг изменилось сердцебиение плода, оно стало редким – 100-90 уд/минуту, глухим и аритмичным. При осмотре: открытие шейки полное, головка плода в узкой части полости малого таза, стреловидный шов в правом косом размере, малый родничок кпереди. НАИБОЛЕЕ вероятная акушерская тактика?

A. введение физиологического раствораB. кесарево сечение

Page 134: postgraduate.kaznmu.kz€¦ · Web view58 Больная И, 28 лет, доставлена с жалобами на внезапно возникшую боль внизу живота

С.Ж.АСФЕНДИЯРОВАТЫНДАҒЫҚАЗАҚ ҰЛТТЫҚ МЕДИЦИНА

УНИВЕРСИТЕТІ

КАЗАХСКИЙНАЦИОНАЛЬНЫЙМЕДИЦИНСКИЙ УНИВЕРСИТЕТ ИМЕНИ С.Д.АСФЕНДИЯРОВА

КАФЕДРА ИНТЕРНАТУРЫ И РЕЗИДЕНТУРЫ ПО АКУШЕРСТВУ И ГИНЕКОЛОГИИТЕСТОВЫЕ ЗАДАНИЯ ДЛЯ ПРОВЕДЕНИЯ ИТОГОВОЙ ГОСУДАРСТВЕННОЙ АТТЕСТАЦИИ РЕЗИДЕНТОВ

ВЫПУСКА 2012-2013 ГГ. ПО СПЕЦИАЛЬНОСТИ «АКУШЕРСТВО И ГИНЕКОЛОГИЯ»

C. применение кислородаD. вакуум-экстракция плодаE. краниотомия

266. Роженица находится в родах около 10 часов. Воды не изливались. Внезапно роженица побледнела, появилась рвота, сильные распирающие боли в животе, матка приняла асимметричную форму, плотная. Сердцебиение плода глухое. PV: открытие шейки полное, плодный пузырь цел, напряжен, предлежащая головка в широкой части полости малого таза. НАИБОЛЕЕ вероятная акушерская тактика?

A. срочно приступить к операции кесарева сеченияB. вскрыть плодный пузырь и наложить акушерские щипцыC. вскрыть плодный пузырь и вакуум-экстракция плодаD. лечение угрожаемого состояния плодаE. стимуляция окситоцином

267.Первородящая 34 года, находится во втором периоде родов. В детстве перенесла оперативное вмешательство по поводу открытой травмы промежности. Ягодицы плода на тазовом дне. Схватки потужного характера через 2-3 минуты, по 40-45 секунд, средней силы. Предполагаемая масса плода 3500,0 гр. Сердцебиение плода 150 ударов/минуту. Промежность с рубцовыми изменениями. Что НАИБОЛЕЕ вероятно необходимо предусмотреть в плане ведения родов?

A. перинео- или эпизиотомиюB. введение спазмолитиковC. ручное пособие по ЦовьяновуD. введение окситоцинаE. местную анестезию

268.Роды пятые в срок. Схватки начались 12 часов назад. Родовая деятельность хорошая, форма живота округлая, окружность живота 107 см, расстояние от лобка до пупка 21 см, до дна матки - 29 см, до мечевидного отростка - 42 см. Сердцебиение плода 135 ударов/минуту, ясное, слева на уровне пупка. Наружные размеры таза: 25-28-31-20 см. PV: шейка сглажена, раскрытие зева полное, плодный пузырь цел, слегка напряжен. Предлежащая часть не определяется. НАИБОЛЕЕ вероятная тактика?

A. классический внутренний поворот плодаB. кесарево сечениеC. акушерские щипцыD. вакуум-экстракция плодаE. декапитация

269.Повторнородящая 43 года. Имела нормальных 3-е родов. Настоящие роды продолжаются 48 часов. Воды отошли 16 часов назад. Температура тела 37,6°С, пульс 90 ударов/минуту. В начале схватки были очень сильными, а затем они стали все короче и слабее. Головка плода очень большая, не вставляется ко входу в малый таз. PV: открытие шейки матки полное, швы широкие, роднички очень большие, при прощупывании головка эластической консистенции. НАИБОЛЕЕ вероятная тактика?

A. кесарево сечениеB. стимуляция окситоциномC. акушерские щипцыD. краниотомияE. вакуум-экстракция плода

270.Доставлена роженица при 4-х срочных родах с отошедшими дома 12 часов тому назад водами и свисающей из половых путей правой ручкой плода, которая выпала 4 часа назад. Поперечное

Page 135: postgraduate.kaznmu.kz€¦ · Web view58 Больная И, 28 лет, доставлена с жалобами на внезапно возникшую боль внизу живота

С.Ж.АСФЕНДИЯРОВАТЫНДАҒЫҚАЗАҚ ҰЛТТЫҚ МЕДИЦИНА

УНИВЕРСИТЕТІ

КАЗАХСКИЙНАЦИОНАЛЬНЫЙМЕДИЦИНСКИЙ УНИВЕРСИТЕТ ИМЕНИ С.Д.АСФЕНДИЯРОВА

КАФЕДРА ИНТЕРНАТУРЫ И РЕЗИДЕНТУРЫ ПО АКУШЕРСТВУ И ГИНЕКОЛОГИИТЕСТОВЫЕ ЗАДАНИЯ ДЛЯ ПРОВЕДЕНИЯ ИТОГОВОЙ ГОСУДАРСТВЕННОЙ АТТЕСТАЦИИ РЕЗИДЕНТОВ

ВЫПУСКА 2012-2013 ГГ. ПО СПЕЦИАЛЬНОСТИ «АКУШЕРСТВО И ГИНЕКОЛОГИЯ»

положение плода, подвижности плода нет. Температура тела у роженицы 38,5°С. Сердцебиение плода глухое, аритмичное, до 100 ударов/минуту. Потуги частые. НАИБОЛЕЕ вероятная тактика?

A. акушерский поворотB. интраперитонеальное кесарево сечениеC. экстраперитонеальное кесарево сечениеD. краниотомияE. краниоклазия

271.Через 30 минут после начала второго периода родов у повторнородящей появились кровянистые выделения из половых путей. АД 120/65 мм рт. ст. Схватки через 2-3 минуты по 50-55 секунд. Матка плохо расслабляется вне схватки, болезненна при пальпации в нижнем сегменте, размеры большого таза 25-28-31-18 см. Признак Вастена «вровень». Сердцебиение плода глухое, 150-160 ударов/минуту. Попытка произвести катетеризацию мочевого пузыря женщины не удалась из-за механического препятствия. PV: открытие шейки матки полное, отёк передней губы шейки матки; головка плода прижата ко входу в малый таз, на головке большая родовая опухоль. Сагиттальный шов отклонён кпереди, достигаются большой и малый роднички, расположенные на одном уровне. Наиболее вероятный диагноз?

A. клинически узкий тазB. дискоордированная родовая деятельностьC. крупный плодD. задний асинклитизмE. передний асинклитизм

272.Через 30 минут после начала второго периода родов у повторнородящей появились кровянистые выделения из половых путей. АД 120/65 мм рт. ст. Схватки через 2-3 минуты по 50-55 секунд. Матка плохо расслабляется вне схватки, болезненна при пальпации в нижнем сегменте, размеры большого таза 25-28-31-18 см. Признак Вастена «вровень». Сердцебиение плода глухое, 150-160 ударов/минуту. Попытка произвести катетеризацию мочевого пузыря женщины не удалась из-за механического препятствия. PV: открытие шейки матки полное, отёк передней губы шейки матки; головка плода прижата ко входу в малый таз, на головке большая родовая опухоль. Сагиттальный шов отклонён кпереди, достигаются большой и малый роднички, расположенные на одном уровне. Наиболее вероятная тактика?

A. акушерские щипцыB. вакуум-экстракция плодаC. кесарево сечениеD. региональная анестезияE. стимуляция окситоцином

273.У роженицы 19 лет схватки начались 10 часов назад. Беременность первая, протекала удовлетворительно. Схватки через 5-6 минут по 30 секунд, слабые. Роженица утомлена, дремлет между схватками. Объективно: рост 163 см, весь 65 кг. Размеры таза нормальные. А/Д 110/70, 115/70 мм рт. ст. Положение плода продольное, предлежит головка, прижата ко входу в таз. Сердцебиение плода ясное, ритмичное, 136 ударов в минуту. Воды целы. Предполагаемая масса плода 3400,0 граммов. PV: влагалище нерожавшей, шейка сглажена, края мягкие, растяжимые, открытие зева 4 см. Плодный пузырь цел, во время схватки наливается слабо. Головка прижата ко входу в малый таз. Стреловидный шов в правом косом размере таза, малый родничок слева у лона. Мыс не достигается. Наиболее вероятный метод, который позволяет подтвердить клинический диагноз:

A. партограммаB. структурные изменения шейки маткиC. гистерография

Page 136: postgraduate.kaznmu.kz€¦ · Web view58 Больная И, 28 лет, доставлена с жалобами на внезапно возникшую боль внизу живота

С.Ж.АСФЕНДИЯРОВАТЫНДАҒЫҚАЗАҚ ҰЛТТЫҚ МЕДИЦИНА

УНИВЕРСИТЕТІ

КАЗАХСКИЙНАЦИОНАЛЬНЫЙМЕДИЦИНСКИЙ УНИВЕРСИТЕТ ИМЕНИ С.Д.АСФЕНДИЯРОВА

КАФЕДРА ИНТЕРНАТУРЫ И РЕЗИДЕНТУРЫ ПО АКУШЕРСТВУ И ГИНЕКОЛОГИИТЕСТОВЫЕ ЗАДАНИЯ ДЛЯ ПРОВЕДЕНИЯ ИТОГОВОЙ ГОСУДАРСТВЕННОЙ АТТЕСТАЦИИ РЕЗИДЕНТОВ

ВЫПУСКА 2012-2013 ГГ. ПО СПЕЦИАЛЬНОСТИ «АКУШЕРСТВО И ГИНЕКОЛОГИЯ»

D. кардиотахографияE. наблюдение

274.Повторнородящая с беременностью 39-40 нед. поступила в родильный дом с жалобами на нерегулярные тянущие боли внизу живота и пояснице. Состояние удовлетворительное. Положение плода продольное, предлежит головка. Сердцебиение плода ясное, ритмичное до 150 ударов в мин. PV: влагалище рожавшей, шейка матки расположена центрально, укорочена до 2,5 см, мягкой консистенции, цервикальный канал проходим для 1 п/п за внутренний зев, «переходного» валика нет, плодный пузырь цел, предлежит головка, прижата ко входу в малый таз. Какая тактика ведения является НАИБОЛЕЕ целесообразной:А) вмешательства не требуетсяB) произвести амниотомию, с последующим родовозбуждением C) сразу начать родовозбуждение окситоциномD) подготовка родовых путей простагландинамиE) подготовка родовых путей мезопростолом275.У роженицы 20 лет во втором периоде своевременных родов, когда головка плода была расположена в узкой части полости малого таза, произошел припадок эклампсии. Какая тактика ведения является НАИБОЛЕЕ целесообразной?А) продолжать роды на фоне магнезиальной терапииB) начать родостимуляцию на фоне магнезиальной терапииC) наложить акушерские щипцы на фоне магнезиальной терапииD) родоразрешить путем кесарева сечения E) вмешательства не требуется276.Первородящая во II периоде родов 2,5 часа. Роженица ведет себя чрезвычайно беспокойно, жалуется на распирающие боли в животе. Бурные схватки, вне схваток матка почти не расслабляется. Сердцебиение плода до 170 ударов в минуту. При пальпации живота определяются напряжение и резкая болезненность. Контракционное кольцо находится на уровне пупка, матка в форме песочных часов. PV: полное открытие зева, плодного пузыря нет, головка находится во входе в малый таз, отмечается большая родовая опухоль и выраженный отек губы зева.Какой из перечисленных предварительных диагнозов НАИБОЛЕЕ вероятен? A) дискоординированная родовая деятельность B) чрезмерная родовая деятельностьC) преждевременная отслойка плацентыD) угрожающий разрыв маткиE) свершившийся разрыв матки277.У повторнородящей во II периоде родов, нет прогресса в продвижении головки по плоскостям малого таза в течение 1 часа 30 мин. Состояние удовлетворительное. Сердцебиение плода не страдает. Потуги через 1-2 мин, по 60 сек, хорошей силы. PV: открытие маточного зева полное, плодного пузыря нет, предлежит головка, во входе в малый таз, стреловидный шов отклонен кпереди к лону.Наиболее вероятная тактика ведения родов: A) вмешательства не требуетB) родостимуляция окситоциномC) акушерские щипцыD) вакуум-экстракция плодаE) кесарево сечение

Page 137: postgraduate.kaznmu.kz€¦ · Web view58 Больная И, 28 лет, доставлена с жалобами на внезапно возникшую боль внизу живота

С.Ж.АСФЕНДИЯРОВАТЫНДАҒЫҚАЗАҚ ҰЛТТЫҚ МЕДИЦИНА

УНИВЕРСИТЕТІ

КАЗАХСКИЙНАЦИОНАЛЬНЫЙМЕДИЦИНСКИЙ УНИВЕРСИТЕТ ИМЕНИ С.Д.АСФЕНДИЯРОВА

КАФЕДРА ИНТЕРНАТУРЫ И РЕЗИДЕНТУРЫ ПО АКУШЕРСТВУ И ГИНЕКОЛОГИИТЕСТОВЫЕ ЗАДАНИЯ ДЛЯ ПРОВЕДЕНИЯ ИТОГОВОЙ ГОСУДАРСТВЕННОЙ АТТЕСТАЦИИ РЕЗИДЕНТОВ

ВЫПУСКА 2012-2013 ГГ. ПО СПЕЦИАЛЬНОСТИ «АКУШЕРСТВО И ГИНЕКОЛОГИЯ»

278Первородящая в родах 15 часов. Потуги через 2 минуты до 50-55 сек., хорошей силы. PV: шейка матки сглажена, открытие маточного зева полное, плодного пузыря нет, головка прижата ко входу в малый таз. Стреловидный шов слегка отклонен к крестцу.Какой из перечисленных предварительных диагнозов НАИБОЛЕЕ вероятен? A) II период родов. Слабость родовой деятельности.B) II период родов. Высокое прямое стояние головки.C) II период родов. Клинически узкий таз.D) II период родов. Переднетеменной асинклитизм.E) II период родов. Заднетеменной асинклитизм.279.При влагалищном исследовании женщины определяется: 2/3 внутренней поверхности лонного сочленения и верхняя половина крестцовой впадины заняты головкой. Свободно прощупывается 4-5 крестцовые позвонки и седалищные кости. Стреловидный шов стоит в правом косом размере. Определите, в какой плоскости таза находится головка?А) головка во входе в малый таз большим сегментомВ) головка в узкой части полости малого тазаС) головка в широкой части полости малого таза D) головка в плоскости выхода малого тазаЕ) головка во входе в малый таз малым сегментом280.3 минуты назад произошли роды живым доношенным плодом от 2 беременности и 2-х родов. Женщина выбрала выжидательную тактику ведения 3-го периода. Через 10 минут дно матки поднялось выше пупка, тело матки отклонилось вправо, зажим, наложенный на пуповину у половой щели опустился на 10 см, появился позыв к потуге. Какие признаки наступления 3-го периода вы отмечаете (по авторам)?

A. признак Шредера, признак Кюстнера-Чукалова, признак АльфельдаB. признак Шредера, признак Кюстнера-Чукалова, признак Альфельда, признак КлейнаC. признак Шредера, признак Кюстнера-Чукалова, признак КлейнаD. признак Шредера, признак Альфельда, признак Микулича,E. признак Микулича, признак Гентера, признак Кюстнера-Чукалова

281.Скорой помощью доставлена роженица через 36 часов после излития вод, с запущенным поперечным положением плода, сердцебиение плода не прослушивается. PV: открытие полное, шейка плода достижима. НАИБОЛЕЕ правильная тактика врача?А) продолжать консервативное ведение родовB) произвести кесарево сечениеC) произвести внутренний акушерский поворотD) провести 1 тур родостимуляцииE) произвести плодоразрушающую операцию 282.Определите патологию раннего послеродового периода по следующей клинической картине: состояние относительно удовлетворительное. АД 100/70 мм рт. ст., пульс 98 ударов/минуту. Кожные покровы обычной окраски, из половых путей умеренные кровянистые выделения, достигли 500,0 мл и продолжаются. Послед цел. При осмотре родовых путей – разрывов нет. При наружном массаже матка приходит в тонус и через некоторое время расслабляется. Какой наиболее вероятный диагноз?

A. нарушение свертывающей системы кровиB. разрыв маткиC. трофобластическая болезньD. атоническое кровотечениеE. ДВС-синдром

Page 138: postgraduate.kaznmu.kz€¦ · Web view58 Больная И, 28 лет, доставлена с жалобами на внезапно возникшую боль внизу живота

С.Ж.АСФЕНДИЯРОВАТЫНДАҒЫҚАЗАҚ ҰЛТТЫҚ МЕДИЦИНА

УНИВЕРСИТЕТІ

КАЗАХСКИЙНАЦИОНАЛЬНЫЙМЕДИЦИНСКИЙ УНИВЕРСИТЕТ ИМЕНИ С.Д.АСФЕНДИЯРОВА

КАФЕДРА ИНТЕРНАТУРЫ И РЕЗИДЕНТУРЫ ПО АКУШЕРСТВУ И ГИНЕКОЛОГИИТЕСТОВЫЕ ЗАДАНИЯ ДЛЯ ПРОВЕДЕНИЯ ИТОГОВОЙ ГОСУДАРСТВЕННОЙ АТТЕСТАЦИИ РЕЗИДЕНТОВ

ВЫПУСКА 2012-2013 ГГ. ПО СПЕЦИАЛЬНОСТИ «АКУШЕРСТВО И ГИНЕКОЛОГИЯ»

283.У родильницы после родов переношенным плодом в раннем послеродовом периоде возникло атоническое кровотечение. После принятых мер кровотечение прекратилось, матка плотная, общая кровопотеря достигла 1000,0 мл. Состояние родильницы средней степени тяжести, кожные покровы бледные, пульс 112 ударов/минуту, слабого наполнения. АД 80/50 мм рт. ст. Проба Ли – Уайта 8 минут. Наиболее вероятный диагноз? A) Атоническое кровотечение. Геморрагический шок III степени. B) Атоническое кровотечение. Геморрагический шок II степени. C) Атоническое кровотечение. Геморрагический шок I степени. D) Атоническое кровотечение. ДВС-синдромE) Атоническое кровотечение284.Ранний послеродовый период. Кровопотеря 250,0 мл и продолжается. Матка на уровне пупка, мягкая. После наружного массажа матка сократилась, но затем вновь расслабилась. Кровопотеря 400,0 мл, женщина побледнела, появилось головокружение, АД 90/50 мм рт. ст, пульс 100 уд/ минуту. Наиболее вероятная тактика ведения родильницы? A) приступить к переливанию кровиB) вводить внутривенно окситоцинC) хирургический гемостазD) ручное обследование полости маткиE) бимануальная компрессия285.У первородящей 20 лет с тяжелой анемией через 10 минут последового периода, кровопотеря 300,0 мл и продолжается. Наиболее вероятная Ваша тактика? A) введение окситоцинаB) ручное отделение и выделение последа C) введение мизопростола 1000 мг D) наружный массаж маткиE) введение физиологического раствора286.У родильницы 30 лет через 20 минут после рождения последа началось кровотечение из родовых путей. Послед цел. Родовые пути целы. Матка не сокращается. Общая кровопотеря 300,0 мл. Наиболее вероятный диагноз?A) выворот маткиB) разрыв маткиC) трофобластическая болезньD) ДВС-синдром E) атоническое кровотечение287.При головке плода в полости малого таза вагинально определяются:А) головка прикрывает верхнюю треть симфиза и крестца, мыс недостижим, седалищные ости прощупываются. Головка согнута, малый родничок ниже большого, стреловидный шов находится в одном из косых размеров.B) крестовидная впадина свободна, к мысу можно подойти согнутым пальцем, внутренняя поверхность симфиза доступна, стреловидный шов в косом размере.C) внутренняя поверхность крестцовой впадины и 2/3 внутренней поверхности симфиза занята головкой, стреловидный шов косом размере.D) внутренняя поверхность крестовой впадины и симфиза заняты головкой, прощупываются седалищные ости, стреловидный шов в косом размереE) крестцовая впадина и внутренняя поверхность симфиза заняты головкой, седалищные ости не прощупываются, стреловидный шов в прямом размере.288.При головке плода в полости малого таза вагинально определяются:

Page 139: postgraduate.kaznmu.kz€¦ · Web view58 Больная И, 28 лет, доставлена с жалобами на внезапно возникшую боль внизу живота

С.Ж.АСФЕНДИЯРОВАТЫНДАҒЫҚАЗАҚ ҰЛТТЫҚ МЕДИЦИНА

УНИВЕРСИТЕТІ

КАЗАХСКИЙНАЦИОНАЛЬНЫЙМЕДИЦИНСКИЙ УНИВЕРСИТЕТ ИМЕНИ С.Д.АСФЕНДИЯРОВА

КАФЕДРА ИНТЕРНАТУРЫ И РЕЗИДЕНТУРЫ ПО АКУШЕРСТВУ И ГИНЕКОЛОГИИТЕСТОВЫЕ ЗАДАНИЯ ДЛЯ ПРОВЕДЕНИЯ ИТОГОВОЙ ГОСУДАРСТВЕННОЙ АТТЕСТАЦИИ РЕЗИДЕНТОВ

ВЫПУСКА 2012-2013 ГГ. ПО СПЕЦИАЛЬНОСТИ «АКУШЕРСТВО И ГИНЕКОЛОГИЯ»

A) головка прикрывает верхнюю треть симфиза и крестца, мыс недостижим, седалищные ости прощупываются, согнута, малый родничок ниже большого, стреловидный шов находится в одном из косых размеровB) крестцовая впадина свободна, к мысу можно подойти согнутым пальцем, внутренняя поверхность симфиза доступна, стреловидный шов в косом размереC) внутренняя поверхность крестцовой впадины и 2/3 внутренней поверхности симфиза занята головкой, стреловидный шов в косом размере D) внутренняя поверхность крестцовой впадины и симфиза заняты головкой, прощупываются седалищные ости, стреловидный шов в косом размереE) крестцовая впадина и внутренняя поверхность симфиза заняты головкой, седалищные ости не прощупываются, стреловидный шов в прямом размере289.У первородящей 22 лет, через 6 часов от начала схваток произошло излитие околоплодных вод. Что делать?A) Назначить антибиотикиB) Выслушать сердцебиение плодаC) Назначить стимуляцию родовой деятельностиD) Определить, где находится головкаE) Сделать триаду по Николаеву290.Проводная ось таза это:А) линия, соединяющая середину всех косых размеров малого тазаB) линия, соединяющая середину всех прямых размеров классических плоскостей малого таза С) линия, параллельная терминальной линииD) линия, соединяющая середину всех прямых размеров параллельных плоскостей малого тазаE) расстояние от верхневнутреннего края симфиза до крестцового мыса291.Повторнородящая с доношенной беременностью поступила в роддом с указанием на излитие вод 4 часа назад, cхваток нет. В анамнезе 2 года назад операция кесарева сечения при доношенной беременности по поводу узкого таза. Тактика врача:А) начать родовозбуждениеB) создать глюкозо-витаминно-гормональный фонC) экстренно операция кесарева сечения D) ждать начала спонтанных cхватокE) кесарево сечение при 10 часах безводного периода и слабых cхватках292.Какое из условий является основным при наложении акушерских щипцов:А) нахождение головки в полости таза или на тазовом дне B) полное открытие маточного зеваC) соответствие размеров головки плода таза женщиныD) живой плодE) отсутствие плодного пузыря293.У первородящей 23 лет, с доношенной беременностью и предполагаемой массой плода 3200,0, общеравномерносуженный таз І степени. II период 40 минут, продвижения головки нет. Сердцебиение плода ясное, 140 уд/мин. Потуги болезненные, матка имеет форму песочных часов, пальпация нижнего сегмента вне схваток болезненна. Диагноз: А) II период родов. Слабость потугB) II период родов. Угрожающий разрыв матки C) II периол родов. Клинически узкий таз. Начавшийся разрыв маткиD) II период родов. Клинически узкий таз. Угрожающий разрыв матки Е. срочные роды. Клинически узкий таз. Совершившийся разрыв матки

Page 140: postgraduate.kaznmu.kz€¦ · Web view58 Больная И, 28 лет, доставлена с жалобами на внезапно возникшую боль внизу живота

С.Ж.АСФЕНДИЯРОВАТЫНДАҒЫҚАЗАҚ ҰЛТТЫҚ МЕДИЦИНА

УНИВЕРСИТЕТІ

КАЗАХСКИЙНАЦИОНАЛЬНЫЙМЕДИЦИНСКИЙ УНИВЕРСИТЕТ ИМЕНИ С.Д.АСФЕНДИЯРОВА

КАФЕДРА ИНТЕРНАТУРЫ И РЕЗИДЕНТУРЫ ПО АКУШЕРСТВУ И ГИНЕКОЛОГИИТЕСТОВЫЕ ЗАДАНИЯ ДЛЯ ПРОВЕДЕНИЯ ИТОГОВОЙ ГОСУДАРСТВЕННОЙ АТТЕСТАЦИИ РЕЗИДЕНТОВ

ВЫПУСКА 2012-2013 ГГ. ПО СПЕЦИАЛЬНОСТИ «АКУШЕРСТВО И ГИНЕКОЛОГИЯ»

294.Через 7 часов от начала родовой деятельности у повторнородящей излились околоплодные воды и начались потуги. При вагинальном исследовании открытие маточного зева полное, плодного пузыря нет. Головка занимает всю крестовую впадину и лонное сочленение, седалищные ости пальпируются, прощупывается копчик. Стреловидный шов в левом косом размере, малый родничок справа спереди. Определите местоположение головки:

A. над входом в малый тазB. прижата ко входу в малый тазC. в широкой части малого тазаD. в узкой части малого тазаE. на тазовом дне

295.Поперечное положение плода при доношенном сроке беременности является показанием для родоразрешения:

A. путем операции кесарева сечения в экстренном порядке B. путем операции кесарева сечения в плановом порядке C. через естественные родовые пути после поворота плода на ножкуD. через естественные родовые пути после поворота плода на головкуE. путем плодоразрушающей операции

296.При пролонгировании беременности в сроке до 36 недель при излитии околоплодных вод антибактериальную терапию следует начать через:

A. 12 часов от момента излития вод B. 18 часов от момента излития вод C. сразу при поступлении в стационарD. 8 часов от момента излития вод E. 6 часов от момента излития вод

297.Первобеременная Н., 22 лет, поступила в роддом с жалобами на схваткообразные боли внизу живота в течение 4-х часов. Срок беременности 39-40 недель. Объективно схватки через 5 минут до 35 секунд. Вагинально: шейка матки сглажена, открытие маточного зева на 2 см, плодный пузырь цел, предлежит головка, прижата ко входу в малый таз. Наиболее вероятный диагноз:

A. Беременность 39-40 недель. Прелиминарный период.B. Беременность 39-40 недель. Патологический прелиминарный период.C. Беременность 39-40 недель. Ложные схватки.D. Беременность 39-40 недель. Первый период родов, латентная фаза.E. Беременность 39-40 недель. Первый период родов, активная фаза.

298.В партограмме в графе «количество схваток за 10 минут» закрашены 2 клетки косыми штрихами. Это означает, что в данный момент у роженицы родовая деятельность протекает со следующей частотой и характером схваток:

A. 2 схватки слабой силыB. 2 схватки средней силыC. 2 схватки хорошей силы D. Нерегулярные, слабые схватки E. Прелиминарные схватки

299.В партограмме в графе «количество схваток за 10 минут» закрашены 4 клетки косыми штрихами. Это означает, что в данный момент у роженицы родовая деятельность протекает со следующей частотой и характером схваток:

A. 4 схватки слабой силыB. 4 схватки средней силыC. 4 схватки хорошей силы D. Регулярные, слабые схватки

Page 141: postgraduate.kaznmu.kz€¦ · Web view58 Больная И, 28 лет, доставлена с жалобами на внезапно возникшую боль внизу живота

С.Ж.АСФЕНДИЯРОВАТЫНДАҒЫҚАЗАҚ ҰЛТТЫҚ МЕДИЦИНА

УНИВЕРСИТЕТІ

КАЗАХСКИЙНАЦИОНАЛЬНЫЙМЕДИЦИНСКИЙ УНИВЕРСИТЕТ ИМЕНИ С.Д.АСФЕНДИЯРОВА

КАФЕДРА ИНТЕРНАТУРЫ И РЕЗИДЕНТУРЫ ПО АКУШЕРСТВУ И ГИНЕКОЛОГИИТЕСТОВЫЕ ЗАДАНИЯ ДЛЯ ПРОВЕДЕНИЯ ИТОГОВОЙ ГОСУДАРСТВЕННОЙ АТТЕСТАЦИИ РЕЗИДЕНТОВ

ВЫПУСКА 2012-2013 ГГ. ПО СПЕЦИАЛЬНОСТИ «АКУШЕРСТВО И ГИНЕКОЛОГИЯ»

E. Прелиминарные схватки 300.В партограмме в графе «количество схваток за 10 минут» произведено сплошное закрашивание 5-ти клеток. Это означает, что в данный момент у роженицы родовая деятельность протекает со следующей частотой и характером схваток:

A. 5 схваток слабой силыB. 5 схваток средней силыC. 5 схваток хорошей силыD. Регулярные потугиE. Нерегулярные потуги

301.В отделении патологии беременности поступила беременная К., 29 лет, с диагнозом «Беременность 36-37 недель. Преэклампсия легкой степени». В последние сутки самочувствие пациентки ухудшилось. К гипертензии, отекам, протеинурии присоединились головная боль, ощущение тяжести в области лба и затылка, появилось расстройство зрения в виде мелькания мушек перед глазами. Сердцебиение плода ясное, ритмичное, до 140 уд/минуту. Ваш НАИБОЛЕЕ вероятный диагноз?

A. преэклампсия легкой степениB. преэклампсия тяжелой степениC. гипертонический кризD. НЦД по гипертоническому типуE. новообразование головного мозга

302.В отделении патологии беременности поступила беременная К.,29 лет, с диагнозом «Беременность 36-37 недель. Преэклампсия легкой степени». В последние сутки самочувствие пациентки ухудшилось. К гипертензии, отекам, протеинурии присоединились головная боль, ощущение тяжести в области лба и затылка, появилось расстройство зрения в виде мелькания мушек перед глазами. Сердцебиение плода ясное, ритмичное, до 140 уд/минуту. Ваша НАИБОЛЕЕ вероятная тактика?

A. наблюдение в динамикеB. перевод в палату интенсивной терапииC. перевод в нейрохирургическое отделениеD. амниотомия с последующим родовозбуждениемE. абдоминальное кесарево сечение

303.Срок беременности 32-33 недели. Жалобы на головную боль, отеки. Объективно: нижние конечности, передняя брюшная стенка, лицо отечны. А/Д 190/120 мм рт. ст. (исходное АД 110/70 мм рт. ст.). При исследовании глазного дна обнаружены дистрофические изменения. Дефицит выделяемой жидкости 30%. Белок мочи 0,99%. В течение, какого времени НАИБОЛЕЕ вероятно консервативное лечение?

A. до 6 часовB. 6-12 часовC. 24-48 часовD. до 5 сутокE. до 10 суток

304.Беременность 32 недель. Головка над входом в малый таз. Мажущие кровянистые выделения из половых путей. Кожные покровы розовые. Пульс 72 уд/минуту. АД 110/70 мм рт. ст. Матка не напряжена, безболезненная. Влагалище и шейка матки без изменений, незначительно выраженная пастозность в заднем своде. НАИБОЛЕЕ вероятная тактика?

A. кесарево сечениеB. амниотомияC. сохраняющая терапия

Page 142: postgraduate.kaznmu.kz€¦ · Web view58 Больная И, 28 лет, доставлена с жалобами на внезапно возникшую боль внизу живота

С.Ж.АСФЕНДИЯРОВАТЫНДАҒЫҚАЗАҚ ҰЛТТЫҚ МЕДИЦИНА

УНИВЕРСИТЕТІ

КАЗАХСКИЙНАЦИОНАЛЬНЫЙМЕДИЦИНСКИЙ УНИВЕРСИТЕТ ИМЕНИ С.Д.АСФЕНДИЯРОВА

КАФЕДРА ИНТЕРНАТУРЫ И РЕЗИДЕНТУРЫ ПО АКУШЕРСТВУ И ГИНЕКОЛОГИИТЕСТОВЫЕ ЗАДАНИЯ ДЛЯ ПРОВЕДЕНИЯ ИТОГОВОЙ ГОСУДАРСТВЕННОЙ АТТЕСТАЦИИ РЕЗИДЕНТОВ

ВЫПУСКА 2012-2013 ГГ. ПО СПЕЦИАЛЬНОСТИ «АКУШЕРСТВО И ГИНЕКОЛОГИЯ»

D. стимуляция окситоциномE. наблюдение в динамике

305.При беременности 38 недель началось кровотечение. НАИБОЛЕЕ вероятные действия врача?A. осмотр в развернутой операционнойB. кесарево сечениеC. гемостатическая терапияD. амниотомияE. стимуляция окситоцином

306.Беременность 32 недели. Неделю назад в течение 3 дней были кровяные выделения без видимой причины. Сейчас кровяные выделения в умеренном количестве. Головка высоко над входом в малый таз. Схваток нет. НАИБОЛЕЕ вероятная тактика?

A. кесарево сечениеB. амниотомияC. консервативные методы леченияD. родовозбуждение окситоциномE. акушерские щипцы

307.Беременность 32 недели. Неделю назад в течение 3 дней были кровянистые выделения без видимых причин. Сейчас выделения в умеренном количестве. Головка высоко над входом в малый таз. Схваток нет. НАИБОЛЕЕ вероятный диагноз?

A. отслойка нормально расположенной плацентыB. полное предлежание плацентыC. неполное предлежание плацентыD. угрожающий разрыв маткиE. начавшийся разрыв матки

308.Беременная М.,27 лет, срок беременности 36 недель доставлена в родильный дом бригадой "скорой помощи" с жалобами на головную боль, мелькание мошек перед глазами, тошноту. В приемном покое появились мелкие фибриллярные подергивания мышц лица, затем тонические сокращения всей скелетной мускулатуры, остановка дыхания, потеря сознания. Через 20-25 секунд возникли клонические судороги, изо рта выделилась пена. НАИБОЛЕЕ вероятные первые действия врача по окончании припадка?

A. транспортировка в палату интенсивной терапииB. вдыхание кислородно-воздушной смеси 40-60% по 10 минутC. наркозD. транспортировка в операционнуюE. магнезиальная терапия

309.В отделении патологии беременности находится на лечении женщина К.,29 лет, с диагнозом «Беременность 36-37 недель. Преэклампсия легкой степени». В последние сутки самочувствие больной ухудшилось, присоединились головная боль, ощущение тяжести в области лба и затылка, появилось расстройство зрения в виде мелькания мушек перед глазами. Беременная возбуждена. Сердцебиение плода ясное, ритмичное, до 140 ударов/минуты. Околоплодные воды не отходили. НАИБОЛЕЕ вероятный диагноз?

A. отеки беременныхB. преэклампсия легкой степениC. преэклампсия тяжелой степениD. эклампсияE. гипертонический криз

Page 143: postgraduate.kaznmu.kz€¦ · Web view58 Больная И, 28 лет, доставлена с жалобами на внезапно возникшую боль внизу живота

С.Ж.АСФЕНДИЯРОВАТЫНДАҒЫҚАЗАҚ ҰЛТТЫҚ МЕДИЦИНА

УНИВЕРСИТЕТІ

КАЗАХСКИЙНАЦИОНАЛЬНЫЙМЕДИЦИНСКИЙ УНИВЕРСИТЕТ ИМЕНИ С.Д.АСФЕНДИЯРОВА

КАФЕДРА ИНТЕРНАТУРЫ И РЕЗИДЕНТУРЫ ПО АКУШЕРСТВУ И ГИНЕКОЛОГИИТЕСТОВЫЕ ЗАДАНИЯ ДЛЯ ПРОВЕДЕНИЯ ИТОГОВОЙ ГОСУДАРСТВЕННОЙ АТТЕСТАЦИИ РЕЗИДЕНТОВ

ВЫПУСКА 2012-2013 ГГ. ПО СПЕЦИАЛЬНОСТИ «АКУШЕРСТВО И ГИНЕКОЛОГИЯ»

310.У беременной в сроке 32 недели беременности появились признаки прогрессирующей преждевременной отслойки плаценты. Произведена срочная госпитализация в роддом. НАИБОЛЕЕ вероятная акушерская тактика:

A. интенсивная терапияB. родовозбуждение C. кесарево сечениеD. амниотомияE. динамическое наблюдение

311.Повторнобеременная при сроке 36 недель поступила в дородовое отделение с незначительными яркими кровянистыми выделениями из половых путей. Тонус матки повышен. Сердцебиение плода ясное. НАИБОЛЕЕ вероятный диагноз?

A. преждевременная отслойка нормально расположенной плацентыB. неполное предлежание плацентыC. полное предлежание плацентыD. разрыв маткиE. начавшиеся преждевременные роды

312.В женскую консультацию обратилась первобеременная с жалобами на слабое шевеление плода. Срок беременности 35-36 недель, дно матки между пупком и мечевидным отростком, сердцебиение плода глухое, ритмичное. На ногах отеки, прибавка в весе 10 кг. Какой метод исследования НАИБОЛЕЕ информативен в данной ситуации?

A. гормональный метод исследования (определение эстриола)B. кардиотахография с использованием функциональных пробC. метод наружного акушерского исследованияD. УЗ-метод исследованияE. анализ мочи на наличия белка

313.Первородящая женщина 30 лет, поступила в отделение патологии беременности с жалобами на головную боль, боли в подложечной области, нарушение сна. АД 140/80 мм рт. ст., в моче белок, голени пастозны. Срок беременности – 37 недель. Предлежание головное. Сердцебиение плода ясное, до 140 ударов/минуту. Ваш НАИБОЛЕЕ вероятный диагноз:

A. отеки беременныхB. преэклампсия легкой степениC. преэклампсия тяжелой степениD. эклампсияE. НЦД по гипертоническому типу

314.Первородящая 23 лет доставлена в роддом после припадка эклампсии, который произошел дома. Беременность 37-38 недель, состояние тяжелое, сознание заторможенное. АД 150/100 мм рт. ст., пульс 98 уд/минуту. Выраженные отеки на ногах. Определите наиболее вероятную тактику ведения беременной:

A. пролонгирование беременности на фоне леченияB. проведение комплексной интенсивной терапии в течение 2-3 днейC. кесарево сечение на фоне комплексной интенсивной терапииD. родовозбуждение с последующим наложением акушерских щипцовE. подготовка родовых путей с целью индукции родов

315.Беременная 28 лет поступила в роддом с кровянистыми выделениями. Беременность третья, 2 родов. В 10 недель перенесла грипп с высокой температурой, в 12 недель лечилась в стационаре с диагнозом начавшийся выкидыш. Последняя менструация 4 месяца назад. При объективном исследовании: АД 120/65 мм рт. ст., пульс 72 уд/минуту. Дно матки на уровне пупка. Матка неравномерной консистенции, безболезненная. Части плода, движения, сердцебиение не

Page 144: postgraduate.kaznmu.kz€¦ · Web view58 Больная И, 28 лет, доставлена с жалобами на внезапно возникшую боль внизу живота

С.Ж.АСФЕНДИЯРОВАТЫНДАҒЫҚАЗАҚ ҰЛТТЫҚ МЕДИЦИНА

УНИВЕРСИТЕТІ

КАЗАХСКИЙНАЦИОНАЛЬНЫЙМЕДИЦИНСКИЙ УНИВЕРСИТЕТ ИМЕНИ С.Д.АСФЕНДИЯРОВА

КАФЕДРА ИНТЕРНАТУРЫ И РЕЗИДЕНТУРЫ ПО АКУШЕРСТВУ И ГИНЕКОЛОГИИТЕСТОВЫЕ ЗАДАНИЯ ДЛЯ ПРОВЕДЕНИЯ ИТОГОВОЙ ГОСУДАРСТВЕННОЙ АТТЕСТАЦИИ РЕЗИДЕНТОВ

ВЫПУСКА 2012-2013 ГГ. ПО СПЕЦИАЛЬНОСТИ «АКУШЕРСТВО И ГИНЕКОЛОГИЯ»

определяются. PV: шейка матки укорочена, цервикальный канал пропускает палец, определяется мягкая ткань, расположенная над внутренним зевом. Части плода не определяются. Ваш наиболее вероятный диагноз:

A. начавшийся поздний выкидышB. предлежание плацентыC. миома маткиD. отслойка нормально расположенной плацентыE. пузырный занос

316.Беременная 28 лет. Срок беременности 38 недель. В анамнезе 1,5 года назад кесарево сечение. При поступлении жалобы на постоянные боли внизу живота, которые усиливаются при шевелении плода. Матка в нормотонусе. Предлежит головка. Сердцебиение плода ясное, 136 уд/минуту. При пальпации области послеоперационного рубца боли усиливаются. PV: шейка мягкая, длина 1,5 см, цервикальный канал пропускает 1 поперечный палец за внутренний зев, в области которого определяется «переходной валик». Предлежит головка. Мыс не достижим. Выделения слизистые. Наиболее вероятный диагноз:

A. Рубец на матке. Предвестники родовB. Рубец на матке. Преждевременная отслойка нормально расположенной плацентыC. Рубец на матке. I период родовD. Угрожающий разрыв матки по рубцуE. Начавшийся разрыв матки по рубцу

317.Беременная 28 лет. Беременность четвертая, в анамнезе три искусственных аборта. В сроке 28 недель в покое появились скудные кровянистые выделения из половых путей. Кровопотеря около 30,0 мл. Положение плода поперечное. Матка в нормотонусе. При влагалищном исследовании обнаружено краевое предлежание плаценты. Наиболее вероятная тактика?

A. произвести операцию кесарево сечениеB. произвести амниотомиюC. вызвать родовую деятельностьD. назначить спазмолитики и пролонгировать беременностьE. наблюдение в динамике под контролем анализов крови

318.Первобеременная 19 лет находится в родильном доме с диагнозом «Беременность 35 недель. Преэклампсия тяжелой степени». Проводимая комплексная терапия в течение 48 часов с положительным эффектом. Ваша наиболее вероятная тактика:

A. продолжить комплексную терапию и пролонгировать беременностьB. произвести операцию кесарево сечениеC. подготовка для досрочного родоразрешения через естественные родовые путиD. выписать под строгим контролем женской консультацииE. отменить комплексную терапию и пролонгировать беременность

319.Повторнобеременная, со сроком беременности 32 недели, поступила в отделение патологии беременности, поперечное положение плода, жалобы на тянущие боли внизу живота. Матка возбудима. Сердцебиение плода ясное, ритмичное, до 140 ударов/минуту. PV: шейка матки слегка укорочена, цервикальный канал пропускает кончик пальца, предлежащая часть не определяется. Наиболее вероятная акушерская тактика:

A. кесарево сечениеB. наружный поворот плодаC. сохранение беременностиD. родовозбуждение E. амниотомия

Page 145: postgraduate.kaznmu.kz€¦ · Web view58 Больная И, 28 лет, доставлена с жалобами на внезапно возникшую боль внизу живота

С.Ж.АСФЕНДИЯРОВАТЫНДАҒЫҚАЗАҚ ҰЛТТЫҚ МЕДИЦИНА

УНИВЕРСИТЕТІ

КАЗАХСКИЙНАЦИОНАЛЬНЫЙМЕДИЦИНСКИЙ УНИВЕРСИТЕТ ИМЕНИ С.Д.АСФЕНДИЯРОВА

КАФЕДРА ИНТЕРНАТУРЫ И РЕЗИДЕНТУРЫ ПО АКУШЕРСТВУ И ГИНЕКОЛОГИИТЕСТОВЫЕ ЗАДАНИЯ ДЛЯ ПРОВЕДЕНИЯ ИТОГОВОЙ ГОСУДАРСТВЕННОЙ АТТЕСТАЦИИ РЕЗИДЕНТОВ

ВЫПУСКА 2012-2013 ГГ. ПО СПЕЦИАЛЬНОСТИ «АКУШЕРСТВО И ГИНЕКОЛОГИЯ»

320.У повторнобеременной, первородящей женщины 26 лет произошел полный разрыв матки в нижнем сегменте. Плод погиб, признаков инфекции нет. Наиболее вероятно пациентке показано:

A. лапаротомия, ушивание разрыва матки в нижнем сегментеB. лапаротомия, надвлагалищная ампутация матки без придатковC. лапаротомия, экстирпация матки без придатковD. лапаротомия, надвлагалищная ампутация матки с трубамиE. лапаротомия, экстирпация матки с трубами

321.При наружном акушерском обследовании беременной в дне матки пальпируется крупная часть мягкой консистенции, у входа в малый таз - крупная часть плотной консистенции, шаровидной формы, слева и кпереди - ровная без выступов поверхность, справа и кзади - бугристая поверхность. Наиболее вероятный диагноз?

A. продольное положение, тазовое предлежание, 1-я позиция, передний видB. продольное положение, головное предлежание, 1-я позиция, задний видC. продольное положение, головное предлежание, 1-я позиция, передний видD. продольное положение, головное предлежание, 2-я позиция, передний видE. продольное положение, головное предлежание, 2-я позиция, задний вид

322.У повторнородящей 36 лет, в доношенном сроке, появились кровянистые выделения из половых путей. Матка в тонусе, не расслабляется. Сердцебиение плода до 80 ударов в минуту. PV: шейка матки «зрелая», плодный пузырь цел, предлежит головка, прижата к входу в малый таз. Наиболее вероятная тактика врача:А) Родоразрешение через естественные родовые путиВ) Произвести вскрытие плодного пузыряС) Абдоминальное родоразрешение (кесарево сечение)D) Наложить акушерские щипцы (полостные)Е) Применить утеротонические средства (окситоцин)323.У повторнородящей 36 лет, в доношенном сроке, появились кровянистые выделения из половых путей. Матка в тонусе, не расслабляется. Сердцебиение плода до 80 ударов в минуту. PV: шейка матки «зрелая», плодный пузырь цел, предлежит головка, прижата к входу в малый таз. Наиболее вероятный диагноз:А) Предлежание плацентыВ) Низкая плацентацияС) Разрыв шейки маткиD) Разрыв маткиЕ) Отслойка плаценты324.Наиболее характерное осложнение родовой деятельности у беременных с гипотериозом:А) Дискоординированная родовая деятельностьВ) Быстрые родыС) Стремительные родыD) Слабость родовой деятельностиЕ) Несвоевременное излитие околоплодных вод325.У беременной в 34 недель появились боли в области лонного сочленения, поясничной области, "утиная" походка. Наиболее вероятный диагноз:А) Угроза преждевременных родовВ) Флебит нижних конечностейС) Радикулит поясничного отделаD) Воспаление лонного сочленения (симфизит)E) Размягчение лонного сочленения (симзиопатия)

Page 146: postgraduate.kaznmu.kz€¦ · Web view58 Больная И, 28 лет, доставлена с жалобами на внезапно возникшую боль внизу живота

С.Ж.АСФЕНДИЯРОВАТЫНДАҒЫҚАЗАҚ ҰЛТТЫҚ МЕДИЦИНА

УНИВЕРСИТЕТІ

КАЗАХСКИЙНАЦИОНАЛЬНЫЙМЕДИЦИНСКИЙ УНИВЕРСИТЕТ ИМЕНИ С.Д.АСФЕНДИЯРОВА

КАФЕДРА ИНТЕРНАТУРЫ И РЕЗИДЕНТУРЫ ПО АКУШЕРСТВУ И ГИНЕКОЛОГИИТЕСТОВЫЕ ЗАДАНИЯ ДЛЯ ПРОВЕДЕНИЯ ИТОГОВОЙ ГОСУДАРСТВЕННОЙ АТТЕСТАЦИИ РЕЗИДЕНТОВ

ВЫПУСКА 2012-2013 ГГ. ПО СПЕЦИАЛЬНОСТИ «АКУШЕРСТВО И ГИНЕКОЛОГИЯ»

326.В родильный дом машиной «скорой помощи» доставлена беременная 25 лет. Жалобы на жажду, плохой сон, головную боль, тошноту. 2 недели назад при очередном посещении женской консультации была предложена госпитализация в связи с повышением АД. Беременная от госпитализации отказалась. Объективно: рост 163 см, вес 78 кг. Кожа чистая, бледная, отмечается одутловатость лица. АД 160/110, 175/120 мм рт. ст, пульс 96 уд/ минуту. Матка соответствует сроку 33-34 недели беременности (гестационный срок 36-37 недель). Положение плода продольное, предлежит головка, над входом в таз. Сердцебиение плода ритмичное, приглушено, 144 — 150 ударов/минуту. На передней брюшной стенке и голенях умеренные отеки. В моче обнаружен белок 1,65 г/л. Наиболее вероятный диагноз?

A. преэклампсия легкой степениB. преэклампсия тяжелой степениC. хроническая артериальная гипертензияD. гестационная артериальная гипертензия E. эклампсия

327. Децидуальная оболочка-это:А)трансформированный в связи с беременностью функциональный слой эндометрия. В) трансформированный в связи с беременностью миометрийС) слой эндометрия, отпадающий во время менструацииD) трансформированные в связи с беременностью эндо- и миометрийЕ) трансформированные в связи с беременностью все слои матки328. Определение уровня б-фетопротеина в крови беременных проводится для: A) диагностики токсоплазмоза B) выявления …врожденных и наследственных заболеваний плода …C) определения степени тяжести гестозаD) диагностики беременностиE) определения срока беременности329.Диагональная коньюгат - это расстояние:А) от середины верхнего края симфиза до крестцового мысаВ) от середины верхнего края симфиза до верхнего угла ромба МихаэлисаС) от крестцового мыса до нижнего края симфиза D) между седалищными буграмиЕ) между остями седалищных костей330.На приеме у участкового врача беременная предъявляет жалобы на головную боль и мелькание мушек перед глазами. Артериальное давление 170/80мм.рт.ст. Что необходимо сделать в первую очередь?А) ввести спазмолитикиВ) необходима консультация окулистаС) произвести влагалищное исследованиеD) назначить повторную явку и отпустить домойЕ) в/в вести сульфат магния и доставить беременную на машине скорой помощи в род. дом 331.Наименее вероятные клинические проявления HЕLLР-синдрома:А) желтухаВ) боли в правом подреберьеС) рвота с кровьюD) анурия Е)кровоизлияния в местах инъекций332.Диурез при тяжелой преэклампсии зависит, главным образом от:А) канальцевой реабсорбции

Page 147: postgraduate.kaznmu.kz€¦ · Web view58 Больная И, 28 лет, доставлена с жалобами на внезапно возникшую боль внизу живота

С.Ж.АСФЕНДИЯРОВАТЫНДАҒЫҚАЗАҚ ҰЛТТЫҚ МЕДИЦИНА

УНИВЕРСИТЕТІ

КАЗАХСКИЙНАЦИОНАЛЬНЫЙМЕДИЦИНСКИЙ УНИВЕРСИТЕТ ИМЕНИ С.Д.АСФЕНДИЯРОВА

КАФЕДРА ИНТЕРНАТУРЫ И РЕЗИДЕНТУРЫ ПО АКУШЕРСТВУ И ГИНЕКОЛОГИИТЕСТОВЫЕ ЗАДАНИЯ ДЛЯ ПРОВЕДЕНИЯ ИТОГОВОЙ ГОСУДАРСТВЕННОЙ АТТЕСТАЦИИ РЕЗИДЕНТОВ

ВЫПУСКА 2012-2013 ГГ. ПО СПЕЦИАЛЬНОСТИ «АКУШЕРСТВО И ГИНЕКОЛОГИЯ»

B) изменений почечной паренхимыC) периферического сопротивления сосудовD) сужения сосудов почекE) клубочковой фильтрации 333.Наименее вероятные показания к искусственной вентиляции легких при преэклампсии:A) отеки беременных B) эклампсический статус C) острая дыхательная недостаточность D) печеночно-почечная недостаточность E) после операции кесарева сечения с тяжелой преэклампсией334.На скорой помощи доставлена первородящая 22 лет после приступа эклампсии дома с доношенной беременностью. Акушерский статус: матка увеличена соответственно доношенному сроку беременности, с четкими контурами. Положение плода продольное. Предлежит головка. Сердцебиение плода ясное, ритмичное. PV: открытие маточного зева полное, плодного пузыря нет, головка на тазовом дне. Мыс не достижим. Тактика родоразрешения:А) продолжить период изгнанияB) ввести сокращающие препаратыC) …. наложить выходные акушерские щипцы D) произвести краниотомиюE) родоразрешить путем операции кесарева сечения335.У роженицы с тяжелой преэклампсией при беременности 33 недели началась преждевременная отслойка нормально расположенной плаценты. Сердцебиение плода выслушивается. Действия врача:A) консервативное родоразрешение B) интенсивная терапия и родовозбуждениеC) применение спазмолитиков и токолитиковD) операция кесарева сечения E) профилактика гипоксии плода в/в актовегином336.Первобеременная 34-х лет с беременностью 32 недель находится в отделении реанимации в течение 1 суток с диагнозом: Беременность 32 недель. Преэклампсия тяжелой степени. Эффекта от лечения нет. Не подготовленные родовые пути. Тактика врача:А) продолжить лечение до суток В) проводить созревание шейки матки С) родоразрешить путем операции кесарево сечение D) продолжить магнезиальную терапию до 3-х сутокE) пролонгировать беременность до срока родов в условиях стационара337.Скорой помощью доставлена в роддом первородящая 22 лет, с болями в эпигастральной области, ухудшением зрения, срок беременности 37 недель, АД 160/100 мм. рт. ст., незрелая шейка матки. ОАМ: уд.вес 1020, суточная протеинурия 5 грамм/л. Что делать?А) проводить лечение в течение сутокB) проводить лечение в течение 24-48 часов, если нет эффекта, родоразрешить. C) провести амниотомию с последующим родовозбуждениемD) проводить лечение преэклампсии до полного излеченияE) проводить лечение преэклампсии в течение 12 часов при отсутствии эффекта, родоразрешить338.Во время операции кесарева сечения по поводу полной преждевременной отслойки нормально расположенной плаценты выявлена матка Кювелера, присоединился ДВС-синдром. Что делать: А) ушить разрез на матке и проводить борьбу с ДВС-синдромомВ) произвести надвлагалищную ампутацию матки и борьбу с ДВС-синдромом

Page 148: postgraduate.kaznmu.kz€¦ · Web view58 Больная И, 28 лет, доставлена с жалобами на внезапно возникшую боль внизу живота

С.Ж.АСФЕНДИЯРОВАТЫНДАҒЫҚАЗАҚ ҰЛТТЫҚ МЕДИЦИНА

УНИВЕРСИТЕТІ

КАЗАХСКИЙНАЦИОНАЛЬНЫЙМЕДИЦИНСКИЙ УНИВЕРСИТЕТ ИМЕНИ С.Д.АСФЕНДИЯРОВА

КАФЕДРА ИНТЕРНАТУРЫ И РЕЗИДЕНТУРЫ ПО АКУШЕРСТВУ И ГИНЕКОЛОГИИТЕСТОВЫЕ ЗАДАНИЯ ДЛЯ ПРОВЕДЕНИЯ ИТОГОВОЙ ГОСУДАРСТВЕННОЙ АТТЕСТАЦИИ РЕЗИДЕНТОВ

ВЫПУСКА 2012-2013 ГГ. ПО СПЕЦИАЛЬНОСТИ «АКУШЕРСТВО И ГИНЕКОЛОГИЯ»

С) произвести перевязку a. iliaca interna, экстирпацию матки и борьбу с ДВС-синдромом D) произвести перевязку a. iliaca interna и борьбу с ДВС-синдромомЕ) прошить магистральные сосуды339.У повторнобеременной с резус-отрицательной кровью и беременностью 37 недель титр антител 1:32. Что делать?А) родоразрешить путем амниотомии с последующим родовозбуждением B) проводить десенсибилизирующую терапиюC) родоразрешить путем операции кесарево сечениеD) произвести пересадку кожного лоскута мужаE) произвести плазмофорез340.Беременность при заболеваниях печени противопоказана или должна быть прервана наименее вероятно при: A) портальном циррозеB) острой жировой дистрофии печениC) желчнокаменной болезни и частых приступах печеночной колики D) печеночной недостаточности, обусловленной гестозомE) холестатическом гепатозе беременных 341.Какова тактика у беременных с туберкулезным менингитом: А) активное лечение и прерывание беременности на этом фоне при любом сроке B) прервание беременности в сроке 35 –36 недель на фоне активного леченияC) пролонгирование беременности, проведение противотуберкулезной терапииD) прервание беременности до 12 недель без лечения E) наблюдение фтизиатра, донашивание беременности 342.При какой форме туберкулеза беременность необходимо прервать:А) очаговый туберкулез легкихB) фиброзно-кавернозный и кавернозный туберкулез легких, туберкулез почек C) диссеминированный туберкулез легких в острой и подострой форме, туберкулезный менингитD) туберкулезное поражение костейE) эксудативный плеврит343.Тактика участкового врача при обращении беременной во II половине беременности с жалобами на кровянистые выделения из половых путей:A) ввести в/венно окситоцин 1,0 мл и срочно госпитализировать в роддомB) осмотреть на зеркалах для исключения травмы половых органовC) произвести влагалищное исследование для исключения начала преждевременных родовD) срочно госпитализировать в роддом, вызвав машину скорой помощиE) при незначительных выделениях назначить свечи с папаверином, контрольная явка через 3 дня344.Формула Ланковца для определения предполагаемой массы внутриутробного плода:A) окружность живота (см) x высота стояния дна матки над лоном (см) / 10B) высота стояния дна матки х полуокружность живота поперечник матки (см) / 10C) окружность живота (см) высота стояния дна матки (см) поперечник матки (см) / 10D) высота стояния дна матки (см)) х поперечник матки (см) длина плода тазомером (см) / 10E) рост беременной (см) масса беременной (кг) окружность живота (см)высота стояния дна матки (см) х 10345.Тактика врача у беременных с хроническим гематогенно-диссеминирующим туберкулезом легких:A) Прервать беременность в сроке 35-36 недель на фоне специфической терапии.B) Прервать беременность в сроке 12 недельC) Сохранить беременность, наблюдение акушера-гинеколога

Page 149: postgraduate.kaznmu.kz€¦ · Web view58 Больная И, 28 лет, доставлена с жалобами на внезапно возникшую боль внизу живота

С.Ж.АСФЕНДИЯРОВАТЫНДАҒЫҚАЗАҚ ҰЛТТЫҚ МЕДИЦИНА

УНИВЕРСИТЕТІ

КАЗАХСКИЙНАЦИОНАЛЬНЫЙМЕДИЦИНСКИЙ УНИВЕРСИТЕТ ИМЕНИ С.Д.АСФЕНДИЯРОВА

КАФЕДРА ИНТЕРНАТУРЫ И РЕЗИДЕНТУРЫ ПО АКУШЕРСТВУ И ГИНЕКОЛОГИИТЕСТОВЫЕ ЗАДАНИЯ ДЛЯ ПРОВЕДЕНИЯ ИТОГОВОЙ ГОСУДАРСТВЕННОЙ АТТЕСТАЦИИ РЕЗИДЕНТОВ

ВЫПУСКА 2012-2013 ГГ. ПО СПЕЦИАЛЬНОСТИ «АКУШЕРСТВО И ГИНЕКОЛОГИЯ»

D) Проводить противотуберкулезное лечение до конца беременности.E) Направить на хирургическое лечение и пролонгировать беременность.346.Показанием к искусственной вентиляции легких при преэклампсии является:A) Эклампсический статусB) Острая дыхательная недостаточностьC) Печеночно-почечная недостаточностьD) Холестатический гепатоз E) Преэклампсия 347.Тактика врача при обращении беременной с головной болью:A) Назначить амбулаторно гипотензивные средства, анальгетики.B) Направить в дневной стационар. C) Вызвать машину скорой помощи и направить в роддом.D) Госпитализировать в терапевтический стационар.E) Срочно направить к невропатологу.348.Скорой помощью доставлена в роддом повторнородящая 32 лет, с болями в эпигастральной области, ухудшение зрения, срок беременности 39-40 недель. Артериальное давление 180/110 мм рт ст. родовые пути закрыты. Что делать?A) Проводить лечение в течение суток, если нет эффекта родоразрешить.B) проводить лечение в течение 1-2 часов, затем амниотомия с последующим родовозбуждением.C) провести лечение в течении 24-48 часов, после чего родоразрешить D) лечить до полного излечения.E) лечить в течение 12 часов, при отсутствии эффекта родоразрешить.349.Повторнобеременная, первородящая 32 лет. Беременность 37 недель. Длительность преэклмапсии с 30 недель. В течение 2 дней проводится комплексная терапия. Положительной динамики показателей нет. Шейка матки незрелая. По данным эхографического исследования обнаружено: ЗВУР плода, умеренное маловодие. По данным допплерографии: критические показатели кровотока в артериях пуповины, снижение кровотока в маточных артериях с обеих сторон с "дикротической выемкой". Какова дальнейшая тактика ведения беременной?

A. досрочное родоразрешение через естественные родовые пути после подготовки организма к родамB. усилить и продолжить начатую терапию, после подготовки организма к родам родоразрешить через естественные родовые пути ближе к сроку родовC. экстренное абдоминальное родоразрешение путем операции кесарева сеченияD. усилить и продолжить начатую терапию в течение 48 часов, родоразрешить в плановом порядке путем кесарева сеченияE. начать ускоренную подготовку шейки матки к родам. Способ родоразрешения избрать в зависимости от состояния беременной.

350.НЕLLР-синдром клинически может проявляться: А) клиникой преждевременной отслойки нормально расположенной плацентыВ) разрывом печени с кровотечением в брюшную полостьC) коагулопатическим кровотечениемD) быстрым формированием печеночно-почечной недостаточностиE) отеком легких

351.Невынашивание беременности — это самопроизвольное прерывание беременностиа) от зачатия до 37 недельb) от 22 недель до 37 недельc) в сроки до 22 недель

Page 150: postgraduate.kaznmu.kz€¦ · Web view58 Больная И, 28 лет, доставлена с жалобами на внезапно возникшую боль внизу живота

С.Ж.АСФЕНДИЯРОВАТЫНДАҒЫҚАЗАҚ ҰЛТТЫҚ МЕДИЦИНА

УНИВЕРСИТЕТІ

КАЗАХСКИЙНАЦИОНАЛЬНЫЙМЕДИЦИНСКИЙ УНИВЕРСИТЕТ ИМЕНИ С.Д.АСФЕНДИЯРОВА

КАФЕДРА ИНТЕРНАТУРЫ И РЕЗИДЕНТУРЫ ПО АКУШЕРСТВУ И ГИНЕКОЛОГИИТЕСТОВЫЕ ЗАДАНИЯ ДЛЯ ПРОВЕДЕНИЯ ИТОГОВОЙ ГОСУДАРСТВЕННОЙ АТТЕСТАЦИИ РЕЗИДЕНТОВ

ВЫПУСКА 2012-2013 ГГ. ПО СПЕЦИАЛЬНОСТИ «АКУШЕРСТВО И ГИНЕКОЛОГИЯ»

d) от зачатия до 28 недельe) в сроки 22-28 недель

352.Невынашивание беременности это: a) Самопроизвольное прерывание беременности до 34 недель; b) Самопроизвольное прерывание беременности с момента зачатия и до 22 недель включительно; c) Самопроизвольное прерывание беременности от момента зачатия и до 37 недель, считая с первого дня последней менструации; d) Самопроизвольное прерывание беременности с момента зачатия и до 37 недель с момента зачатия; e) Прерывание беременности по медицинским показаниям со стороны матери и плода.

353.Наиболее частой причиной прерывания беременности на ранних сроках является: a) Хромосомные аномалии b) Инфекции c) Пороки развития маткиd) Гормональные нарушения e) Иммунные нарушения

354.Большинство хромосомных нарушений при невынашивании беременности составляют: a) Триплодииb) Полиплоидный набор хромосом c) Трисомииd) Транслокацииe) Инверсии 355.Наименее характерная причина невынашивания беременности при экстрагенитальной патологии: a) Предшествующие беременности патологические изменения в организме материb) Осложнения беременности, часто развивающиеся у женщин с соматической патологией c) Первичная плацентарная недостаточностьd) Ятрогенное действие фармакологических средств, используемых в терапии основных заболеванийe) Медицинские показания для преждевременной индукции

356.Для какого эндокринного нарушения НАИМЕНЕЕ характерно невынашивание беременности:А) гиперандрогенияB) гиперпролактинемияC) недостаточность лютеиновой фазыD) гипертиреозE) гипотиреоз

357.У пациентки 25 лет в анамнезе гиперпролактинемия. Метаболизм каких гормонов нарушается?А. прогестеронаБ. АКТГВ. андрогеновГ. Т4Д.Т3

Page 151: postgraduate.kaznmu.kz€¦ · Web view58 Больная И, 28 лет, доставлена с жалобами на внезапно возникшую боль внизу живота

С.Ж.АСФЕНДИЯРОВАТЫНДАҒЫҚАЗАҚ ҰЛТТЫҚ МЕДИЦИНА

УНИВЕРСИТЕТІ

КАЗАХСКИЙНАЦИОНАЛЬНЫЙМЕДИЦИНСКИЙ УНИВЕРСИТЕТ ИМЕНИ С.Д.АСФЕНДИЯРОВА

КАФЕДРА ИНТЕРНАТУРЫ И РЕЗИДЕНТУРЫ ПО АКУШЕРСТВУ И ГИНЕКОЛОГИИТЕСТОВЫЕ ЗАДАНИЯ ДЛЯ ПРОВЕДЕНИЯ ИТОГОВОЙ ГОСУДАРСТВЕННОЙ АТТЕСТАЦИИ РЕЗИДЕНТОВ

ВЫПУСКА 2012-2013 ГГ. ПО СПЕЦИАЛЬНОСТИ «АКУШЕРСТВО И ГИНЕКОЛОГИЯ»

358.Какой инфекционный фактор на современном этапе становится основной причиной невынашивания беременностиА) токсоплазмозB) краснухаC) сифилисD) условно-патогенная флораE) хламидиоз

359.Лидирующий фактор в этиологии спорадического самопроизвольного прерывания беременности в ранние сроки выступаетa). хромосомная патологияb). инфекцияc). пороки развития половых органовd). экстрагенитальная патологияe). эндокринная патология

360.Обобщенное понятие «Синдром потери плода» НЕ включает в себя:А) один или более самопроизвольных выкидышей в сроки до 12 недель беременности B) одну или более неразвивающихся беременностей в анамнезеC) поздние самопроизвольные выкидышиD) мертворождение и неонатальную смертностьE) младенческую смертность

361.Самопроизвольный аборт - это прерывание беременности:А) до 28 недельВ) до 24 недельC) до 22 недельD) до 16 недельE) до 12 недель

362.Частота невынашивания в популяцииА) 10 % от общего числа беременностейB) 20 % от общего числа беременностейC) 30 % от общего числа беременностейD) 40 % от общего числа беременностейE) 50 % от общего числа беременностей

363.Частота самопроизвольного прерывания беременности после инвазивных методов пренатальной диагностики (хориоцентез, амниоцентез, кордоцентез) составляет:А. 10%.Б. 25%.В. 80%.Г. 3–5%.Д. 15%.

364.Частота самопроизвольного прерывания беременности при хромосомной

Page 152: postgraduate.kaznmu.kz€¦ · Web view58 Больная И, 28 лет, доставлена с жалобами на внезапно возникшую боль внизу живота

С.Ж.АСФЕНДИЯРОВАТЫНДАҒЫҚАЗАҚ ҰЛТТЫҚ МЕДИЦИНА

УНИВЕРСИТЕТІ

КАЗАХСКИЙНАЦИОНАЛЬНЫЙМЕДИЦИНСКИЙ УНИВЕРСИТЕТ ИМЕНИ С.Д.АСФЕНДИЯРОВА

КАФЕДРА ИНТЕРНАТУРЫ И РЕЗИДЕНТУРЫ ПО АКУШЕРСТВУ И ГИНЕКОЛОГИИТЕСТОВЫЕ ЗАДАНИЯ ДЛЯ ПРОВЕДЕНИЯ ИТОГОВОЙ ГОСУДАРСТВЕННОЙ АТТЕСТАЦИИ РЕЗИДЕНТОВ

ВЫПУСКА 2012-2013 ГГ. ПО СПЕЦИАЛЬНОСТИ «АКУШЕРСТВО И ГИНЕКОЛОГИЯ»

патологии составляет:А. 80-100%.Б. 60–80%В. 40-60%.Г. 20–40%.Д. 10-20%

365.Первобеременная сообщает, что она выпивает 4–5 чашек кофе в день. Чем может осложниться течение беременности:А. риском ранних выкидышейБ. гипоксией плодаВ. аномалией развития плодаГ. снижением двигательной активностиД. плацентарной недостаточностью

366.Самопроизвольные выкидыши - самопроизвольное прерывание беременности, которое заканчивается рождением…А). незрелого и не жизнеспособного плода массой менее 500гр.B). незрелого, но жизнеспособного плода массой менее 500гр.C). зрелого и жизнеспособного плода массой менее 500гр.D). незрелого и не жизнеспособного плода массой более 500гр.E). рождением незрелого и не жизнеспособного плода массой 500гр.

367.НАИБОЛЕЕ характерная причина ранних самопроизвольных выкидышей аномалия развития матки заболевание почекзаболевание печеникурениеплохое питание

368.Наиболее частая причина невынашивания среди эндокринологических этиологических факторов:А. недостаточность лютеиновой фазыБ. гиперандрогенияВ. гиперпролактинемияГ. гиперинсулинемияД. неполноценность рецепторов эндометрия к половым гормонам

369.При подозрении на инфекционную причину невынашивания беременности вне беременности проводят следующие методы исследований за исключением:А. микроскопии мазков из влагалища и канала шейки маткиБ. бактериологического исследования отделяемого канала шейки матки В. определения антител к фосфолипидамГ. определения IgG и IgM к ВПГ и ЦМВ в крови методом ИФАД. определения активированных NK-клеток и провоспалительных цитокинов

Page 153: postgraduate.kaznmu.kz€¦ · Web view58 Больная И, 28 лет, доставлена с жалобами на внезапно возникшую боль внизу живота

С.Ж.АСФЕНДИЯРОВАТЫНДАҒЫҚАЗАҚ ҰЛТТЫҚ МЕДИЦИНА

УНИВЕРСИТЕТІ

КАЗАХСКИЙНАЦИОНАЛЬНЫЙМЕДИЦИНСКИЙ УНИВЕРСИТЕТ ИМЕНИ С.Д.АСФЕНДИЯРОВА

КАФЕДРА ИНТЕРНАТУРЫ И РЕЗИДЕНТУРЫ ПО АКУШЕРСТВУ И ГИНЕКОЛОГИИТЕСТОВЫЕ ЗАДАНИЯ ДЛЯ ПРОВЕДЕНИЯ ИТОГОВОЙ ГОСУДАРСТВЕННОЙ АТТЕСТАЦИИ РЕЗИДЕНТОВ

ВЫПУСКА 2012-2013 ГГ. ПО СПЕЦИАЛЬНОСТИ «АКУШЕРСТВО И ГИНЕКОЛОГИЯ»

370.Подготовка к беременности пациенток с привычным невынашиванием беременности и ИЦН не предусматривает:А. санацию половых путейБ. нормализацию микрофлоры влагалищаВ. лечение хронического эндометритаГ. лечение супругаД. психотерапевтическую коррекцию психосоматических нарушений

371.НАИМЕНЕЕЕ характерное клиническое проявление андрогенных влиянийА) гирсутизмB) гипертрихозC) акнеD) себореяE) нигроидный акантоз

372.НАИМЕНЕЕЕ характерное клиническое проявление андрогенных влиянийА) андроидное телосложение B) нарушения менструального циклаC) анаболический эффектD) запоздалое закрытие зон роста трубчатых костейE) уменьшение расстояния между уретрой и входом во влагалище

373.Пороки развития матки нередко как причина невынашивания комбинируются с пороками развития:А. мочевыводящих путейБ. нервной системыВ. сердца Г. ЖКТД. дыхательной системы

374.Для верификации анатомических причин невынашивания беременности используют:А. УЗИ органов малого тазаБ. МРТ органов малого тазаВ. Рентген органов малого тазаГ.осмотр на зеркалахД. вагинальное исследование

375.Какое нарушение репродуктивной функции наиболее характерно для пациенток с гипотиреозомА) аменореяB) бесплодиеC) самопроизвольный абортD) неразвивающаяся беременностьE) антенатальная гибель плода

376.При каких видах аномалий развития матки НАИМЕНЕЕ характерно невынашивание беременностиА) неполная внутриматочная перегородка

Page 154: postgraduate.kaznmu.kz€¦ · Web view58 Больная И, 28 лет, доставлена с жалобами на внезапно возникшую боль внизу живота

С.Ж.АСФЕНДИЯРОВАТЫНДАҒЫҚАЗАҚ ҰЛТТЫҚ МЕДИЦИНА

УНИВЕРСИТЕТІ

КАЗАХСКИЙНАЦИОНАЛЬНЫЙМЕДИЦИНСКИЙ УНИВЕРСИТЕТ ИМЕНИ С.Д.АСФЕНДИЯРОВА

КАФЕДРА ИНТЕРНАТУРЫ И РЕЗИДЕНТУРЫ ПО АКУШЕРСТВУ И ГИНЕКОЛОГИИТЕСТОВЫЕ ЗАДАНИЯ ДЛЯ ПРОВЕДЕНИЯ ИТОГОВОЙ ГОСУДАРСТВЕННОЙ АТТЕСТАЦИИ РЕЗИДЕНТОВ

ВЫПУСКА 2012-2013 ГГ. ПО СПЕЦИАЛЬНОСТИ «АКУШЕРСТВО И ГИНЕКОЛОГИЯ»

B) седловидная маткаC) двурогая маткаD) однорогая маткаE) рудиментарная матка

377.НАИМЕНЕЕ оправданное мероприятие по коррекции биоценоза после лечения бактериального вагиноза А) ликвидация запоровB) диета (квашеная капуста, йогурт, биокефир)C) эубиотики местноD) применение гестагеновE) фитоэстрогены

Клиника самопроизвольного аборта378.Симптомокомплекс: схваткообразные боли внизу живота, умеренные кровянистые выделения из половых путей, матка соответствует сроку гестации, укорочение шейки матки, характерен для:А) несостоявшегося абортаВ) неполного абортаC) аборта в ходуD) начавшегося абортаE) угрожающего аборта

379.Симптомокомплекс: схваткообразные боли в внизу живота, обильные кровянистые выделения из половых путей, в цервикальном канале определяется плодное яйцо, - характерен для:А) несостоявшегося абортаВ) неполного абортаC) аборта в ходуD) начавшегося абортаE) угрожающего аборта

380.Симптомокомплекс: схваткообразные боли в внизу живота, шейка матки сохранена, зев закрыт, характерен для:А) несостоявшегося абортаВ) неполного абортаC) аборта в ходуD) начавшегося абортаE) угрожающего аборта

381.Беременная С. 30 лет поступила в стационар с жалобами на кровянистые выделения из половых путей и тянущие боли внизу живота. В анамнезе 2 самопроизвольных выкидыша в сроке 7-8 недель беременности. Срок беременности 6-7 нед. Состоит на учете по поводу фибромиомы матки. Состоит на учете у терапевта по поводу эндемического зоба и НЦД по смешанному типу. Ваш диагноз:А) Угрожающий привычный аборт в сроке 6-7 недель. ОАА. Миома матки. Эндемический зоб. НЦД по смешанному типу

Page 155: postgraduate.kaznmu.kz€¦ · Web view58 Больная И, 28 лет, доставлена с жалобами на внезапно возникшую боль внизу живота

С.Ж.АСФЕНДИЯРОВАТЫНДАҒЫҚАЗАҚ ҰЛТТЫҚ МЕДИЦИНА

УНИВЕРСИТЕТІ

КАЗАХСКИЙНАЦИОНАЛЬНЫЙМЕДИЦИНСКИЙ УНИВЕРСИТЕТ ИМЕНИ С.Д.АСФЕНДИЯРОВА

КАФЕДРА ИНТЕРНАТУРЫ И РЕЗИДЕНТУРЫ ПО АКУШЕРСТВУ И ГИНЕКОЛОГИИТЕСТОВЫЕ ЗАДАНИЯ ДЛЯ ПРОВЕДЕНИЯ ИТОГОВОЙ ГОСУДАРСТВЕННОЙ АТТЕСТАЦИИ РЕЗИДЕНТОВ

ВЫПУСКА 2012-2013 ГГ. ПО СПЕЦИАЛЬНОСТИ «АКУШЕРСТВО И ГИНЕКОЛОГИЯ»

В) Начавшийся привычный аборт в сроке 6-7 недель. ОАА. Миома матки. Эндемический зоб. НЦД по смешанному типуC) Привычный аборт в ходу в сроке 6-7 недель. ОАА. Миома матки. Эндемический зоб. НЦД по смешанному типуD) Угрожающий аборт в сроке 6-7 недель. ОАА. Миома матки. Эндемический зоб. НЦД по смешанному типуE) Начавшийся аборт в сроке 6-7 недель. ОАА. Миома матки. Эндемический зоб. НЦД по смешанному типу

382.Беременная С. 30 лет поступила в стационар с жалобами на кровянистые выделения из половых путей и тянущие боли внизу живота. В анамнезе 2 самопроизвольных выкидыша в сроке 7-8 недель беременности. Срок беременности 6-7 нед. Состоит на учете по поводу фибромиомы матки. Состоит на учете у терапевта по поводу эндемического зоба и НЦД по смешанному типу. НАИБОЛЕЕ оправданное лечение?А) Постельный режимВ) Применение дицинонаC) Применение дюфастонаD) Применение спазмолитиковE) Применение электрорелаксации матки

383.В стационар поступила беременная К. с жалобами на сильные схваткообразные боли внизу живота и обильные кровянистые выделения из половых путей. Задержка менструации 2 месяца. В анамнезе 1 самопроизвольный аборт, состоит на учете по поводу миомы матки. При осмотре матка увеличена до 8-9 недель беременности, шейка матки укорочена, в шеечном канале определяется плодное яйцо. Какова ваша тактика:А) сохраняющая терапияВ) удаление плодного яйца и выскабливание полости маткиC) применение спазмолитиков D) применение дюфастона 40 мг, затем по 10мг через 8 часов E) применение кровоостанавливающих препаратов

384.К клинической форме самопроизвольного аборта НЕ относится:А) угрожающий абортВ) начавшийся абортС) аборт в ходуD) несостоявшийся абортЕ) привычный аборт

385.На машине скорой помощи обратилась беременная женщина в сроке 7-8 недель беременности. Жалуется на схваткообразные боли внизу живота. При бимануальном исследовании: внутренний зев закрыт, матка увеличена соответственно сроку беременности, размягчена, иногда слегка болезненна, кровянистых выделений нет. Наиболее вероятный диагноз:А). угрожающий абортB). начавшийся абортC). аборт в ходуD). несостоявшийся абортE). инфицированный аборт

Page 156: postgraduate.kaznmu.kz€¦ · Web view58 Больная И, 28 лет, доставлена с жалобами на внезапно возникшую боль внизу живота

С.Ж.АСФЕНДИЯРОВАТЫНДАҒЫҚАЗАҚ ҰЛТТЫҚ МЕДИЦИНА

УНИВЕРСИТЕТІ

КАЗАХСКИЙНАЦИОНАЛЬНЫЙМЕДИЦИНСКИЙ УНИВЕРСИТЕТ ИМЕНИ С.Д.АСФЕНДИЯРОВА

КАФЕДРА ИНТЕРНАТУРЫ И РЕЗИДЕНТУРЫ ПО АКУШЕРСТВУ И ГИНЕКОЛОГИИТЕСТОВЫЕ ЗАДАНИЯ ДЛЯ ПРОВЕДЕНИЯ ИТОГОВОЙ ГОСУДАРСТВЕННОЙ АТТЕСТАЦИИ РЕЗИДЕНТОВ

ВЫПУСКА 2012-2013 ГГ. ПО СПЕЦИАЛЬНОСТИ «АКУШЕРСТВО И ГИНЕКОЛОГИЯ»

386.При осмотре на зеркалах у беременной женщины в сроке 10-11 недель беременности отмечается кровотечение из канала шейки матки, сглаживание шейки матки; при вагинальном исследовании - раскрытие внутреннего зева; в сводах влагалища обнаруживаются околоплодные воды. Наиболее вероятный диагноз:А). угрожающий абортB). начавшийся абортC). аборт в ходуD). несостоявшийся абортE). инфицированный аборт

387.Повторнобеременная 28 лет, обратилась к врачу в женскую консультацию с жалобами на схваткообразные боли внизу живота, скудные кровянистые выделения из половых путей. В анамнезе 2 самопроизвольных выкидыша. При влагалищном исследовании: шейка матки сохранена, наружный зев пропускает кончик пальца. Матка соответствует 16-17 нед. сроку беременности. Ваш диагноз:А. беременность 16-17 недель. Привычное невынашивание.Б. беременность 16-17 недель. Угрожающий самопроизвольный выкидыш.В. беременность 16-17 недель. Неполный самопроизвольный выкидыш. Привычное невынашивание.Г. беременность 16-17 недель. Начавшийся аборт. Привычное невынашивание.Д. беременность 16-17 недель. Начавшийся аборт

388.Повторнобеременная 28 лет, обратилась к врачу в женскую консультацию с жалобами на схваткообразные боли внизу живота. В анамнезе 2 самопроизвольных выкидыша. При влагалищном исследовании: шейка матки сохранена, наружный зев закрыт. Матка соответствует 16-17 нед. сроку беременности. Ваш диагноз:А. беременность 16-17 недель. Привычное невынашивание.Б. беременность 16-17 недель. Угрожающий самопроизвольный выкидыш. Привычное невынашивание.В. беременность 16-17 недель. Неполный самопроизвольный выкидыш. Привычное невынашивание.Г. беременность 16-17 недель. Начавшийся аборт. Привычное невынашивание.Д. беременность 16-17 недель. Начавшийся аборт

389.Повторнобеременная 28 лет, обратилась к врачу в женскую консультацию с жалобами на схваткообразные боли внизу живота, обильные кровянистые выделения из половых путей. В анамнезе 2 самопроизвольных выкидыша. При влагалищном исследовании: шейка матки укорочена, цервикальный канал пропускает 1 поперечный палец. Определяется нижний полюс плодного яйца. Матка не соответствует 16-17 нед. сроку беременности. Ваш диагноз:А. беременность 16-17 недель. Аборт входу. Привычное невынашивание.Б. беременность 16-17 недель. Угрожающий самопроизвольный выкидыш.В.беременность 16-17 недель. Неполный самопроизвольный выкидыш. Привычное невынашивание.Г. беременность 16-17 недель. Начавшийся аборт. Привычное невынашивание.Д. беременность 16-17 недель. Начавшийся аборт

Page 157: postgraduate.kaznmu.kz€¦ · Web view58 Больная И, 28 лет, доставлена с жалобами на внезапно возникшую боль внизу живота

С.Ж.АСФЕНДИЯРОВАТЫНДАҒЫҚАЗАҚ ҰЛТТЫҚ МЕДИЦИНА

УНИВЕРСИТЕТІ

КАЗАХСКИЙНАЦИОНАЛЬНЫЙМЕДИЦИНСКИЙ УНИВЕРСИТЕТ ИМЕНИ С.Д.АСФЕНДИЯРОВА

КАФЕДРА ИНТЕРНАТУРЫ И РЕЗИДЕНТУРЫ ПО АКУШЕРСТВУ И ГИНЕКОЛОГИИТЕСТОВЫЕ ЗАДАНИЯ ДЛЯ ПРОВЕДЕНИЯ ИТОГОВОЙ ГОСУДАРСТВЕННОЙ АТТЕСТАЦИИ РЕЗИДЕНТОВ

ВЫПУСКА 2012-2013 ГГ. ПО СПЕЦИАЛЬНОСТИ «АКУШЕРСТВО И ГИНЕКОЛОГИЯ»

390.Повторнобеременная 28 лет, со сроком беременности 16-17 недель, обратилась в гинекологическое отделение с жалобами на схваткообразные боли внизу живота, обильные кровянистые выделения из половых путей. В анамнезе 2 самопроизвольных выкидыша. При влагалищном исследовании: шейка матки укорочена, цервикальный канал свободно пропускает 1 поперечный палец. Матка не соответствует сроку беременности. Ваш диагноз:А. беременность 16-17 недель. Привычное невынашивание.Б. беременность 16-17 недель. Угрожающий самопроизвольный выкидыш.В. беременность 16-17 недель. Неполный самопроизвольный выкидыш. Привычное невынашивание.Г. беременность 16-17 недель. Начавшийся аборт. Привычное невынашивание.Д.беременность 16-17 недель. Начавшийся аборт

391.Беременная 27 лет обратилась к врачу в женскую консультацию с жалобами на ноющие боли внизу живота. В анамнезе 2 самопроизвольных выкидыша в раннем сроке беременности. Данная беременность – 3. На зеркалах – слизистая влагалища и шейки матки чистая, выделения бели. При бимануальном исследовании: шейка матки сохранена, наружный зев закрыт. Матка увеличена до 13-14 недель беременности. Ваш диагноз:А.беременность 13-14 недель. Угрожающий выкидыш. Привычное невынашиваниеБ. беременность 13-14 недель. Начавшийся выкидыш. Привычное невынашиваниеВ. беременность 13-14 недель. Угрожающий выкидыш Г. беременность 13-14 недель. Начавшийся выкидыш Д. беременность 13-14 недель. Привычное невынашивание

392.Беременная 27 лет обратилась к врачу в женскую консультацию с жалобами на ноющие боли внизу живота. В анамнезе 2 самопроизвольных выкидыша в раннем сроке беременности. Данная беременность – 3. На зеркалах – слизистая влагалища и шейки матки чистая, выделения бели. При бимануальном исследовании: шейка матки сохранена, наружный зев закрыт. Матка увеличена до 13-14 недель беременности. Ваша тактика:А. госпитализация в стационарБ. лечение в дневном стационареВ.наблюдениеГ. спазмолитикиД. токолитики

393.Беременная 27 лет обратилась к врачу в женскую консультацию с жалобами на ноющие боли внизу живота, скудные кровянистые выделения. В анамнезе 2 самопроизвольных выкидыша в раннем сроке беременности. Данная беременность – 3. На зеркалах – слизистая влагалища и шейки матки чистая, выделения бели. При бимануальном исследовании: шейка матки сохранена, наружный зев пропускает кончик пальца. Матка увеличена до 13-14 недель беременности. Ваша тактика:А. госпитализация в стационарБ. лечение в дневном стационареВ.наблюдениеГ. спазмолитикиД. токолитики

394.Беременная, состоящая на учете при сроке беременности 11-12 недель, обратилась с -жалобами на незначительные кровянистые выделения из половых путей. Ваша тактика?

Page 158: postgraduate.kaznmu.kz€¦ · Web view58 Больная И, 28 лет, доставлена с жалобами на внезапно возникшую боль внизу живота

С.Ж.АСФЕНДИЯРОВАТЫНДАҒЫҚАЗАҚ ҰЛТТЫҚ МЕДИЦИНА

УНИВЕРСИТЕТІ

КАЗАХСКИЙНАЦИОНАЛЬНЫЙМЕДИЦИНСКИЙ УНИВЕРСИТЕТ ИМЕНИ С.Д.АСФЕНДИЯРОВА

КАФЕДРА ИНТЕРНАТУРЫ И РЕЗИДЕНТУРЫ ПО АКУШЕРСТВУ И ГИНЕКОЛОГИИТЕСТОВЫЕ ЗАДАНИЯ ДЛЯ ПРОВЕДЕНИЯ ИТОГОВОЙ ГОСУДАРСТВЕННОЙ АТТЕСТАЦИИ РЕЗИДЕНТОВ

ВЫПУСКА 2012-2013 ГГ. ПО СПЕЦИАЛЬНОСТИ «АКУШЕРСТВО И ГИНЕКОЛОГИЯ»

А.осмотр шейки матки на зеркалахБ. УЗИ исследованиеВ. направление в гинекологическое отделениеГ. направление в онкодиспансерД. госпитализация в дневной стационар и наблюдение

395.Клинические варианты самопроизвольных выкидышей:А.угрожающий, начавшийся, аборт входу, неполный, полный выкидыш Б. угрожающий, неполный и полный выкидышВ. начавшийся, неполный и полный выкидышГ. угрожающий, аборт входу, полный выкидыш Д. несостоявшийся, начавшийся, полный выкидыш

396.Обильное кровотечение из половых путей бывает при ... .А). неполном выкидышеB). угрожающем выкидышеC). начавшемся выкидышеD). полном выкидышеE). разрыве рудиментарного рога

397.Скудные кровянистые выделения из половых путей бывают при ... . А. начавшемся выкидышеБ. угрожающем выкидышеВ. полном выкидышеГ. разрыве рудиментарного рогаД. неполном выкидыше

398.Абортом в ходу называется - А. отслоение плодного яйца и продвижение по родовому каналуБ. отслоение плодного яйцаВ. рождение плодного яйцаГ. рождение плода без плодных оболочекД.частичное отслоение плодного яйца

399.С каким состоянием нет необходимости дифференцировать начавшийся абортА) внематочная беременностьB) рак шейки матки C) пузырный заносD) предлежание плацентыE) разрыв варикозного узла во влагалище

400.Наиболее часто встречающаяся форма гиперандрогении при привычном невынашивании беременности: a) Надпочечниковаяb) Яичниковаяc) Смешанная, яичниково-надпочечниковая.

Page 159: postgraduate.kaznmu.kz€¦ · Web view58 Больная И, 28 лет, доставлена с жалобами на внезапно возникшую боль внизу живота

С.Ж.АСФЕНДИЯРОВАТЫНДАҒЫҚАЗАҚ ҰЛТТЫҚ МЕДИЦИНА

УНИВЕРСИТЕТІ

КАЗАХСКИЙНАЦИОНАЛЬНЫЙМЕДИЦИНСКИЙ УНИВЕРСИТЕТ ИМЕНИ С.Д.АСФЕНДИЯРОВА

КАФЕДРА ИНТЕРНАТУРЫ И РЕЗИДЕНТУРЫ ПО АКУШЕРСТВУ И ГИНЕКОЛОГИИТЕСТОВЫЕ ЗАДАНИЯ ДЛЯ ПРОВЕДЕНИЯ ИТОГОВОЙ ГОСУДАРСТВЕННОЙ АТТЕСТАЦИИ РЕЗИДЕНТОВ

ВЫПУСКА 2012-2013 ГГ. ПО СПЕЦИАЛЬНОСТИ «АКУШЕРСТВО И ГИНЕКОЛОГИЯ»

d) Центральнаяe) Периферическая (идиопатическая)

401.Что НАИБОЛЕЕ подходит к описанию «привычного невынашивания»А) 1 выкидыш в анамнезеB) 2 выкидыша подряд и более в анамнезе C) более 2 выкидышей подряд в анамнезеD) более 3 выкидышей подряд в анамнезеE) более 4 выкидышей подряд в анамнезе

402.«Привычная потеря беременности » - это:А. прерывание двух и более беременностейБ. прерывание первой беременностиВ. прерывание беременности до 12 недельГ. прерывание беременности до 22 недельД. прерывание беременности до 37 недель

403.В структуре невынашивания привычный выкидыш составляетА) 1-10%B) 5-20%C) 10-30%D) 15-40%E) 20-50% 404.В женскую консультацию обратилась пациентка с привычным невынашиванием во 2-м триместре в анамнезе. НАИБОЛЕЕ частая причина данной патологии?А. инфекцияБ. эндокриннаяВ. истмико-цервикальная недостаточностьГ. аутоиммуннаяД. хромосомные аномалии

405.К анатомическим факторам привычного невынашивания беременности относят:А) генитальный инфантилизмБ) хронический эндометритВ) врожденные аномалии развития маткиГ) внутриматочные синехииД) органическая ИЦН

406.Что НАИМЕНЕЕ показано пациенткам с привычным выкидышем на фоне гиперандрогении яичникового генеза на этапе прегравидарной подготовки:А. снижение массы телаБ. коррекция нарушений углеводного и жирового обменаВ. иммуномодулирующая терапияГ. дексаметазон в большой дозеД. стимуляция овуляции и поддержание лютеиновой фазы.

Page 160: postgraduate.kaznmu.kz€¦ · Web view58 Больная И, 28 лет, доставлена с жалобами на внезапно возникшую боль внизу живота

С.Ж.АСФЕНДИЯРОВАТЫНДАҒЫҚАЗАҚ ҰЛТТЫҚ МЕДИЦИНА

УНИВЕРСИТЕТІ

КАЗАХСКИЙНАЦИОНАЛЬНЫЙМЕДИЦИНСКИЙ УНИВЕРСИТЕТ ИМЕНИ С.Д.АСФЕНДИЯРОВА

КАФЕДРА ИНТЕРНАТУРЫ И РЕЗИДЕНТУРЫ ПО АКУШЕРСТВУ И ГИНЕКОЛОГИИТЕСТОВЫЕ ЗАДАНИЯ ДЛЯ ПРОВЕДЕНИЯ ИТОГОВОЙ ГОСУДАРСТВЕННОЙ АТТЕСТАЦИИ РЕЗИДЕНТОВ

ВЫПУСКА 2012-2013 ГГ. ПО СПЕЦИАЛЬНОСТИ «АКУШЕРСТВО И ГИНЕКОЛОГИЯ»

407.Преждевременные родыЧто отвечает описанию «недонашивание беременности» А) раннее самопроизвольное прерывание беременностиB) неразвивающаяся беременностьC) несостоявшийся выкидышD) поздний самопроизвольный выкидышE) преждевременные роды

408.Актуальность проблемы недонашивания заключается в высокой частоте:А. внутриутробного инфицированияБ. перинатальной заболеваемостиВ. перинатальной смертностиГ. родовой травмыД. гипотрофии плода

409.Преждевременные роды – это роды, наступившие при сроке беременности А) до 22 недельB) от 22 до 34 недель C) от 28 до 34 недельD) от 22 до 37 недельE) от 28 до 37 недель

410.НАИМЕНЕЕ характерный фактор риска преждевременных родовА) курениеB) низкий росто-весовой показательC) инфекционные заболеванияD) избыточная масса телаE) возраст 16 лет и младше

411.НАИМЕНЕЕ характерный фактор риска преждевременных родовА) истмико -цервикальная недостаточностьB) рубец на матке после кесарева сеченияC) многоплодная беременностьD) аномалии развития маткиE) преждевременные роды в анамнезе

412.Прерывание беременности при сроке 22- 28 недель, когда рождается незрелый жизнеспособный ребенок с массой тела от 500 до 1000 г., согласно номенклатуре ВОЗ, это А) самопроизвольный выкидышВ) срочные родыС) преждевременные родыD) ранние преждевременные родыЕ) патологические роды

413.Пролонгирование беременности при угрожающих и начинающихся преждевременных родах противопоказано при: А. невозможности проведения профилактики дистресс-синдрома у плодаБ. отсутствии у женщины тяжелой соматической и акушерской патологии

Page 161: postgraduate.kaznmu.kz€¦ · Web view58 Больная И, 28 лет, доставлена с жалобами на внезапно возникшую боль внизу живота

С.Ж.АСФЕНДИЯРОВАТЫНДАҒЫҚАЗАҚ ҰЛТТЫҚ МЕДИЦИНА

УНИВЕРСИТЕТІ

КАЗАХСКИЙНАЦИОНАЛЬНЫЙМЕДИЦИНСКИЙ УНИВЕРСИТЕТ ИМЕНИ С.Д.АСФЕНДИЯРОВА

КАФЕДРА ИНТЕРНАТУРЫ И РЕЗИДЕНТУРЫ ПО АКУШЕРСТВУ И ГИНЕКОЛОГИИТЕСТОВЫЕ ЗАДАНИЯ ДЛЯ ПРОВЕДЕНИЯ ИТОГОВОЙ ГОСУДАРСТВЕННОЙ АТТЕСТАЦИИ РЕЗИДЕНТОВ

ВЫПУСКА 2012-2013 ГГ. ПО СПЕЦИАЛЬНОСТИ «АКУШЕРСТВО И ГИНЕКОЛОГИЯ»

В. подтекании околоплодных водГ. раскрытии маточного зева не более 2-4 смД. воспалительной реакции влагалищных мазков

414.НАИМЕНЕЕ оправданное мероприятие при ведении преждевременных родовА) обязательное присутствие неонатологаB) строгое соблюдение «тепловой цепочки»C) пережатие пуповины после прекращения пульсацииD) обязательное проведение эпидуральной анестезииE) раннее грудное вскармливание по возможности

415.Абсолютное противопоказание к индуцированным преждевременным родамА) тазовое предлежание плодаB) многоплодная беременностьC) ЗВРПD) первичный генитальный герпесE) рубец на матке

416.С целью индукции преждевременных родов при антенатальной гибели плода применяютА) мизопростолB) мифепристонC) динопростонD) ламинарииE) энзапрост

417.Какие профилактические мероприятия согласно доказательной медицины НАИБОЛЕЕ эффективны для снижения частоты преждевременных родовА) усиленный антенатальный уход, госпитализация в «критические сроки»B) увеличение числа дородовых посещенийC) половое воздержаниеD) диагностика и лечение бактериального вагиноза до 20 недели беременностиE) диагностика и лечение бактериального вагиноза после 20 недели беременности

418.Истмико-цервикальная недостаточностьПо какой бальной шкале определяется степень выраженности ИЦНА.шкала АльговераБ.шкала ШтембераВ.шкала БишопГ.шкала ИЦНД.шкала Штарка

419.На диспансерном учете в женской консультации состоит беременная 26 лет. Жалоб не предъявляет. В анамнезе – 1 медицинский аборт, 2 самопроизвольных выкидыша в сроки 21 и 25 недель. При влагалищном исследовании: матка увеличена до 17 недельного срока беременности. Шейка матки укорочена до 1,5 см, размягчена на всем протяжении, канал шейки свободно пропускает 1 палец. Тактика врача женской консультации:А. амбулаторное наблюдение

Page 162: postgraduate.kaznmu.kz€¦ · Web view58 Больная И, 28 лет, доставлена с жалобами на внезапно возникшую боль внизу живота

С.Ж.АСФЕНДИЯРОВАТЫНДАҒЫҚАЗАҚ ҰЛТТЫҚ МЕДИЦИНА

УНИВЕРСИТЕТІ

КАЗАХСКИЙНАЦИОНАЛЬНЫЙМЕДИЦИНСКИЙ УНИВЕРСИТЕТ ИМЕНИ С.Д.АСФЕНДИЯРОВА

КАФЕДРА ИНТЕРНАТУРЫ И РЕЗИДЕНТУРЫ ПО АКУШЕРСТВУ И ГИНЕКОЛОГИИТЕСТОВЫЕ ЗАДАНИЯ ДЛЯ ПРОВЕДЕНИЯ ИТОГОВОЙ ГОСУДАРСТВЕННОЙ АТТЕСТАЦИИ РЕЗИДЕНТОВ

ВЫПУСКА 2012-2013 ГГ. ПО СПЕЦИАЛЬНОСТИ «АКУШЕРСТВО И ГИНЕКОЛОГИЯ»

Б. назначить спазмолитическую терапиюВ. провести курс лечения токолитикамиГ. хирургическая коррекцию ИЦНД. постельный режим дома

420.Метод хирургической коррекции ИЦН вне беременностиА) по Мак-Дональду B) по Любимовой – Мамедалиевой C) по СендиD) по ШиродкаруE) по Ельцову-Стрелкову

421.НАИБОЛЕЕ оптимальный метод хирургической коррекции ИЦН во время беременности на современном этапе А) по Любимовой B) по Любимовой – Мамедалиевой C) по СцендиD) по ШиродкаруE) по Мак-Дональду

422.Методика проведения хирургической коррекции ИЦН по Любимовой – Мамедалиевой А) иссечение слизистой оболочки вокруг наружного зеваB) наложение кисетного шва путем многократного прокалывания стенки влагалищаC) наложение циркулярного шва на шейку матки в области внутреннего зева после предварительного рассечения слизистой влагалищаD) наложение кругового проволочного шва на шейку матки в области внутреннего зева без рассечения слизистой влагалищаE) наложение двойного П-образного лавсанового шва на шейку матки

423.В каком году была предложена методика хирургической коррекции ИЦН по Любимовой – Мамедалиевой А) в 1965 годуB) в 1967 годуC) в 1978 годуD) в 1987 годуE) в 1990 году

424.Какой шовный материал используют при проведении хирургической коррекции ИЦН по Любимовой – Мамедалиевой А) хромированный кетгутB) шелкC) фасциальная полоскаD) лавсанE) металлическая проволока в полиэтиленовой оболочке

425.Какой метод хирургической коррекции ИЦН НАИБОЛЕЕ оправдан при пролабировании плодного пузыряА) по Любимовой

Page 163: postgraduate.kaznmu.kz€¦ · Web view58 Больная И, 28 лет, доставлена с жалобами на внезапно возникшую боль внизу живота

С.Ж.АСФЕНДИЯРОВАТЫНДАҒЫҚАЗАҚ ҰЛТТЫҚ МЕДИЦИНА

УНИВЕРСИТЕТІ

КАЗАХСКИЙНАЦИОНАЛЬНЫЙМЕДИЦИНСКИЙ УНИВЕРСИТЕТ ИМЕНИ С.Д.АСФЕНДИЯРОВА

КАФЕДРА ИНТЕРНАТУРЫ И РЕЗИДЕНТУРЫ ПО АКУШЕРСТВУ И ГИНЕКОЛОГИИТЕСТОВЫЕ ЗАДАНИЯ ДЛЯ ПРОВЕДЕНИЯ ИТОГОВОЙ ГОСУДАРСТВЕННОЙ АТТЕСТАЦИИ РЕЗИДЕНТОВ

ВЫПУСКА 2012-2013 ГГ. ПО СПЕЦИАЛЬНОСТИ «АКУШЕРСТВО И ГИНЕКОЛОГИЯ»

B) по Любимовой – Мамедалиевой C) по СцендиD) по ШиродкаруE) по Мак-Дональду

426.Возможное осложнение хирургической коррекции ИЦН по Любимовой – Мамедалиевой А) кровотечение при проведении операцииB) образование пролежней с последующим возникновением свищейC) поперечные или круговые отрывы шейки маткиD) высокий процент оперативного родоразрешения вследствие рубцовой деформации шейки маткиE) частичное неглубокое прорезывание швов

427.Противопоказание для проведении хирургической коррекции ИЦН по Любимовой – Мамедалиевой А) эрозия шейки маткиB) эндоцервицитC) пролабирование плодного пузыряD) повышенная возбудимость матки, не снимающаяся под воздействием медикаментозных средствE) трихомониаз во время данной беременности, пролеченный, с отрицательными микробиологическими результатами

428.НАИМЕНЕЕ характерная особенность хирургической коррекции ИЦН по Любимовой – Мамедалиевой А) сохранение дренажного отверстия при вероятности развития внутриматочной инфекцииB) меньшее количество койко-дней после проведения операцииC) возможность проведения коррекции при пролабировании плодного пузыряD) активизация женщины сразу после окончания операцииE) отсутствие возможности проведения повторной коррекции ИЦН в случае прорезывания швов на шейке матки

429.Проведение хирургической коррекции ИЦН по Любимовой – Мамедалиевой НАИБОЛЕЕ целесообразно в срокиА) до 12 недель беременностиB) 14 -16 недель беременностиC) 18-20 недель беременностиD) 22-24 недель беременностиE) 26-28 недель беременности

430.Оптимальные сроки для заблаговременного снятия швов с шейки матки после коррекции ИЦН по Любимовой – Мамедалиевой при неосложненном течении беременностиА) 36-37 недель беременностиB) 37-38 недель беременностиC) 38-39 недель беременностиD) 39-40 недель беременностиE) с началом родовой деятельности

Page 164: postgraduate.kaznmu.kz€¦ · Web view58 Больная И, 28 лет, доставлена с жалобами на внезапно возникшую боль внизу живота

С.Ж.АСФЕНДИЯРОВАТЫНДАҒЫҚАЗАҚ ҰЛТТЫҚ МЕДИЦИНА

УНИВЕРСИТЕТІ

КАЗАХСКИЙНАЦИОНАЛЬНЫЙМЕДИЦИНСКИЙ УНИВЕРСИТЕТ ИМЕНИ С.Д.АСФЕНДИЯРОВА

КАФЕДРА ИНТЕРНАТУРЫ И РЕЗИДЕНТУРЫ ПО АКУШЕРСТВУ И ГИНЕКОЛОГИИТЕСТОВЫЕ ЗАДАНИЯ ДЛЯ ПРОВЕДЕНИЯ ИТОГОВОЙ ГОСУДАРСТВЕННОЙ АТТЕСТАЦИИ РЕЗИДЕНТОВ

ВЫПУСКА 2012-2013 ГГ. ПО СПЕЦИАЛЬНОСТИ «АКУШЕРСТВО И ГИНЕКОЛОГИЯ»

431.НАИМЕНЕЕ оправданное показание для экстренного снятия швов с шейки матки после коррекции ИЦН по Любимовой – МамедалиевойА) начало преждевременных родовB) кровотечениеC) излитие околоплодных водD) частичное прорезывание швов на шейке маткиE) полное прорезывание швов на шейке матки

432.НАИБОЛЕЕ частая причина позднего выкидыша и преждевременных родовА) истмико -цервикальная недостаточностьB) гиперандрогенияC) хромосомные аберрацииD) аномалии развития маткиE) недостаточность лютеиновой фазы

433.НАИБОЛЕЕ эффективный нехирургический метод лечения истмико-цервикальной недостаточности во время беременностиА) постельный режим с возвышенным положением ножного концаB) профилактическое применение спазмалитиковC) применение токолитиков D) применение разгружающего влагалищного пессарияE) длительное применение гестагенов и дексаметазона

434.В отделение поступила беременная А., 26 лет с жалобами на тянущие боли внизу живота. В детстве перенесла корь, скарлатину. Данная беременность 3, срок 19-20 недель, 1 беременность - роды крупным плодом, осложнилась разрывом шейки матки, 2 беременность через 2 года закончилась поздним абортом. При осмотре матка в тонусе. Шейка матки деформирована, наружный зев зияет, шейка укорочена, канал свободно проходим для 1 пальца за внутренний зев. Ваш диагноз:А) Беременность 19-20 недель. Угрожающий поздний самопроизвольный аборт В) Беременность 19-20 недель. Начавшийся поздний самопроизвольный аборт. ИЦН C) Беременность 19-20 недель. Угрожающий поздний самопроизвольный аборт. ИЦНD) Беременность 19-20 недель. Поздний самопроизвольный аборт в ходу. ИЦНE) Беременность 19-20 недель. Угрожающий поздний привычный самопроизвольный аборт.

435.В женскую консультацию обратилась повторнобеременная с жалобами на дискомфорт внизу живота и в пояснице, скудные кровянистые выделения, чувство давления во влагалище. Пациентка работает медицинским работником. В анамнезе два самопроизвольных выкидыша в сроке 15-16 и 20-21 недель. Выберите наиболее вероятную причину выкидыша:А. ИЦНБ. стрессВ.физическая нагрузкаГ. инфекцияД. психологическая нагрузка

436.В женскую консультацию обратилась повторнобеременная с жалобами на дискомфорт внизу живота и в пояснице, скудные кровянистые выделения, чувство давления во влагалище. В

Page 165: postgraduate.kaznmu.kz€¦ · Web view58 Больная И, 28 лет, доставлена с жалобами на внезапно возникшую боль внизу живота

С.Ж.АСФЕНДИЯРОВАТЫНДАҒЫҚАЗАҚ ҰЛТТЫҚ МЕДИЦИНА

УНИВЕРСИТЕТІ

КАЗАХСКИЙНАЦИОНАЛЬНЫЙМЕДИЦИНСКИЙ УНИВЕРСИТЕТ ИМЕНИ С.Д.АСФЕНДИЯРОВА

КАФЕДРА ИНТЕРНАТУРЫ И РЕЗИДЕНТУРЫ ПО АКУШЕРСТВУ И ГИНЕКОЛОГИИТЕСТОВЫЕ ЗАДАНИЯ ДЛЯ ПРОВЕДЕНИЯ ИТОГОВОЙ ГОСУДАРСТВЕННОЙ АТТЕСТАЦИИ РЕЗИДЕНТОВ

ВЫПУСКА 2012-2013 ГГ. ПО СПЕЦИАЛЬНОСТИ «АКУШЕРСТВО И ГИНЕКОЛОГИЯ»

анамнезе два самопроизвольных выкидыша в сроке 15-16 и 20-21 недель. При вагинальном обследовании: Матка увеличена до 14-15 недель, шейка матки укорочена до 2,0 см, мягкая. Наружный зев пропускает кончик пальца. Выберите наиболее правильный диагноз:А. беременность 15-16 недель. Угрожающий выкидыш. Привычное невынашиваниеБ. беременность 15-16 недель. Начавшийся выкидыш. ИЦН. Привычное невынашиваниеВ. беременность 15-16 недель. Угрожающий выкидыш. ОАА. Привычное невынашивание Г. беременность 15-16 недель. Начавшийся выкидыш. ОАА. Привычное невынашивание Д. беременность 15-16 недель. ИЦН. Привычное невынашивание

437.В женскую консультацию обратилась повторнобеременная с жалобами на дискомфорт внизу живота и в пояснице, чувство давления во влагалище. В анамнезе два самопроизвольных выкидыша в сроке 15-16 и 20-21 недель. При вагинальном обследовании: Матка увеличена до 14-15 недель, шейка матки укорочена до 2,0 см, мягкая. Наружный зев пропускает кончик пальца. Выберите оптимальную тактику ведения:А. наблюдениеБ. госпитализация в дневной стационар для токолизаВ. хирургическая коррекция шейки маткиГ. седативная терапия амбулаторноД. спазмолитики амбулаторно

438.К противопоказаниям к хирургическому лечению ИЦН НЕ относятся: А. тяжёлые формы ЭГЗБ. II степень чистоты влагалищной флорыВ. психические заболеванияГ. острые инфекцииД. ВПР плода

439.НАИМЕНЕЕ оправданное показание к снятию швов с шейки матки при ИЦН:А. срок беременности 37 недельБ. подтекание околоплодных вод В. кровянистые выделения из полости маткиГ. ложные схватки до 37 недельД. прорезывание швов

440.НАИМЕНЕЕ оправданное действие в послеоперационном периоде после хирургической коррекции шейки матки А. бактериоскопия вагинального отделяемогоБ. коррекция микробиоценоза влагалищаВ. влагалищное исследованиеГ. токолизД. осмотр швов на шейке матки.

441.Согласно протоколу препаратами для токолиза являютсяА) спазмалитики и атосибанB) бета-миметики и спазмалитикиC) нифедипин и атосибанD) нифедипин и гинипралE) атосибан и магнезия

Page 166: postgraduate.kaznmu.kz€¦ · Web view58 Больная И, 28 лет, доставлена с жалобами на внезапно возникшую боль внизу живота

С.Ж.АСФЕНДИЯРОВАТЫНДАҒЫҚАЗАҚ ҰЛТТЫҚ МЕДИЦИНА

УНИВЕРСИТЕТІ

КАЗАХСКИЙНАЦИОНАЛЬНЫЙМЕДИЦИНСКИЙ УНИВЕРСИТЕТ ИМЕНИ С.Д.АСФЕНДИЯРОВА

КАФЕДРА ИНТЕРНАТУРЫ И РЕЗИДЕНТУРЫ ПО АКУШЕРСТВУ И ГИНЕКОЛОГИИТЕСТОВЫЕ ЗАДАНИЯ ДЛЯ ПРОВЕДЕНИЯ ИТОГОВОЙ ГОСУДАРСТВЕННОЙ АТТЕСТАЦИИ РЕЗИДЕНТОВ

ВЫПУСКА 2012-2013 ГГ. ПО СПЕЦИАЛЬНОСТИ «АКУШЕРСТВО И ГИНЕКОЛОГИЯ»

442.Антагонист окситоциновых рецепторовА) гинипралB) партусистенC) карбетоцинD) нифедипинE) атосибан

443.Какой из ниже перечисленных препаратов применяется с целью токолиза, согласно протоколуА) гинипралB) партусистенC) карбетоцинD) папаверинE) трактоцил

444.Токолитики при угрозе преждевременного прерывания беременности применяют в срокиА) с ранних сроков беременности до 34 неделиB) с ранних сроков беременности до 37 неделиC) с 22 недель до 34 недельD) с 22 недель до 37 неделиE) при угрозе позднего выкидыша

445.Какое условие является противопоказанием к применению трактоцилаА) целый плодный пузырьB) наличие живого плодаC) открытие шейки матки от 1 до 3 смD) нарушение сердечного ритма плодаE) регулярные маточные сокращения продолжительностью не меньше 30 секунд и частотой более 4 в течение 30 минут

446.Схемы применения атосибана А) внутривенное введение в 3 последовательных этапа B) внутривенное однократное введение C) внутрь по схеме в течение 2-х днейD) внутривенное введение с последующим переходом на таблетированные формыE) внутримышечное введение по схеме447.Болюсное введение атосибана А) внутривенное введение раствора для инъекций в дозе 6,75 мгB) длительная инфузия концентрата в высокой дозе 300 мкг/мин в течение 3 часовC) длительная инфузия концентрата в низкой дозе 100 мкг/мин в течение 45 часовD) внутривенное введение с последующим переходом на таблетированные формыE) внутримышечное введение по схеме

448.Второй этап введения трактоцилаА) внутривенное введение раствора для инъекций в дозе 6,75 мгB) длительная инфузия концентрата в высокой дозе 300 мкг/мин в течение 3 часовC) длительная инфузия концентрата в низкой дозе 100 мкг/мин в течение 45 часовD) внутривенное введение с последующим переходом на таблетированные формы

Page 167: postgraduate.kaznmu.kz€¦ · Web view58 Больная И, 28 лет, доставлена с жалобами на внезапно возникшую боль внизу живота

С.Ж.АСФЕНДИЯРОВАТЫНДАҒЫҚАЗАҚ ҰЛТТЫҚ МЕДИЦИНА

УНИВЕРСИТЕТІ

КАЗАХСКИЙНАЦИОНАЛЬНЫЙМЕДИЦИНСКИЙ УНИВЕРСИТЕТ ИМЕНИ С.Д.АСФЕНДИЯРОВА

КАФЕДРА ИНТЕРНАТУРЫ И РЕЗИДЕНТУРЫ ПО АКУШЕРСТВУ И ГИНЕКОЛОГИИТЕСТОВЫЕ ЗАДАНИЯ ДЛЯ ПРОВЕДЕНИЯ ИТОГОВОЙ ГОСУДАРСТВЕННОЙ АТТЕСТАЦИИ РЕЗИДЕНТОВ

ВЫПУСКА 2012-2013 ГГ. ПО СПЕЦИАЛЬНОСТИ «АКУШЕРСТВО И ГИНЕКОЛОГИЯ»

E) внутримышечное введение по схеме449.Третий этап введения трактоцилаА) внутривенное введение раствора для инъекций в дозе 6,75 мгB) длительная инфузия концентрата в высокой дозе 300 мкг/мин в течение 3 часовC) длительная инфузия концентрата в низкой дозе 100 мкг/мин в течение 45 часовD) внутривенное введение с последующим переходом на таблетированные формыE) внутримышечное введение по схеме

450.Какой из ниже перечисленных гестагенов НАИМЕНЕЕ популярен при лечении угрожающего аборта на современном этапе:А) дюфастон B) утрожестан C) туринал D) микронизированный прогестеронE) кристаллический прогестерон

451.Основная цель токолиза при преждевременных родахА) пролонгирование беременности до доношенного срокаB) проведение профилактики РДС и перевод на соответствующий уровень оказания неонатальной помощиC) снятие возбудимости маткиD) уменьшение болевой реакцииE) профилактика чрезмерной родовой деятельности

452.НАИМЕНЕЕ оправданный препарат для прегравидарной подготовки пациенток с функциональной гиперпролактинемиейА) парлоделB) норпролакC) достинекD) абергинE) даназол

453.При наступлении беременности у женщин с функциональной гиперпролактинемией на фоне лечения дофаминомиметиками прием препаратовА) продолжается всю беременностьB) продолжается под контролем уровня пролактина до нормализации показателейC) отменяется всем женщинам при диагностике беременностиD) отменяется выборочно в зависимости от уровня пролактинаE) продолжается с увеличением дозы в связи с беременностью

454.Схема применения нифедипина с целью токолиза А) по 10 мг через 8 часов в течение 7 дней. Максимальная курсовая доза – 210 мг в деньB) по 10 мг каждые 15 минут в течение 4 часов. Максимальная доза – 160 мг в деньC) по 10 мг каждые 3 часа до прекращения схваток. Максимальная доза – 160 мг в деньD) 10 мг внутрь, при сохранении сокращений матки через 15 минут 10 мг повторно, затем по 10 мг каждые 3-8 часов в течение 48 часов до исчезновения схваток. Максимальная доза – 160 мг в день

Page 168: postgraduate.kaznmu.kz€¦ · Web view58 Больная И, 28 лет, доставлена с жалобами на внезапно возникшую боль внизу живота

С.Ж.АСФЕНДИЯРОВАТЫНДАҒЫҚАЗАҚ ҰЛТТЫҚ МЕДИЦИНА

УНИВЕРСИТЕТІ

КАЗАХСКИЙНАЦИОНАЛЬНЫЙМЕДИЦИНСКИЙ УНИВЕРСИТЕТ ИМЕНИ С.Д.АСФЕНДИЯРОВА

КАФЕДРА ИНТЕРНАТУРЫ И РЕЗИДЕНТУРЫ ПО АКУШЕРСТВУ И ГИНЕКОЛОГИИТЕСТОВЫЕ ЗАДАНИЯ ДЛЯ ПРОВЕДЕНИЯ ИТОГОВОЙ ГОСУДАРСТВЕННОЙ АТТЕСТАЦИИ РЕЗИДЕНТОВ

ВЫПУСКА 2012-2013 ГГ. ПО СПЕЦИАЛЬНОСТИ «АКУШЕРСТВО И ГИНЕКОЛОГИЯ»

E) 10 мг внутрь, при сохранении сокращений матки через 15 минут 10 мг повторно, затем по 10 мг каждые 3-8 часов в течение 72 часов до исчезновения схваток. Максимальная доза – 200 мг в день

455.Профилактика РДС проводится А) с 22 до 36 недель беременностиB) с 24 до 36 недель беременностиC) с 22 до 34 недель беременностиD) с 24 до 34 недель беременностиE) с 22 до 24 недель беременности

456.Выберете подходящее описание проведения профилактики РДСА) применение глюкокортикоидов эффективно с 22 недель беременностиB) повторный курс глюкокортикоидов при рецидиве угрозы преждевременных родовC) доза дексаметазона 24 мг в течение 48 часов однократно во время беременности D) доза дексаметазона 24 мг в течение 24 часов однократно во время беременности E) обязательное применение при хориоамнионите

457. Антибиотикопрофилактика при преждевременных родахА) не показанаB) показана при любом виде преждевременных родовC) показана только при сроке до 34 недельD) показана только при сроке от 34 до 37 недельE) показана только при признаках инфекции

458.Оптимальный режим применения токолитических препаратов при преждевременных родахА) монотерапия в течение не более 48 часов B) комбинированное назначение токолитиковC) индивидуальный подходD) режим смены препаратов при неэффективностиE) длительная терапия от 1 недели и более459.Оптимальный срок применения дюфастона при угрозе невынашивания беременности в группе риска А) до 12 недель беременностиB) до 16 недель беременностиC) до 20 недель беременностиD) до 24 недель беременностиE) до 28 недель беременности

460.Современные рекомендации по коррекции гиперандрогении во время беременностиА) дексаметазон под контролем экскреции 17 КС до 35-36 недельB) дексаметазон под контролем экскреции 17 КС до 30-32 недельC) дексаметазон под контролем экскреции 17 КС до 18-20 недельD) дексаметазон под контролем экскреции 17 КС до нормализации показателей 17КС и ДЭАE) после наступления беременности необходимости в лечении гиперандрогении нет

461.Что отвечает описанию «несостоявшегося выкидыша»

Page 169: postgraduate.kaznmu.kz€¦ · Web view58 Больная И, 28 лет, доставлена с жалобами на внезапно возникшую боль внизу живота

С.Ж.АСФЕНДИЯРОВАТЫНДАҒЫҚАЗАҚ ҰЛТТЫҚ МЕДИЦИНА

УНИВЕРСИТЕТІ

КАЗАХСКИЙНАЦИОНАЛЬНЫЙМЕДИЦИНСКИЙ УНИВЕРСИТЕТ ИМЕНИ С.Д.АСФЕНДИЯРОВА

КАФЕДРА ИНТЕРНАТУРЫ И РЕЗИДЕНТУРЫ ПО АКУШЕРСТВУ И ГИНЕКОЛОГИИТЕСТОВЫЕ ЗАДАНИЯ ДЛЯ ПРОВЕДЕНИЯ ИТОГОВОЙ ГОСУДАРСТВЕННОЙ АТТЕСТАЦИИ РЕЗИДЕНТОВ

ВЫПУСКА 2012-2013 ГГ. ПО СПЕЦИАЛЬНОСТИ «АКУШЕРСТВО И ГИНЕКОЛОГИЯ»

А) раннее самопроизвольное прерывание беременностиB) неразвивающаяся беременностьC) поздний самопроизвольный выкидышD) аборт в ходуE) неполный аборт

462.Несостоявшийся выкидыш характеризуется:А. гибелью плода (эмбриона) с задержкой его самопроизвольного изгнания из полости маткиБ. самопроизвольным абортом, осложненным острым эндометритомВ. излитием вод и признаками хориоамнионитаГ. антенатальной гибелью плодаД. урежением сердцебиения плода 463.НАИМЕНЕЕ характерное осложнение для неразвивающейся беременностиА) коагулопатическое кровотечениеB) аутоинтоксикация продуктами аутолиза эмбриона (плода)C) хронический эндометритD) повторная неразвивающаяся беременностьE) острое воспалительное заболевание органов малого таза

464.Среди гормональных нарушений, приводящих к неразвивающейся беременности, НАИБОЛЕЕ часто встречается: A) Гиперандрогения B) Сахарный диабетC) Гипофункция яичниковD) ГиперпролактинемияE) Нарушение функции щитовидной железы

465.Какое ВЗОМТ НАИБОЛЕЕ часто является причиной неразвивающейся беременности А) кольпитB) цервицитC) хронический эндометритD) сальпиногоофоритE) пельвиоперитонит

466.НАИБОЛЕЕ информативный метод диагностики хронического эндометрита как причины невынашивания вне беременностиА) УЗИB) допплерометрияC) гистеросальпингографияD) гистероскопияE) бактериологическое исследование

467.Неразвивающаяся беременность по типу анэмбрионии чаще встречается в срокиА) 5-8 недель беременностиB) 9-11 недель беременностиC) 11-12 недель беременностиD) 12-15 недель беременности

Page 170: postgraduate.kaznmu.kz€¦ · Web view58 Больная И, 28 лет, доставлена с жалобами на внезапно возникшую боль внизу живота

С.Ж.АСФЕНДИЯРОВАТЫНДАҒЫҚАЗАҚ ҰЛТТЫҚ МЕДИЦИНА

УНИВЕРСИТЕТІ

КАЗАХСКИЙНАЦИОНАЛЬНЫЙМЕДИЦИНСКИЙ УНИВЕРСИТЕТ ИМЕНИ С.Д.АСФЕНДИЯРОВА

КАФЕДРА ИНТЕРНАТУРЫ И РЕЗИДЕНТУРЫ ПО АКУШЕРСТВУ И ГИНЕКОЛОГИИТЕСТОВЫЕ ЗАДАНИЯ ДЛЯ ПРОВЕДЕНИЯ ИТОГОВОЙ ГОСУДАРСТВЕННОЙ АТТЕСТАЦИИ РЕЗИДЕНТОВ

ВЫПУСКА 2012-2013 ГГ. ПО СПЕЦИАЛЬНОСТИ «АКУШЕРСТВО И ГИНЕКОЛОГИЯ»

E) 15-18 недель беременности

468.Наиболее частая форма тромбофилии, приводящая к невынашиванию беременности: a) Недостаток XII фактора; b) Дефицит протеина С; c) Дефицит кофактора II гепарина; d) АФС; e) Дефицит антитромбина;

469. НАИБОЛЕЕ информативный метод диагностики неразвивающейся беременности А. УЗИБ. определение ХГЧВ. влагалищное исследованиеГ. осмотр в зеркалахД. наружное акушерское исследование

470.Для АФС как причины невынашивания НАИМЕНЕЕ характерно: А. тромбоз плацентарных сосудовБ. антенатальная гибель плодаВ. внутриутробная задержка плодаГ.укорочение времени свертывания кровиД. удлинение времени свертывания крови

471.НАИМЕНЕЕ характерная причина неразвивающейся беременностиА) истмико -цервикальная недостаточностьB) эндокринные факторыC) хромосомная аномалияD) аутоиммунные процессыE) инфекция

472.НАИМЕНЕЕ характерное осложнение беременности при АФСА) неразвивающаяся беременностьB) антенатальная гибель плодаC) ЗВРПD) преэклампсияE) преждевременный разрыв плодных оболочек 473.Согласно данным литературы при АФС без проведения лечения гибель эмбриона/плода наблюдается А) у 10-15% женщинB) у 30-35% женщинC) у 50-55% женщинD) у 70-75% женщинE) у 90-95% женщин

474.НАИМЕНЕЕ подходящее описание для АФСА) аутоиммунная патологияB) гиперпродукция антител к собственным фосфолипидам

Page 171: postgraduate.kaznmu.kz€¦ · Web view58 Больная И, 28 лет, доставлена с жалобами на внезапно возникшую боль внизу живота

С.Ж.АСФЕНДИЯРОВАТЫНДАҒЫҚАЗАҚ ҰЛТТЫҚ МЕДИЦИНА

УНИВЕРСИТЕТІ

КАЗАХСКИЙНАЦИОНАЛЬНЫЙМЕДИЦИНСКИЙ УНИВЕРСИТЕТ ИМЕНИ С.Д.АСФЕНДИЯРОВА

КАФЕДРА ИНТЕРНАТУРЫ И РЕЗИДЕНТУРЫ ПО АКУШЕРСТВУ И ГИНЕКОЛОГИИТЕСТОВЫЕ ЗАДАНИЯ ДЛЯ ПРОВЕДЕНИЯ ИТОГОВОЙ ГОСУДАРСТВЕННОЙ АТТЕСТАЦИИ РЕЗИДЕНТОВ

ВЫПУСКА 2012-2013 ГГ. ПО СПЕЦИАЛЬНОСТИ «АКУШЕРСТВО И ГИНЕКОЛОГИЯ»

C) активизация процесса тромбообразованияD) увеличение активности антитромбина IIIE) спазм мелких артерий и артериол

475.НАИМЕНЕЕ характерные лабораторные изменения при АФСА) выявление в крови антикардиолипиновых антителB) наличие волчаночного антикоагулянтаC) увеличение количества тромбоцитовD) тромбоцитопенияE) ложноположительная RW

476.Какой, из ниже перечисленных факторов невынашивания, НАИМЕНЕЕ опасен тромботическими осложнениями во время беременностиА) антифосфолипидный синдромB) анти ХГ- сенсибилизацияC) генетические формы тромбофил ииD) гипергомоцистеинемияE) гиперандрогения

477.Пациентке с «классическим» АФС, с тромбозами в анамнезе во время беременности НАИМЕНЕЕ показано назначение:А. гепаринотерапии Б. ацетилсалициловой кислотыВ. препаратов кальций Г. преднизолона Д. витаминов группы D

478.К показаниям к проведению плазмафереза при АФС во время беременности НЕ относятся:А. гиперкоагуляция, не поддающаяся коррекции медикаментозными средствамиБ. аллергические реакции на введение антикоагулянтов и антиагрегантовВ. активация инфекции во время беременности Г. обострение язвенной болезни желудка, когда необходимо прекратить иммуносупрессивную терапию.Д. угроза прерывания беременности

479.Ведение беременности у женщин с АФС представлено ниже. Выберите наименее значимое обследование:А. с ранних сроков гестации определение волчаночного антикоагулянтаБ. гемостазиологический контрольВ. КТГ плодаГ. во II и III триместрах исследование функций печени и почекД.УЗИ в допплеровском режиме

480.Определите наименее значимую тактику ведения пациентки с АФС в родах и после родов:А. в родах кардиомониторный контроль плодаБ. контроль гемостазиограммы перед родами и в родах.В. отмена гепарина за 12-24 часа до родоразрешения и возобновление терапии через 12 часовГ. после родов глюкокортикоидная терапия в течение 2 недель

Page 172: postgraduate.kaznmu.kz€¦ · Web view58 Больная И, 28 лет, доставлена с жалобами на внезапно возникшую боль внизу живота

С.Ж.АСФЕНДИЯРОВАТЫНДАҒЫҚАЗАҚ ҰЛТТЫҚ МЕДИЦИНА

УНИВЕРСИТЕТІ

КАЗАХСКИЙНАЦИОНАЛЬНЫЙМЕДИЦИНСКИЙ УНИВЕРСИТЕТ ИМЕНИ С.Д.АСФЕНДИЯРОВА

КАФЕДРА ИНТЕРНАТУРЫ И РЕЗИДЕНТУРЫ ПО АКУШЕРСТВУ И ГИНЕКОЛОГИИТЕСТОВЫЕ ЗАДАНИЯ ДЛЯ ПРОВЕДЕНИЯ ИТОГОВОЙ ГОСУДАРСТВЕННОЙ АТТЕСТАЦИИ РЕЗИДЕНТОВ

ВЫПУСКА 2012-2013 ГГ. ПО СПЕЦИАЛЬНОСТИ «АКУШЕРСТВО И ГИНЕКОЛОГИЯ»

Д. переливание тромбоцитарной массы

481.У повторнобеременной в анамнезе 4 выкидыша подряд. При сроке беременности 18-19 недель появились жалобы на покраснение, отёк, болезненность по ходу вен ног. Ваш предположительный диагноз:А. беременность 18-19 недель. Привычное невынашивание. Венозный тромбоз сосудов ногБ. беременность 18-19 недель. АФС. Привычное невынашивание. Венозный тромбоз сосудов ногВ. беременность 18-19 недель. ОАА. Венозный тромбоз сосудов ногГ. беременность 18-19 недель. Привычное невынашивание. Тромбофлебит сосудов ногД. беременность 18-19 недель. Привычное невынашивание. Варикозная болезень

482.НАИМЕНЕЕ физиологичные изменения во время беременности, увеличивающие риск венозных тромбозовА) сдавление беременной маткой нижней полой вены и подвздошных вен B) увеличение объема крови и недостаточность венозных клапановC) стаз крови в результате гормональных измененийD) тромбоцитопения с гиперагрегацией тромбоцитов E) ингибиция фибринолиза

483.НАИМЕНЕЕ обоснованная терапия при АФС во время беременностиА) низкомолекулярные гепарины (фраксипарин, клексан)B) антиагреганты и антикоагулянты (курантил, тромбоасс, сулодексид)C) антикоагулянты непрямого действия (варфарин) в первом триместре беременности D) глюкокортикоиды (преднизолон, метипред) при вторичном АФСE) плазмоферез

484.Какой препарат в первом триместре беременности при лечении АФС может вызвать эмбриопатию с нарушением роста эпифизов и гипоплазией носовой перегородкиА) клексанB) курантилC) варфаринD) метипредE) сулодексид

485.НАИМЕНЕЕ характерное нарушение плазменного звена гемостаза при АФСА) удлинение АЧТВ более 10% B) положительный этаноловый тестC) повышение ПДФD) повышение уровня АТ III E) снижение протеина С

486.Прегравидарная подготовка женщин с АФС НЕ включает:А) проведение антибактериальной и противовирусной терапии B) иммуномодулирующую и системную энзимотерапиюC) коррекцию гемостаза (антикоагулянты и антиагреганты)D) назначение глюкокортикоидовE) переливание тромбоцитарной массы

Page 173: postgraduate.kaznmu.kz€¦ · Web view58 Больная И, 28 лет, доставлена с жалобами на внезапно возникшую боль внизу живота

С.Ж.АСФЕНДИЯРОВАТЫНДАҒЫҚАЗАҚ ҰЛТТЫҚ МЕДИЦИНА

УНИВЕРСИТЕТІ

КАЗАХСКИЙНАЦИОНАЛЬНЫЙМЕДИЦИНСКИЙ УНИВЕРСИТЕТ ИМЕНИ С.Д.АСФЕНДИЯРОВА

КАФЕДРА ИНТЕРНАТУРЫ И РЕЗИДЕНТУРЫ ПО АКУШЕРСТВУ И ГИНЕКОЛОГИИТЕСТОВЫЕ ЗАДАНИЯ ДЛЯ ПРОВЕДЕНИЯ ИТОГОВОЙ ГОСУДАРСТВЕННОЙ АТТЕСТАЦИИ РЕЗИДЕНТОВ

ВЫПУСКА 2012-2013 ГГ. ПО СПЕЦИАЛЬНОСТИ «АКУШЕРСТВО И ГИНЕКОЛОГИЯ»

487.Преждевременный дородовый разрыв плодных оболочек (ПДРПО) – происходит в срокиА) до 22 недель беременностиB) от 22 до 34 недель C) от 28 до 34 недельD) от 22 до 37 недельE) от 28 до 37 недель

488.НАИМЕНЕЕ оправданная диагностическая манипуляция при ПДРПО:А) осмотр в зеркалахB) влагалищное исследованиеC) УЗИ плодаD) «нитразиновый тест» E) контрольная подкладная

489.Из дополнительных диагностических мероприятий при ПДРПО НАИМЕНЕЕ показано проведение: А) ОАКB) УЗИ плодаC) измерения температуры, АД, пульсаD) контроля ЧСС плодаE) допплерометрии

490.Антибиотикопрофилактика при ПДРПО в сроке 24-34 недели беременностиА) не проводитсяB) начинается сразу после постановки диагноза – эритромицин 250 мг х 3 раза в течение 7 днейC) начинается через 18 часов безводного периода – эритромицин 250 мг х 3 раза до начала родовой деятельностиD) начинается сразу после постановки диагноза – ампициллин 2 г в/в каждые 6 часов до рождения плодаE) начинается сразу после постановки диагноза – цефазолин 2 г в/в х 3 раза до начала родовой деятельности

491.Антибиотикопрофилактика при ПДРПО в сроке 34-37 недель беременностиА) не проводитсяB) начинается сразу после постановки диагноза – эритромицин 250 мг х 3 раза в течение 7 днейC) начинается через 18 часов безводного периода – эритромицин 250 мг х 3 раза до начала родовой деятельностиD) начинается с началом родовой деятельности – ампициллин 2 г в/в каждые 6 часов до рождения ребенкаE) начинается сразу после постановки диагноза – цефазолин 2 г в/в х 3 раза до начала родовой деятельности

492.Продолжительность выжидательной тактики при ПДРПО не зависит от: А) срока беременностиB) желания женщиныC) состояния плодаD) наличия инфекции

Page 174: postgraduate.kaznmu.kz€¦ · Web view58 Больная И, 28 лет, доставлена с жалобами на внезапно возникшую боль внизу живота

С.Ж.АСФЕНДИЯРОВАТЫНДАҒЫҚАЗАҚ ҰЛТТЫҚ МЕДИЦИНА

УНИВЕРСИТЕТІ

КАЗАХСКИЙНАЦИОНАЛЬНЫЙМЕДИЦИНСКИЙ УНИВЕРСИТЕТ ИМЕНИ С.Д.АСФЕНДИЯРОВА

КАФЕДРА ИНТЕРНАТУРЫ И РЕЗИДЕНТУРЫ ПО АКУШЕРСТВУ И ГИНЕКОЛОГИИТЕСТОВЫЕ ЗАДАНИЯ ДЛЯ ПРОВЕДЕНИЯ ИТОГОВОЙ ГОСУДАРСТВЕННОЙ АТТЕСТАЦИИ РЕЗИДЕНТОВ

ВЫПУСКА 2012-2013 ГГ. ПО СПЕЦИАЛЬНОСТИ «АКУШЕРСТВО И ГИНЕКОЛОГИЯ»

E) готовности родовых путей

493.НАИМЕНЕЕ информативный признак хориоамнионита при ПДРПОА) лихорадка матери (выше 37,8 С)B) тахикардия у плода (выше 160)C) тахикардия у матери (более 100 уд/мин)D) лейкоцитоз 11 тысE) болезненность матки

494.НАИБОЛЕЕ оптимальная тактика при хориоамнионите при ПДРПОА) пролонгирование беременности на фоне антибактериальной терапииB) немедленное родовозбуждение с консервативным ведением родовC) немедленное кесарево сечениеD) немедленное родовозбуждение, при отсутствии возможности родоразрешения в течение 4 часов – кесарево сечение по обычной методикеE) подготовка к родам простагландинами с последующим родовозбуждением

495.Безводный период при преждевременном разрыве плодных оболочек – этоА) период между разрывом плодных оболочек и началом родовой деятельностиB) период между разрывом плодных оболочек и полным открытием маточного зеваC) период между разрывом плодных оболочек и рождением плодаD) период между разрывом плодных оболочек и окончанием родовE) период между разрывом плодных оболочек и окончанием раннего послеродового периода

496.Состояние недоношенных новорожденныхК категории экстремально низкой массы тела (ЭНМТ) относятся новорожденные, рожденные с массой А) от 350,0 до 499,0B) от 500,0 до 1000,0C) от 1001,0 до 1499,0 D) от 1500,0 до 2000,0E) от 2001,0 до 2500,0

497.К категории очень низкой массы тела (ОНМТ) относятся новорожденные, рожденные с массой А) от 350,0 до 499,0B) от 500,0 до 1000,0C) от 1001,0 до 1499,0 D) от 1500,0 до 2000,0E) от 2001,0 до 2500,0 498.По какой шкале проводят оценку функции дыхания у недоношенных новорожденных А) по АпгарB) по СильвермануC) по Штарку D) по Мак-ДональдуE) по Ферриману-Голвею

Page 175: postgraduate.kaznmu.kz€¦ · Web view58 Больная И, 28 лет, доставлена с жалобами на внезапно возникшую боль внизу живота

С.Ж.АСФЕНДИЯРОВАТЫНДАҒЫҚАЗАҚ ҰЛТТЫҚ МЕДИЦИНА

УНИВЕРСИТЕТІ

КАЗАХСКИЙНАЦИОНАЛЬНЫЙМЕДИЦИНСКИЙ УНИВЕРСИТЕТ ИМЕНИ С.Д.АСФЕНДИЯРОВА

КАФЕДРА ИНТЕРНАТУРЫ И РЕЗИДЕНТУРЫ ПО АКУШЕРСТВУ И ГИНЕКОЛОГИИТЕСТОВЫЕ ЗАДАНИЯ ДЛЯ ПРОВЕДЕНИЯ ИТОГОВОЙ ГОСУДАРСТВЕННОЙ АТТЕСТАЦИИ РЕЗИДЕНТОВ

ВЫПУСКА 2012-2013 ГГ. ПО СПЕЦИАЛЬНОСТИ «АКУШЕРСТВО И ГИНЕКОЛОГИЯ»

499.Какой признак НЕ оценивают по шкале СильверманаА) движение грудной клетки и втяжение передней брюшной стенки на вдохеB) втяжение межреберийC) мышечный тонусD) положение нижней челюсти и участие крыльев носа в акте дыханияE) втяжение грудины

500.Какой признак НАИМЕНЕЕ информативен при определении степени зрелости недоношенных детей по ВОЗА) исчерченность стопB) сосок грудной железыC) половые органыD) «сыровидную» смазку на кожеE) положение головы при тракции за руки

501.НАИМЕНЕЕ информативный признак определения степени зрелости недоношенных детей по ВОЗА) масса тела и длинаB) состояние швов и родничковC) ушные раковиныD) поза на руке исследователяE) исчерченность мошонки

502.НАИБОЛЕЕ распространенная причина перинатальной смертности среди недоношенных новорожденныхА) асфиксияВ) респираторный дистресс–синдромС) генерализованная инфекцияD) врожденные пороки развитияЕ) внутричерепные кровоизлияния

503.При каких сроках беременности можно говорить о перенашивании А) 40 недель и болееB) 41 неделя и болееC) 42 недели и болееD) 43 недели и более E) после даты родов

504.Какое средство НАИМЕНЕЕ оправдано при тенденции к перенашиванию беременности с целью индукции родовА) мизопростолB) динопростонC) ламинарии D) мифепристонE) пальцевая отслойка нижнего полюса плодного пузыря

505.НАИМЕНЕЕ характерное осложнение при запоздалых родах

Page 176: postgraduate.kaznmu.kz€¦ · Web view58 Больная И, 28 лет, доставлена с жалобами на внезапно возникшую боль внизу живота

С.Ж.АСФЕНДИЯРОВАТЫНДАҒЫҚАЗАҚ ҰЛТТЫҚ МЕДИЦИНА

УНИВЕРСИТЕТІ

КАЗАХСКИЙНАЦИОНАЛЬНЫЙМЕДИЦИНСКИЙ УНИВЕРСИТЕТ ИМЕНИ С.Д.АСФЕНДИЯРОВА

КАФЕДРА ИНТЕРНАТУРЫ И РЕЗИДЕНТУРЫ ПО АКУШЕРСТВУ И ГИНЕКОЛОГИИТЕСТОВЫЕ ЗАДАНИЯ ДЛЯ ПРОВЕДЕНИЯ ИТОГОВОЙ ГОСУДАРСТВЕННОЙ АТТЕСТАЦИИ РЕЗИДЕНТОВ

ВЫПУСКА 2012-2013 ГГ. ПО СПЕЦИАЛЬНОСТИ «АКУШЕРСТВО И ГИНЕКОЛОГИЯ»

А) дистресс плодаB) синдром мекониальной аспирацииC) клинически узкий тазD) родовой травматизмE) стремительные роды

506.Редкое осложнение при перенашиванииА) внутриутробное инфицирование плодаB) ЗВРПC) тромбоэмболические осложнения D) ПОНРП E) СДР и развитие пневмопатии

507.НАИБОЛЕЕ информативный метод определения состояния плода при перенашивании беременностиА) амниоскопияB) УЗИC) допплерографияD) КТГE) определение суточной экскреции эстриола

508.НАИБОЛЕЕ определяющий фактор повышенной чувствительности переношенного плода к гипоксии и родовым травмамА) старение плацентыB) снижение способности к конфигурации головкиC) уменьшение количества околоплодных водD) тощая пуповинаE) более выраженная зрелость ЦНС

509.НАИМЕНЕЕ характерное изменение в плаценте при перенашиванииА) белые инфарктыB) кальцификаты, участки жирового перерожденияC) уменьшение толщины плацентыD) уменьшение соотношения массы плаценты и массы плодаE) прокрашивание тканей плаценты, оболочек, пуповины «зеленью»

510.Для каких ВПР плода НАИМЕНЕЕ характерно перенашивание беременностиА) анэнцефалия B) поликистоз почекC) болезнь ДаунаD) гидроцефалия E) атрезия пищевода

511.НАИМЕНЕЕ характерный для пролонгированной беременности УЗ критерийА) крупные размеры плодаB) нормальное количество водC) обычные контуры головкиD) отсутствие дыхательных движений плода

Page 177: postgraduate.kaznmu.kz€¦ · Web view58 Больная И, 28 лет, доставлена с жалобами на внезапно возникшую боль внизу живота

С.Ж.АСФЕНДИЯРОВАТЫНДАҒЫҚАЗАҚ ҰЛТТЫҚ МЕДИЦИНА

УНИВЕРСИТЕТІ

КАЗАХСКИЙНАЦИОНАЛЬНЫЙМЕДИЦИНСКИЙ УНИВЕРСИТЕТ ИМЕНИ С.Д.АСФЕНДИЯРОВА

КАФЕДРА ИНТЕРНАТУРЫ И РЕЗИДЕНТУРЫ ПО АКУШЕРСТВУ И ГИНЕКОЛОГИИТЕСТОВЫЕ ЗАДАНИЯ ДЛЯ ПРОВЕДЕНИЯ ИТОГОВОЙ ГОСУДАРСТВЕННОЙ АТТЕСТАЦИИ РЕЗИДЕНТОВ

ВЫПУСКА 2012-2013 ГГ. ПО СПЕЦИАЛЬНОСТИ «АКУШЕРСТВО И ГИНЕКОЛОГИЯ»

E) плацента без выраженных признаков старения

512.Какой признак НАИМЕНЕЕ характерен для синдрома Беллентайна -РунгеА) наличие пушковых волосB) повышенная плотность костейC) снижение тургора кожиD) сухая «пергаментная» кожаE) слабая выраженность слоя подкожно-жировой клетчатки

513.НАИМЕНЕЕ характерный признак пролонгированной беременностиА) крупные размеры плодаB) ребенок без признаков переношенностиC) физиологически удлиненная беременностьD) ЗВРПE) плацента без выраженных признаков старения

514.Диагноз запоздалых родов устанавливают на основании:А) осмотра последаB) после осмотра новорожденногоC) толщины плацентыD) количества околоплодных водE) срока беременности

515.При переношенной беременности количество околоплодных вод:А) уменьшаетсяB) умеренно увеличиваетсяC) резко увеличиваетсяD) остается неизмененнымE) отсутствуют

516. Уменьшение толщины плаценты, наличие петрификатов, маловодие характерны для: А) переношенной беременностиB) недоношенной беременностиC) острой гипоксии плодаD) гемолитической болезни плодаE) неиммунной водянки плода

517.При возникновении угрожаемого состояния плода на фоне переношенной беременности целесообразно произвести:А) амниоцентез для определения состояния плодаB) определение pH крови плодаC) подготовку организма к родам D) родовозбуждениеE) кесарево сечение 518. К схеме использования мизопростола с целью индукции родов при перенашивании беременности НАИМЕНЕЕ всего подходит следующееА) введение 25-50 мкг мизопростола каждые 6 часов до созревания шейки матки

Page 178: postgraduate.kaznmu.kz€¦ · Web view58 Больная И, 28 лет, доставлена с жалобами на внезапно возникшую боль внизу живота

С.Ж.АСФЕНДИЯРОВАТЫНДАҒЫҚАЗАҚ ҰЛТТЫҚ МЕДИЦИНА

УНИВЕРСИТЕТІ

КАЗАХСКИЙНАЦИОНАЛЬНЫЙМЕДИЦИНСКИЙ УНИВЕРСИТЕТ ИМЕНИ С.Д.АСФЕНДИЯРОВА

КАФЕДРА ИНТЕРНАТУРЫ И РЕЗИДЕНТУРЫ ПО АКУШЕРСТВУ И ГИНЕКОЛОГИИТЕСТОВЫЕ ЗАДАНИЯ ДЛЯ ПРОВЕДЕНИЯ ИТОГОВОЙ ГОСУДАРСТВЕННОЙ АТТЕСТАЦИИ РЕЗИДЕНТОВ

ВЫПУСКА 2012-2013 ГГ. ПО СПЕЦИАЛЬНОСТИ «АКУШЕРСТВО И ГИНЕКОЛОГИЯ»

B) не более 50 мкг на одно введениеC) интравагинальное введение (в задний свод влагалища)D) интрацервикальное введениеE) общая суточная доза не более 200 мкг

519. К правилам использования мизопростола с целью индукции родов при перенашивании беременности НАИМЕНЕЕ всего подходит следующееА) проинформировать беременную и получить письменное согласиеB) после введения простагландина необходимо уложить беременную на 30 минутC) провести КТГ контроль или аускультацию плода через 30 минутD) при появлении условий (зрелая шейка матки) перевести в родильный блок, провести амниотомиюE) при безэффективности повторно применить мизопростол по схеме

520. Основное преимущество метода пальцевой отслойки нижнего полюса плодного пузыря с целью индукции родов при перенашивании беременности А) безболезненный методB) дородовый разрыв плодных оболочек не характеренC) эффективный методD) метод простой в исполнении и не требует затратE) отсутствует риск кровотечения

521. К схеме введения окситоцина с целью индукции родов при перенашивании беременности НАИМЕНЕЕ всего подходит следующееА) 5 ЕД окситоцина развести в 400 мл изотонического раствораB) введение начать с 4 капель в минуту (12 мл/ч)C) увеличивать скорость инфузии каждые 15 минутD) продолжить введение окситоцина до родоразрешения и первые 30 минут после родовE) максимальная скорость введения окситоцина в исключительных случаях не должна быть более 64 капель в минуту

522. Эффективность индукции родов оценивается А) через 30 минут от начала введения окситоцинаB) через 1 час от начала введения окситоцинаC) через 2 часа от начала введения окситоцинаD) через 4 часа от начала введения окситоцинаE) через 6 часов от начала введения окситоцина

523. К правилам инфузии окситоцина с целью индукции родов при перенашивании беременности НАИМЕНЕЕ всего подходит следующееА) инфузия только при вскрытом плодном пузыреB) контроль инфузии по возможности инфузоматомC) аускультация сердцебиения плода каждые 30 минутD) оценка схваток каждые 30 минутE) при возникновении гипертонуса или признаков угрожающего состояния плода уменьшение скорости инфузии

524. При гиперстимуляции матки на фоне инфузии окситоцина НАИМЕНЕЕ всего показано

Page 179: postgraduate.kaznmu.kz€¦ · Web view58 Больная И, 28 лет, доставлена с жалобами на внезапно возникшую боль внизу живота

С.Ж.АСФЕНДИЯРОВАТЫНДАҒЫҚАЗАҚ ҰЛТТЫҚ МЕДИЦИНА

УНИВЕРСИТЕТІ

КАЗАХСКИЙНАЦИОНАЛЬНЫЙМЕДИЦИНСКИЙ УНИВЕРСИТЕТ ИМЕНИ С.Д.АСФЕНДИЯРОВА

КАФЕДРА ИНТЕРНАТУРЫ И РЕЗИДЕНТУРЫ ПО АКУШЕРСТВУ И ГИНЕКОЛОГИИТЕСТОВЫЕ ЗАДАНИЯ ДЛЯ ПРОВЕДЕНИЯ ИТОГОВОЙ ГОСУДАРСТВЕННОЙ АТТЕСТАЦИИ РЕЗИДЕНТОВ

ВЫПУСКА 2012-2013 ГГ. ПО СПЕЦИАЛЬНОСТИ «АКУШЕРСТВО И ГИНЕКОЛОГИЯ»

А) уложить беременную на левый бокB) уменьшить скорость инфузииC) прекратить введение окситоцинаD) обеспечить подачу кислородаE) провести инфузию физиологического раствора 500 мл за 15 минут

525. НАИБОЛЕЕ характерная причина формирования клинически узкого таза при запоздалых родахА) анатомическое сужение тазаB) крупный плодC) разгибание головкиD) гидроцефалия E) плохая конфигурация головки

526. Среди перечисленных ниже акушерских осложнений перенашивание НАИБОЛЕЕ характерно дляА) предлежания плацентыB) преэклампсииC) гемолитической болезни плодаD) анэнцефалииЕ) многоплодной беременности

527. При перенашивании беременности с целью подготовки к родам противопоказано применение А) мизопростолаB) динопростонаC) энзапростаD) ламинарийE) пальцевой отслойки нижнего полюса плодного пузыря

528. Простагландин Е 2А) мизопростолB) динопростонC) энзапростD) мифепристонE) сайтотек

529. Форма препарата ДИНОПРОСТОН для индукции родовА) таблетка для интравагинального введенияB) таблетка для внутреннего примененияC) гель для интравагинального и интрацервикального примененияD) свечи для ректального применения E) раствор для парентерального введения

530. В стационаре первого уровня допускается выхаживание недоношенных детей со стабильными функциями дыхания и кровообращения, если их масса превышаетА) 500,0 граммB) 1000,0 граммC) 1500,0 грамм

Page 180: postgraduate.kaznmu.kz€¦ · Web view58 Больная И, 28 лет, доставлена с жалобами на внезапно возникшую боль внизу живота

С.Ж.АСФЕНДИЯРОВАТЫНДАҒЫҚАЗАҚ ҰЛТТЫҚ МЕДИЦИНА

УНИВЕРСИТЕТІ

КАЗАХСКИЙНАЦИОНАЛЬНЫЙМЕДИЦИНСКИЙ УНИВЕРСИТЕТ ИМЕНИ С.Д.АСФЕНДИЯРОВА

КАФЕДРА ИНТЕРНАТУРЫ И РЕЗИДЕНТУРЫ ПО АКУШЕРСТВУ И ГИНЕКОЛОГИИТЕСТОВЫЕ ЗАДАНИЯ ДЛЯ ПРОВЕДЕНИЯ ИТОГОВОЙ ГОСУДАРСТВЕННОЙ АТТЕСТАЦИИ РЕЗИДЕНТОВ

ВЫПУСКА 2012-2013 ГГ. ПО СПЕЦИАЛЬНОСТИ «АКУШЕРСТВО И ГИНЕКОЛОГИЯ»

D) 2000,0 граммE) 2500,0 грамм

531.Наиболее лучшей профилактикой послеродового кровотечения вероятно является:А) постоянная катетеризация мочевого пузыря во время родовБ) постоянная катетеризация периферических вен во время родовВ) выжидательная тактика ведения ведение третьего периода родовГ) активное ведение третьего периода родовД) наружный массаж матки после выделения последа532. К этиотропной терапии при травме родовых путей наиболее вероятно относится:А) наружный массаж маткиБ) ручное отделение частей последаВ)ушивание разрывов родовых путейГ) бимануальная компрессия маткиД) переливание свежезамороженной плазмы533.Какая манипуляция наиболее вероятно относится к временным методам остановки послеродового кровотечения:А) наружный массаж маткиб) 1000 мг мизопростола в прямую кишкув) введение утеротониковг) ручное обследование полости маткид)бимануальная компрессия матки534.К временным методам остановки послеродового кровотечения вероятнее всего относится:А) наружный массаж маткиб)прижатие брюшной аортыв) наложение компрессионных швовг) ручное обследование полости маткид) 1000 мг мизопростола в прямую кишку535.Роженица 38 лет находится в родовом зале после срочных родов живым ребенком массой 4500. Через 10 минут после самостоятельного отделения и выделения целого последа из половых путей началось обильное кровотечение, достигшее 300 мл, и продолжается. Родовые пути целы. Матка при пальпации мягкая, дно на уровне пупка. Что вероятнее всего нужно сделать в первую очередь: а) внутривенно капельное введение 10 ЕД окситоцина б) прижатие брюшной аортыв) бимануальная компрессия маткиг) ручное обследование полости маткид) 1000 мг мизопростола в прямую кишку536.У родильницы О. 32 лет ранний послеродовый период осложнился атоническим кровотечением. Начаты мероприятия по остановке кровотечения. Какие из перечисленных показаний являются наиболее вероятным показанием для лапаротомии? А) отсутствие эффекта от наружного массажа матки Б) неэффективность ручного обследования маткиВ) неэффективность бимануальной компрессии матки Г) неэффективность введение мизопростола 1000 мг perrectumД) отсутствие эффекта от сдавливания брюшной аорты

Page 181: postgraduate.kaznmu.kz€¦ · Web view58 Больная И, 28 лет, доставлена с жалобами на внезапно возникшую боль внизу живота

С.Ж.АСФЕНДИЯРОВАТЫНДАҒЫҚАЗАҚ ҰЛТТЫҚ МЕДИЦИНА

УНИВЕРСИТЕТІ

КАЗАХСКИЙНАЦИОНАЛЬНЫЙМЕДИЦИНСКИЙ УНИВЕРСИТЕТ ИМЕНИ С.Д.АСФЕНДИЯРОВА

КАФЕДРА ИНТЕРНАТУРЫ И РЕЗИДЕНТУРЫ ПО АКУШЕРСТВУ И ГИНЕКОЛОГИИТЕСТОВЫЕ ЗАДАНИЯ ДЛЯ ПРОВЕДЕНИЯ ИТОГОВОЙ ГОСУДАРСТВЕННОЙ АТТЕСТАЦИИ РЕЗИДЕНТОВ

ВЫПУСКА 2012-2013 ГГ. ПО СПЕЦИАЛЬНОСТИ «АКУШЕРСТВО И ГИНЕКОЛОГИЯ»

537.Ранний послеродовый период. Кровопотеря 250,0 мл и продолжается. Матка на уровне пупка, мягкая. После наружного массажа матка сократилась, но затем вновь расслабилась. Кровопотеря 400,0 мл, АД 90/50 мм рт. ст, пульс 100 ударов в минуту. Наиболее вероятная тактика ведения? А) приступить к переливанию кровиБ) вводить внутривенно окситоцинВ) провести хирургический гемостазГ) ручное обследование полости маткиД) бимануальная компрессия матки538. У роженицы Д., 28 лет, родившей плод массой 4300,0, при осмотре шейки матки на зеркалах обнаружен кровоточащий разрыв шейки матки с переходом на свод влагалища. Что в первую очередь наиболее вероятно нужно сделать:А) ушивание разрыва шейки матки Б) бимануальную компрессию маткиВ)ручное обследование полости маткиГ) экстирпацию матки без придатков Д) наружное акушерское исследование матки539. В последовом периоде в течение 30 минут, плацента не отделилась. Кровотечение из половых путей 250,0 мл и продолжается. Какова наиболее вероятная Ваша тактика?А) внутривенное введение окситоцинаБ) контролируемое подтягивание за пуповинуВ) введение физиологического раствораГ) ручное отделение и выделение последа Д) наружный массаж матки540.У многорожавшей 32 лет, произведено ручное обследование полости матки в связи с сомнением ее целостности, но матка плохо сокращается, кровопотеря достигла 700,0 мл и продолжается. Наиболее вероятная Ваша тактика?А) повторное ручное обследование полости маткиБ) введение мизопростола 1000 мг perrectumВ) внутривенное введение окситоцинаГ) бимануальная компрессия маткиД) наложение гемостатических швов541.После рождения последа, целостность оболочек под сомнением. Началось кровотечение. Матка периодически расслабляется. Наиболее вероятная Ваша тактика?А) введение окситоцинаБ) наружный массаж маткиВ) прижать аорту кулакомГ)ручное обследование полости матки Д) проверить целостность последа542.У первородящей 20 лет с тяжелой анемией через 10 минут последового периода, кровопотеря 300,0 мл и продолжается. Наиболее вероятная Ваша тактика? А) внутривенное введение окситоцинаБ) ручное отделение и выделение последа В) введение мизопростола 1000 мг Г) наружный массаж маткиД) бимануальная компрессия матки543. При обнаружении дефекта последа наиболее вероятная Ваша тактика:А) внутривенное введение окситоцинаБ) введение мизопростола 1000 мг

Page 182: postgraduate.kaznmu.kz€¦ · Web view58 Больная И, 28 лет, доставлена с жалобами на внезапно возникшую боль внизу живота

С.Ж.АСФЕНДИЯРОВАТЫНДАҒЫҚАЗАҚ ҰЛТТЫҚ МЕДИЦИНА

УНИВЕРСИТЕТІ

КАЗАХСКИЙНАЦИОНАЛЬНЫЙМЕДИЦИНСКИЙ УНИВЕРСИТЕТ ИМЕНИ С.Д.АСФЕНДИЯРОВА

КАФЕДРА ИНТЕРНАТУРЫ И РЕЗИДЕНТУРЫ ПО АКУШЕРСТВУ И ГИНЕКОЛОГИИТЕСТОВЫЕ ЗАДАНИЯ ДЛЯ ПРОВЕДЕНИЯ ИТОГОВОЙ ГОСУДАРСТВЕННОЙ АТТЕСТАЦИИ РЕЗИДЕНТОВ

ВЫПУСКА 2012-2013 ГГ. ПО СПЕЦИАЛЬНОСТИ «АКУШЕРСТВО И ГИНЕКОЛОГИЯ»

В) наружный массаж маткиГ) ручное отделение и выделение последа Д) бимануальная компрессия матки544.Наиболее вероятная тактика при частичном истинном приращении плаценты:А) введение окситоцинаБ) наблюдение за тонусом маткиВ) наружный массаж маткиГ) ручное обследование полости матки Д)ампутация или экстирпация матки545.Операция ручного отделения плаценты и ее выделения показана при:а) отсутствии признаков отделения плаценты в течение 30 мин после рождения плодаб) появлении кровянистых выделений и наличии признаков отделения плацентыв) кровопотере в объеме 100 мл и отсутствии признаков отделения плацентыг) появлении признаков отделения плаценты и отсутствии кровянистых выделенийд) сразу после рождения плода у женщин с рубцом на матке546.Физиологическая остановка кровотечения с плацентарной площадки вероятно всего обусловлена:А) внутривенным введением сокращающих средствB)уменьшением объема матки C) ускореннымтромбообразованием на плацентарной площадкеD) особенностью строения спиралевидных артерий плацентарной площадкиE)миотампонадой, тромботампонадой, вазотампонадой547. В раннем послеродовом периоде появилось обильное кровотечение из половых путей. Кровопотеря 700 мл и продолжается. Матка дряблая медикаментозные методы лечения без эффекта. Послед цел. Родовые пути целы.Наиболее вероятная правильная тактика на данном этапе:А) наружный массаж матки Б) введение мизопростола 1000 мг В)наложение компрессионных швовГ) ручное обследование полости маткиД) бимануальная компрессия матки548.В раннем послеродовом периоде появилось обильное кровотечение из половых путей. Кровопотеря 1500 мл и продолжается. Матка дряблая. Гемостатические швы без эффекта. Послед цел. Родовые пути целы.Наиболее вероятная правильная тактика на данном этапе:А) прижатие аорты брюшиныБ) введение мизопростола 1000 мг В)ампутация или экстирпация маткиГ) ручное обследование полости маткиД) бимануальная компрессия матки549.У роженицы после рождения плода сразу появилось кровотечение из родовых путей. Наиболее правильная тактика врача:A) вводить сокращающие, B) осмотреть родовые путиC)ручное отделение последа D) осмотреть родовые пути E) наружный массаж матки

Page 183: postgraduate.kaznmu.kz€¦ · Web view58 Больная И, 28 лет, доставлена с жалобами на внезапно возникшую боль внизу живота

С.Ж.АСФЕНДИЯРОВАТЫНДАҒЫҚАЗАҚ ҰЛТТЫҚ МЕДИЦИНА

УНИВЕРСИТЕТІ

КАЗАХСКИЙНАЦИОНАЛЬНЫЙМЕДИЦИНСКИЙ УНИВЕРСИТЕТ ИМЕНИ С.Д.АСФЕНДИЯРОВА

КАФЕДРА ИНТЕРНАТУРЫ И РЕЗИДЕНТУРЫ ПО АКУШЕРСТВУ И ГИНЕКОЛОГИИТЕСТОВЫЕ ЗАДАНИЯ ДЛЯ ПРОВЕДЕНИЯ ИТОГОВОЙ ГОСУДАРСТВЕННОЙ АТТЕСТАЦИИ РЕЗИДЕНТОВ

ВЫПУСКА 2012-2013 ГГ. ПО СПЕЦИАЛЬНОСТИ «АКУШЕРСТВО И ГИНЕКОЛОГИЯ»

550.У роженицы в раннем послеродовом периоде кровопотеря 1000 мл и продолжается, наложение компрессионных швов без эффекта, признаков ДВС синдрома нет.Наиболее вероятная тактика:А) перевязка внутренних подвздошных артерийБ) перевязка маточных артерий поО’ЛириВ)ампутация матки без придатковГ) ампутация матки с придаткамиД) экстирпация матки